Anda di halaman 1dari 761
SOLUTIONS TO THE PROBLEMS in TRANSPORT PHEOMENA Second Edition (2002) by R. Byron Bird Warren E. Stewart Edwin N. Lightfoot Department of Chemical Engineering University of Wisconsin Madison, Wisconsin 53706 USA This solutions manual has been prepared by the authors of the textbook for use by professors teaching courses in transport phenomena. It contains the solutions to 500 of the unsolved problems in the textbook. No part of this material may be reproduced in any form, electronic, mechanical, photocopying, recording, scanning, or otherwise. JOHN WILEY & SONS, Inc. New York, New York 1A.1 Estimation of dense-gas viscosity. a. Table E.1 gives T, = 126.2 K, p. = 33.5 atm, and pre = 180 x 10~® g/em-s for Nz. The reduced conditions for the viscosity estimation are then: Pr = P/Pe = (1000 + 14.7)/33.5 x 14.7 = 2.06 T, = T/T. = (273.15 + (68 ~ 32)/1.8)/126.2 At this reduced state, Fig. 1.3-1 gives ty = 1.15. Hence, the predicted viscosity is = fy / fe = 1.15x 180 x 10-* = 2.071074 g/em:s. This result is then converted into the requested units by use of Table F.3-4: 2.32 f= 2.07 x 1074 x 6.7197 x 107? = 1.4 x 10™4 Iba,/ft-s 1A.2 Estimation of the viscosity of methyl fluoride. a. CHF has M = 16.04—1.008 + 19.00 = 34.03 g/g-mole, T. = 4.554273, 277.70 K, p, = 58.0 atm, and ¥, = 34.03/0.300 = 113.4 em*/g-mole. The critical viscosity is then estimated as He = 61.6(34.03 x 27.70)'/2(113.4)-2/? = 255.6 micropoise from Eq. 1.3-1a, and Me = 7.70(34.03)"/?(58.0)/9(277.7)-/% = 263.5 micropoise from Eq. 1.3-1b. ‘The reduced conditions for the viscosity estimate are T, = (370+ 273.15)/277.70 = 2.32, pp = 120/58.0 = 2.07, and the predicted yu, from Fig. 1.3-1 is 1.1. The resulting predicted viscosity is M = fle =1.1 x 255.6 x 107° = 2.8 x 107 g/em-s via Eq.1.3-la, or 1.1 x 263.5 x 107° = 2.9 x 107*g/em-s via Eq.1.3-1b. 1A.3 Computa\ n of the viscosities of gases at low density. Equation 1.4-14, with molecular parameters from Table E.1 and collision integrals from Table E.2, gives the following results: For Oz: M = 32.00, 6 = 3.433A, c/k = 113 K. Then at 20°C, xT/e = 293.15/113 = 2.594 and Q, = 1.086, Equation 1.4-14 then gives 6693 x 107°. (3.433)? x 1.086 2.02 x 10-4 g/em-s 2.02 x 107 Pas = 2.02 x 107? mPas. 3 is 2.04 x 10-? mPas. For Na: M = 28.01, o = 3.667A, ¢/x = 99.8 K. Then at 20°C, xT/e = 293.15 /99.8 = 2.937 and 9, = 1.0447. Equation 14-14 then gives ‘The reported value in Table 1. /8.01 x 393.15 # = 2.6693 x 10-5 (3.007 x L04aT = 1.72 x 10~* g/ems = 172x107 Pas 1.72 x 107? mPa-s. ‘The reported value in Table 1.1-3 is 1.75 x 10-? mPa:s. For CHi, M = 16.04, o = 3.780A, e/x = 154 K. Then at 20°C, xT/e = 298.15/154 = 904 and Q, = 1.197. Equation 1.4-14 then gives VI60E x DORIS (3.7807 = 1.07 x 1074 g/ems 1.07 x 107% Pas 1.07 x 10-? mPa:s. = 2.6693 x 10~ ‘The reported value in Table 1.1-3 is 1.09 x 10-? mPa-s. 1A.4 Gas-mixture viscosities at low density. ‘The data for this problem are as follows: Component = M 14, poise x 10° (Hz) 2.016 88.4 2CChF2) 120.92 124.0 Insertion of these data into Eq. 1.4-16 gives the foloowing coefficients for mixtures of Hy and Freon-12 at this temperature: $y =O =10 by - (14 208)” _ 1 1390,92\"4]" oa ee 120102 124.0 2016 = 3.934 1 4 120.92 1) 1a (1240 2 7 2.016 \¥4]" 2016 384 720.92 .0920 Equation 1.4-15 then gives the predicted mixture viscosities: 2 Di= D= A Bi= A+B= 1-2 LezsFig LxsGog cipi/ Dy z2p2/ DO, mix X 10° pobs,poise x 10° 0.00 3.934 1.000 0.0 124.0 (124.0) 124.0 0.25 3.200 0.773 6.9 120.3 127.2 128.1 0.50 2.467 0.546 18.1 113.6 131.7 131.9 0.75 1.734 0.319 38.2 97.2 135.4 135.1 1.00 1.000 0.092 88.4 0.0 (88.4) 88.4 1A.5 Viscosities of chlorine-air mixtures at low density. Equation 1.4-14 and Tables E.1, E.2 give the following viscosities at 75°F (= 273.15 + (75 — 32)/1.8 = 297.03 K) and 1 atm: 0.91, 0) = 4.115A, e1/K = 357K; hence, 1.754, and For component 1, (Clz), Mi = KT /e, = 297.03/357 = 0.832 and Q, fa = 2.6693 x 10-* = 1.304 x 10-4 g/em-s = 0.01304 ep. (a5 For component 2, (air), Mz = 28.97, = 3.617A, e,/« = 97.0K; hence, kT/e1 = 297.03/97.0 = 3.062 and %q,1 = 1.033, and 28.97 X 297.08 2 = 2.6693 x 10-5 = 1.832 x 107“ g/em-s = 0.01832 ep. (3.617)? x 1.033, Eq. 1.4-16 then gives the following coefficients for Eq. 1.4-15 at this temperature: Oy, = by = 10 1 70. m) ( 908) 2 rog.g7\ V8)? $y =— (1+ 1+ V8 \'* 28.97, 3) (3 vist) 70.51 = 0.5339 2 1 28. st) ~e (ques) 1 770.91\1/* $y =— (14 1+ SET V8 \'* 70.91 m1) ( ‘i30i) 38.97 = 1.8360 Equation 1.4-16 then gives the predicted mixture viscosities: m= Me Ys A Bis AtB= l—2 Lephig Lepboe cipn/Dy z2p2/ Ly mix. x 10° 0.00 0.5389 «1.000 0.0 0.01832 0.0183 0.25 0.6504 1.2090 0.005012 0.011365 0.0164 0.50 0.7670 1.4180 0.008501 0.006460 0.0150 0.75 0.8835 1.6270 0.011070 0.002815 0.0139 1.00 1.000 1.8360 (0.01304 (0.0 0.0130 1A.6 Estimation of liquid viscosity. a. The calculated values for Eq. 1.5-9 at 0°C and 100°C are as follows: TK 273.15 373.15 py g/em? 0.9998 0.9584 _ V=M/p, cm/g-mole 18.01 18.80 AU\vap,74, cal/g-mole= 897.5 x 18.016 x 252.16/453.59 8989. 8089. AU vap.r4/ RT = 8989/1.98721/T 16.560 12.120 exp 0.40840 vap,r4/RT 859.6 140.5 Nh/V, g/ems 2.22 x 10-4 = 2.12 x 10-# Predicted liquid viscosity, g/em-s 0.19 0.0298 6. The predicted values for Eq. 1.5-11 at 0°C and 100°C are: _ TK 273.15 373.15 Nh/V, g/em-s 2.22% 10-* 2.12 x 10-4 exp(3.87;/T) 179.7 44.70 Predicted liquid viscosity, g/em-s 0.0398 0.0095 Summary of results: ‘Temperature, °C 0 100 Observed viscosity, centipoise|=}g/em-s%100 1.787 0.2821 Prediction of Eq. 1.5-9 19. 2.98 Prediction of Eq. 1.5-11 3.98 0.95 Both equations give poor predictions. This is not surprising, since the empirical formulas in Eqs. 1.5-8 et seq. are inaccurate for water and for other associated liquids. 1A.7 Molecular velocity and mean free path. From eq. 1.4-1, the mean molecular velocity in O2 at 273.2 K is SRT [Sx 831451 x 10" x 273.2 7M = x 32.00 From eq. 14-3, the mean free path in Oz at 1 atm and 273.2 K is = 4.25 x 10* cm/s RT 82.0578 x 273.2 TT = 9.3 x 107% Vand?pN ~ Von(3 x 10-8)? x 1 x 6.02214 x 108 ae Hence, the ratio of the mean free path to the molecular diameter is (9.3 x 1074/3 x 10-*) = 3.1 x 10* under these conditions. At liquid states, on the other hand, the corresponding ratio would be on the order of unity or even less. I-7 1B.1 Velocity profiles and stress components 4. Ty = Ty, = —Hb, and all other 7, are zero. 0,0, = pb*y?, and all other 0,0, are zero. b. ty = Ty =-2ub, and all other ty are zero. 0,0, = pb y’, pv,y = pryd, = pb’xy, po,v, = pb?x?, and all other pv,0, are zero. c. All ty are zero 12,0, = phy’, pV,Vy = pryv, = —pb xy, PLyv, = pb?x? and all other po,0, are zero. d. Tey = Ty = Ub, Ta, =-2h, and all others are zero. the components of pvv may be given in the matrix: 0,0, =4pb’x? —pv,v, =tpb*xy pv,0, pyv =| poyv, =4pb xy pv,v,=}pb y? poy, pv,0, =—tpb?xz pv,vy=-}pb yz pv,v, = pb?z? 1B.2 A fluid in a state of rigid rotation a. A particle within a rigid body rotating with an angular velocity vector w has a velocity given by v=[wxr]. If the angular velocity vector is in the +z-direction, then there are two nonzero velocity components given by 0, =-w,y and v, = +w,x. Hence the magnitude of the angular velocity vector is b in Problem 1B.1(c). b. For the velocity components of Problem 1B.1(c), a, a, Pe sg ang Se % = 2» ox oy ox oy c. In Eq. 1.2-4, we selected only the linear symmetric combinations of derivatives of the velocity, so that in pure rotation there would be no viscous forces present. In (b) we see that the antisymmetric combination is nonzero in a purely rotational motion. 1B.3 Viscosity of suspensions Expanding the Mooney expression, we get (with € = ¢/¢,) 2 3 et -14( 2%), 1/ 20) 1/20), Ho l-e) 2\l-e 3il-e =1+$o(l+ete?+--)+Bg"(14 2e+--) 418 93 (1e-)t = 14394942 3125 1 segeee Saag) (le aa aa)" The first two terms match exactly with the first two terms in Eq. 1B.3-1. We can make the third term match exactly, by setting 2 8 +Stega whence @, = 0.618 2% and the coefficient of ¢° becomes aeoinoope me ent 2G 48 40.618 20.382 If we try ¢ = 0.70, the coefficients of ¢? and ¢° become 6.70 and 17.6 respectively. This gives a somewhat better find of Vand's data. {-10 1C.1 Some consequences of the Maxwell-Boltzmann equation a. The mean speed is fee atu w= em RT gi, b. First rewrite Eq. 1C.1-4 as gerry, [oer 5PH ay, ai a gy, r = md BAT du, The integral over u, in the numerator of the first factor is zero because the integrand is the product of a factor "u," (an odd function of the integration variable about u, = 0) and an exponential function (an even function), and the range of integration extends equally far in the positive and negative directions. c. The mean kinetic energy per molecule is futemetau | oer sede ave tome Om feet ge XL 2 eer Pay Tm PaePag We and is thus }xT for each degree of freedom. I-\ 1C.2 The wall collision frequency When we change to dimensionless variables in the second line of Eq. 1C.2-1, we get Zan( stig) AEG gach (Gas) PAT (ee) “ey CE ACE Pa oN [+12 1C.3 The pressure in an ideal gas a. The dimensions of the quantities in Eq. 1C.3-1 are 2 if Using these units, one finds that the expression on the right of Eq. 1C.3-1 has units of M/Lt? (which are the same as the units of force per area). b. Combining Eqs. 1C.1-1 and 1C.3-1 we get pean)" . anm( se) (Cider a, fee MAP ay, [er mePeT Ay, nam EY) ete Pas Le tan Lee tas ann 2222)" 68 vee) = mer amidst) ext ute aE ay dud, 1-13 1D.1 Uniform rotation of a fluid a. For the special case that w =5,w, we get =[W xr] = 2,22, 645,10 /x, = W(8,€1ny + 82€1%) = w(—8,y + 85x) Then using Eqs. A.6-1, 2, 13 and 14, we can get the velocity components in cylindrical coordinates 2, =(v-8,) (—8.y+8,x)-8,)=w(-ycos6 + xsin 6) -rsin @cos@ + xcosOsin®) = 0 0% =(v-85)=w((—8,y+8,x)-89)=w((-y)(-sin 6) +xc0s 6) = w(rsin Osin 0 +rcos Ocos 6) = wr Therefore, the angular velocity of every point in the fluid is v,/r=w, which is a constant, and there is no radial velocity. This is the way a rigid body rotates at constant angular velocity. b. The vector operations are (using the abbreviated notation of §A.9 and the Einstein summation convention) (VV) = 9,2, = IE nV Xp = Egy 5 Enmn®; {Vv}y ={V[w xt]}, = ean nd n = EjantOn Sin = & =-{Vv},=-{Vv}} jimi Om and from this last result we see that Vv +(Vv)* =0. c. The results above indicate that for a fluid is a state of pure rotation, the tensor + is identically zero. That is, there are no viscous stresses present in the fluid. This was the assertion made just before Eq. 1.2-4. I-14 1D.2 Force on a surface of arbitrary orientation. a. We can specify the surface area and the orientation of the surface of AOBC as ndS. To project this surface onto the yz-plane, we take the dot product with 5,, so that the area of AOBC is (n-3,)ds. b. The force per unit area on three triangles perpendicular to the three coordinate axes are Force on AOBC =8 71, +8 yyy +3272 Force on AOCA =8, ity, +8, Ay +8. Ty. Force on AOAB=8,ft., +5 fia +87. c. Force balance on the volume OABC is then HAS =(8 hy +8, +8,%.)(0-8,)dS +(8, ty +8 yy +3,My.)(m-By )AdS + (8,7, +B yy +8.7.)(m-3, )aS or m, =[10-8,8, 7% ]+[n-8,5 7, ]+[n-5,8.7%.] +[n-8,8,7,,]+[n-5,8,7]+[n-3,8,7,,] +{n-8,8,7,.]+[n-8,8,7.4]+[n-8,5,7.,] =2E[e88))]=[0 2] \-15 2A.1 Thickness of a falling film. a, The volume flow rate w/p per unit wall width W is obtained from Eq. 2.2.25: _w_ _ vRe _ (1.0037 x 107*)(10) pw 4 4 = 2.509 x 107? em?/s Here the kinematic viscosity v for liquid water at 20°C was obtained from Table 1.1- 2. Since 1 ft=12x2.54 cm, 1 hr =3600 s, and 1 gal=231.00 in’ x(2.54cm/in)*=3785.4 em? (see Appendix F), the result in the requested units is Ww _ a g9sn9 em? 1 ap = 0.02509 em*/s x sree = 0.727 US. gal/hr-ft gal/em? x 30.48 em/ft x 3600 s/hr 6. The film thickness is calculated from Eqs. 2.2-25 and 2.2-22 as s-( wy” geos i pW 3v_vRe\V* = (aa) _ (31.0087 « 1 ~~ \°"(980.665)(1.0) = 0.00361 in. 2 Ws (2.509 10) = 0.009167 cm 2A.2 Determination of capillary radius by flow measurement. Assuming the flow to be laminar, we solve Eq. 2.3-21 for the capillary radius: pa [Sule _ «Beka “\ixpap ~ V aap Insertion of the data in mks units gives (8.1416)(4.829 x 105) = ¥3.186 x 10-18 7.51 x 1074 m R 51 x 107? em ‘As a check on this result, we calculate the corresponding Reynolds number: * qr ZBL = 10-4)(4.03 x 10-5)(0.9552 x 103) — oe ‘This value supports our assumption of laminar flow. Since the entrance length, L. = 0.035DRe = 0.35 cm is less than L, the entrance-effect correction to Ris at most of the order of |(1-(L_/L)]'/4—1], or 0.2 percent of R in the present example. Difficulties with this method include: (1) Inability to account for departures from a straight, circular cylindrical wall geometry. (2) Inability to account for in- advertent spatial and temporal variations of temperature, hence of the fluid density and viscosity. A simpler method is to measure the length L and mass m of a small slug of liquid mercury (or another liquid of known density) injected into the tube, and calculate the mean radius R of the slug as (m/[prZ])'/?, on the assumption that the slug is a right circular cylinder. This method allows comparisons of mean R values for various intervals of the tube length. 2A.3 Volume rate of flow thrugh an annulus. Assuming the flow to be laminar, we use Eq. 2.4-17 to calculate the volume flow rate w/p, with the specifications ie = 0.498/1.1 = 0.45 } = 136.8 (Ib, /ft-hr)(1 hr/3600s) = 3.80 x 107? Ibm /ft-s (Po ~ Pz) = (5.39 psi)(4.6330 x 10° poundals/ft?/psi) = 2.497 x 10* Ibm /ft- R=Llin.=11/12f Here Appendix F has been used for the conversions of units. With these specifica- tions, Eq. 2.4-17 gives w _ (m)(2.497 x 10*)(1.1/12)* 1 = (0.45)?)? 2 ; ‘Eee x Ta [o ~ 49))— Gina | (1 = 0.2025)? 1n(1/0.495) = (0.6748){0.1625] = 0.110 ft3/s = (0.49242) [a — 0.04101) — As a check on our assumption of laminar flow, we calculate the Reynolds num- ber: - ruse _ 2 x mRyu(l + «) _ 2(0.110)(80.3) ~ (31416)(1-1/12)(3.80 x 10-7)(1.45) ‘This value is well within the laminar range, so our assumption of laminar flow is confirmed. 1110 2A.4 Loss of catalyst particles in stack gas. a, Rearrangement of Eq. 2.6-17 gives the terminal velocity ve = D'(p. — p)g/184 in which D is the sphere diameter. Particles settling at vz greater than the centerline gas velocity will not go up the stack. Hence, the value of D that corresponds to be = 1.0 ft/s will be the maximum diameter of particles that can be lost in the stack gas of the present system. Conversions of data to egs units give ve = (1 ft/s)(12 x 2.54 em/ft) = 30.48 em/s p = (0.045 Ibm /ft?)(453.59 g/lbym)((12 x 2.54)~* ft? fem?) =7.2x 10 g/em> Hence, _ [484% __ [(48)(0.00026)(30.48) ~ Vi (1.2 —7.2 x 10-4)(980.7) = 1.1 x 107? em = 110 microns b, Equation 2.6-17 was derived for Re<< 1, but holds approximately up to Re=1. For the system at hand, Deep _ (1.1 x 10-7)(30.5)( # (0.00026) aT Re x10) < 0.93 Hence, the result in a. is approximately correct. Methods are given in Chapter 6 for solving problems of this type without the ereeping-flow assumption. 2-4 2B.1 Different choice of coordinates for the falling film problem. Set up a momentum balance as before, and obtain the differential equation Since no momentum is transferred at ¥=6, then at that plane . This boundary condition enables us to find that C, =-pgécosB, and the momentum flux distribution is Fe =~pgdcos6 1- Note that the momentum flux is in the negative ¥-direction. Insertion of Newton's law of viscosity t,, =—H(dv, /d%) into the foregoing equation gives the differential equation for the velocity distribution: (eset) This first-order differential equation can be integrated to give (HO) The constant C, is zero, because v, =0 at ¥=0. We note that ¥ and x are related by 7/6 =1-(x/8). When this is substituted into the velocity distribution above, we get (fF) which can be rearranged to give Eq. 2.2-18. 2-5 2B.2 Alternate procedure for solving flow problems Substituting Eq. 2.2-14 into Eq. 2.2-10 gives d(_, dv,)_ #v, __ pgcosB 4 12s) = pgcosp or Gap = PEs Integrate twice with respect to x (see Eq. C.1-10) and get = PROB 2 we +C.x+C, Then use the no-slip boundary condition that v, =0 at x=6, and the zero momentum flux boundary condition that dv,/dx=0 at x=0. The second gives C,=0, and the first gives C, =(pgcosB/2)5?. Substitution of these constants into the general solution and rearranging then gives Eq. 2.2-18. 2B.3 Laminar flow in a narrow slit a. The momentum balance leads to d _ (P. -P,) acs (Po-P.) ieee and 1, x L x+C, Substitution of Newton's law T,, =— a into the above gives do, (Po-P) Gg yon (Po-Pi)e* GC dx HL H : 2uL —tx+C,. Hu 2 Use of the no-slip boundary conditions at x= +B gives the expres- sions in Eq. 2B.3-1 and 2. One can also see that C, = 0 directly, since we know that the velocity distribution must be symmetrical about the plane x =0. b. The maximum velocity is at the middle of the slit and is The ratio of the average to the maximum velocity is then (ex) Fotos) Fry f-8)8 yg Pome Eyedy Kae c. The mass rate of flow is (Po-P.)B* _ 2 (Po-P,)oBW w= p(2BW Xe.) = p(2BW)(3) ee TEE = aE d, In Eq, 25-22, set both viscosities equal to j1. set b equal to B, and multiply by BWp. 2B.4 Laminar slit flow with a moving wall (“plane Couette flow") Start with the velocity distribution from part (a) of Problem 2B.3 (in terms of the integration constants). Determine C, and C; from the boundary conditions that v, =0 at x=—B, and v,=V at x= B. This leads to P.-P, )B 2 p, = CoP) 1-(2) +¥ (144) 2uL B 20"B This expression can be differentiated with respect to x and then Newton's law of viscosity t,, =-4(dv,/dx) can be used to get the expression for the stress tensor. Notice that the velocity distribution is no longer symmetric about the midplane, so that C, #0. 2B.5 Interrelation of slit and annulus formulas From Eq. 2.4-17 we get of Bul Je(-e-or pelea a(P,-P,)Rip In(1-e) 1+2e-e)" =(1-1+4e-6¢7 +4e9-e*)+ ze -3 (4e2-4e7 +e") le2ylgiyigt etherslerstety.. =(4e-6e7 +49 —e*)-(4e-6e7 + Se9 Let =(4e-6e7 +46? —e*)- This gives, finally, a result in agreement with Eq. 2B.5-1 wa Po- PL RP (ge 3 44.0) Sul gets 2B.6 Flow of a film on the outside of a circular tube a. A momentum balance on the film gives drt) d| ( ao, at +pgr=0 or ae 2) + pero The latter may be integrated to give a +CInr+C, Next use the boundary conditions that at r=R, v, =0 (no slip) and that at r=aR, dv,/dr =0. When the integration constants have been found, we get for the velocity distribution = PSR? (5) aint. v, 4u 1 R + 2a’ Ing b. The mass rate of flow in the film is then an raR . w=," fp pe.rdrd@ = 2nR* pf v.Edé in which a dimensionless radial coordinate €=1r/R has been introduced. Then pier! 2u Sf(a- 8? +207 In )Edé = mk (3 EP -1E4 4202 [-4e2+ ye*Ing))f 2 sae ~3a" + 4a* Ina) c. If we set a=1+€ (where € is small) and expand in powers of € using §C.2, we get 2-lo ops 2 opie? ya PBR (ge +0(¢*)) =2%2 gRte 8u 3h This is in agreement with Eq. 2.2-21 if we make the identifications W =2nR and 5~=R (and furthermore consider only the case that cosB =1. 2-11 2B.7 Annular flow with inner cylinder moving axially a. The momentum balance is the same as that in Eq. 2.3-11 or Eq. 2.4-2, but with the pressure-difference term omitted. We can substitute Newton's law of viscosity into this equation to get -u 22: = whence v, =-Ginrec, or 2 = -DiIng +D, dr or % That is, we select new integration constants, so that they are dimensionless. These integration constants are determined from the no-slip conditions at the cylindrical surfaces: v,(KR)=v) and v,(R)=0. The constants of integration are D, =0 and D, =-1/Ink. This leads then directly to the result given in the book. b. The mass rate of flow is w= fo fi Ps = 2m aot ~( @ing-1E ee emcee) which is equivalent to the answer in the text. c. The force on a length L of the rod di Faft(w Se) which gives the expression in the book. d. When we replace x by 1—€ and expand in a Taylor series, we get kRdOdz= 2nKRLUv, (x8) F=2aL(-p)0, = 2MLw)o(1_ ge — she. ) € To get this last result one has to do a long division involving the polynomial in the next-to-last step. 2B.8 Analysis of a capillary flowmeter Designate the water by fluid "I" and the carbon tetrachloride by "II". Label the distance from B to C as "J". The mass rate of flow in the tube section "AB" is given by mPa-Py)R%p, _ A[(Ps Pa) + Pgh] Ro, 8uL 8uL Since the fluid in the manometer is not moving, the pressures at D and E must be equal; hence Pa + Pigh + P18] + PSH = Pp + PS) + PugH from which we get Pa— Pa + Pgit= (Py - P1)SH Insertion of this into the first equation above gives the expression for the mass rate of flow in terms of the difference in the densities of the two fluids, the acceleration of gravity, and the height H. 213 2B.9 Low-density phenomena in compressible tube flow When we replace no-slip boundary condition of Eq. 2.3-17 by Eq. 2B.9-1, we get - 2 - c= Cos BuIRE , Po=Pu)RE aL 2QuL so that the velocity distribution in the tube is (Po se 2 . (zy |: (Po SBN Next we write the expression for w, but consider only the flow through a length dz of the tube: we in 7 p(z)v,(r,z)rdrd@ = zane 2 ai édé where we have introduced the ideal gas law, with R, being the gas constant (we use a subscript g here to distinguish the gas constant from the tube radius). We have also introduced a dimensionless radial coordinate. When we introduce the velocity distribution above, we get ‘ a (a aC Ba 5 +2 bug = ARYY M_\_ ap) , 4S. "Bu (#5 Bla 4a) This is now integrated over the length of the tube, keeping mind that the mass flow rate w is constant over the entire length ee eee Rg Mel oe acoh fowdz = (ee p+ io 2-14 = 2R*(_M \pi-pi, 400/, _ 2H 2 to R (Port) This gives a. Ales | gee» A) 8uL RT) 2 = B(Po=Pu)R*( PavgM (4b 8uL Rey Roe which leads then to Eq. 2B.9-2. 2-15 2B.10 Incompressible flow in a slightly tapered tube a. The radius at any downstream distance is R(z)=Ry +(R, ~Ry)(2/L) b. Changing the independent variable proceeds as follows: i -e) ee az) 8u dR )\ dz 8u dR L c. First we rearrange the equation in (b) to get Then we integrate this equation to get feo aaa) whence we can get the pressure difference in terms of the mass rate of flow 8uwL RS 3p J\ Ry Next we solve to get the mass flow rate _(32(Po-Pi)P\(_Ro-R, \_( 2(Po-Pi)Rop | 3 Ry-Ry 8uL RE -R 8uL Ri RO -RS This is the result, with the first factor being the solution for a straight tube, the second factor being a correction factor. It would be better to write the correction factor as "1-X ", so that the quantity X gives the deviation from straight-tube behavior. The quantity X is then Po er 2-16 X=l-2r Ral Rs 34 3 Ro Rv -R (Ro/Ri)’ =1 1-(R,/Ro) . 3(R,/Ro)_____1+(R,/Ro)+(R./Ro) ~3(Ri/Ro)” 1+(R,/Ro)+(Ri/Ro)” 1+(R,/Ro)+(Ri/Ro)” which then leads to the desired result in Eq. 2B.10-3. 3_Ry-R 1 SLE=(R/Ro)] 1 SLL (R/Ro)I(RL/Ro)” 2-1 2B.11 The cone-and-plate viscometer a. In a parallel-plate system with rectilinear flow, the velocity distribution is just 0, /2) = y/b, where b is the plate spacing and % is the velocity of the upper plate. We now make the following correspondences between the parallel-plate system and the cone- plate system (using @ as the usual variable in spherical coordinates measured downward from the z-axis, and y as the variable measured upward from the plate surface): 2%; 0) Or; bersinyy; yorsiny=ry= n-6) When this correspondence is made, Eq. 2B.11-1 results. b. From Eq, B.1-19, we get for the force per unit area in the @-direction on a face perpendicular to the @-direction « =y8in® a % 00 9 0\ sind Here we have used the fact that the angle between the cone and plate is so tiny that @ is very nearly 47 so that sin @ is very close to unity. c. The torque is obtained by integrating the force times lever arm over the entire plate area: _ iQ Q\( RE Toe fe A gran iar ae( tO) which leads to Eq. 2B.11-3. 2-18 2B.12 Flow of a fluid in a network of tubes At A the pipe splits into three pipes, and at the next set of junctions the fluid flows equally in six pipes, and then at the next set of junctions the fluid flows back into three pipes, and finally at B the fluid is all returned to a single pipe. Call the modified pressure at the junctions where three pipes split into six pipes P.,,, and that where six pipes join to form three pipes P,_,5. Then in each of the first set of three pipes PA —Pass)RA a 2(P 4 ~Prso)RiP a peut} 3 Bul 3aR'p In each of the batch of six pipes w _ (Ps 46—Pois)Rip 8uLw WTA! 396 7 B63 AP P. -P,,, =e 6 SuL or 346 76493 Eoin and in each of the final batch of 3 pipes w _ (Pog ~Pa)Rip 8yLw w _ Peis Pa)Rip -P,= 3 SuL or Poa 3aR*p When all the pressure differences are added together, the unknown quantities P ,, and P,.,, cancel out, and we get BuLw/5 3a(P,-P,)RYp 2-8 = eee) or sad Me ai 2C.1 Performance of an electric dust collector a, First we solve the problem of the vertical motion of the particle as it falls under the action of the electromagnetic field. The equation of motion for the particle (without gravitational acceleration or Stokes drag) is This equation may be integrated with the initial conditions that x= x, and dx/dt =0 at £=0, to give 06? 2m X= Xy- From this we can get the time f, required for the particle to fall to x=-B: 2m(B + Xo) Te Next we look at the horizontal motion. From Eq. 2B.3-2 and the expression for x(t), we find that (with 0,, = (Po — p.)B?/2uL [Oa es) This may be integrated to give eet Lafitemegft[1- (882 +28 }e| m Am’ (Po- Pr) (ge _ 52), 20°C a) QL (B36) Sint Next, square this expression and then insert the expression for ty above to get Po = Pr) [2m(B +0) (55 ia 1 Feo (9B~2x—)\B+%) Then, in order to remove the radical, we square this, thereby getting 8(Po - pi) m 2 3 Cog OB~ 2%) (B+%0) Next, we set dL#/dx, equal to zero, and this yields 4 values for x9: 4B, 3B, B, and B. It is only the first of these that is physically acceptable. When that value is put back into the expression for L, we get finally 1. [12 @o-putma® J" min 1350 we gu 2C.2 Residence-time distribution in tube flow a. A fluid element at a radius r within the tube will require a time f to reach the tube outlet L L Oo Pan d= (7/8) ] All the fluid with a radius less than r will have left the tube at this time. Hence the fraction of the flow that will have left the tube is voy Aree (2) | fFordrda “\R) AR When the first equation is solved for (r/R)* and substituted into the expression for F(t), we get roti at)(at) “Ge (at) b, The mean residence time is then obtained by solving the last equation in (a) for t. and substituting into the Eq. 2C.2-1: Dap dee be a ar 2C3 Velocity distribution in a tube The derivation in §2.3 is valid up through Eq. 2.3-15. If the viscosity is dependent on the radial coordinate, however, Eq. 2.3-16 is inappropriate. Instead we get (Po-P 1) pr aay C2 Application of the no-slip boundary condition at the tube wall gives (®-P, 0-8 wis ant’ This may be solved for the integration constant, and the velocity distribution is then (7) This is the same as Eq. 2.3-18 if the viscosity is a constant. Next we get an expression for the average velocity v,rdrd@ (v,)= com aera ali _ (oP)? Py =Fue 1 id ee ay = (PPO GE (rt _SGiayy (7° /u er v, Then we find the dimensionless ratio Der 2C.4 Falling-cylinder viscometer a. Equation 2.4-2 is valid for this problem, but the pressure difference is not known. When Newton's law of viscosity is substituted into Eq. 2.4-2 we get 2, --([ FPL) cine sc, 4uL The two constants of integration and the (unknown) pressure difference can be obtained from two boundary conditions and a mass-conservation condition: At r=«R, 0, =—0p; at r=R, v,=0; and f-"J'v.rdrd0 = m(KR)* vp. This states that the fluid displaced by the falling cylinder must be compensated for by a net motion upward through the annular slit. These three unknown constants may be obtained from these conditions (lengthy!) and the result is _(1-52)=(1+«?)In(Vé) 2% (1- «*)—(1+ x?)In(1/x) b. The force acting downward on the cylindrical slug of height H is (py —p)g-7(KR)?H. The difference in the pressures acting on the top and bottom of the slug is an upward force (Po-Pu)-m aR)? = -[- (dP/dz)te-a( wR)? =-— 40H -a(KRY R°[(1- x?) - (1+ «?)In(yx)] In addition, there is an upward force associated with the frictional drag by the fluid 2n(kR)H(-t,,)| bea = 2an(xeyHn( Ze) 2x -(1+ (un). ] . amo =e )-(1F)in(Ve) ote When these are equated and the result solved for the viscosity, the expression in Eq. 22C.4-2 is obtained. c. Next put k=1~-€, and expand in powers of ¢, keeping terms up to €°. Use §C.2, and obtain Eq. 2C.4-3. 935 2C.5 Falling film on a conical surface a. A mass balance on a ring of liquid contained between s and s+As gives (2ns(sin)3(s){o))|, ~(2s(sinB)6(s)(0))),,,, =0 Letting As— 0 then gives d d 3 (25(2)) =0 whence, from Eq. 2.2-20 ag (55°)=0 Equation 2.2-20 is valid strictly for a flat plate with constant film thickness, but we apply it here approximately to a different geometry. b. When the equation in (a) is integrated, we get s6°=C, in which C is an integration constant. This constant is determined by requiring that the mass flow down the conical surface be the same as that flowing up the central tube (i.e., wv). We hence write (width of film) x (thickness of film) x (mass flow rate), and then use Eq. 2.2-20: w= (2nssinB)-5-plv)= erssing (5) of CEI cs 8) From this we get C; c=——__3Hw es __ St _ (2asinB)-(p’gcosB) mp®gsin2B The film thickness as a function of the distance down the cone from the apex is thus Vip? gssin2B 2C.6 Rotating cone pump a. Inner cone not rotating. For sufficiently small values of B, the flow will resemble very much that for a thin slit (see Problem 2B.3), for which the mass rate of flow is given as the answer to part (b). This formula may be adapted to the flow in the annular space of height dz. as follows, if the inner cone is not rotating and if the gravitational force is not included: -}{-£) B?(2nzsin0)p BU az L where we have made the identification (py -p,)/L->-dp/dz and W > 2ar=2azsin@. Across any plane z = constant, the mass flow rate will be a constant. Hence the above equation can be integrated to give m3 ww pdz 3 ww yy P= ae Bpsindln 2 P24 rE Bpsing ™L, b. Effect of the gravitational force and the centrifugal force The result in (a) may be modified to include the effect of gravitational acceleration g and the angular velocity Q of the inner cone. The gravity force in the z-direction (per unit volume) is given Dy Fyn, = P8608 B. The centrifugal force (per unit volume) acting in the middle of the slit will be, approximately, Fey = e(42r)*/r = }Q"zsinB, where ris the distance from the centerline of the cones to the middle of the slit. The component of this force in the 2- direction is then F,..,, = 4Q°zsin? B. Then the first equation in (a) can be modified to give B?(2nzsinB)p L 2(-#. + 4on%ssin? pcos This equation can be integrated to give 9-24 4nB*psin, 5 “Sate ~ Pr) + ($0? sin’ 6)(13 - 13) (pg cosB)(L, ~L,)] Many assumptions have been used to get this solution: (1) laminar flow (turbulent flow analog is not difficult to work out); (2) curvature effects have been neglected (correction for this is easy to do); (3) entrance effects have been ignored (this can probably be handled approximately by introducing an "equivalent length"); (4) instanta- neous accommodation of velocity profiles to the changing cross- section (it would be difficult to correct for this in a simple way). 2-28 2C.7 A simple rate-of-climb indicator a. Consider two planes of area $ parallel to the earth's surface at heights z and z+ Az. The pressure force in the z-direction acting on the plane at height z and that acting at the plane at height z+ Az will be just the mass of air in the layer of height Az: Spl, — SPla ax = P8SAZ Division by SAz and then letting Az—0 gives the differential equation which describes the decrease in the atmospheric pressure with increased elevation; here R, is the gas constant. b. Let p, be the pressure inside the Bourdon element and p, be the pressure outside (i., the ambient atmospheric pressure). We now write an equation of conservation of mass for the entire instrument: =Po)R* Bul yd i — Po )R* Pact Views aul PS d ; Vip, = Pi; at i Vie Moe = at Ril Here m,,, is the total mass of air within the system (Bourdon element plus capillary tube), wv, is the mass rate of flow of the air exiting to the outside, p, is the density of the air inside the Bourdon elements, and Pyyy is the arithmetic average of the inlet and outlet densities within the capillary tube (see Eq. 2.3-29). The third form of the mass balance written above has made use of the ideal gas law, p=pR,T/M. If we neglect changes in the arithmetic average pressure P,yy and use the abbreviation B= 2R*p,y,/8uLV, we can integrate the mass balance above and get p,=e™([Bp,e"dt +C) 2-29 To get p,(t) we make use of the fact that there is a constant upward velocity, so that dp, _ dp, dz __{ PM), dt dz dt (eH 8°? Ap, whence p, =p%e* Then the mass-balance equation becomes el ee BPE gat 4 co ‘ ° esse Determine the constant of integration, C, from the initial condition that p? =p? at f=0. Then -p,_ A are oe Bohan AU (ee) In the limit that t > ©, we get fir B>>A A_vMg 8Vul p»Ai tes, TBR T AR Pg Hence for p, = Payg, the gauge pressure is ~p, =0,{ SHE Mav n-ne nn( Me Mee) Hence the pressure difference approaches an asymptotic value that varies only slightly with altitude. c. To get the relaxation time, note that P Fale) Boo P.( -e*) whence 2-30 ei av, sothat - AR"Pseg It is necessary to have tj <<100 to insure the plausibility of the assumption that B>>A. 2-31 2D.1 Rolling-ball viscometer The rolling-ball viscometer consists of an inclined tube containing a sphere whose diameter is but slightly smaller than the internal diameter of the tube. The fluid viscosity is determined by observing the speed with which the ball rolls down the tube, when the latter is filled with liquid. We want to interrelate the viscosity and the terminal velocity of the rolling ball. The flow between the sphere and the cylinder can be treated locally as slit flow (see Problem 2B.3) and hence the only hydro- dynamic result we need is dp _12p(v.) az oF (*) But we must allow the slit width o to vary with @ and z. From the figure we see that R?=(R=r) + (r/ +0)? -2(R=r)(r" + 0)cos8 where r’ = Vr? — 2”, Solving for o we get o=-r'+(R=r)cos@+Ry1+[(R-r)/R} (-sin? 6) The second term under the square-root sign will be very small for the tightly fitting sphere-cylinder system and will hence be neglected. Furthermore we replace Vr? - 2? by VR? -z? and add compensating terms R-VR?=2 +(\R?=2? - Vi? =2?) o=(R-1)| coso+ R-VR? =z? +(R=r)-3(R=r)(2/R) R-r ae | =(R- nfo =(R- n[lome +1)+ The omission of the term containing (z/R)’ and the higher-order terms is possible, since the greatest contribution to the viscous drag occurs at the plane z = 0, and hence less accuracy is required for regions of larger z. Note that the above result gives correctly o=0 atz=0, 0=2,and o=2(R-r) atz=0, 0=0. Next we assert that dp/dz will be independent of 6, which is probably a good approximation. Then according to (*) (v,) must have the form (v.)=Ba)o* — (**) Next, the volume rate of flow across any plane z will be = ["%(o,)o(0,z)RdO= RB(2)[""[o(0,2)] do = BRB(2)(R~r)*f'"[cos? $6 + a] d= BRB(2(R-7)°1(a) in which a =(R-VR?~2?)/2(R-7). The volume rate of flow Q at all cross-sections will be the same, and its value will be, to a very good approximation Q = 2R?vp, where vy is the translational speed of the rolling ball. Equating the two expressions for Q gives Rv B@)= FR) 1a) (my Combining (*), (**), and (***) we get dp ___ 3muRvy dz 2(R-r) (a) The total pressure drop across the slit is then wore dz d nih gaat ap= [Pa into which we have to insert dz/da. Virtually no error is introduced by making the upper limit infinite. From the definition of a 2? =-4(R-r) a? +4R(R-r)o The first term on the right is smaller than the second, at least for small z. Then dz=R(R-r)da/ Vo, and the pressure drop expression becomes (with £? = o) Ap =2R(R=7) yt fo da= 4,/R(R—7) yp Bas Leo 3/2 = RR) eh ie Te)" See) where =2[" =4/a-+ a T= 2l reat 3[2 yg (v0 +2) Jr osa1 ‘The pressure drop multiplied by the tube cross-section must, according to an overall force balance, be equal to the net force acting on the sphere by gravity and buoyancy 4R°(p, ~ p)gsinB = mR?Ap where p, and p are the densities of the sphere and fluid respectively. Combining the last three results gives the equation for the viscosity 4 Ri(e.-0) pein Ron” ony % R 234 2D.2 Drainage of liquids a. The unsteady mass balance is a 2 (paw) =(ole.)W8}, -(ole.)W9)., Divide by pWaz and take the limit as Az— 0, to get Eq. 2D.2-1. b. Then use Eq. 2.2-22 to get Eq. 2D.2-1: 05 _ pg ds __ pgs? 05 a Bu dz Hoe which is a first-order partial differential equation. c. First let A = \/pg/it , so that the equation in (b) becomes: dA _ dA ==" a Inspection of the equation suggests that A =./2/f, which can be seen to satisfy the differential equation exactly. Therefore Eq. 2D.2-3 follows at once. This equation has a reasonable form, since for long times the boundary layer is thin, whereas for short times the boundary layer is thick. 9-25 3A.1 Torque required to turn a friction bearing. Equation 3.6-31 describes the torque required to turn an outer cylinder at an angular velocity 2,. The corresponding expression for the torque required to turn an inner rotating cylinder at an angular velocity (2; is given by a formally similar expression, = 4rpQ RL ( derivable in like manner from the corresponding velocity profile in Eq. 3.6-32. The specifications for this problem (converted into SI units via Appendix F) 0.998004; x’ ior 0.996012 0.996012 0.003088 200 cp)(10-* kg/m-s/ep = 0.200 kg/m-s Qj = (200 rpm)(1 min/60 s)(2x radians/revolution) = 20/3 radians/s (1 in?)(1 m/39.37 in)? = 0.000645 m? Z = 2 in = 2/39.37 m = 0.0508 m p= (50 Ibm /ft*)(0.45359 kg /Ibm(39.370/12 ft/m)* = 800.9 kg/m? Hence, the required torque is T. = (4x)(0.200 kg/m-s)(207/Sradians/s)(0.000645m?)(0.0508 m)(249.8) = 0.431 kg-m?/s? = 0.32 ft-lby = M98 and the power required is P =T,Qj = (0.32 Iby-ft)(207/3 s71)(3600 s/hr)(5.0505 x 10-7 hp-hr/Iby-ft) = 0.012 hp In these calculations we have tacitly assumed the flow to be stable and laminar. To test this assumption, we formulate a transition criterion based on the critical angular velocity expression given under Fig. 3.6-2: Dp — KP? # Insertion of numerical values for the present system gives Re < about 41.3 for = 1 (207/3 radians/s)(800.9 kg/m*)(0.000645 m*)(1 — 0.998004)*/? (0.200 kg/m-s This Re value is well below the transition value of 41.3 for this geometry; therefore, the foregoing predictions of T, and P are realistic. Re= = 0.0048 3-1 3A.2 Friction loss in bearings. ‘The power expended to overcome the bearing friction is ) in which L is the total bearing length of 2 x 20 x 1 = 40 ft for the two shafts. The ications for this problem (converted to ST units via Appendix F) are: P=T.O;= 4xpO RL G x _ 16 ** F642 x 0.008 0.998751 (3) = Sooiaia = 706 = (5000 cp)(107* kg/m-s/cp) = 5 kg/m-s 0; = (50/60 rev/s)(2x radians/rev) = 5/3 radians/s R® = (8/39.37 m)? = 0.04129 m? L = 40 ft = (40 x 12/39.37 m) = 12.2 m 0.998751 .999375; With these values, the calculated power requirement is P = (4)(5 kg/m-s)(5x/3 rad/s)? (0.04129 x 12.2 m*)(799.6) = 6.938 x 10° kg-m?/s* ‘This result is then expressed in horsepower by use of Table F.3-3: P = (6.938 x 10° kg-m*/s®)(3.7251 x 10-7 hp-hr [kg-m?/: = 930 hp 1)(3600 s/hr) ‘Thus, the fraction of the available power that is lost in bearing friction is 930/(4000+ 4000) = 0.116. 3A.3 Effect of altitude on air pressure. For a stationary atmosphere (i.¢., no wind currents), the vertical component of the equation of motion gives dp _ ae °9 The air is treated as an ideal gas, a Tae with M = 29, and with temperature in °R given by T(z) 130 —- 0.0032 at elevation 2 ft above Lake Superior. The pressure pz at 22 = 2023 — 602 = 1421 ft above lake level is to be calculated, given that p) = 750 mm Hg at 7 = 0. The foregoing equations give dinp _ dz ~~ R30 — 0.0032) Integration gives wn tniraten) = Fe fae R Mg 1 is = == —— ]n(530 — 0.0032] ol . _ _Mg_,,, [525.737 ~ 0.003R "| 530 Insertion of numerical values in Ibm-ft-s units gives . (29 tb /lb-mol)(32.17 ft/s?) In(p2/P0) = 505 R/)(4.9686 x 10" Tf? /s?Tb-molR) —0.0505 In (525.737 /over530] Hence, P2 = pr exp(—0.0505) = 750 x 0.9507 = 713 mm Hg Since the fractional change in P is small, one gets a good approximation (and a. quicker solution) by neglecting it. That method gives pz = 712 mm Hg. BAA cosity determnation with a rotating-cylinder viscometer. Here it is desirable to use a sufficiently high torque that the precision of viscosity determinations is limited mainly by that of the measurement of angular velocity. A torque of 10* dyn-cm, corresponding to a torque uncertainty of 1%, appears reasonable if the resulting Reynolds number is in the stable laminar range. ‘The geometric specifications of the viscometer are: R=22%5cm; «R=2.00cm = 2.00/2.25 = 0.888889; K 1—n? = 0.209877; (R)? = 4.00 cm? L=4em; = 5.0625 em? 0.790123 ‘The angular velocity corresponding to this torque value is: TAC 2) (204 g-cm?/s?)(0.209877) © = Feu(aRFL ~ Sa(0.57 g/em-s)(4 em?)(4 om) ~ 18 Tadians/s ‘The Reynolds number at this condition is: MQoRp _ (18.3)(6.0625)(1.29) ae H 0.57 = 20 Accordng to Fig. 3.6-2, this Re value is well within the stable laminar range; there- fore, a torque of 10* dyn-cm is acceptable 3A.5 Fabrication of a parabolic mirror. Equation 3.6-44 gives the shape of the free surface as : vn (8)" ‘The required derivatives of this function at the axis of rotation are dz &:_ @ Roo ad Setting the desired focal length equal to half the radius of curvature of the mirror surface at r = 0, and using Eq. 3A.5-1, we obtain ‘Thus, the required angular velocity to produce a mirror with focal length f = 100 cin at standard terrestrial gravity is __, [980.665 cm/s? 214 radians/s which corresponds to 609/2x = 21.1 revolutions per minute, 3-5 3A.6 Scale-up of an agitated tank. ‘The specifications for the operation in the large tank (Tank I) are ‘Ny = 120 rpm; and the tank is to be operated with an uncovered liquid surface. To allow direct. prediction of the operation of Tank I from experiments in the smaller system (Tank II), the systems must be geometrically similar and must run at the same values of Re and Fr. To meet the latter requirement, Eqs. 3.7-40,/41 must be satisfied, Equation 3.7-41 requires Dy N} = DiN?P when, as usual, the gravitational fields for the two systems are equal. Then the model must operate at Nu = MVDy/ Dy = 120710 = 380 rpm. and Eq. 3.7-40 requires are = (13.5/0.9)(0.1)*(V'10) = 0.474 ep From Table 1.1-1, we see that this value of viz corresponds closely to the value for liquid water at 60°C, Thus, the model should operate at 380 rpm, with liquid water at very nearly 60°C. 3A.7 Air entrainment in a draining tank. As this system is too complex for analytic treatment, we use dimensional analy- sis. We must establish operating conditions such that both systems satisfy the same dimensionless differential equations and boundary conditions. This means that the large and small systems must be geometrically similar, and that the Froude and Reynolds numbers must be respectively the same for each. Choose D (tank diameter) as characteristic length, and (4Q/xD*) as charac- teristic velocity, where Q is the volumetric draw-off rate. Then 4Qe =e and =e _ Dig Subscripts L and $ will be used to identify quantities associated with the large and small tanks, respectively. We take the gravitational field g to be the same for both. ‘Then the requirement of equal Reynolds numbers gives 1) (B)- (2) (2)- (a) eam oe and the requirement of equal Froude numbers gives (&)- (Bi) Combining these requirements, we obtain (32) = (0.02277)"/* = 0.080 1 Hence, Ds = (0.080)(60 ft) = 4.8 ft Qs = (0.080)*/?(800 gal/min) = 1.46 gal/min ‘Therefore: a, The model tank should be 4.8 ft in diameter. b. Its draw-off tube should be 0.080 ft in diameter and 0.080 ft high. c. Its draw-off tube should have its axis 0.32 ft from the wall of the tank. Furthermore, if water at 68°F (20°C) is withdrawn from the model tank at 1.46 gal/min, air entrainment will begin when the liquid level is (4.8/60) of the level at which entrainment would begin in the large tank at its withdrawal rate of 800 gal/min. 3-7 3B.1 Flow between concentric cylinders and spheres a. The derivation proceeds as in Example 3.6-3 up to Eq. 3.6- 26, which we choose to rewrite as 2 2% =p, +D,{4) 7 7 The boundary conditions are that v(KR)=Q,KR and v9(R)= Q,R. Putting these boundary conditions into the above equation for the angular velocity gives =D, +D,—y and Q,=D,+D, These equations can be solved for the integration constants x? 1 1/x? ba xs Hence the solution to the differential equation is (2,-2,x7) _(Q, =84)( 28) 1-K t= Vr. The z-components of the torques on the outer and inner cylinders are T= foe" (-toR) ap RAOdz = aair?| sur (*2)] Irak =2aLRuR (1-8) («8)'(-275s) --4m(@, -9,) va T, = oly" (+t KR), ¢g KRAOd2= 2nt(se)| -ar 2(%2)] r (wry = #4muL(Q;-2,) > b. In Example 3.6-5 it is shown how to get Eq. 3.6-53 for the velocity distribution. The boundary conditions are : v,(KR) = KRQ, sin@ and v4(R) = RQ, sin 9. Equation 3.6-53 can be written in the form ° 5 poe Dr +D{5) rsin@ r The constants can be obtained according to the method of (a) and the final expression is 2% = 2.0) (01-2) rsin@—1-K i= Ur ‘The torques at the outer and inner cylinders are then = ff] eurH( 22 _ (Reina) sin ee o | tH al (KR) - EI, F T.= 0° IF (-t.),..(Rsin @)R? sin ododg 2s o =-82u (2, -2 (xR)? T.= I" (+t), gq (KRSin 0)(KR)? sin OdOdG = +8244(2, - 2.) 3 3-4 3B.2 Laminar flow in a triangular duct a. It is clear that the boundary conditions that v, =0 at y=H and at y=+V3x. Therefore the no-slip boundary conditions are satisfied. Next it has to be shown that the equation of motion =P, | (Po, , Po, Lae ay? is satisfied. Substituting the solution into the second-derivative terms, we get Bo-PL a? & of |S + Zleowy- y° - 3H? + Hy?) -{ Po- Pu) _6H- -( Aa \(6y - 6H -6y +2H) and this just exactly cancels the pressure-difference term. b. To get the mass rate of flow we integrate over half the cross-section and multiply by 2: wa 2g SarFe je Ly -H\x? -¥"jaeay = of Fae) (y= =x)" A Psat )o- Hy ) Sat) a) ) The average velocity is then the volume rate of flow (w/p), divided by the cross-sectional area H?/,3 so that ay (®o-P,)H* 60uL 7 3-10 ‘The maximum velocity will be at the tube center, or at x = 0 and y = y=2H/3, so that 9 PoP or 3B.3 Laminar flow in a square duct a. The boundary conditions at x= +B and y= +B are seen to be satisfied by direct substitution into Eq. 3B.3-1. Next we have to see whether the differential equation . av, . ao, Hae ae is satisfied. Substituting the derivatives from Eq. 3B.3-1 into this differential equation gives eet of 2a) Hence the differential equation is not satisfied. b. The expression for the mass flow rate from Eq. 3B.3-1 is given by 4 times the flow rate for one quadrant: wn Basta epel-(5) [o-(3) fo _(Po-P,)Bip =a FPG 1-2) nin mu “Fie [ca-e ef (Py-P,)BIp .)2_ 0.444(Py -P,)Bip = 2/7 8) ie 3B.4 Creeping flow between two concentric spheres a. From Eq. B4-3, there is only surviving term on the left side aS sind)=0 whence vy sin@=u(r) rein b. From Eq. B.6-8 (omitting the left side for creeping flow) the only surviving terms are : Hey] op gu td®, [1 4 (nit) 7 Pil ar Fae sind ar\” dr c. When the equation in (b) is multiplied by rsin 6, the left side is a function of @ alone, and the right side contains only r. This means that both sides must be equal to some constant, which we call B. This gives Eqs. 3B.4-2 and 3. Integration of the pressure equation proceeds as follows: P. ae dO cotye Ip. =I? ano Ok Pa P= Bln ae tante From this we get the constant B se alee P, = Une “Incot™ he Zincot} € Next we integrate the velocity equation a (au) Br BR GR ) (PB) or weS(2-S* ec, Gi ae) a QR or where we have selected the constants of integration in such as way that they will be dimensionless: C, =-xK and C,=-Kx-1 (from the no-slip condition at the walls). When this solution is combined with the expression for B we get: voll Oo) A Bete Alr-to(i-8] which leads to Eq. 3B.4-5. d. The mass rate of flow must be the same through any cross- section. It is easiest to get w at @=4x where vg =u(r)/sind= u(r)/sin} a =u(r): 0 LflaPPalgn Tid = 2AR' pf uEdE PR a = 2nip Pia loa) o(1-4) fea P,-P)R 1 =27R? . =| P4uincotte 6 (1- x)? (see Eq. 3B.4-6) H14 3B.5 Parallel-disk viscometer a, The equation of continuity (Eq. B.6-5) just gives 0 = 0. Equation B.6-5 gives for the 8-component of the equation of motion 1a Fv6 SEH? ot a b. When the postulated form for the tangential component of the velocity is inserted into this, we get simply d?f/dz? =0, which has to be solved with the boundary conditions that f(0)=0 and f(B)= which are just statements of the no-slip conditions on the wetted surfaces of the disks. It is easily shown that f = Qz/B. and therefore that vy =Qrz/B. c. The z-component of the torque exerted on the fluid by the upper rotating disk is Be (-taor)|,_grardo = 2m [Se] arse Biter We can now do the -integration and solve for the viscosity to get the desired formula, y= 2BT,/70R*. 3B.6 Circulating flow in an annulus a. Neglect of curvature gives for the z-component of the equation of motion dP do, @v, _dP or H pls a ae dr? dz The left side is a function of r alone and the right side is a function of z alone, so that both sides must equal a constant. Therefore 2 oe =C’ with boundary conditions 0, (KR) = % and v,(R)=0 Use the dimensionless variables ¢ = v,/v and & =1/R; instead of the latter, it is more convenient to use {= (E-x)/(1- x) in this part of the problem. Then = 2C, with boundary conditions ¢(0)=1 and 9(1)=0 Integration of this equation gives O=C,07 +O,64C, The boundary conditions give C; =1 and C, -(1+C,) so that = QC? -(14Q)E41 The mass-balance condition [}¢d¢=0 gives C, =3, and the velocity distribution is 9=307 -4041 b. We use exactly the same procedure when the thin-slit ap- proximation is not made, but the algebraic manipulations -are messier. The equation to be solved is 1a (te) ae Bra dr) dz with the same no-slip boundary conditions as before. Use the same dimensionless variables as in (a) and find that the solution has the form @=C,6? +C, INE +C, The three constants of integration are determined from the no-slip boundary conditions (k)=1 and @(1)=0, along with the mass- conservation condition {." J! ¢&déd0 = . The results are 2ei(t—e')fin(ve) Lem (1-«?)= (14 «?)in(x)’ * (1-2) - (1+ x?) n(x) With these expressions for the integration constants, Eq. 3B.6-2 follows. 3-17 3B.7 Momentum fluxes for creeping flow into a slot a. Inside the slot, the nonzero component of pv is mor cia) 3) | Outside the slot, the nonzero component of pwv are poy, ed 2”) yy apf 20) ee awe) G+y)*’ ry" awe) G+y?yt : _ _ 2w xy BE ta0t Az) @ryy b. Atx=-a,y=0 pe (aoua\aiy as Wp) a? This quantity is positive as we would expect, since positive x-momentum is being transported in the x-direction. c. Atx=-a, y-+a. 2 of 2v) 4 mWp) 2*a* This is negative, since at the point in question the y-momentum is negative and being transported in the x-direction. d. The total flow of kinetic energy in the slot is (if we use = y/ B): PY,Vy fi (dor? vaya = aw dov2ndy= paw 2 aon aa) (=m )dn 3 =!8 ppw{ 3” 35" aBWp. ‘The total flow of kinetic energy across the plane at x = -a is 348 K [dev atin v +v2)v,dy =lp iia ay a mp ior *@ayyP 2105 py 2 7 768°" | Wp i ‘The integrals appearing here can be found in integral tables. We conclude that the total flow of kinetic energy across the plane at x = -a is not the same as that in the slit. As a>, the flow of kinetic energy tends toward zero, since the fluid velocity tends to zero as x» —00. This emphasizes that kinetic energy is not conserved. e. Eq. 3B.7-1 clearly satisfies the equation of continuity, since for incompressible flow (dv, /dx) + (av, /dy) + (9v,/92)=0. When the derivatives are calculated from Eqs. 3B.7-2, 3, and 4, it is found that these expressions also satisfy the incompressible equation of continuity as well. f. From Eqs. 3B.7-2 and B.1-1 we get (3) 3x27 xt ___ 4m Np. (@+y') (@+¥) 0 mWpx* auf 22) 3x? At -0 mol(r+y) (Psy) |, The second of these is an illustration of Example 3.1-1. g. From Eqs. 3B.7-2, 3B.7-3, and B.1-4, we get, after evaluating the derivatives 3B.8 Velocity distribution for creeping flow toward a slot a. For the given postulates, the equation of continuity gives 19 (y,)=0 from which it follows that v, = 4f(@) ror r Since the flow is symmetric about 0=0, df/d@=0 at @=0; and since the fluid velocity is zero at @=+}7, it follows that f=0 at @=+47. b. The components of the equation of motion given in Eqs. B.6-4 and 5, appropriately simplified are he d 0= 2H an 0 7? dO c. When the first equation is differentiated with respect to 0 and the second with respect to r and the two results subtracted we get Eq. 3B.8-1. d. The equation in Eq. 3B.8-1 can be integrated once to get e TE sapec, A particular integral is fp; = }C,, and the complementary function is (according to Eq. C.1-3) for =C,c0s20+C;sin2@ . The complete solution is then the sum of these two functions. e. The integration constants are determined from the boundary conditions. It is found that C; = 0, and that }C, =C). Then from w=-Wp fee, rd = WoI7 fe = ~2WpC, f°"cos?* 040 = WC, we get C,=—w/Wpz and the velocity distribution is given by Eq. 3B.8-2. f. From the velocity distribution and the equations obtained in (b) we can get 320 oP uf 4w\ 29 aa z. £( Se le 0-sin? 0) and hence 2w nWp cos @-sin? @)+F() (*) Furthermore om (#22 30-7 d6 or P zp }O> 2 0+G(r) Here F and G are arbitrary functions of their arguments. The second expression for the modified pressure can be rewritten as mW, a (35 ey loo? 0+(1-sin? 6)) +G(r) E(B oe 0~sin?6)+H(r) (*) By comparing the two (*) expressions for the modified pressure, we see that they are the same except for the functions F and H. Since the first is a function of @ alone and the second a function of r alone, they must both be equal to a constant, which we call P... This is the value of the modified pressure at r= se. g. The total normal stress exerted on the wall at @= 7/2 is (when one uses the result of Example 3.1-1) Tealoana = (P+ Too gaga= (P ~ PSM) ape me D = Bat prt PRN Pom + oye h. From Eq. B.1-11 3-2! D == 2 (20) 41 lone a) 7 0 Nan = -1fo-( 28 \-acosdsin | =0 een]? The first term is zero since v,/r=f since vg=0 was one of the postulates. The second term is zero, as can be seen by using Eq. 3B.8- 2, and the fact that cos$=0. This is agreement with the result in Problem 3B.7(g). i. Since the z-component of the velocity is zero, we can expand the velocity vector in either the cylindrical coordinate system or the Cartesian system thus V=5,0, +350, =8,0, +3,0, Since vy =0 was one of the postulates, when we take the dot product of this equation with 3,, we get the x-component of the velocity 2, =(8,-8,)=0, cosa =-—2” Wor Similarly for the y-component of the velocity 5 2w : 2wx?y 2, =(8,8,)=0,sind=-= cos? Osin@ Wor __ 2wx?y mWp(x? +y?)” These results are in agreement with Eqs. 3B.7-2 and 3. 322 3B.9 Slow transverse flow around a cylinder a. At the cylinder surface we get by using Eq. B.1-11 Cu, cos8 : P\ig = Po — HG Oe — pgRsin 8 fp 2{%) .190,)] _ Cuo. sind Faber = a{ Ze) +2 | R Also from Example 3.1-1 we know that 7, b. If n is the outwardly directed unit normal vector for the cylinder, then the force per unit area acting on the surface is -[n-n] evaluated at the surface. But n=8,, so that the x-component of the force per unit area at every point is 6. [8,-*)), =. [8, (5 +4) ‘The pressure and stress terms are evaluated thus (using Eq. A.6-13): ~(8. [5,75], =-(8.-3,7) lr=R cos@ j= Phen ~(8 18, -t)), gp =“(8e [8,648 8o%0#))) = (8, Bo) hag = el an SiO These expressions lead to Eq. 3B.9-5. c. The total force on a length L of the cylinder is then F,= [0 (-Pl..g 6058+ Trel,.g Sin 0)RdOdz +pgRsin @cos 0+ 2 Cuv,,cos? 0 fo R = nefe(-. cos 0+ Cuo,,) p22 = Ruf R Ve dO =2nCpo,.L The first and third integrals in the next-to-last line are zero since the integrands are odd. Cuo,, sin? @ R 3B.10 Radial flow between parallel disks a. The continuity equation is, for v, =, (r,2) , from Eq. B4-2 1a sg (7%-)=0 — fromwhich 70, = 6(2) The equation of motion is obtained from Eq. B.6-4 a, de [aria ay, 00 ae oH SEZ, i} =| b. When the results for the equation of continuity in (a) are used in the equation of motion, we get Eq. 3B.10-1. c. With the creeping flow assumption, Eq. 3B.10-1 gives a> . dP ay Ee ty hi ase ee ie om which r dr aoe =B and ve since the left side is a function of r alone and the right side a function of z alone, and therefore both sides must be equal to a constant, B. When the pressure equation is integrated from r, to r,, we get dr P,-P dP =B iio whence B=? 4 i f In(r,/7) d. When this result is substituted into the ¢@ equation we get @o_ P,-P, . P,-P, 2 =f =-—1-2,. from which =a yg tet de® ln(rs/n) 9“ Tia(rajrs) 2 OFFS The integration constants are obtained from applying two boundary conditions. We could require that ¢=0 at z=+b, and thereby determine the integration constants. Another method is to recognize that the flow is symmetric about z = 0, and use as one of the boundary conditions d$/dz=0 at z = 0. Either method will give C, =0, and then C, is easily obtained. The final result is 3-24 oe Bctiels(3)] 2uin(r,/r;) b Division by r then gives Eq. 3B.10-3. e. The mass rate of flow at any cylindrical surface in the system must be the same. Select the surface at r =r, and obtain w= fy" [ee ary = (Pi-Po)b ayy _ ge der d= 2apb-2- hl eg The integral gives 2/3, so that Eq. 3B.10-4 is obtained. 35 3B.11 Radial flow between two coaxial cylinders a, From Eq. B.4-2 we get for this flow, with 2,(r), 1d =—(r,)=0 whence v, = C where Cis a constant rdr r Atr=R, 0,(R)=C/R so that C = Ro,(R). b. The relations in Eq. 3B.11-1 follow immediately from Eqs. B.6-4, 5, and 6 for the velocity profile v,(r) in (a). c. Integration from r to R gives R teed PR)-F()=0C(-3E] This gives, making use of the meaning of C obtained in (a) ; PP) POR)=$o[Re (RIE (de) = sate, [s-(8) ] d. The only nonzero components are (from Eqs. B.1-8 to 13) 3-26 3B.12 Pressure distribution in incompressible fluids The equation of motion method to get the pressure distribution is correct. On the other hand, the second method gives nonsense, as one can see from Fig. 3.5-1. For an incompressible fluid (the vertical straight line), specifying the density does not give any information about the pressure. 321 3B.13 Flow of a fluid through a sudden contraction a, For an incompressible fluid, Eq. 3.5-12 becomes 40(v3 - 07) +(P2 - pi) +g(l2—m1)=0 or 4o(v -v7)+(P,-F,)=0 If "1" is the large tube and "2" the small one, then the fluid velocity in "2" must be greater than in "1." Then the modified pressure in "2" must be less than that in "1." Thus the modified pressure decreases as the fluid moves from the large cross-section region to the small cross-section region, in agreement with experimental observations. b. For an ideal gas, Eq. 3.5-12 becomes Lolo? — v2) 4 RE iy Pe nS) 4p(v} ot) + pin (he h)=0 The pressure and elevation terms may no longer be combined. If the elevation does not change, the pressure decreases as the fluid moves into the contracted part of the tube. 3-28 3B.14 Torricelli's equation for efflux from a tank From Eq. 3.5-12 we get 1 4 (Pena —9) + (Porm ~ Pat) + (0h) =0 Here it has been assumed that the velocity at the surface is virtually zero, that the pressure is atmospheric at both "1" and "2", and that the datum plane for the height is at the exit tube. When the above equation is solved for the efflux velocity we get Torricelli's equation. 374 3B.15 Shape of free surface in tangential annular flow a. The velocity distribution is given by Eq. 3.6-32, and the equations corresponding to Eqs. 3.6-38 and 39 are: 2-422) (0) GQ] om Bon Integration of these equations gives (see Eqs. 3.6-40, 41, and 42) 2 2 2 =f 28) mr (QT) Pp 1 SES ; 4lnr + R pgz+C Now let p=Pym atr =R and z=zp, where zp is the height of the liquid at the outer-cylinder surface. Then we can write at r= R and Z=Zp oe =16 2028) a 4InR+1]-pgz_+C which is the equation that determines C. When we subtract the last equation from the equation for p, we get P~ Pain = +f 2252) (-2 Fraing +6 *) -aete~2n) The equation of the liquid surface is then the locus of all points for which p = jim, OF . nang SE) [Breme-e) b. When the outer cylinder is rotating, we can use Eq. 3.6-29 for the velocity distribution to get 3-40 _ (QR) [17 )? 1( KR) poo 28) [2 2) -amr-2() Les Then, we select r = R and z =z, as the point to determine C. Q,KR\f11) 12 Paton = 25) E@) ~2nR-5(x) — P8ZR+C Subtracting, we get P-Paim -5(Pe8) (8) -(2) -ang-(4) +(e feat -2) From this equation we can get Eq. 3B.15-2 by setting the left side of the equation equal to zero and solving for za —z- 33) 3B.16 Flow in a slit with uniform cross flow From Eq. B.6-1, for this problem we have dv, (Po-P,), dv, @o do wae a ee ee 0 (2% dy +H ay or ap aaa in which n=y/B, ¢=0,/[(P)~P,)B°/uL], and A=Buyp/p. This equation has to be solved with the no-slip condition at = 0,1. We write the solution as the sum of a complementary function and a particular integral. The equation for the complementary function is a d , , q oe -Age =0 —withsolution — gcp = Ste" +C, By inspection, the particular integral is ¢», = n/A. Application of the boundary conditions then gives the constants of integration. The final solution is then (with A= Buyo/1) 1 ean 1 (Po-P,)B ify e4¥8-1 a(n ) or 2 EPPS b. The mass rate of flow in the x-direction is then ne (Po-P.)B Wp a w=) {) ev.dydz =-* ~. Sjoan (P,-P,)B°Wo 1 BL A lt ¢ By making a Taylor-series expansion about A = 0, from (a) we get @=4(n-7*)+O(A). When A- 0, this result can be shown to be equivalent to Eq. 2B.3-2. Similarly, A Taylor-series expansion about A = 0 yields from the result in (b) w é 3 = el 0 Ble 3-32 ii, 1 2 A’ (1+A+}A +148 +-)-1 i141, 1 AeA ) sifit Al2 A =4+0(A) But the "B" in this problem is twice the "B" of problem 2B.3. If we switch to the "B” of Problem 2B.3, it is found that the answers agree exactly. d. For the coordinate system here, we select as the dimensionless quantities y Vy bop yet, vo ga oP) al a Then the differential equation and boundary conditions are ave dy ev 12¥ wa with V(4l)=0 The solution is then the sum of a complementary function and a particular integral (as before in (c)) Application of the boundary conditions then leads to *Y +cosha+ Ysinha asinha Then the average value of this over the cross-section of flow is Wye [ivay _ifce” +cosha+Ysinha)dY _ -(1/a)sinha + cosha tr a 1 fray 2 asinha = asinha Then we can form the ratio given in Eq. 3B.16-3: V__ ef ~Ysinha~cosha (V)(Wa)sinha— cosha As a check on this we can go directly from Eqs. 3B.16-1 and 2 to Eq. 3B.16-3. From the first two equations we get . 2al(e* -1)y-B(e*#/* -1)] BI(A-2)(e* ~ 1) +24] Next, we make the connections between the notations in the two different approaches: y=z+b; B=2b; A=2a (the "y” of part () is called "z" here, and {= z/b). Then a _af(et- 1)(z+b)- Db(etairx=+) 1) ,) ofa 1)(e** -1) +20] (e4 =1)(6+1)-2(e' Se), (e* -e*)(0+1)-2(e% -e"*) (Ya)[(a= fe -1) +20] (Ya)[(a-1)(e*-e*) + 20e*] _glet—er*) +(e +e"%) 26% _csinhar+ cose (Yalfa(et +e*)-(e*-e*)] coshar—(I/a)sinh 3C.1 Parallel-disk compression viscometer a. The equation of continuity of Eq. B4-2 for incompressible fluids, taking into account the symmetry about the z-axis is just Eq. 3C.1-6. The equation of motion in Eq. 3C.1-7 comes from Eq. B.6-4 ignoring the hydrostatic pressure, the inertial force terms, and omitting the terms that are small. b. Equation 3C.1-7 can be integrated with respect to z to give 1 dj grape TOG The constant C, is found to be zero from the boundary condition in Eq. 3C.1-8, and C, is found from Eq. 3C.1-9. c. Integrating of Eq. 3C.1-6 with respect to z from 0 to H 2(2-H Nae do, Performing the integrations then gives H? 4, ‘). Hl Ty rar" ar d. Integration of the equation in (c) then yields 2 fa F+Cnr+C, The integration constant C, must be zero, since the pressure is finite at the center of the disks, and C, is determined from Eq. 3C.1-10. Equation 3C.1-13 is thus obtained. e. The force on the upper plate is then 2n (RSUVR’ (ty _ 3uv9R? py, _ F=f", iHwP 1 (5) ertom aan HOF (1-7 Bas The integral is 1/4, and this leads to the result in Eq. 3C.1-14. 3-35 f.In this situation, the radius of the glob of liquid R(t) and the instantaneous disk separation H(t) are related to the sample volume Vby V = x[R(t)} H(#). Then the force acting on the upper disk is roe feed [:- (at) ose _ Smpvo[R(H)}* __3H00V? {HOP — 2afH(P g. If, in Eq. 3C.1-14 we replace vy by —dH/dt, we then have an ordinary, separable differential equation for H(t). Integration gives Fy para —pt ae BmpRE IH FP whence 1 _ 1, 4st [HOE He” Smur™ 3-3t 3C.2 Normal stresses at solid surfaces for compressible fluids First write the equation of continuity for a compressible fluid as rele SZinp=-(9-¥)-2(w-¥p) The normal stress on a surface perpendicular to the z-axis is Calan =(-2¢ Re +Gu-Khv-v)) le=0 The terms containing v drop out by the no-slip condition at the surface, and their derivatives with respect to x and y drop out on the surface as explained in Example 3.1-1. This result shows that the normal stresses at surfaces are zero for compressible flow if the flow is at steady state. 337 3C.3 Deformation of a fluid line The curve at any time t is 6(r,t)=(v9/r)t, which in tangential annular flow is (from Eq. 3.6-32) O(r,t)= (Rey =1 Oe and do = RIO 5, (Wx) -1 (x) -1 The differential element of length along the curve is given by (at)? = (dr + (rdo? = (dr)'| 14 ABD (2a) Mos (xt) The total length of the curve is then I= (tal =f 1s MRD CANT ep! 1+ 162 NT gg tees es (eye) (vx)? -a)'e* To get a rough, order-of-magnitude estimate assume that N is large and then the "1" can be neglected and the integral performed analytically l_4naNKx R ltk (limit of large N) 3-38 3C.4 Alternative methods of solving the Couette viscometer problem by use of angular momentum concepts a. By making an angular momentum balance (actually the z- component of the angular momentum balance) over an annular region of thickness Ar and height L we obtain (2nrL)-(rt,9)|, ~(2a(r + Ar)L)-(rt,9)} 0 IrtAr ad Dividing through by 27LAr and rearranging we get (Pee)... “(Pe0) Ar whence d alte) 0 the second form resulting from taking the limit as Ar —> 0. Then using Eq. B.1-11 for the stress-tensor component, we have Af pd 2a) = f(r $(% =0 whence ve=-SL+ Cr From this Eq 3.6-20 follows. b. Here we start with Eq. 3.4-1, which simplifies to the following for the symmetric stress tensor [¥-{rxt}]=0 The z-component of this equation is 334 14 (rfexeyt)=0 or 14 ({rxe},)=0 where, in cylindrical coordinates, r=3,r +8,z. We now work out the cross product, which is DDE (8,7 +3,2), FC sol Cor +E coe(0)F 7 Hence the equation of change for angular momentum simplifies to and the development proceeds further as in (2). 3C.5 Two-phase interfacial boundary conditions a. This result follows at once from Eq. 3C.5-1, when the viscous-stress-tensor terms are omitted. b. To get the right side of Eq. 3C.5-3, it is evident that Eq. 3C.5-1 had to be multiplied by 1/p'vj . The interfacial-tension term in Eq. 3C.5-3 is then “Gall Galles] The terms involving the viscous stress tensor are togty|_# 7 ws l [a'-2"] eee] ele a ta gy - po ome Hy Mio | And finally, the pressure terms are converted to modified pressure terms plus terms involving the gravitational acceleration nlf P=Pote'g(t hy) _p™—po+p'g(t-ho)) __ 8(t=No)(o'-e") pos p'05 To? piv We see that the Reynolds numbers for the two phases, the Weber number (Eq. 3.7-12), and the Froude number (Eq. 3.7-11) appear as well as the density ratios for the two phases. Bal 3D.1 Derivation of the equation of motion from Newton's second law of motion a. Equation 3D.1-1 is the statement that the time rate-of- change of momentum is equal to the sum of the surface forces and the gravity forces acting on a small blob of fluid. When the Leibniz formula (Eq. A.5-5) is applied to the left side of Eq. 3D.1-1, we get 4 fovav = J Soudv + Slonnv- n)dS= Jgenve Jlrewlss at yo rw? E =f 2 pwav + Iv. pvv]dV (using Eq. A5-3) ) ot veo The term containing the stress tensor in Eq. 3D.1-1 can also be rewritten as a volume integral using Eq. A.5-3 to give a f aevav = — JlV-evv]av - f[V-n]Jav+ fogav vit) vit) Vit) vit) Since the choice of the blob volume was arbitrary, all the volume integral operations may be removed, and we obtain the equation of motion of Eq, 3.2-9. b, If the blob is fixed, then we can write a momentum balance over the blob as follows: E | pvav = —[[n-pev]as— f[n-n]éS + [pga This states that the rate of increase of momentum within the fixed volume equals the rate of increase because of convective transport, the rate of increase because of molecular momentum transport, and the force acting on the system by gravity. The time derivative can be taken inside, since the volume is fixed, and the surface integrals can be converted to volume integrals. The result is an equation containing only volume integrals over the fixed volume: [Zovav =-[[v-pev]av — f[v-a]av + [pgav 3-de Since the volume was chosen arbitrarily, the volume integrals can be removed, and, once again, the result in Eq. 3.2-9 is obtained. 3-43 3D.2 The equation of change for vorticity Method I: Start with the Navier-Stokes equation in the D/Dt-form, but rearranged thus: Iv-vv]-59p+ wWvtg =-Viv? [vv v]]-29p + wWvtg Next we take the curl and introduce the vorticity w =[Vx v] ow =[Vx[vxw]]+ Wow ot or & <[y-wy]-[¥ -vw]+ Ww Then using Eq. A.4-24 and the fact that (V-v)=0 for incompressible fluids and (V-w)=0 always (since the divergence of a curl is always zero, we get Eq. 3D.2-1. Method II: Start now with the Navier-Stokes equation in 0/dt-form x A¥-w]- 0p wWvtg Take the curl of this equation and introduce the vorticity to get & =-[Vx[V-wv]]+ vv2w or & @. The local Reynolds number at the trailing edge (z = L = 20 ft) is: Re = Lveo/v = (3ft)(20 ft/s/(1.62 x 10~* ft?/s) 7 x 10° 4. According to Eq. 4.4-17, the boundary layer thickness at the trailing edge is 62) = 4.04/22 _ (1.62 x 10-4 ft?/s)(3 ft) aoe 20 ft/s = 24x 1075 ft ¢. According to Eq. 4.4-30, the total drag force of the fluid on both sides of the plate is Fe = 1.328/ppL Wd, = 1.328y/(0.075 Ibyn/ft®)(1.22 x 107 Ib,/ft-s)(3 ft)(10 ft)?(20 ft/s) = 0.62 Ibyy-ft/s? = 0.019 Iby A-G 44.7 Entrance flow in conduits. a. With the indicated substitutions, Eq. 4.4-17 gives vk. (DY? 2(v) ~ (9.28 2 D% oar = 0.023DRe or ) which is similar to the expression given in §2.3, except that the coefficient is about 2/3 as large. b. At the typical transition locus 7v.0/v © 3 x 10° for flow along a flat plate, Eq. 44-17 gives = 4.64(3.5 x 10°)"!/? = 0.00847 and the transition Reynolds number based on the characteristic length 5 is = (3 x 10°)(0.00847) = 2542 For flow in tubes, with transition occurring when 6 = D/2 and with vx = 2(v), the latter result gives D(v)/v=2542 as the minimum transition Reynolds number, in fair agreement with the reported value of 2100. c. For laminar flow between parallel planes, the method in Problem 4.C gives & = B and vmax = (3/2)(vz) at the end of the entrance region. Insertion of these results into Eq. 4.4-17 gives whence 1.5 B{v,) Gaye = 0.070B?(vz)/v =0.070BRe with Re=B(v.)/v 4B.1 Flow of a fluid with a suddenly applied constant wall stress a. Differentiation of Eq. 4.1-1 with respect to y gives Fe, IF 4 Af _ e)_ B(_ a, ayat “ay ay at ay) Va ey Then using Newton's law of viscosity, we get or, ar, ue 7 Tw ot oy’ b, This equation is to be solved with the initial condition that Ty, =0 for t<0, and the boundary conditions that t,, =t% at y=0, and that t,, =0 at t=. c. The solution is exactly as in Example 4.1-1 with appropriate changes of notation, and the solution is given in Eq. 4B.1-1. d, To get the velocity profile, we integrate Newton's law of viscosity: 0 on . ho [, SP tty or ath (1-err 4B.2 Flow near a wall suddenly set in motion (approximate solution) a, Integration of Eq. 4.1-1 over y gives - Wy Pv, dtm of, Ge =n; ye whence a pv,dz =m | Since the velocity gradient is zero at infinite distance from the plate, we end up with Eq. 4B.2-2. b. We introduce the variable’ = y/é(t). Then when Eqs. 4B.2-3 and 4 are substituted in Eq. 4B.2-2, we find d Spf er—(I- 3nd n= non-B 3M) ss Then after dividing by po,, and evaluating of the integral, we get 34 53, 1 —— Eq. 4B.2-£ 8dt 2a «ee Fa 4825) c. Eq. 4B.2-5 when integrated gives fpodb=4vpdt or 5(¢)=V8vE Then the velocity profile is given by =1-3(-4)42(LY i 1 (a) 3) for Osys8vt for y28vt 4-9 4B.3. Creeping flow around a spherical bubble a. According to Eq. B.1-18, the vanishing of the shear stress is (%) 100, _ a(1f1 aw)),1af 1 av\_ ro + jt goo ri, or 43{ Psind lao In the second form, we have inserted the expressions for the velocity components in terms of the stream function from the last line in Table 4.2-1. Next we insert y= f(r)sin?@ and obtain Eq. 4B.-1. b, Equations 4.2-7 through 10 are still valid for this problem, as well as the values of C;=-4v, and C,=0. Hence we have to determine the remaining constants in f(r)= Cyr + Cr by requiring that Eq. 4B.3-1 be valid at r = R, as well as f= 0 at r= R (Eq. 4.2-3). These boundary conditions lead to C,=0 and C, = 4v,R. Then Eqs. 4B.3-2 and 3 follow directly. c. When the velocity distributions in Eq. 4B.3-2 and 3 are substituted into the equations of motion (Eqs. B.6-7 and 8), we get : Safee|EY oe ot Z(H Sf Integration of these two equations gives (#2-\() cos0+F(8) and P= (#2) 2) cos +G(r) In order for the solution to be unique, F(@) and G(r) must be equal to a constant. If we require that the modified pressure be equal to pp at z =O infinitely far from the sphere, we then get (RY rorcro-( |S) me d. The z-component of the force acting on the sphere is Pe Efe (8--[8,-{e5+=}))]_.R?sin odode = -2AR? [*(pcos0+1,,cosO—t,»sin9)|,_, sin dd0 where Eqs. A.6-28 and 29 have been used for getting the dot products of the unit vectors. Then we use the result of (c) and Eqs. B.1-15 and 18 for the components of the stress tensor (along with Eqs. 4B.3-2 and 3) to get the three contributions to the force: -ann’ feo ~ pgz -(#\8) cos o| =4aR°pg +4 mpRo., cos @sin 6d eR sin ede lek = H4auR?o,, g(4 -cos' 70) Fie, = 2a? |5( 24 2m, , v,.8,, or by using Eqs. A.6-28 and 29 as ro, v, =0,,c0s0 and vy =-v.,sind Gi) at r=R, v, =0 (iil) at z=0 as ra, PP, b. Since v=—V@, we have as r—> <0 —0/dr = 0.,cos8 and -(1/r)39/00 = —v., sin® When these equations are integrated we find that ¢=-v.rcos0, Thus we feel that $= f(r)cos@ may be an appropriate trial function. c. We next write the 3-dimensional Laplace equation in spherical coordinates (for a system with symmetry about the z-axis) 4.2(%). 71 _( snot) 7 ar) * Fain"? 36) =° Into this equation we substitute the trial velocity potential and get z e(eZ)-2 Sa(rd £20 whichasthesolution f=Cyr+C,r? since this is an “equidimensional" equation of the form of Eq. C.1-14. d. Application of the boundary conditions then gives pool 03(7) Jane ov o3(7) fo e. Then from the components of v =—V@. we get Eqs. 4B.5-2 and 3 by differentiation. _P =e [1-(8) Joo oft . 32) fs osc, The integration constant is then obtained from the boundary condition given in (a): C;=-P,-pv2. Then when the modified pressure is evaluated at the surface of the sphere, Eq. 4B.5-4 is obtained directly. 4-15 4B.6 Potential flow near a stagnation point a. At the origin of coordinates (z = 0) the complex velocity dw/dz = ~20q2 is zero, which is a stagnation point. b. By taking the real and imaginary parts of the complex velocity, we get from Eq. 4.3-12: v, = 2v9x and v, = ~2vpy. c. When 2 is positive, the fluid is flowing toward the surface y=0 in the upper half plane. The magnitude of v9 specifies the speed with which the fluid is flowing: v= fv? +03 = 2ugr. 4-16 4B.7 Vortex flow a. By using Eq. 4.3-12, we find that dw (2) (2) ir (_x-iy . ee a alee =-0, +0, dz 2n\z) 2nlaz) 2al ey a! in which the overbar indicates the complex conjugate. Equating the real and imaginary parts gives ° Eft y}--E (282); 0, E(t) -E(se02) Unley) alr al atay) alr The components in cylindrical coordinates are o-tino-+,co80= (2) ¥, = 0, coSO+v, sin =0; v a\r b. The forced vortex is given by v = Qr in Eq. 3.6-37. 4-17 4B.8 The flow field around a line source a. For this purely radial flow v, =v,(r) and the other two components are zero. Then V7 = 0 simplifies at once to Eq. 4B.8-1. b. Integration of Eq. 4B.8-1 gives ¢=C,Inr+C,. Then, since v, =-dd/dr, we find v, =-C,/r. Next we calculate the volumetric flow rate per unit length thus T=[}"0,rd0=~2nC, whence v, = = The pressure distribution is then obtained from of the equation of motion po, fa 4P gy (X)-4) ae “ar dr Paar) Dart) ae Integration then gives P. TY) pf 1 7 ae-( 2) f (-4}r or PL-P the radial component 4-18 4B.9 Checking solutions to unsteady flow problems a. Substituting the solution of Eq. 4.1-14 (or 15) into Eq. 4.1-5, we have to verify that deer a) Bl grera) We have to use the chain rule of partial differentiation along with the Leibniz formula inl or peta ood (teea (2) "\-aata) He" onlaie) When the definition of 7 is used, the above is found to be an identity. 4-19 4C.1 Laminar entrance flow in a duct. a. Calculation of the mass flow rate according to Eqs. 4C.1-1 and 4C.1-2 gives w = p(v_)WB ; . 7 = mw [’he(2)-(2 = pvW J Be) () Jere aw f dy = pW [e- 5] + poeW1B— 8 = maw | sy al Equating the first and last expressions for w, we get (with 6 < B and A = 6/B), ve) = (ve) gag = edge a ‘The following related equations will prove useful in part b: ey de @- a) a 25 GE = (oe) as Leiee [( “i wal =o ag] *b, ‘The boundary layer in this system lies between y = 0 and y = 6, so those limits suffice for the integrals in Eq. 4.4-13. Evaluation of the terms in Eq. 44-13 (divided by p) according to the results in a then gives: te(ve = ve)dy = Leto [120 = w(1 20+ uPyIdu with w= y/6 a Les) [iv — du? + 2u9 — u? + 2u8 — uf}du = Zero —4/3)+ (2/4) — (1/3) + (2/4) - @/5)] T+9A] dA, = Biv)? la - 3 PAs) 4-10 FE [tenn dy = v6 Fe we ft [1-2u-+u2}du 7 vo ~14(1/3)|du A dA Ga ay ae t9) (ve) B. With these substitutions, Eq. 44-13 gives 7 dA +512 [2/15] + 9(v.)? B-—— a [ee saessa A aaa ~ Ay de __Blvz)? dd [544634 GB oe | 15 | Multiplication of both members by (5/3)A(3 — A)/(B(ve)? gives wo (—”%_) — 844747 da (v,)B? ) ~ (3— A)? dz in agreement with Eq. 4C.1-4. c. Integration of the last equation with the initial condition that A = 0 at = 0 gives ve 1 8 6s +73? 5 wR? Of, Bsr From integral tables we get the formulas s a a f wmy* =p fines bs) + sal + 3? 1 a J tage [et teem 02) - 2] 40 which yield the definite integrals 42 and the solution vr 3-A A 3-A 9 oa = 6 [m(254) i= al 7[e- A)~ om(25* 7 asa 3 6, 63 =7a4sin(254 ya Se +1-35-y 6A ~ 21(3 ~ A)— 63 Aa 3-4 Aa A 214 +a A 3 =7A4+48In a A 3 A = =7a+43m (254 in agreement with Eq. 4C.1-5. d. Setting A=1 and 2 = L, in the last equation gives Le 7 7 il? + 481n(2/3) + #I = (0.1) [7 — 19.462325 + 13.5] = 0.104 in agreement with answer (4). ¢. Application of Eq. 4.3-5 to the region 6(x) < y < B, with vy neglected so that v = ve there, gives 5Pre + P= constant Insertion of the result of part a gives 2 Fetes)? (<4) +P = constant Evaluating the constant at 2 = 0, where A = 0 and P = P,, we get 5oten) (525) #P = Jobe +P or A-2t 4C.2 Torsional oscillatory viscometer a. The equation for the rotating bob in a vacuum is just “moment-of-inertia times angular acceleration equals the the sum of the torques.” In mathematical terms this is #08 I ar =—-kOg in which —K0, is the "restoring torque." The solution to this equation is rc On =C, cos | C, sin Ks =C, cos @ot +Cz sin @gt This states that the bob is oscillating with a frequency w= yk/I, which is called the natural frequency. b. If there is a fluid in the gap, the equation of motion of the bob must contain both the "restoring torque" and the torque exerted by the fluid in the @ direction on the solid surface that is perpendicular to the r direction: #05 I ar = -KOg —(2ARL(RM-Ty,.) Here (27RL)(—t,9|,,.,) is the force, and R is the lever arm. Next one uses Eq. B.1-11 to replace the shear stress by the appropriate product of viscosity and velocity gradient. This gives Eq. 4C.2-1. c. Equation 4C.2-3 is obtained from simplifying Eq. B.6-5. d. The choice of the variable x is convenient, since x = 0 at the surface of the bob, and x = 1 is the inner surface of the cup. There are many ways to select the other dimensionless variables, but the choices we have made allow the viscosity to appear only in the parameter called M. Equations 4C.2-11 and 12 follow immediately. e. From Eq. 4C.2-14 we get for the bob angular displacement 0, = R{ 03 exp(i@z)} = (08, +108, )(coswr + isin dr) = 0%, cos@r— 02, sinae 4-23 Here 6%, and 62, are the real and imaginary parts of the complex amplitude 6%. The angular displacement of the bob can also be written in terms of an amplitude and a phase shift: 6, = Acos(Gr ~ a) == Acos@tcosa — AsinOrsina When the two expressions for the bob displacement are equated, we find that Acos@t = 02, and Asin@t = 08, from which we get and tano = O& = S{a%]} 9%, {eR} A= (0% + 0% for the amplitude and the phase angle respectively. The ratio of the amplitudes of the cup and bob is then |02|/ 02. dg° dx ae f. wg? mis and (it)? 0° = —08 + M. l=0 g. The differential equation for $° is then Pe? dx? - (Be =0 with ¢°(0)= AOgi@ and $°(1) = AO%ia The solution to this equation with the boundary conditions is then © = Amae iD. 4. 90 po i | sinh Jia/Mx 6° = Aiwoe cons [Baal x ~ OR cosh 7 Seat Differentiation of this with respect to x gives Eq. 4C.2-16. h. From Egs. 4C.2-12, 14, and 16 it follows that 290 _po, MAID fi@(p. a fio (i@)" OR = OR + Sn AE | ORcoshy 7 This equation may be solved for Oz, and Eq. 4C.2-17 results. 4-24 i, When the hyperbolic functions are expanded in Taylor series and the terms arranged in powers of 1/M, we get 8 — cosh fio, (1-*) sinh, ji@/M 6 VM" Maio JiayM “fs@)-368) 45 From this, Eq. 4C.2-18 follows. The results of part (e) can then be used to get the amplitude and phase angle. 4-25 4C.3 Darcy’s equation for flow through porous media. (a) In Case 1, the equation of state is p = po. Hence, Eq. (4C.3-1) gives (Yew) = Insertion of Eq. (4C.3-2) for vo, with p = po and g = —V4, gives (v *[Vp-+poV8}) =0 or VP in which P = p+ pod is the “modified pressure” defined in Chapters 2 and 3 for systems with constant p. 0 (b) In Case 2, the equation of state is p = pye#?. Hence, Vp = poBe®°Vp = pBVp, so that pVp = #v Expressing the divergence term of Eq. (4C.3-1) via Darcy's equation, we get ~(V # pe) = +2 (9 # Vp eg) Inserting this result, multiplied by 8/s, into the smoothed continuity equation, we get = Vip B(Y # 64) (c) In Case 3, the equation of state is p = pop. Hence, Vp = poVp and Vp = (1/p0)Vp. Then Eqs. (4C.3-1,2), with the pg term neglected, give a eB = V8 l-Z VP) =+(V*p—Vp) HP = (Vepy, a pVp) 5 1 = 2 (v6 v5? Teo) Kee =v up 4-16 Hence, 2eupo IP _ oa, ZeHpo OP _ vi ea (d) In Case 4, the equation of state is p = pop”, giving p = (p/po)'/". Thus, 1 majmn afm) vp = dpm pimp whenee eee aim ting Coote so that ~(¥ #0) = + ph ( pV) Tag g(m4)/m m+ ie With this result, Eq. 4C-1 gives (m+ Vewpy!”\ Op _ oe jm4nim ( a a @ 421 4C.4 Radial flow through a porous medium a. For the radial flow (in cylindrical coordinates) of an incompressible fluid Eqs. 4C.3-1 and 2 become d xd? (rm) andy, = ESE Integrating the first of these equations gives 0, =C,/r. Substituting this into the second equation and integrating gives or DyInr+D,=-P where new constants of integration have been introduced. These constants are determined from the boundary conditions: When the integration constants have been determined, the pressure distribution is found to be P-P, _ _In(e/R) P,-P, In(R,/R;) b. The velocity distribution is then given by using Darcy's law Kd? K Or ear aer 1__ x (P,-P,) war op 1 -p,)—1_1._* (Ps Dim )r br In(R,/R,) c. The mass rate of flow through the system is 2nKh(P, - P,)p w= pPor(Ri PPaRie= RR 4-28 4D.1. Flow near an oscillating wall The problem is to solve Eq. 4.1-1, with the initial condition that v, =0 at f= 0, and the boundary conditions that v, =0 at y= and v, =v) coswt at y=0. When we take the Laplace transform of Eq. 4.1-1 and the boundary and initial conditions, we get 1 p,= Ge with 8, 0ST pp at y=O and 9, =O at y= This ordinary differential equation is easily solved with the boundary conditions to give = rose h rere e/¥y) This may be inverted by using the convolution theorem, or else by consulting a table of transforms (see, for example, Formula #11, on p. 246 of Vol. 1 of A. Erdélyi, et al., Table of Integral Transforms, McGraw-Hill, New York (1954)). The use of the table leads directly to Eq. 4D.1-1. 4D.2 Start-up of laminar flow in a circular tube a. The partial differential equation, initial condition, and boundary conditions are aL ws) with v.(r,0)=0, 9, (0,£)= finite, and v,(R,t)=0. We now introduce the following dimensionless variables e 2 Ht g oR °° JR ant” Then Eq. 4D.2-1 is obtained, along with the initial and boundary conditions: ¢(£,0)=0, 9(0,1)= finite, and 9(R,t)=0. b. The asymptotic solution is obtained by setting the time derivative equal to zero and solving the ordinary differential equation with the boundary conditions. Then the partial differential equation for $,(£,7) is 43 ( 62) at EGE” 0 with ancillary conditions: @,(,0)=.(), 9,(0,t)= finite, and ¢,(1,2)=0. We now try a solution of the form @,(£,1) =(£)T(r). This leads to two ordinary differential equations aT_ on 1d ene d id) 48 fe T an fa (s 3 0 which have as their solutions T =Cyexp(-a?z) and B=C,Jy(aé)+C,Yo(a€) in which the Cs are constants, and Jy and Y are zero-order Bessel functions. Since Yq is not finite at the tube axis, we must set C, equal to zero. Since = must be zero at the tube wall, this will occur only if 4-20 Jo(ct) = 0.This will happen only at @,,@),03,---, that is, at the zeroes of Jy. Hence there are many solutions to the E-equation that will satisfy the boundary conditions: ©, =C,,Jo(a,€)- Therefore, the general solution to the partial differential equation must be = 2B, exp(-ar)Jo(a,6) The constants B,, are to be determined from the initial condition, (1-€7)= DB, Jo(an$) This is done by multiplying both sides of the last equation by Jo(Gm&)& and integrating from 0 to 1: [plolaind)6(1- £2} = DB, [JOE (Om )EAE Because of the orthogonality properties of the Bessel functions, the only term on the right side that contributes is the term for m =n. The integrals may then be evaluated using some standard relations for Bessel functions. This gives AG =B,,-4[J,(@,)) from which and this leads directly to Eq. 4D.2-2. 4-3) 4D.3 Flows in the disk-and-tube system a. If the tangential component of the velocity depends only on rand z, then the equation of motion simplifies to 2, Pv, re)) + i We try a solution of the product form: v9(r,0) = f(r)g(z). When this is substituted into the partial differential equation above, and the resulting equation divided by f(r)g(z) we get ld(1d/,)__1@g paral) gas The left side is a function of r only and the right side a function of z, only and therefore both sides must equal a constant; we call the constant ~c?. Then we have two ordinary differential equations to solve: ald 2fe 8 ree ipa) e790 eae ee The second equation has the solution g= Acoshcz + Bsinhcz, where A and B are constants. The requirement that the velocity be zero at z = L, gives sinh{cL(1-[z/L])] coshcL & where B is a constant. The first equation can also be written as Sd (e-Dyeo ar rar which is now recognized as a Bessel equation of first order, with the solution f = MJ,(cr)+NY; (cr), where M and N are constants. But since Y, becomes infinite at r = 0, only the J, term is needed, Since f = 4-32 0 at at r= R, we must have J, (c,)=0. There are infinitely many c, that satisfy this equation. Hence the solution is of the form sinhe,L(1- ¢) PolE,6)= SBN e6) cet The coefficients B, are determined from the initial condition ROE = YB Ji (nS) a This is done by multiplying the equation by J,(c,,€)& and integrating over € from 0 to 1 and making use of the orthogonality relation. This gives ROP I (enE)EAE = 5B, fol lCn8 a (Cn8) EAE or __2RQ " Cndo (Cm) Ron) B, Hale)? or 8 Thus the steady-state velocity distribution is £,c) = 2005 alcaé) sinhe,b(1~6) al witle(C,) coshe,L 453 4D.4 Unsteady annular flows For the tangential annular flow (part (b)), the equation of motion for (r,t) is piteny,9]22 ar 05] »)| with v9(r,0)=0, v9(KR,t) = KRQ;, V(R,t) = RQ,. We introduce these dimensionless variables: r A b= Ones Re, @) Then the partial differential equation for ¢(£,7) becomes Sealer with 9(£,0)=0, $(«,t)=—Ka, $(1,t)=1-a@. For the steady state (ie., at infinite time), we have the solution _{1-a(1-x?) we). 1 4 -( ES) e-(peas a Hence the time-dependent solution is 9(E,7)= 9..(€)- (6,2) Here ¢,(&,t) is the transient contribution, which satisfies the partial differential equation for 9(€,7) , but with boundary and initial conditions: @,(«,t) = @,(1,7)=0 and @,(E,0)= ¢.(€) Application of the method of separation of variables with 9.(6,7)= f($)3(t) gives 434 1d _ pd (1 a, gar fil gae®) where -b? is the separation constant. Thus we arrive at two ordinary differential equations which can be solved (the second- order equation is a Bessel function ). Since this is a Sturm-Liouville equation, the complete solution is a sum of eigenfunctions multiplied by the appropriate exponential function: 96:1) = BC,2(b,8)e% in which Z,(b,€) is the superposition of the two solutions to the Bessel equation: Z (nS) = Jn(BuE)¥ (bn) Tn(Ba x) (b,6) These functions satisfy the boundary conditions in that Z,(b,€)=0 at & =k. The conditions that Z,(b,£)=0 até=1 determine the eigenvalues b,. The C, can then be determined from the initial condition, which is 9-(8) = ECyza(b8) When this equation is multiplied by Z,(b,£)€dé and then integrated over the domain of interest, we get * fZa(ene)) 648 3[Z3(0,)~ 725 (6,%)] The integrals are performed by making use of a mathematical handbook. The expression above may be simplified by using 4-35 i. The expressions for A and B ii, The defining equation for the b, iii. The relation Jo(x)¥, (x) J,(x)Yo(x) =-2/a Then it may be shown that Z5(t,)=- 5A and altyk)= he and also C4 (1e ct) = Alba) A= ah (ux) + oF (2n)] [F@.")-F(6,)] in which o- is the dimensionless angular velocity. Then the complete expression for the transient behavior in tangential annular flow of a Newtonian fluid is 1-a(1- x?) re )1 as.o-{ Hoe) (ale an § (N= 2a) + 2 (b)]2a(n8) ate ct [i (6.x) -F?(ba)] Complete tables are given in the original reference. as well as some typical velocity profiles. Also, a Laplace-transform solution is given for small times, for which the expression above converges too slowly to be of much value. 4-36 D.5 Stream functions for three-dimensional flow. (a) The divergence of the curl of A is \(0/0z,)|V A]; which gives, according to Eq. A.4-10, Lope E Lein/0r,) A = >> Lewmar may A OA, =LU Len [a pe mel BOT iw Thus, the mass flux function pu = [Vx] satisfies the continuity equation for steady flow (or for unsteady flow with constant density). Here use has been made of the relations eijx = —€yik and eisk = ein = 0 contained in Eq. A.2-15. ‘The divergence of the product expression for pv is a a (Fe to)xCPH)) =D (: 2. [age 59n,* DS #]) 8 (ddr Or = “Loy 5; 6 (6, Oa (3 3) yr Ihe = LL Dibecmbige (GEE) =LYT Vein dlGvrdvs) = OY Weil nh deve + dr2aven) = €128(Diavi1 dave + Gry Aiavo) + €231(O2a41 O12 + Osvh1O21 v2) + €312(8s1 th O22 + O11 As2H2) + £132(Aisti O22 + Ast Aad) ( + €321(Os2v1 ite + Av abo) + €213(Barh B52 + O11 Oza b2) = Drow Os ha(eras + €213) + ovr istpa(Er2s + £321) + dead Y2(ena1 + e321) + Isr O21 ¥2(E251 + €132) + dar Ooy2(ea12 + e132) + Ar Asavo(ca12 + €213) =0 ‘Thus, the product expression for pv is divergenceless. Here Eq. A.2-15 and the symmetry relations 0;; = 95: have been used. Stream functions of this form have been used by several authors; see footnote 1 of §4.2. 4-31 (b) In each of the coordinate systems shown in Table 4.2-1, the two nontrivial velocity components for incompressible flow are the corresponding components of curl(—834s)/p = curl(—83y)/hs), as may be verified by use of Eqs. G, H and I in ‘Tables A.7-1,2,3. Thus, for Cartesian coordinates with v, = 0 and no z-dependence, hg = hy = 1, and Eq, A.7-18 (with v read as 6443/hs) gives the velocity components ob v, = eurl,(—A)/p ie curly(—A)/p = +: For cylindrical coordinates with v; = 0 and no 2-dependence, hy = hz = 1 and Eq. A.7-18 (with v read as 65y3/hs) gives the velocity components 1a ay ve = euel(—A)/p=—2 5 and v9 = curle(—A)/p = +5 For cylindrical coordinates with vg = 0 and no é-dependence, hg = hg =r and Eq. A.7-18 (with v read as 631)/h3) gives the velocity components 1a 10: v; = curl,(—A)/p ie and tp =curl,(—A)/p = +258 For spherical coordinates with vg = 0 and no ¢-dependence, hs = hg = rsin@ and Eq. A.7-18 (with v read as 631/hg) gives the velocity components a 1 ob 1 oy ee = eurl(—A)/p =F and vy = curlo(A)/p = 5 5 (c) Consider two surfaces, 1(1,:22,23) = C1 and Yo(1, 72,03) = C2, which intersect along a line £. At each point on LC, the vectors Vix and Vy are normal to both surfaces, and the velocity vector v = [(Vih1)x(V2)] is consequently tangent to £. Thus, the intersection of any such pair of surfaces is a streamline. In Fig. 4,3-1, we may choose 1 = U(r, 8) and pz = z; the resulting streamlines in a plane of constant 12 are shown in the figure. (4) Read v in Eq. A.5-4 as the vector A whose curl is the local mass flux pu. Then the net mass flow through S is [retwxapus = f (e— pode for steady flow, or for unsteady incompressible flow. A no-slip condition v = 0 on C requires [Vx A] = 0 there, but this derivative condition does not require the vanishing of A nor of the net mass flow. 438 5A.1 Pressure drop needed for laminar-turbulent transition. ‘The minimum value of Re= 4w/(xDys) needed to produce turbulent flow in a long, smooth tube is about 2100. Poiseuille’s law, Bq. 2.3-21, holds until this critical Re value, giving CT Hence, the pressure gradient needed to initiate the laminar-turbulent transition is \ap/dz| = Se ‘The other specifications for this problem are: w= 18.3 cp=0.183 g/ems; p=1.32g/em*; R= 2.67 cm ‘The pressure gradient required to initiate turbulence at the given conditions is then: (4)(0.183 g/em-s)? (1.32 g/em?)(2.67 cm)? % (11.2 dyne/cm*)(0.1 Pa/(dyne/cm?))(10° cm/km) 1.1 x 105 Pa/km (1.1 x 10° Pa/km)(1.45 x 107 psi/Pa)(1.602 km/mile) dp) ; z (2.1 x 10°) 26 psi/mile 5A.2 Velocity distribution in turbulent pipe flow. a. Application of Eq. 5.4-4 gives _ @o=pu)R _ (1.0 psil(0.25 £1) _ a m= OL = ( (2)(6280 fty ) = 2.367 x 107° psi = (2.367 x 1075 psi)(6.8947 x 10° Pa/psi) = 0.1633 Pa 4. For use of Fig. 5.5-3 we need the following additional values: = 1.0019 x 107% Pas vy = 1.0087 x 1072 em?/s = 1.0037 x 10~* m*/s p = 0.9992 g/cm* = 0.9992 x 10° kg/m* v, = Vt0/p = yf (0.1633 ke/m-s?)/(0.9992 x 103 kg/m) = 0.01278 m/s v,/v = (0.01278 m/s)/(1.0037 x 10-° m?/s = 1.273 x 104 m=") R = (3 in/(39.37 in/m) = 0.0762 m At the tube center, y+ = Rv. /v = (0.0762)(1.27 x 10) = 970 and Fig. 5.5-3 gives ve|y on = 22-7. Consequently, imax = 22.7v. = 0.290 m/s, vt =22.78./T:max, and y/R=s*t/970 We can now tabulate the time-smoothed velocity profile. Asterisked values of ¥./Temax are added here to give a better calculation of the mass flow rate in part (a). Ds /Tamax vt yt y/R o r/R (from Fig. 5.5-3) 0.0 0.0 0.0 0.0 1.0 04 2.27 2.27 0.0023 0.9977 0.2 4.54 4.55 0.0047 0.9953 0.3* 681 73 0.0075 0.9925 04 9.08 12.0 0.0124 0.9876 0.5* 11.35 18.0 0.0186 0.9814 0.6* 13.62 275 0.0284 0.9716 07 15.89 62.0 0.064 0.936 0.8" 18.16 170 0.175, 0.825 0.85 19.30 250 0.258 0.742 0.9" 20.43 392 0.404 0.596 1.00 2.7 970 1.00 0.000 ¢. See the graph on the following page. Velocity profile in Problem 5A.2 1.0 0.9 + |] os a 07 0.6 04 0.3 0.3 p— ff} ff 01 u/R d. The estimate (7) 85 zmax gives (5) Re — Die) _ (2 x 0.0762 m)(0.85)(0.290 m/s) v (1.0037 x 10 m?/s) so the flow is certainly turbulent. =37« ¢. The mean value of v+ over the flow cross-section is 1 Q@wt)= f vtd(r/RY (0.85)(0.290 m/s. Hence, Applying trapezoidal quadrature to the values calculated in (b), we get 1 f vtd(r/RY = (4) (1.0000? — 0.99772] lo 2.27 + 4.54 > 4.54+6.81 ) [0.99777 — 0.99537] ) (0.9953? — 0.99257) [0.99257 — 0.98767} 11.35 ——= } (0.9876? — 0.98147] [0.98147 — 0.97167] aS ws nee {0.97167 — 0.9367} [0.9367 — 0.8257] 18.1 e ns 9m {0.8257 — 0.7427} 10 4200 749" - 0.5964] eS) OT) 15, os 18. ) “TT ) TP + wasn) {0.596 — 0.0007] = 188 m/s Hence, (wt )v, = (18.8)(0.01278 m/s) = 0.241 m/s and w= aR p(o,) = x(0.0762 m)?(999.2 kg/m*)(0.241 m/s) 4.4 kg/s = 9.7 Ibm /s 5B.1 Average flow velocity in turbulent tube flow a. The ratio of average to maximum velocity is obtained as follows for the power-law expression: Bmax )rdrdO 2 (1 2 ’ oy dr olsmi®) = eRe = 2f,(1- 6)!" 848 = 2f0""(1- )ag goon gine J (n)+1 (I/n)+2)), _nl(2n+1)-(n+)] | 2n? eeCas) CLES) CES) CLES) b. For the logarithmic profile we have "he. [v.)rdrd9 Bema fpf rdrao De = ah (2 in 6A \(R-y)ly where y=R-r _ 2% gery Ro + Yes ear [ewe a rE 20. [28% lie -jy") “(zl op env g(ayiiy _ 2% aes K = = [2in2e-— ny In the above we have used the fact that in the limit as x goes to zero, xInx vanishes. 5B.2 Mass flow rate in a turbulent circular jet a. Immediately starting to differentiate the velocity compon- ents with respect to the position coordinates is not the way to solve this problem. It is much easier to solve if one introduces some abbreviations in order to minimize writing: C=C u=Z (= [1447] v, eon Zac ORE In terms of these quantities, the velocity components are Then we can calculate the derivatives appearing in the equation of continuity: WZ __ 2C?_— 2c? Cr)__2C? 27? aap ap OS) Sy 2 ~ Jr p(t) ) ~ spl?) 19, —)_10(w-fut)_1f(2u-w)c a aydu(C ra F5L iF )3(EFAS-20e w(9) = Pl s(t aun") ule? —404)) = 25 (0-402) Thus the equation of continuity is satisfied. Next we get the expres- sions for the terms on the left side of the equation of motion: 0 _( tb) eee) a | ar zal 2p 2x a zp zp az ,az__4C*(1-}u?-4ut) — 4c*(1+4u?)(1-4u7) Ra aR 4C*u(1-4u?) re{y The term on the right side is 13(,22). 10 2C°u)\)__ 10 (207 ror\ or rar" “PIP ror\ 2p =-1(sG9.¢. 2c"), (ee) apoz aif z) + Pai Therefore the equation of motion is satisfied. b. The mass flow rate is ach) rdr ® fuea(Cor/2h'] w(2)= fp" [F 08, (r,2)rdrdo = 2p. n2G: wd ne eT [rea (c3/2) Ps] pee FA Cay f+C/y sf}, 2y() = 2p POE Sa 8aps z 2 5B.3 The eddy viscosity expression in the viscous sublayer We start with Eq. 5.3-13 is dimensionless form ray is) from which + +\3 do” _y_(“\yr_(¥) 4... dy vB 14.5 For slit flow with thickness 2B, we have 2 =P 4 Me Y\_ 9) 2, in WP Ay =n 1-H) = (urn) where the definition of turbulent viscosity in Eq. 5.4-1 has been used. If now we write this last equation in dimensionless form, we have Ty. vi. wo) do* —*=-1 =4144- por “(say { pay Substituting the dimensionless velocity gradient obtained from Eq. 5.3-13 into this expression gives 3 ats % lye =-14( yf) s.. 1425 )y (ely (5) ye Of yr ale aval eo evaee HO LOB H 145. Terms 1 and 2 on the left exactly balance terms 1 and 2 on the right. Term 3 on the right will exactly cancel term 4 if the expression for the turbulent viscosity given in Eq. 5.4-2 is used. With the same substitution, terms'5 and 6 on the right side are of higher order and can be neglected in the vicinity of the wall. 5C.1 Two-dimensional turbulent jet a. The total flow of z momentum in the jet is given by J= ff [Spo2dudy = Woe? 2) fede Since J is constant for all values of z, we conclude that 9, ema 6 Y/VE. b. By integrating one of the stream-function equations we get Yc [ B.A OD, spay | fx 0 27? | fdE +z 0¢ 24? The momentum flow J [=] Lim/t? and JJ/p[=]L’/t. But y[=]L?/t also. This suggests that viee)= [LE ree) is the only form that can be put together for the stream function. c. According to Eq. 5.4-3, the kinematic viscosity is the product of b and @, 3. in which b has dimensions of length and must be some function of z. This leads to the fact that the kinematic viscosity (which has dimensions of 1?/t) can be constructed thus: vtsa)a [EE re) wea LE The third factor has been included in order to make the dimensions come out correctly. The dimensionless constant is then included, but this quantity must be determined experimentally. d, The z component of the equation of motion is , _ W, +5, 7 n= Bf nF) Oe Oe ak” Ox or, in terms of the stream function, 5-9 a hCrr—SC Oz Ox?” Ax Oxdz Ox ox? Inserting the expressions for dimensionless stream function from (b) and the kinematic viscosity from (c), we get {5 852) aa) where f is a function of €. We next convert the derivatives with respect to x and z to derivatives with respect to € (indicated by primes), to get (5- var Fr") Multiplying by z” and canceling two terms in the equation gives JRF’ +45? = ar” When we change to a new variable 1 = £/42, this equation becomes @E (dF) _1@F d(,dF)_1a°F F—>+|—| == —| F—|=-— a? (5) pyrite an\" dn) 2am f. The final result in (d) can be integrated at once to give : 2 re aac, or Ap FF ocr dn dn? But at 7 =0, both F and and its second derivative are zero so that C’ g. Integration of the result iin (f) gives 5-10 dF aE P=2 yc = an* or an h. Integrating this last result gives F dF 1 F lpreen Lan or ~garetanh == 0 or F=~-CtanhCn Then the time-smoothed axial velocity is - a ara)=- 3 [Ea)) a2 #61. [LaF 1 Bw as pW dn 4a TC Cx = one at bot pwe 4a °° gaz i. The mass rate of flow at plane z is w=) {90, a =Wo. 42; pe Eo ps ‘sech*Cndn- 4Az I = Woz,|——C - 2tanh x|) = 2CW¢ Pe yg Btanhaly = 2CWpz1) = 205A |JpWz Thus we see that the mass rate of flow increases with the distance from the exit of the slit. This is a result of the fact that additional fluid is dragged along by the jet. Sil 5C.2 Axial turbulent flow in an annulus a. We divide the annular region into two parts, depending on whether or not r is less than or greater than BR: Region <: aR: bR_ [te _ (Po-PL)R 7 A Region >: R= ea op (Ba ee vd b where v,, = \(P)—P,)R/2Lp. Then the velocity distribution in the two regions may be obtained from Eq. 5.3-3: iT Region <: c= /2= 8 ii vs Region <: Fe = Jel CBE) as = Lan 2) oa v my Region >: [oan eae eu (Sea K y being defined as y =7—aR in Region < and y= R-r in Region >. b. The continuity of the velocity profile at r=bR gives a relation between A“ and 4”: F-i2 lk AS = pul \0-eice |r c. The mass rate of flow through the annulus is w= 2np|pOsrdr + 2np[e D7rdr The first integral is: anon [al (2) “fe 1-2) 1 7 =2npo; f° | Zem( +45 bv +aR)dy 7 1 7 rotaTsoce) es +4 AS? + +(#2)5| a = nttpos| L(b-a) + 2a(0-a)fin(o-a[ us) +A°[(b-a)? +2a(b-a)]- z[je-aF i ma] = AR? pos (b? - a oo =In(b- ( Elsah Te -t Sl The second integral in the expression for the mass flow rate is then: 2npv? fal (22) ® | 5B = 2npve “lz (22-2 [e- ne see ee cane} [3% LR (eine —x)- K we ( "} rh ." v2 /¥) x= = aR*poz[ 2 -{0- Byin(a nf a). (- a| -Aoesnua-a( 22-409 +24°(1-b)- 27 (1-b)'] =a’ (1-¥ | Linco 2) 2°24] We now combine the two integrals and use the result in (b) to eliminate A< in favor of A”; furthermore, we use the result in (a) to introduce ¥,,. This leads to: w= AR*p0,, {af Sm( S2=0-ayi-e) + x ]-al in which Aza? (2 ~ a)” + (1-b2)* ed! ae - Boat (te ay Ht) o-Ps -(3+z35) The expression for A does not agree with that of Meter and Bird, and we conclude that their expression is incorrect. 5C.3 Instability in a simple mechanical system a, The centrifugal force acting on the mass is mQ*r= mQ?Lsin@. The gravitational force acting downward is mg. These forces must have a resultant in the direction of the rod, and therefore _ tang = m2 bsine Geos = 8 mg OL If the angular velocity goes to zero, it would appear that cos @ would go to infinity! b. However, this formula describes the relation among the various quantities when the system is rotating and @>0. When as Q decreases, the right side of the equation attains a value of 1 when Q=Q,,,, and then cos@=1 and @ is zero. For for value of Q less than its "threshold value", the value of @ remains at 0. When one starts up the system from rest, 0 will always be zero. However, if Q>Q,, and there is any disturbance on the system, then the system will move up to the stable curve (given by the equation in part (a). It has to be understood that the graph we have given is only for the steady state, and that to understand the system fully, it is necessary to examine the full unsteady-state equation. c. According to p. 12 of L. D. Landau and E. M. Lifshitz, Mechanics (Pergamon, 1990) the Lagrangian for this problem is L=1mL? (6? +Q? sin? 0) + mgLcos@ Then Lagrange's equation of motion gives for the system we are considering the following equation of motion #0 mL a? = mQ?LsinOcos 6—mgsin9 GAS d. Consider the lower (unstable) branch in the diagram. For very small perturbations (6,) to the steady state (8), we have then sin @ =sin 6, = 0, and cos 0 = 1. Then the equation of motion becomes PO _(o2_8 =|9?-£ Io, dP ( ‘) ‘ Now we try a small perturbation of the form 0, = AR{e™} When we substitute this function into the differential equation we get @, = #i/Q? ~ (g/L). If Q g/L, @, is positive imaginary and e' will increase indefinitely with time. Hence the branch 8 = 0 is unstable with respect to infinitestimal disturbances. e. Next we consider the upper branch for which 0s =g/Q7L and sin@ =1~(g/Q7L). Then the equation of motion becomes mL a o = mQ?L(sin 8) + 8, cos. )(cos 8, ~ 8, sin 8) — mg(sin 6) + 0, cos) =—mQ?L6, sin? 8, where we have neglected terms quadratic in 0,. Hence the equation of motion becomes : T8e + Aa(O? (g/L) (2? -(e/L))0, Wenow try a solution of the form 6; = AS{e’) and get all (a? +(g/L)(? ~(g/L)) For the upper branch, Q? > (g/L), and hence both quantities @, are real. Hence the system is stable to small perturbations. Os 5-6 Oy, = qg/t 5-17 5D.1 Derivation of the equations of change for the Reynolds stress Multiplication of the ith component of Eq. 5.25 by 0; and time-smoothing gives for constant p gives (in the Cartesian tensor notation of §A.9, with the Einstein summation convention and with the shorthand notation 9, = 0/dt): 070,P" ~p( 01, 5.0, + 079,0,8, + 0(9,0,0,) + #7,9,0,0, 2) @) @) 6) © Then we write the same equation with i and j interchanged. When the two equations are added we get, term by term: Q)—— p(070,0; + 970,27) = pa, (7a, + oe) = p( 2700; + 70,07) The second term on the right side is the negative of the left side. The first term on the right side can also be written as 2pd, 010 . Therefore the left side is just 4,070), @) ~(oap FAP’) @) = ~p(878,9,07 + 975,0,01) (used 5.2-10) A) ~p( 079,078, + 159,078, ) = -(017,9,, + 07040, ) (used 5.2-11) ©) ~0( 779,0(07 + 07,0707) =-0( 9,070) - 070,907) -p( 0700, 0% + 0707,4,07) Note that the third term is zero by Eq. 5.2-11, and that the second and fourth terms cancel giving -p(0,;0;27) © +u(070,0,0 + 079,9,07) Combining the above gives: 1, 070; = ~(70,p" + v,0,p") -o(5,9,0,01) -p( 110,9,5, + 170,0,5,) ~p( a,0(0707) +u(f0,9,07 + 079,,77) or 1d, 0707, +0(3,9,0707) TOIT, + V;0,9,5; ) -p(I,0,0707) ~ (ap FoF’) +u(07,9,07 + 079,0,07) The two terms on the left side are the substantial derivative term in Eq. 5D.1-1. The remainder of the terms are set out in the same order as in Eq. 5D.1-1. Ba 5D.2_ Kinetic energy of turbulence Taking the trace of Eq. 5D.1-1, we get PLT) = 2077.09) -o(v-W VW) -2(v’- Vp’) + 2u(v’-V2v’) Modify the third term on the right as follows: 27) = 20) 1207-V) The last term on the right is zero according to Eq. 5.2-11. Then divide the first equation above by 2 to get Ripe? = -0(vV-vv) -(V-Gpo7)v'] (Ww) +nv ve) which is Eq. 5D.2-1. For an interpretation of this equation, see pp. 63 et seq. in the book by Tennekes and Lumley cited on p. 176. 520 GA.1 Pressure drop required for a pipe with fittings. ‘The average velocity at the given conditions is 4(w/p) _ (4)(097 m?/ ()= De = Ge)(0.25 my and the Reynolds number is Pe) = (0.25 m(40.1 m/s)/(1.0037 x 107° m?/s) = 9.99 x 10° ‘Thus, the flow is turbulent, and Fig. 6.2-1 gives £=0.0020 for hydraulically smooth pipe. The total equivalent length of the pipe and fittings is L, = 1234 m of pipe + (4)(32)(0.25) m equivalent for 4 90° elbows + (2)(15)(0, 25) m equivalent for 2 90° elbows = 1274.5 m The required pressure drop, according to Eq. 6.1-4 with L replaced by Le, is then _ 9 1273.5 m 5 q = 2 Te ae (098 keg/m*)(40.1 m/s)*(0.0020) =3.3 x 10" Pa = 4.7 x 10° psi GA.2 Pressure difference required for flow in pipe with elevation change. In this problem the pipe diameter is (3.068 in.)/(39.37 in./m) = 0.07793 m, the mass flow rate is w = (18/60 gal/s)(3.7853 lit/gal)(0.9982 kg/lit) = 1.13 kg/s, the average velocity is - (4)(1.13 kg/s (0.07793 m)?(998.2 kg/m" 0.237 m/s, and the Reynolds number is D(v)p _ (0.07793 m)(0.237 m/s)(998.2 kg/m*) _ Re = ~ = 1.84 x 10% (1.002 x 10-? kg/m-s) * From Fig. 6.2-2 we find that for this Re value, f = 0.0066 for smooth tubes. Hence, Po — pL = ~pg(ho — hr) +255 plo)? f = —(998.2 kg/m*)(9.807 m/s?)((—50 x 0.3048//2) m) + 2{(50 x 12/3.008) + (2 x 15)] - (998.2 kg/m*)(0.237 m/s)*(0.0066) 1.055 x 10° Pa+ 167 Pa 055 x 10° Pa = 15.3 psi ‘The corresponding calculation in terms of Ibm, ft and s, neglecting the small friction term, is Po ~ Pt, = (62.4.% 0.9982 Ibm /ft®)(32.174 ft/s?)((50/ V2) ft) = 7.09 x 108 Ibm /ft/s? = 15.3 psia G2 6A.3 Flow rate for a given pressure drop. ‘The quantities needed for this calculation are as follows, in units of Ibm, ft, and s: (0.25 My /in?)(144 in? /ft?)(32.174 Tb ft/s?-Iby) 16 x 10° Ibm /ft-55 D=05ft; p= 624 lbm/ft?; L = 1320 ft; = 6.73 x 10 Ibm /ft-s Po~ PL Hence, the solution must lie on the locus _ (0.5)(62.4) [1.15 x 10-8)(0.5) ~ 673 x10-tY 2(1320)(62.4) = (4.64 x 10*)/3.52 x 10-3 = 2.74 x 10° Method B: The last equation gives a straight line on the logarithmic plot of f vs. Re, passing through f = 1 at Re= 2.74 x 10° and through f = 0.01 at Re= 2.74 x 10*, and intersecting the f curve for smooth tubes at Re= 3.6 x 10, Hence, the average velocity is 3.6 x 104 = Welw = ceri = ORY and the volume rate of flow is (v) gay = GANV05) 0.75) = 0.152 £13 /s = 68 U.S. gal/hr Method A gives the same result if the plot of f vs. Re/F is accurately drawn. 6-> 6A.4 Motion of a sphere in a liquid. The force of gravity on the sphere is Feray = mg = (0.0500 ¢)(980.665 cm/s?) = 49.03 dynes The buoyant force of the fluid on the sphere is Fhuoy = (4/3) R° pg = (4/3)(0.25 cm)*(0.900 g/em*)(980.665 cm/s?) = 57.77 dynes a. The resultant upward force is Racy — Feray = 57.77 — 49.03 = 8.74 dynes and is balanced, at steady state, by an equal and opposite drag force Fy = 8.74 dynes. 5. The friction factor is defined by Fea RY) ‘Thus, for this system, Fi _ _ 8Fe (2) (Gov) *D¥ov3, 8(8.74 dynes) * (0.500 em)#(0.900 g/em*)(0.500 cm/s)? = 3.95 x 10? f c. From Fig, 6.31 we see that f is very close to its creeping-flow asymptote, 24/Re. To the same approximation, Peso x. 24/f = 0.061 Hence, _ Dvsopf _ (0.5 em)(0.5 em/s)(0.9g/em*)(3.95 x 10?) “84 4 = 3.7 g/cmss = 3.7 x 10° ep 4 6A.5 Sphere diameter for a given terminal velocity. a. Method A: Replot the f-curve of Fig. 6.3-1 as f/Re (which does not contain D) vs. Re. Then from this curve we can find the value of Re for any calculated value of f/Re, and determine D as Rept/pva >- 2, Method B: On the log-log plot of f = f(Re), plot also the locus f = (f/Re)Re, which will be a line of slope 1, and find the desired Re at the intersection of the two loci. This method avoids any need to prepare an auxiliary plot. b. The data of Problem 24.4 give Veo = (1 ft/s)(12 x 2.54 em/ft) = 30.48 cm/s p= 0.045 Ibm /ft*)(453.59 g/Ibm)(12 x 2.54 m/f) =7.2 x 10-4 g/em- ps = 1.2 g/em* w= 2.6 x 10 g/em-s g = 980.7 cm/s? from which we calculate Ps=P tte 3a (°*) :980.7)(0.00026) ( “3 1.203, =278 = We therefore draw a line of slope 1 through f = 27.8, Re= lon Fig. 6.3-1. This line intersects the f vs. Re curve at Re= 0.95. The particle diameter is then calculated as Rey _ (0.95)(2.6 x 104) D = Voop — (30.48)(7.2 x 10-4) (0.0112 cm)(10* microns/em) = 112 microns ¢. Here vee is 10 times larger, giving f/Re= 27.8 x 10%. This locus intersects the f(Re) curve at Re= 75. Hence, at this gas velocity the diameter of the largest particle that can be lost is _ _(75)(2.6 x 1074 ™ (30.48)(7.2 x 10-4 D = (0.89 em)(10* microns/em) = 0.89 x 10* microns 6A.6 Estimation of void fraction of a packed column. The superficial velocity is _ (244 Ib/min)(1_min/60 s)(453.59 g/Ib) _ © = “2865 gfem®)(146 in?)(2.54 em/iny? ~ 1°22 m/s According to Eq. 6.4-9 (the Blake-Kozeny equation, developed for laminar flows), 150pLv G-5 + f DL=n)(ve)p Insertion of the previous result for f gives Were)? Bate) = RO w)(Po— Pi) DU = nya ~ T= w)(Po = Pr) Equation 2.4-16 gives (.) (Po-Pr) Bul Rfi-st 1-* 1—K? in(/n). Combining the last two results, we get the relation 1 fl-«t 1-4? as [i =e main ar needed to make Eqs. GA.7-1 and 3 consistent with Eq. 2.4-16 for laminar flow. The K values given by this formula are in excellent agreement with those tabulated on page 194 and recommended for turbulent flow as well. }. The data for this operation are # = 1.139 cp(=mPa-s) = 7.66 x 10~* Ibm /ft-s; 62.2 Ibm /ft3; D=15in=125f; =Gin/15in=04; G=3.801; #H =0.131; K = 0.6759; ‘w/p = 1500 ft? /s; ___ Aw/p) 4(1600 ft°/s) ; (2) = pe = (wD) ~ wae — 0.5 wey ~ 388 fis; neg = DORE _ (1.25 ft)(1 — 0.4)(888 ft/s)(62.3 Ibm /ft® : = 0.6750 Tae) 1.6 x 10) Equation GA.7-2 then gives: 7 = 8.801 logyo(1.6 x 107 VF) — 0.131 1 Solving by iteration, we get f = 0.00204. The longitudinal pressure gradient in a horizontal flow is then (po =p) _ 2fp(v:)? L Dil=*) __ 2(0.00204)(62.3 Ibm /ft*)(388 ft/s)? “0.25 #0 = 0.4) = 5.06 x 10* poundals/ft? = 10.9 psi/ft ¢. The mean hydraulic radius for this system is _ xD(1~62)/4 _ (1) 7D) = 4 and the correspnding Reynolds number is Re, = 22aC@e)e _ DOL- w)s}0 = Re/K = 2.37 x 10" # # Equation 6.2-15 then gives f = 0.00181, which is 0.885 times the value found in part (b). The predicted pressure gradient is reduced correspondingly, giving (Po =P1) _ 4.48 poundals/ft? = 9.7 psi/ft 6A.8 Force on a water tower in a gale. The data for this problem are: D = 40 ft for spherical tank; v9 = (100 mi/hr)(5280/3600) = 147 ft/s; p = 0.08 lbyn /ft3; 4 = 0.017 cp = 1.14 x 1075 Ibm /ft-s ‘The Reynolds number for the tank is Dvoop _ (40 ft)(147 ft/s)(0.08 lbm/ft*) ee (1.14 X 10-5 Thy /f-s) =4.1% 107 ‘The friction factor is approximated as 0.5 by extrapolation of Fig. 6.3-1. ‘The horizontal force of the wind on the tank is then ea = @&®) (Go 08 Ibm /f2)(147 9?) (0.5) = 5.4 x 10° poundals = 1.7 x 10* Iby = 7.5 x 10* Newtons 6A.9 Flow of gas through a packed column. For this compressible-flow problem, we write Eq. 6.4-12 in differential form with v0 = Go/p: > 2 8 _ yy tGo—e? 7G de pbh Inserting the ideal gas formula Me RT to z = L, we get the following implicit expression for Pp and integrating from 2 = the superficial mass flux Go: sotGoG 2 | TAHI-3)),_ Mion oe tap, @ [LO apr to P) ‘The terms are then calculated for this system in egs units: 9 HtGe =<)" wep aE = ppp (1-495. x 1074 g/cm-s)(Go g/em?-s) (1 — 0.41)? , = 150 Gia/ie am CaF (5.5 x 30.48 cm) = (753.4Gp) g*/cm*-s? _ 7 (Go g/em?-s)? (1 — 0.41) (2.54/16 em 0.418 (15820G3) g?/cm!.s? My 2 44.01 g/g-mol apr? — Pt) = @3raB1 x10? x 300 gem®/s-g-mol) x [(25 x 1.0133 x 10°)? ~ (3 x 1.0133 x 10°)? g?/em?-s*] = (1.116 x 10°) g?/em*-s Combining these results, we get the following quadratic equation for Go, 15820G3 + 753.4Go = 1.116 x 10° (5.5 x 30.48 cm) 0 which has the roots Gq = TTB AG VTSBAT FA TSSLOVT. TG x 10") ° 2 x 15820 753.46 +t 265746.2 2 x 15820 The positive root is 8.375 g/cm?/s. the mass flow rate is then w = (1 /4)D* Go = (0.7854)(4 x 2.54 cm)?(8.375 g/em?/s = 679 g/s An identical result for w is obtained if p is assumed constant at the value 7 = (po + px)M/RT. 6-l0 6B.1 Effect of error in friction factor calculations The Blasius formula for the friction factor for turbulent flow in circular tubes is Eq. 6.2-12: 0.0791 Re¥# f= We now form the differential of f, thus gp= 19-0791 ape 4 Re? and regard the differentials as the errors in the relevant quantities. This result may be rearranged thus 0.0791 1 of =~ Soe ape Re=-f- “gage From this we find af __1dRe f 4Re The quantity df/f represents the fractional error in f, and dRe/Re is the fractional error in Re. Therefore, if the Reynolds number is too low by 4%, then the friction factor will be too high by 1%. g-ll 6B.2 Friction factor for flow along a flat plate a, Equation 4.4-30 gives the kinetic force acting on both sides of the flat plate for laminar flow: 1.328) ouLW?03, and an appropriate Reynolds number for a plate of length L is 132g | 2.1328 \Lv_.p Re, b. For turbulent flow, the force acting on the plate is given by Eq. 6B.2-1 as F, = 0.074p02WL(Lv..p/n) ° Then the friction factor is pote _ 0.074902 WL( AK Here we have used the same definition of Reynolds number as in part (2). 6B.3 Friction factor for lamifnar flow in a slit a. The mass flow rate through a slit of width W, thickness 2B, and length L is, for laminar flow _2(Po-P,)B Wo _ =3 a = p(v,)(2BW) and the kinetic force acting on the walls by the fluid is F, =(P)-®,)(2BW) Then the friction factor is pF (Po Pi)(2BW) AK WL)(Je(v.)*) We now replace one of the (v,) in the denominator of this expression by using the above expression for the mass flow rate. This gives pa (PomPi)2BW) Sul Rw WI) a )) (PoP :)B* 2B(v.)p/u Re b Next we try using the mean hydraulic radius empiricism suggested on p. 183. For the plane slit, R,, is 2BW_ 2W +4B In the last step we have made the assumption (consistent with the derivation of the velocity profile for the slit) that B< 1. Hence, the “terminal velocity” is 1/c. A second integration then gives (for 2 = 0 at t = [ dz tanh egtdt or : 1 pet = | tanh pag 9 Sy 1 gay meoshest Now mgc? = 0.221 R?p, so that c= (0220 p]mg 0.220 Rp FTP? poohg (3) 02 (55) (s) Note that this solution assumes that the particle is in the Newton’s drag law region during its entire trajectory. However, the initial condition that » = 0 at t = clearly outside the Newton’s drag law region. (-8 6B.8 Design of an experiment to verify the f vs Re chart for spheres a, We start with the friction-factor expression in Eq. 6.1-7. Since we need to have a D expression containing the Reynolds number (but not the terminal velocity), we eliminate the velocity in favor of Re. This gives f $a (Pam ==) _4.gD (22) (t= =) _4 gD? Burl p JT aRe a) p Ja Reza? Pas PP We now solve this expression for D to get D= J3_fRe? we 4 (bx —p)pg b. Next, substituting numerical values into the formula, we get _ {3 (2)(100)*(107)* P= eee OO 6B.9 Friction factor for flow past an infinite cylinder We select the area A of Eq. 6.1-1 to be the area "seen" by the approaching fluid, which is DL, and K to be 4pv2. Then combining Eqs. 6.1-1 and 6B.9-1 gives 4npv.L Fr = JAK= 77 aire) We now solve for f and insert the expressions for A and K to get po itutal 1b a In(7.4/Re) (DL)(}pv2) In(7.4/Re) (Dv..p/) Therefore 8x f*Rein(7.ayRe) 6-20 6C.1 Two-dimensional particle directories a, We start with Newton's law of motion for a particle Some = SF which states that the rate of change of momentum of the particle is equal to the sum of the various forces acting on the particle. In this instance, the forces are those associated with friction and those related to gravity. Specifically, since m is a constant, we may write ($2R° Pap) Rv = (R)(Seu0(e eH) +($4R pay — 42RD) 88y Here the positive direction for the y axis has been taken to be in the direction of gravity (i.e., downward"). The frictional force has been taken from Eqs. 6.1-5 and 5a, and the gravitational force from Eq. 6.1-6. Dividing v by its absolute value |v|=./02 +03 gives the vector n of Eq. 6.1-5a. When the above equation is divided by the mass of the sphere, we get a fo +0 fy {1-2 Po. “Je, Next we take the x and y components of this equation to get: fe 2. oe frre do, 7 Pais 0, + 0, f +] 1-H yVPx + Py, ( Pas J® “dt in which it is understood that f is a function of the Reynolds number based on the instantaneous velocity: Re = 2R|v|O,ir /Hair + 6-21 b. For the sphere that is dropped vertically, the y component of the equation of motion becomes dey 3 Pa 2 Pa Te BR ogy * oon s Thus the particle that is fired horizontally has a frictional force that is Jo? +07 /v, times greater than that of the particle that moves vertically. Therefore the horizontally fired particle will be slowed down in its descent and the vertically moving particle will land first. c. For the particle moving in the Stokes’ law region, the friction factor is given by Eq. 6.3-15: a Re 2R ot + 0% Py. [ay Then, the x and y components of the equation of motion given in (a) become: de, 9 Mais y dt 2R® pay * 9b e, 9 Haig {4 Pac, 2R? Pan” ( als ‘Thus, the equations of motion are "uncoupled.” Therefore the motion in the y direction is completely independent of the motion in the x direction, and both particles will arrive at their destination simultaneously. 6D.1 Friction factor fora bubble in a clean liquid The rate of energy dissipation in any region is, for a Newtonian fluid with negligible dilatational viscosity E, = [JJ(-t:¥v AV = pfff(Vv + (Vv)")-vav = Aff) (Vv +(Wv)"}(Wv + (Wv)" av For potential flow, [V x v]=0 or Vv =(Vv)', and hence B= Bufff(voe(vv)' av = 24 ff (v-[v-Cov)"]}ev auf (v-v*v)av But for irrotational flow of an incompressible fluid (v?v)=V(¥-v)-[Vx[Vxv]=0 Then using the Gauss divergence theorem on the remaining term E, = 2u f(n-[v-(Wv)" ]}is =n ff (n- Vo? as =n ff 20" 2 sin odd surface at R The surface integral includes the spherical surface at r = R at which n=-8,, and the surface at ‘= at which n=8,. For the potential flow around the sphere, we get from Eqs. 4B.5-2 and 3 Then, when this is substituted into the expression for E,, we get £, =-2au-( 222)? fsin? ado = 12muRo? =F, p= 2m | ~ Fg IR? fp sin? 60 = 12mpRo2 = Fv, Hence, from F, =(7R?)(}pv2)f we conclude that 48 48 G24 TA.1 Pressure rise in a sudden enlargement. According to Eq. 7.6-4 mom= rd ( -1) in which 6 = 2 We compute 8 and vz thus: Si _ (Di\? _ (8)? _ psa (pi) = (5) =o8e _ Q _ (450/60 gal/s)(0.13368 ft*/gal) _ , 9. wes = Gps) THs 1 mom=od (5-1) 1 - 2 5)? = (63 Im /ft?)(7.35 ft/s) (; = 1) = 7610 poundals/ft?) = 1.64 psi Hence, 7A.2 Pumping a hydrochloric acid solution. Equation 7.5-10 reduces to 1y2Z ¢ (pipe friction) + 10} (ext + Lo" FES (pipe friction) + Zu} (ext loss) for the liquid between free surface “1” and free surface “2”, when the liquid is regarded as incompressible and the negligibility of kinetic and potential energy differences between those locations is noted. The major right-hand term is Pa—pr _ (4-1 atm)(6.8087 x 10! Ibm/ft- s2/atm) e 62.4 Ibm /ft? cearanae teas the pipe friction term (with f = 0.0048, from Eq. 6.2-12) is cee 1 =} 2_300 fe eee 3 ppad = 52-80 fils) 5 (0.00484) = 92 £17/s?, 05/2 Ft and the exit loss is | 32-30 ft/s)* = 2.6 ft? /s?. ‘Thus, the required work input per unit mass of fluid pumped is Wr = 3273 +92 +2.6 = 3368 ft? /s? Multiplication by the mass flow rate of 2 w = (R?)p(v) = Og ft?)(62.4 Ibm /ft)(2.30ft/s) = 12.5 Ibm /s gives the power required from the pump: W = Ww = (3368 ft? /s?)(12.5 Ibm/s) 4.22 x 10* ft poundals/s) x (3600 s/br)(1.5698 x 10~® hp-hr/ft poundal) = 2.4 hp = 1.8 kw 7A.3 Compressible gas flow in a cylindrical pipe. For steady isothermal, horizontal pipe flow of an ideal gas at T = 25°C=536.7 R with flat velocity profiles and Wm, = 0, Eq. 7.4-7 gives Caleulation of the terms in ft?/s* gives RT, pr __ (4.9686 x 10! Ib ft2/s?-1b-mol-R)(536.7 R) Snape FO BE = COGS 1 af [sme RYGIET ND ha.2 = 6.60 x 10° 12/5? Mp (8.01 Ib,,/ib-mol) eee and with p = pM/RT in corresponding units, pr = (2 x 2116.2 Iby/ft)(28.01 Ibm /Ib-mol) /(1544.3 ft-lbs /Ib-mol-R)(536.7 R) = 0.1430 Ip /ft® = 2p2 Then rw? (1 1 2G) Ga) _ 1 ( (0.28 Ibm/s)\? i 1 ~ 2 Ge /64 8?) (0.1430 Ib, /f3) 0715 Ibm /ft)2 = = 2.30 x 10° £2 /s? Combining these results, we get By = 6.60 x 10° — 2.39 x 10° = 6.58 x 10° ft?/s? With the aid of Table F.3-3 we get the corresponding values in other units: By = 26.3 Btu/lbm = 6.11 x 104 J/kg 1-3 7A.4 Incompressible flow in an annulus. (i) The mean hydraulic radius is (see Eq. 6.2-16): S_ (R-R) 1 Z~ 2n(R,+ Ri) 2 R= (Ro — Ri) ‘The mass flow rate is w = (241/60 U.S. gal/s)(231/1728 ft? /gal)(62.4 Ibm /ft®) = 33.5 Ibm/s and the average flow velocity is a pS (5 — Ri) _ (83.5 Tom /s)/(62.4 Ibm /ft*) (v) = a((7/12)? — (a/lay? fee) = 0-925 ft/s (ii) The Reynolds number is (see Eq. 6.2-18): Re = Rave _ {Ro Raw ___2w Te aRE Ru = aR + Re _ 2(33.5 Ibm/s) © x((7 + 3)/12 ft)(1.14 x 0.000672 Ibm /ft-s) = 3.34 x 10° Hence, the flow is turbulent and f = 0.0059 according to either Eq. 6.2-15 or Fig. 6.2-2. (ii) From Eq. 7.5-10 we calculate the work required W in ft-poundals per pound [=] ft?/s?: W = o(ha— hy) + SOEs L (Ro — Ri) = (32.2 ft/s?)(5 ft) + (0.615 ft/s)” (in) (0.0059) = g(hz — hn) + (v)? f = (161 + 0.175) = 161.1 ft? /s? ‘The power output required from the pump is then wW = (33.5 Ibm/s)(161.1 ft/s? = 5.4 x 10° ft-poundals/s = 1.9 x 10" ft-poundals/hr = 0.31 hp = 0.23 kw TA.5 Force on a U-bend. According to Eq. 7.2-3, if the z-axis is taken in the direction of the downstream unit vector u; at 51, which is the negative of the downstream unit vector wz at Sz and perpendicular to the gravity vector g, then the force exerted by the water on the U-bend has the z-component (52 © Fy—s) = (viw + pi $1) (52 © 21) — (vow + prSr) (5, © Uz) + Mrot(5e © 9) when the velocity profiles at Planes 1 and 2 are approximated as flat. With (6 ¢ ty) = 1, (62 © uz) = 1, (62 #9) = 0, 51 = So, and v1 = v2 = w/p = Q/S, this gives Fe,joy = (v1 +02) + (pi + p2)S (2Q/5)(Qp) + (pr + P2)S = 2Q?p/S + (pi + P2)S with Q =3 ft°/s. Thus, the horizontal force of the water on the U-bend is Fee,goa = 2(8 f° /5)?(62-4 Vm /ft3)/ (4/144 £0?) + (21419 Iby/in®)(4m in?)(32.174 Ibmft/s?-Iby) = (12871 + 16172 Ibmft/s?) = 29043 Ibmft/s? = 903 Iby 4-5 7A.G Flow-rate calculation. As our system for the mechanical energy balance, we consider the water between plane 1 (the upper liquid surface in the constant-head tank) and plane 2 (just inside the outlet of the exit piping). We neglect the velocity at plane 1, set p = Patm at planes 1 and 2, and treat the water as incompressible. Then Equation 7.5-10 takes the form fe 3M to — a1) = in which the kinetic energy and friction terms all are calculated with the same average velocity v. Collecting the coefficients of v?, we get vit Ep tosss oso, Ry 2g(z1 — 22) in which the coefficients 0.45, 0.4, and 0.4 are the ey; values for the sudden contrac- tion at the pipe inlet and for two smooth 90° elbows. There is no enlargement loss in the system considered, since plane 2 is just inside the exit of the piping. Setting Ry = D/4 and solving for v, we get 2g(z1 — 22) 2.25 + 4fL/D ‘This result is explicit except for the friction factor f, which depends on Re and thus on v. Multiplying the last equation by v/v and evaluating all the dimensional con- stants in egs, we get the working formula, ___12.7%em ~~ 0.010037 cm?/s 1.907 x 10° (A) ‘The last equation is solvable rapidly by iteration. A first trial value 1x10* for Re gives f = 0.0045 from Fig. 6.2.2, and a new value Re= 9.72 x 10° from Eq. (A). At this new trial Re value, we get F = 0.00285 from Fig, 6.2-2, and Eq. (A) then gives Re= 1.056 x 10°. Returning to Fig, 6.2-2 with this new trial Re value, we get f = 0.0028, and Eq. (A) then gives the value Re= 1.059 x 10°, which we accept as final. ‘The mass flow rate can then be calculated as w = RetDy/4, or the volume flow rate as fee Rex q=5- (1.056 x 10°)()(12.7 em)(0.010037 em?/s a RE 11 x 108 em, = 0.11 m3/s Calibration is necessary to get accurate measurements of flow rates in such an apparatus, because of the uncertainties in the geometric details and thus in the energy loss factors (see Table 7.5-1). 7A.7 Evaluation of various velocity averages from Pitot tube data. 1.07—>— — r- + —p-H}¥_] vo it | 0.8 = 0.7 0.6 ma + es gs \ D:,max | 04 0.3 0.2 0.1 0 01 02 03 04 05 06 O7 08 09 10 usr/R This figure shows velocity data from the C. E. thesis of B. Bird (University of Wisconsin, 1915). Bach measurement is labelled with its position number from the table in Problem 7A.7. The flow is evidently turbulent, but not fully developed; if it were, the measured velocities at positions 1, 2, and 3 would show better agreement with those at positions 7, 8, and 9 for the same r values. The solid curve in the figure is a curve-fit of the data, and the dashed curve represents the 1/7th-power model given in Eq, 5.1-4, which is based on extensive experiments in fully developed flow. Application of Simpson’s rule to 11 uniformly spaced points on the solid curve gives the following velocity averages. A finer grid would give more accurate approx- mations to the integrals. _ f 2 udu Pmax Jo Umax = [(1 x 1.0 x 0) + (4 x 0.995 x 0.1) + (2 x 0.986 x 0.2) + (4 x 0.98 x 0.3) + (2 x 0.975 « 0.4) + (4 x 0.96 x 0.5) + (2 x 0.94 x 0.6) + (4 x 0.92 x 0.7) + (2 x 0.87 x 0.8) + (4 x 0.755 x 0.9) + (1 x 0x 1.0)] x 2/30 = 0.832 ‘1 x 1.0? x 0) + (4 x 0.995? x 0.1) + (2 x 0.986? x 0.2) + (4 x 0.98? x 0.3) + (2 x 0.975? x 0.4) + (4 x 0.96? x 0.5) + (2 x 0.94? x 0.6) + (4 x 0.92? x 0.7) + (2 x 0.87? x 0.8) + (4 x 0.755? x 0.9) + (1 x 0? x 1.0)] x 2/30 749 1 4p = f = —2udu ‘0 Umax = [(1 x 1.0% x 0) + (4 x 0.995" x 0.1) + (2 x 0.986° x 0.2) + (4 x 0.98* x 0.3) + (2 x 0.975° x 0.4) + (4 x 0.96% x 0.5) + (2 0.94% x 0.6) + (4 x 0.92% x 0.7) + (2 x 0.87% x 0.8) + (4 x 0.755% x 0.9) + (1 x 0° x 1.0)] x 2/30 = 0.680 ‘These integrals give the ratios (v?)/(v)* = 1.08 and (v*)/{v)* = 1.18. As one might expect from inspection of the plotted curves, these ratios differ significantly from the values 1.02 and 1.06 calculated for a 1/7th-power velocity profile. +4 7B.1 Velocity averages from the }-power law :) _ Vmax ) Vrrax (2) _ Urnax oly K-(R)” rdrd6 a. Average of the velocity . Pita hOB = 2fx(1-x")(7x°)dx=14(2-4)= 2 b, Average of the square of the velocity Ke [b= (oR) )P” redo oe anda a = 2f}x(1- 27? )(x¥? dx =7(3-4) = 8 so that (32)/(2.)° =(#)(8) = # c. Average of the cube of the velocity Gi [1-(e/R))” rarao FF firarae =2ha(t-s"?\G2 d= 88-9) = 8 so that (32)/(8.)° = (#)(8) = 489 == 2 (1- 6)” ede 7B.2 Relation between force and viscous loss for flow in conduits of variable cross section For the variable cross-section, the macroscopic balances in Eqs. 7.5-5 and 6 have to be restated (when the gravitational forces can be neglected) thus: (momentum) 2 ~ 0,) + (P18; — PrS2) (mechanical energy) (ei -28)+ 5 -) To prove that Eq. 7B.2-1 is correct, we multiply the second equation above by pS, (defined in the problem statement) to get PS mE, = 40S,,(07 - 03) +S (P - Pa) = 408 (01 ~%2 (01 + P2) + Su(Pi- Pa) If Eq. 7B.2-1 is to be correct, then in the term containing the velocities, we must have w= 48,,(v, +02). That this is indeed true can be seen as follows: SS, w/(1,1)\_ss wssovon ots) SLB) 2PS(01 + ?2) pS,” pS; \S, +S, PAS, $2 )\S, +52 Next, we have to verify that (p,5, - p,S,)=S,,(p; - P2) + Pm(S: - S2)- Substituting the definitions of S,, and p,, gives for the right side: 28,8 _ PrSi + PrS2\ig _ (B20, r)+( 5, )7%) ~_ 25:S2P1 ~ 25,S2P2 + S7P1 — S3P2 ~ $1521 + S1S2P2 S, +S, = PrSi(S; + $2) P2S2(Si + S2) S, +S, = PSs — P2S2 (Ref: R. B. Bird, CEP Symposium Ser. #58, Vol. 61 (1965), pp. 14-15. +l 7B.3 Flow through a sudden enlargement The pressure rise is given by Eq. 7.6-5, and, in Eq. 7.6-6, in terms of the downstream velocity. We need a similar expression written in terms of the upstream velocity: Pa — P= (60, )[ - (Be,)] = pvzB(1- 8) Then, to get the extremum, we set the first derivative equal to zero: Abe Ses-P poi(1-28)=0 Hence, we get Dz _1 D, =Di li or Po 5 2 Dy 2 dD We still have to verify that this is a maximum for the velocity. To do that, we need the second derivative: #(p.-P (e 1) = 2902 <0 Since this is negative, we have found the location of the maximum. The maximum pressure difference is then P2— Pi = pr; B(1—B)= i pop Fez 7B.4 Flow between two tanks The flow in a tube can be described by the steady-state macroscopic energy balance of Eq. 7.4-7, which for this problem simplifies to or p4~Ps=pe, Then using Eqs. 7.5-8, 6.2-16, and 6.2-12 we get 1 yay 4b 0.0791 pint) peer (Note that we could just as well have started with Eqs. 6.1-4 and 6.2- 12.) The last equation can now be written for each of the two systems, and we omit the factors involving numerical constants, tube length, and the physical properties (since they are the same for the two systems): en i) trx-roh (2) (2) (2) caren) (Pet) "(ey a(He) wy) 1 . -(2) al fr =Pa)y 1 _ 7B.5 Revised design of an air duct a. The flow in a tube can be described by the steady-state macroscopic energy balance of Eq. 7.4-7, which for this problem simplifies to Peon -E, or p)-p,=pb, Then using Eqs. 7.5-8, 6.2-16, and 6.2-12 we get L L 4L 0.0791 R, f= 4(0?) f= 40(0*) Pi-P2 = 40(0?) D kel (Note that we could also have started with Eqs. 6.1-4 and 6.2-12.) b. Label the two systems--the original and the revised--I and II. Then the pressure drop in the two systems will be 4) 0.0791 4040"), (Pi-P2), GR,Opn)™ Ra yy 0.0791 4(v*) PrP GR atolyon)™ Bex Since the numerical constant, the length L, and the values of the physical constants are exactly the same, we now omit those quantities from consideration and concentrate on the quantities that are different. Then we have (with W and H being used for the width and height of the ducts): oy (wis 2 _ [2(Wi +H)” (r:-P2) RY (gz) SF WAP Ont. (w/Su)" zi [2Wa + Ha) Rim (Su/Zu)" Si [Wat (P:-P2)y 4 We have used the fact that w is the same in both systems. Next we make use of the the fact that the pressure drop is the same in both systems: (2m +H)" _[2(Wa + Hn)]” way (Wot c. Next we put in numerical values [24+4f" 2a +2)" (44P awa? Taking the fourth power of both sides gives fie)’ _ [2a +2) fie]? [awa] Thus the equation for 2W, =x becomes 1 _(x+4)° ae 16 x This equation can be solved by trial and error, and the solution is x = 18.4 ft or Wy = 9.2 ft. 7-5 7B.6 Multiple discharge into a common conduit a, Application of the macroscopic mass balance gives: w,=w, or (v,)S,=(v))S,_ (for incompressible flow) Therefore, if the ratio of cross-sectional areas = $,/S, is specified, the relation between the average velocities is 2) y B 2 (, We now define, for later use, a set of quantities K!”) as tn _ (2) Oey! where the index i indicates inlet (1) and (2), and j is the power to which the velocities are raised. For j = 2 and 3, these quantities are (for the 1/7th power law of turbulent flow) 3 and 88° respectively (see Problem 7B.1 b. Application of the macroscopic momentum balance in the direction of flow gives (with no external forces): (vt )S1 ~ (03 )So + PrS1 — P2Se — Fy 4 =0 The force of the fluid on the solid will consist of the viscous force acting tangentially on the walls (which we neglect) and the normal force p,(S;-S;) acting in the direction opposite to the direction of the flow. Thus, when the quantity B = $,/S, is introduced, we have ota{e- aha hot ~P2)=0 Hence the pressure rise is given by ~ Pi = p(n)" (BK? ~B°KP’) c. Finally we apply the steady-state mechanical energy balance (Eq. 7.4-7) for incompressible fluids to get 2) y,\2 3 uy) ~ (01)"(BKP - B°K?) =500 met B - (2) (BK? - 6K) Kk? Ke) For turbulent flow, the ratios of the K() in these expressions are of the order of unity. 4-11 78.7 Inventory variations in a gas reservoir a. The maximum, minimum, and average values of w, are ol (2) pin = A~B 2n (ag = an b. A mass balance over a 24-hour period is obtained by integrating the unsteady mass balance over the 24-hour period: i (ee = fp wat - fwd If the amount of gas in the reservoir is to have the same average value over a 24-hour period, then 0=24w,-24A from which w, = A. c. The total mass of gas in the reservoir as a function of time can be found by integrating the unsteady macroscopic mass balance: di 7 &( Tia it = ffudt - wad to get imy(t)= m9, + [{(A-A~Beosaf)df = m2, ~ 2 sin ot d, In this problem we are assuming that the density of the fluid does not appreciably change with time, and so we assume that it is constant. However, the volume of the gas. V(t), will be a function of time and m,.,(t)= V(t), so that 1-18 B. eV (t)= mb, -Fsin at The criterion for uninterrupted operation of the system is that V(t) must never go below zero. In this limiting situation, the quantity m,(t)=PV(t) must then oscillate between the minimum value, m®, —(B/w) and the maximum value, m?, +(B/o). Therefore, the minimum total mass in the system that can accommodate this kind of oscillation is that PV arin = [hon + (B/00)]— [rey — (B00) or 2B ___(2)(2000) Vin = 09 (0.044)(27/24) =3.48x 10° ft? 3. Add a three-day supply to the amount found above _ 2B 7 sy, (72)(5000) Vanin = * OX28A)= (3.48% 10 008) = (3.48 10° ) + (8.18 10°)= 8.53 x 10°F? 7-19 7B.8 Change in liquid height with time a. We want to calculate the volume between the liquid level at h and the part of the sphere below the liquid level. The sphere is visualized as being generated by a circle in the xz plane, with its bottom at the origin and its center at x = 0,z = R. Such a circle has the equation x? 4(z-R) =R? or x? =2Rz-2? Next we visualize the liquid volume as being made up of thin circular disks of thickness dz, each disk with a volume dV = nx%dz = n(2Rz-z*)dz Then the total volume of the liquid is V=afi(2Re-2?)dz = n(Re? ~42°) = (Rh? - 41°) or 1h vener(1-35) This may be checked by verifying that when the tank is full, h=2R, and the liquid volume is V = 47R°; that when the tank is half full, h=R, and the liquid volume is V = 37R°; and that when the tank is empty, h=0, the liquid volume is V= 0 (Note: This method of obtaining the liquid volume was suggested by Professor L. E. Wedgewood, University of Illinois at Chicago) 4-20 b. To get the liquid height as a function of the time, we start with the differential equation in Eq. 7.1-7 which may be rearranged to give: (H-20+R)+H2R*0)) aot dt A This separable, first-order equation can be integrated to give 4H? ~2(L+R)H +L(2R+L)InH = At+C Att =0,h=2R, and H =2R + L. Therefore H(2R +L) -2(L+ R\(2R+L)+L(2R+L)In(2R+L)=C Subtraction and elimination of the integration constant then gives H? = (QR +L)']-2(L+ RA -QR+1)}+L2R+E)In = At or a[Ge+Ly = OR+L}]-2(L+ RYh-2R)+L2R+ Lin AE =At When h = 2R, this equation gives ¢ = 0, and when h = 0, it gives £ = teggye exactly as in Eq. 7.1-8. We now introduce dimensionless variables: = /2R (dimensionless liquid height) and A=L/2R (dimensionless tube length). Then the above equation becomes: 2 2 nta_ At af(neay -(+ayP]-2a +1n-1)+aisayind = oy c. The parameter A =L/2R is fixed by the geometry of the system. Choose values of 7=h/2R from 0 to 1 and calculate t. from the above equation. These may be plotted to give the curve of the dimensionless liquid height versus the dimensionless time. 7-21 7B.9 Draining of a cylindrical tank with exit pipe a. First we write the unsteady-state mass balance for the tank. The mass of fluid in the tank at any time is 7R?hp, and hence the mass balance is (since there is no inflow stream) d a aR?hp = -w, The quantity w, is the mass rate of flow out of the tank, and this is equal to the mass rate of flow in the tube. The latter is given by the Hagen-Poiseuille formula: _m(Po—P.)D%p _ a(pgh-+ pgL)D‘p On 8 aL 128)L Here pgh is the pressure py exerted by the fluid above the tube entrance, and pgL is the "pgh-term" in the expression for P in fn. 1 on p. 50 and discussed after Eq. 3.5-7 on p. 84. (It is unfortunate that we are dealing with two h's here: the ht in the expression for P and the h which is the height of the fluid in the cylindrical tank; thé‘must not be confused with each other.) ‘The mass balance is then arp lt meg h+L)D'p ,, dh__g(h+L)D*p dt 128uL dt 128uLR? This first-order, separable equation can be integrated thus: 4 2 2 tm 28 at or tg = SHEE (1422) HWh+L 0 128uLR’ ‘pg’ L b. One has many mass-flow-rate vs. pressure-difference relations to choose from in turbulent flow. For purposes of illustration we use Eq. 5.1-6, which can be shown to be identical to the Blasius formula given in Eq, 6.2-12. Thus we replace the second equation in part (a) by Ri 7 _ Sarl [eg(h +L)] ” pri e. yv 2( 0.198 u™4L a Then the mass balance becomes dh 1f_ pitRs4 47 aoa aie) 9" B(h+L)"” This equation can be integrated to give Z(h+L)” =-Bt+C Application of the initial condition gives Z(H+L)" =C Subtraction then gives the expression for the instantaneous liquid level 3[(t +L)” - (n+ 1)” J= Bt The efflux time is then S[(H +L)” - D7] = Bernas or 14( 0.198041)” : tettx = 1 oe) ((H+ Ly” -L”) 7-23 7B.10 Efflux time for draining a conical tank 4. From the unsteady mass balance we get, for the truncated cone system with no input stream d ait = ‘2p ~ 4 7}z)) = ~pv,(zr}) From Fig. 7B.10 we see that r/r, = z/z, so that the mass balance may be rewritten as d qa olre/22)°)=-p0,(m3) or Therefore we get Eee tore eee a 22 a b. We simplify the unsteady mechanical energy balance by (i) omitting the kinetic energy contribution on the left side, because it was shown in Example 7.7-1 to be unimportant, (ii) neglecting the viscous dissipation term E, because it is believed to be small, (iii) assuming incompressibility (which causes the E, term to drop out, (iv) omitting the work term, since there are no moving parts, (v) omitting the pressure terms on the right side, since the pressure is atmospheric at both ends, and (vi) assuming no vortex motion of te fluid. All potential energies are with respect to z = 0 as the datum plane. Hence we get: 2 303 + g2p wv, First we get the total potential energy in the fluid. This is done by integrating pgz over the volume (regarded as a truncated cone): ©, = foddv = JegedV = pg? z-nF dz vie) vo 24 Sih za( 2 2) ae oar! 12) 2822) otf) Then the unsteady mechanical energy balance becomes ated tea 05 + gz )(ov,2r}) or (303 + gz )(po.m3) Then, using the result of the mass balance to eliminate the time derivatve, we get 2 2 r, oo 2) of Hy) (dt aon) % z After dividing through by - pmr3v, we get Torricelli's law: 2) c. Using the results of (a) and (b) we get _& (22) Boca) 288 By making the indicated simplification, we can perform the inte- gration, along with the initial condition that at t = 0, z= 2, to get geahop+gz, or v= /2g( , a2 <4 (RG and tao = 42) Pe To get the efflux time, we have assumed that z= 2, =0 when the container is empty. fie2>) 7B.11 Disintegration of wood chips We start with Eq. 7.5-10, taking plane 1 at the top of the slurry dispersion and plane 2 right at the outlet to the digester; we also take plane 2 to be the datum plane for the calculation of the potential energy: 1 3(e2-0)+(0-g21)+ (P21) =0 Therefore the exit velocity is R= A 4r -r2)+ 81) (44 in®/t)(a 2 poundals/b ¢ 65 Ibyy/ ft? + (22.2 /s?)(20 #t) 124 ft/s Therefore the mass rate of flow at the exit is w, = p0,S, = (65 b,,/£° Jaze ft/ (jz £2 )- 2910 Ib,,/s Next, we apply Eq. 7.2-3 (the momentum balance) to get the impact force (assuming that the pressure and external force terms are quite negligible): Fs, = 01, =(124 ft /s)(2910 Ib,,/s)(1/32 Ib, /poundals) = 10,900 Iby 7-26 7B.12 Criterion for Vapor-Free Flow in a ig System. In the piping system of Fig. 7.5-1, the criterion p > pyap might be violated either (i) in the pump, or (ii) at a plane “A” just downstream of the final elbow. Lacking data on the NPSH (net positive suction head) requirements of the pump at the given operating condition, we can only test for condition (ii); further information on (i) is available in Perry’s Chemical Engineers’ Handbook and in unit operations texts. Applying Eq. 7.5-10 from plane “A” to just above plane “2”, we find (since ¥» and no fittings or enlargement loss occurs in this vertical section), mapa vf ban (ez ~ 24) + RTPA = oe which indicates the minimum pressure to be vf La pa= rte [aor sa)+ Pt jl Using values from page 208 and setting p4 = 1 atm = 6.8087 x 10* poundals/ft?, we find the pressure at plane “A” to be 20 5+ 300+ 100 + 120 }8087 — 40186 + 3 = 27905poundals/ft? 41 atm a = 68087 + 62.4 [s0.2(—20 + (30 poundals/ft? Since the minimum pressure is well above the vapor pressure of water at the system conditions, the pipe will run full if the pump does. For mixtures, one must use the bubble-point pressure rather than the vapor pressure. 7-21 7C.1 End corrections in tube viscometers. We apply the steady-state mechanical energy balance, in the form of Eq. 7.4-7, with the assumptions of incompressible fluid and no mechanical work: 1f (vt) _ (v2) =6 me ha) AP ~p)=E, where "1" and "2" are general labels for the input and output streams. We label the plane at the bottom of the tanks as Plane 5, and designate by pi, the pressure at the outlet plane. Then for Run A, we apply the balance to the region between Plane 5 and Plane 2: 1 (28) _ (22) {- Beat +h (Ps + p84 —Pam) = £_(5— 2) Note that the kinetic energy term is nonzero, because the velocity distribution at the inlet and outlet are not the same. For Run B, we apply the balance to the region between Plane 5 and Plane 0 to get: 1( (28) _ (2%) fS-Soa +5 (Pe + pgls ~Po)=E,(5 0) We now subtract the second equation from the first, noting that the kinetic energy terms will exactly cancel, as will the viscous dissipation terms, since the flow rates (and hence the velocity profiles) in the two systems are equal. This gives alle. ~Ps)*P8(l4-!s)+(Po-Pam)]=9 —(*) Next we apply the mechanical energy balance to the region between Plane 0 and Plane 4 in Run B: 1-28 alle ~La)+ 5 (Pe~Pam)=Es(0-> 4) Here the kinetic energy terms do cancel, because the flow is fully developed at the inlet and outlet planes. The dissipation term can now be calculated by using Eq. 7.5-7 together with Eq. 2.3-22: =F[(P0~ Pan) +8(ls ~Ea)] But, according to Fn. 1 on p 50, Po ~ Py =(Po— Pa) +8 (Mo ~My) = (Po ~ Porm) + 28(Ls ~ La) Therefore, by combining the last three equations we get 1 {le La) + (PoPam)=5(Po-Pa)—() Now Eg. (**) can be rearranged and then combined with Eq. (*) to give: By- Py Po Pam pak, 8 TL la = 08-7 (Pa Ps) +080 -1a)] = pelaele, oe it 1,-La poe) Pe Pa spelt Ip-ly PoP pg ta ea in agreement with Eq. 7C.1-1. 7-24 7D.1 Derivation of the macroscopic balances from the equations of change a, Derivation of the macroscopic mass balance We start by integrating the equation of continuity, Eq.3.1-4 over the macroscopic flow system of Fig. 7.0-1: { Pav =— f(v-pvav vo vo We write V(t) to remind ourselves that the volume may be changing with time because of the presence of moving parts within the system. We now apply the Leibniz formula (Eq. A.5-5) to the left side and the Gauss divergence theorem (Eq. A.5-2) to the right side to get 7 4 fav { o(n-vs)d8=- [(n-oves vy so st) We now combine the two surface integrals thus: da itty =~ §(n-plv—vs))aS dt = J ») Next we divide the surface up into four parts as indicated on p. 221, right after Eq. 7.8-2. We also introduce the assumptions (i) and (ii) listed. in §§7.1 and 7.2. Then we divide the surface into four nonoverlapping parts, $= S, +S, +S; +S,,, and write the right side of the above equation as the sum of four contributions: Line =-J(n-p(v—vg))dS—f(n-p(v-vs))d5— f(n-p(v—vs))4S at ] : 4 - f(n-p(v—vs))ds Sm We now evaluate seriatim the terms on the right side: The surface S, is the inlet plane, which is not moving so that v, =0, and the outwardly directed unit normal vector n is the negative of the vector u,, which indicates the direction of the flow. Since v=v, at this surface is assumed (assumption (i)) to be exactly in the direction of flow, we may say that v=u,v,. The evaluation of the integral proceeds as follows: J(n-p(v-vs))dS = + J(u, -u,0,p)dS = +f pv,dS=p,(0,)S, =v 5 5 5% where it has been assumed that the density is constant over the cross section (assumption (ii)). The integral over the exit plane is evaluated in the same way, the only difference being that n is the same as u,, So that -J(n-plv-vs))d! S=— IC (uy -u,v,0)dS = + Jomas= P2(02)S) = W, S Sy On the fixed surfaces both v and vg are zero, so that S, = 0. Also, on the moving surfaces, v = vs, with the result that S,,. Therefore, Eq. 7.1-1 follows at once: d Sigg = P04)5;~ PaO) and the definitions of w, and w, lead to Eq. 7.1-2. b. Derivation of the macroscopic momentum balance First we integrate Eq. 3.2-9 over the volume of the system: { (See}iv=- [(v-pwonv f(sphiv fv-Kv+ fogiv vit) vit) Vit) We now manipulate seriatim the terms in this equation so that they can be interpreted. In so doing we make use of assumptions i-iv in §§7.1 and 2, and follow the procedures in (a) above. The first term is rearranged by using the Leibniz formula (as applied to a vector function), and then P,,, is introduced as the definition of the integral over the momentum per unit volume, and the second term is rearranged, thus: 731 d § (Zov)iv=4 fpwav - fov(vs-njis=£0,. - {[n-povs}s vit) Vit) S(t) sit) The first term and third terms on the right of the momentum balance are treated Ey, using Eq, A.5-3, and the pressure term is treated by using Eq. A.5-2: -fv- “pevyaV =~ [Ine py iS Vt) S(t) - fIV-t}iv =~ f{n-t}is V(t) S(t) = S(vp}v =~ Japs vu) si Finally, the last term is integrated (with g constant) and the definition of the total mass within the system (given above): + JogdV = +m.8, Vit) When the above contributions to the momentum balance are combined we get the following equation: = f[n-po(v—vs)}8~ [nya fbo-2}9 +m sie) se) sit) The surface integrals are interpreted by partitioning the surface of the system as before: $=S, +5, +5,+,,. The first integral on the right is zero on the fixed and moving surfaces, and the integrations on the inlet and outlet surfaces give the following: = Sle pv(v-v.) HS =- few p(uy2,)(u,2,)]- flex p(u02)(4,02)] =p os- apa [vids = +19, (o? )8,- usps (08) Ss 7-22 where assumption (ii) has been used--that the density of the fluid is constant over the cross-section. Similarly the contributions of the pressure integral at the entry and exit planes are: ~ Jnpds = +u,p,S—u,p,5, S148, where, according to assumption (iv), the pressure is assumed constant over the cross-section. The integral of the pressure over the fixed and moving surfaces contributes to the force of the fluid on the solid surfaces, and the negative of that is the pressure force of the solid on the fluid: F?,,. Similarly the integral over the [n-t] term over the fixed and moving surfaces gives a contribution F‘,. Therefore — Jnpds— f[n-tHS=F?,+FO,=F,,, S45, S48 According to assumption (iii) we neglect the contribution of the [n-t] terms at the entry and exit planes. When all of the contributions above are collected, we get Eq. 7.2-1. Ap, = +p,(02)8,u, —p,(08)Sz +P1S¥) — P25)%, HF + Mo dt and the definitions of w, and w, then lead to Eq. 7.2-2. 1-33 8A.1 Flow of a polyisoprene solution in a pipe We use Eq. 8.3-9, for the mass flow rate of a power-law fluid ina circular pipe: aR°p (@e =FuR)" (/n)+3\0 0 2mL Consider two systems, labeled I and Il differing only in the radius R and the length L. Then, for the same mass flow rate and the same pressure difference, and also the same values of the density and the power-law parameters, from the above equation we know that the dimensions of the two systems will be related by Rew) _ RR) a Ti Ly Ly Solving for the radius of the second system, we get y(3ns1) L ren a) oR ( Inserting the numerical values for this problem gives 58 (e om B =) =1.25 cm R " 10.2 m ry 8-1 8A.2 Pumping of a polyethylene oxide solution The starting point is the same as for Problem 8A.1. The mass rate of flow must be the same for the two designs, so that aR3p ‘/n)+3 aRip (ts = PRY" (Po-Pi)R (1/n)+3\0 0 QmL } 2mL Therefore Ro*C") = RX whence RV 4 R, -2 Therefore DR ae Dy Rea and, for n= 0.6 19s 28 D,=D(5) — =(27 em) =) =23.3.cm 2. 2, @2 8B.1 Flow of a polymeric film a, The momentum flux distribution is the same as Eq. 2.2-13, inasmuch as this result depends only on momentum conservation, and not on the constitutive relation. b. Next we write the power-law model as do, an dv, ( a)" do, ( dv, y ws _ ny =-nf - anf - 3 dx dx dx) dx dx which is the appropriate form, because dv, /dx is negative. c. When the above expression for 1,, is substituted into Eq. 2.2-13, we get a differential equation for the velocity m(-S2)" =(agcosBlx or (e228) 2 This equation is integrated to give _ _(pgcosp)! (vl 2.( m ) (yaya Application of the no-slip condition at the wall gives _(pgcospy\"" alin! o-( mm ) Wari Subtraction of the last two equations gives ° -(exsee8)" gle _ lynn ° m (Yn)+1 ~ (exesney" Th +1 } 3)" d. Then the mass rate of flow in the film is 63 w =f mdr EEE)" Sm = pw esses8)" 5 naman, P*RWS? cosB ™ (Yn)+2 3u 8B.2 Power-law flow in a narrow slit Substituting Eq. 8.3-11 into Eq. 2B.3-1, we get for -B1 and my, we have to use L'H6pital's rule im SEH tim SIME _ NE 3, bt eto1 ER fink ink (see Problem 2B.7) c. The mass flow rate is then 2-(Yfn) we 2nf\. po,rdr = 20R* p09 {Sam A _ 2AR?pv9 ea Wn) 4 4p? | U1 3-(ay 2 (n#3) d. When n=4 use of L'Hépital's rule gives ae 1 a =| ne - 1-K kK 2 gt e. When L'Hépital's rule is used we get = AR? I=? a w= aR ond sa j K 8B.4 Flow of a polymeric liquid in a tapered tube We consider a small region of the tapered tube to be a straight tube over a short distance dz; then we can write "locally" __aR’p [- de af (Yn)+3L" dz 2m Take the nth power of both sides to get _dP _2m/_w (1,,)| dz 2 ( +3)| in which Ris a function of 2: R=R, ({& +); It is easier to integrate the differential equation if we rewrite it as _dP dR ar(R-R) 2m[_w (t+ 3) i dRdz dR L °R [ arp! Then when this equation is integrated with respect to R, we get _pPiyp _(_2mL (fae =( 2m 2 (24s) pe gaara Therefore "(pan _ pon p,-p, =| 2m 2 (443) Ry" = Ro” R,-Ry)L ap\n -3n CasG EE 3n JL ap\n Ro-R, This is the power-law analog of Eq. 2B.10-3. gb 8B.5 Slit flow of a Bingham fluid a. For |x|S xy (ie., in the region where the yield stress is not exceeded), 1 = (according to the upper equation of Eq. 8B.5-1. But the expression for the shear stress is, from Eq. 8.3-2: 1,, =—ndo, /dx. Since the shear stress is finite, the velocity gradient must be equal to zero. This is the plug-flow region. For |x|2xq (ie., in the region where the yield stress is exceeded), the lower equation of Eq. 8B.5-1 has to be used. This means that in the region where x > xp, the velocity will be decreasing in the positive x direction, so that y = do, /dx is required so that 7 will be positive. Similarly, when x<-xy, the velocity will be increasing in the positive x direction, so that y = +do, /dx is needed in order to guarantee that 7 be positive. Hence we have: do. Te =HoGi-% for ~BSxs-xy do. Te = "Ho Gi tt for +x SxS+B Since the flow will be symmetric about the plane x =0, we need solve for the velocity distribution only in one half of the slit. We choose to work with the region 0S x <+B. b. Substitution of the upper relation of Eq. 8B.5-3 into Eq. 2B.3-1 gives — tty Mes = Moet % Integration gives: ~Po- Pi ty Toya 2yol ~ "Ho Applying the no-slip boundary condition at the slit wall (z= B), we get Ponty BC 2poL Ho Subtracting these last two equations eliminates the integration constant and gives o, PUF (2) 21-2) (4x) $x< +B) 2HoL B. M\ B (®.-P,)B* (2) te x = 1-| 2) J-- 4} 1-22 /tg)R The Bingham fluid (see Problem 9B.5) gives for the shear stress component tes (r2%) and #229 (rsm) dr lad In the region (r>79), we can combine the shear-stress expression with the Bingham formula to get do, r Hoa, t= TR R Integration gives icceeaesesee r+ rt 2uoR Uo The integration constant is determined from the boundary condition that the velocity vanishes at the tube wall (the "no slip condition’). Then we get EEE] es Since the velocity must be continuous at r=ro, we can set r= rp in this last expression and get Zea a@] “8 (8) a) 4g) p89 ea Giz The mass rate of flow is then w= fo" [ po.rdrd0 = 2npf} v,rdr We then integrate by parts (cf. Problem 8B.5) to get w= 2p] ro, (Rado, (3h al The first term is zero at both limits, and the second contributes nothing in the range (r< rp). Therefore we get w= noir (Set -20)ar oR ty ‘ = AR teh}, 4( to), 1f Au 3\ ag) 3\7R In getting the final expression, ry has been eliminated by using the relation ry =(7/T,)R. When the yield stress is set equal to zero, this result simplifies to the Hagen-Poiseuille relation for Newtonian fluids. ED 8B.7 The complex-viscosity components for the Jeffreys fluid a. The yx component of Eq. 8.4-4 is dt, . diy, tt BE mm te 4 | Dividing by A,, rearranging, and introducing complex quantitites, we get Wye 1 VP ( gpfot d gy i ap tee ME (fem } vas Ge}) 0 = ted +iA,o)} The only reason for using the complex representation of the trigonometric functions is that then we have to integrate only exponentials, and not products of exponentials and trigonometric functions. Thus, we now have to integrate the above first-order differential equation to which the solution is 0 20 gH ee (14 io)’ +c} Ay - If we assume that the fluid is in an unstressed state at f= —o, then the integration constant C is zero. The integral can now be evaluated to give: ng ong a) gin) al 1 (yA, ) +i (1+ id.) =n Hae 28 oat sina) Then we collect the real and imaginary parts within the braces, and discard the imaginary parts to get $i4 1+A,A,07 |. (A,-A2)0 |,0. Tye =—| My — 225 |p cost —| ny) "24 7° sin wt ° THF | +o) By comparing this result with the defining equation in Eq. 8.2-4, we see that the quantities in the brackets are m’(@) and 1"(w) respectively. Equations (F) and (G) of Table 8.5-1 are obtained by setting 4, =0 and A, =A. This shows that the Giesekus model is a nonlinear generalization of the linear Maxwell model. On the other hand, the Oldroyd model, as may be seen from Eq. 8.5-12 and 13, is a nonlinear generalization of the Maxwell model. b. According to fn. 3 on p. 246, the Jeffreys model can be regarded as a superposition of the Newtonian’ model and the Maxwell model. For the Newtonian model. For the Newtonian fluid (here designated by s) we have n(@)=n, and n’(@)=0 That is, the Newtonian fluid shows no phase difference between the stress and the velocity gradient. For the Maxwell fluid (here designated by p) we have, by taking just one term in Eqs. 8.4-14 and 15 Np no 1+(Ao)° mo hoy and n"(@)= In fn. 3, the connection between the constants of the Jeffreys model and the Newtonian and Maxwell models is given: Ny =, + Ny and Ay “at Pp These relations give my and A, in terms of n, and n,. Presently we will need n, and 7, in terms of ny and 2,: A. and ny = (Ay -A2) els We now get the contributions to the complex viscosity for the Jeffreys fluid by summing the contributions of the Newtonian and Maxwell fluids: Ny 1+A,0)' +n 1+(A,0)° 1+(A,o)" = (1 x 1p) * n(Ayo)" = My + MA A207 14a 1+(Ay@)? arn 1+A,A,07 14 (4,0) {(@)= 7, + TAO _ (A -A,)o y 1+(A,0)? mo oF This is in exact agreement with the results obtained above in (a). gb 8B.8 Stress relaxation after cessation of shear flow a. If one writes out the 3-constant Oldroyd model for the special case of steady-state shearing flow, component by component, one gets (cf. Eqs. 8.5-5, 6, 7, and 8): Tye t BAe BA yy =— MOV) Tax Arye = 07 Ty + ALT Tye = 0 and t,, = 0. When these equations are solved simultaneously, we get , 1 1 Tye =—No¥———s_ and t, =-n)Ayy? ——— ye ™ apy ax =~ NAY Tay This expression agrees only somewhat with the data of Fig. 8.2-4. The equation shows the curve for viscosity starting out with the zero-shear-rate viscosity, and then decreasing--which is correct. However, the equation gives n « 7~* for high shear rates, and this is nonsense. Then one would have 7, /"' at high shear rates; this would mean that as one applies more shear to the fluid, the velocity gradient decreases! Therefore the model has to be labeled as unusable at high shear rates. In other words, it can be used only to describe small deviations from Newtonian behavior. b. For unsteady shear flow, Eq. 8.5-8 describes the shear stress. When the motion is stopped, 7=0 becomes zero, and the equation gives 1,, +A, (dt/dt)=0, which can be solved to give Ty, = Ce”, The constant is then determined by the requirement that the solution must match at ¢ = 0 with the steady state expression in (a). This leads to Eq. 8B.8-2. c. The normal stress in steady flow was given in (a). Arguments similar to those in (b) lead to . 1 ot nA? ee (4.7) This predicts (correctly) that the normal stresses also relax after cessation of flow, but experimentally it is found that the normal stresses relax more slowly than the shear stresses. 8B.9 Draining of a tank with an exit pipe We begin by writing the formula for the mass flow rate through the exit pipe (with radius R,) for a power-law fluid thus - Rip ap fe)". aRip (ae) “(n)+3\ dz 2m)“ (jn)+3\ Lm, Note that this is a quasi-steady-state approximation. We now equate this to the rate of mass depletion in the tank —artp it. #Rip ai Ba)" Oat “(n)+3\ Lm We now divide through by 7R2p(h+L)"" and integrate (for n #1) y Br _po__dh Ro PERo\" ptasnx hte “ane {ye Performing the integration and solving for the efflux time we get ' -(@ ee (+ p09 won"( SR} Hy To check this result we can let n>1 and m— y; we make use of Lhopital's rule to get the result of Problem 7B.9(a): aul) 4k? (djan){(H +L" pon) ft = (25) (aany—(n)} (2) et noe eet pgRy J Rj (Vn?) “(ie 7) £18 8B.10 The Giesekus model a, We have to start by expanding the z? in Eq. (E) of Table 8.5-1 as a function of (A7)?: 2 _1+4[16a(1- aay)? -4[16a(1- af (ay)* we Ba(I- a)(a7F =1-4a(1-a)(ayye+-- and then, the series expansion of the square root of this will be x= fl-4a(1- aay + =1-2a(1- al(ayy?+- Next we get f to the same order (fis defined in Eq. (D) of Table 8.5-1) 1-[1-2a(1- aay) ++] ie 1+(1-2a)[1-2a(1- a)y(ayyr+-] =a(ayy+- Then the viscosity expression (of Eq. (A) in Table 8.5-1) becomes 2 ne (1- aay? +--) 24 mm 1¥(I=2ayatayyr) 2A tae! Similarly, the first normal stress coefficient (Eq. (B) of Table 8.5-1) becomes ¥, _ (ean +-Sfi-(aarye+-)] 1 sa 2mA ——a1-(aAHP+ | GN? ano and the second normal stress coefficient (Eq. (C) of Table 8.5-1) is W, =-amA =-4a°¥, which shows (correctly) that the second normal stress coefficient is smaller than the first normal stress coefficient and has the opposite sign. &14 b. We begin by dividing numerator and denominator of the expression for ? by (Aj)? and then replacing 1/Ay by € (a small quantity): v= eve? +16a(1-a)-e? _ eyfl6a(1- @) /1+[1/16a(1- a)]e? ~ &? 8a(i—a) 8a(1—a) We now expand this last expression in powers of €: _e(t+[Y32a(1-a)Je*+-)-e? _ e(1-et-) 2Ya(1- a) " 2fa(i=a) Then we take the square root and get _ Ve(1-$e+--) **Blati-a)}* Then fis given by facta =g[1-(1-2a)z+--]=1-2x(1- a) Then the viscosity and normal-stress coefficients are 2 n__ (tf _(2x(t- a)? 1m 1+(-2a)f 2(-a)r __ (20-a))'e a))e T-a 1 “Ppa-ayp2fall-ay Va ay ¥ _fllcaf)( 1) __ (sar (1) ; (ay) oext- a) te) 2 eee mA Ar These expressions show (correctly) that the normal stress coefficient has a steeper slope at high shear rates than the viscosity. They also give a second normal stress coefficient that is smaller than the first normal stress coefficient, and that the two coefficients have opposite signs (this is in agreement with the experimental data for flexible polymers). c. For elongational flow, we first get the limit as 420 (please note that this ¢ is not related to the in part ()). In this limit we can expand the square root signs that appear in Eq. (H) of Table 8.5-1 thus: i 1f3, 14 2g- é4-.--1-(1- é-... an afe-4o 2(1-2a)Aé+----1-(1-2a)Aé )| 1 1 . = [3+ gelst-2e pee) |e 1 In the limit that Aé becomes infinite, the quadratic terms under the square root signs dominate and we get 1 2 =(3+2-1)= + a! ' ) 3a aE 31 Thus, the‘elongational viscosity remains finite, unlike the Oldroyd model for which the elongational viscosity becomes infinite. $2) 8C.1 The cone-and-plate viscometer a, According to Eq. 2B.11-1, the velocity distribution in the cone-and-plate system is given approximately by The shear rate is given by the relation two lines above Eq. 8.3-2. In this problem the only nonzero components of the rate of strain tensor are Yo and 49. This can be seen in one of two ways: (a) Look at the right side of Eqs. B.1-15 to 21, but without the factor — 4 and with the div v terms set equal to zero; these are the spherical components of the rate of strain tensor given in Eq. 8.3-1 or Eq. 8.4-1; or (b) add to the components of grad v in Eqs. (S) to (AA) in Table A.7-3 the corresponding transposes in order to construct the rate-of-strain tensor according to Eq. 8.3-1. Therefore, from Eq. B.1-19 iy _8in@ A(_% )_ 19% Yoo = Yoo = 96| sind)” 7 30 since the angle between the cone and plate is extremely small, which means that sin@~sin}=1. When the velocity distribution of Eq. 2B.11-1 is inserted in to the above approximate expression for the nonzero components of the rate-of-strain tensor we get , 9 ~2{%)_ 2 Yoo = Yoo = 36| 5 7 Hence the shear rate is We now have to choose the proper sign. Both Q and yo are positive quantities. Therefore the plus sign must be selected. b. The non-Newtonian viscosity is obtained from the ratio 1= (Top /~Yop)- Hence we have to find a way to get the shear stress G20 from the measured quantities. As pointed out in (a), since the shear rate is constant throughout the gap, all stress components are also constant. This means that the torque can be calculated from the shear stress by integrating shear-stress times lever arm over the surface of the plate from zero to R: 3T, an PR : T.= Spree Toe = DERF lox tard = $mR°t>, whence ‘Then the non-Newtonian viscosity is given by top _ (3T,/2R*) 37, yy (Q/vo) — 2aR?a 9 c. Equation 8C.1-2 follows directly from Eq. B.5-7, after dropping the terms on the left side and the 7, and T,, terms on the tight side; the terms 1, and t,, must, however, be retained, since we know that normal stresses are nonzero in shear flow. Then Eq. 8C.1-3 follows from 7,,=p+t,, and some minor rearranging, and Eq. 8C.1-4 follows from the definitions of the normal stress coefficients and from the results of (a). Next we integrate Eq. 8C.1-4 from the outer rim of the cone- plate system to some arbitrary position r: S400 =[(¥i + 2%) 7? |ffalnr The normal-stress coefficients are constants, because of the constancy of the shear rate over the gap. Therefore we get Holt) = Hoo(R)-[(¥s #22)? Jin = H,,(R)+ [oo R)~ (R= [(¥ +22) Jin = PVF (Hs +22)? Jing $23 Here we have used the boundary condition that the normal stress at the rim is equal to the atmospheric pressure, and we have also used the definition of the second normal stress difference. d. The force exerted by the fluid in the z-direction on the cone is then obtained from the result in (c) as follows: F,= 0" |p Roo(?rdrde — aR?p, R : ant =2af) [>. -¥,7° -[(¥1+ 2¥,)7?|inZ far - mR’, =- AR, 7? ~ 2nR?[(Y, +2, )¥? [p(n E)EdE =o AR, 7? ~20R[(¥, +2)? 13S? Ing 467), = AR Wf? +p ARW, 7? + AR 9? =} ARV, /? Here we made use of the fact that lim In =0. Solving for the first normal stress difference, we get finally PE: Y= Rage e. If one measures m(r)-p, as a function of r/R using flush-mounted pressure transducers, then, knowing W’, from (d), the second normal-stress coefficient can be calculated from Eq. 8C.1-5. Gara 8C.2 Squeezing flow between parallel disks This problem is solved by a quasi-steady-state method. Conservation of mass states that, for an incompressible fluid, the mass rate at which the fluid crosses the cylindrical surface at r should equal the rate at which the mass between the plates within the cylindrical surface at r decreases. The rate of mass displacement caused by the disk motion is: w(r) = ar? p(-H) where (H = dh/dt) To get the mass rate of flow between the two disks, we adapt the result in Eq. 8.3-14 by making the following correspondences locally: W > 2ar, 2B» H, (Py-P,)/L——dp/dr, and w-—>w(r). Then we get for the mass rate of flow emerging from between the disks: wt) = area) dp amy" dr m Equating the two expressions for w(r) we get a differential equation for for the pressure distribution p(r) between the disks _ dp _ [oF] [(yn) +2]" (2) di [ey H This may be integrated to give 2m(-H)"[(n) +2)" fp ay EL a or P~ Patm ent ntl R mcr eel al -(2" | which is the power-law equivalent of the Newtonian result in Eq. 3C.1-13. G05 When a constant force F, is applied to the upper disk, this force must be resisted by the pressure in the fluid at the upper disk, integrated over the disk (we include here also the normal stress T,,, even though we know that it will not contribute for a generalized Newtonian fluid--the proof for this is similar to that given in Example 3.1-1 for Newtonian fluids): = "IS (p- Po + te pardo 2m(-H)"[(1/n)+2]" Re =2n- apn =a Re fla e*)ene _2am(-HY'[(yn) +2)" Rm OPT ng This is now a differential equation for the motion of the upper plate as a function of time: ~qiie (82s a) aye HO? \ 2am") | (An) Y This can be integrated to give: die (a(S) yn ft Sr in ( sot) Capra)" het or Lose t22h)" (Ue ar HOT GT | Dame) | (/n)+2 rs This simplifies properly to Eq. 3C.1-16 for the Newtonian fluid. 26 8C.3 Verification of Giesekus viscosity function a. Eq. 8.5-4 is the same as Eq. 8.5-3 with 2,="4,=4 and Ay = Hz =H =0, but contains in addition a term —(A/ny)a(t-t}. Therefore, for steady shear flow, we may take over Eqs. 8.5-5 to 8 by making the appropriate changes in the constants, and by adding the extra term. To do the latter we need to calculate the components of the product {tt}; this is most easily done by matrix multiplication: Tx Te O\ Tr T, 0 htt, Ta(Tx tty) 0 Tye Ty Of te ty O1=]ta(t+ ty) Ta ttm 0 0 0 zo 0 t 0 0 a When we use this result, Eqs. 8.5-5 to 8 become modified as follows for the Giesekus model: Ty ~ 2A — (2/ Mg a t2, + 73, = 0 Ty —(A/ np )arr?, + 13, =0 T., —(A/mp)artz. = 0 Tgp AT yy —(A/ Mo )OE (Tax + Tey =—Toy When these equations are multiplied by A/ny and dimensionless variables are introduced, we obtain Eqs. 8C.3-1 plus an equation that gives T,,=0 (this equation gives another solutions which is physically unacceptable). b. When these dimensionless equations are written in terms of the dimensionless normal stresses we get N,[1-@(N, +2N,)]=2T yl” N, = a(T}, +2) T,,[1-a(N, +2N,)]=-(1-N, JP c. The second equation in (b) can be solved at once for the dimensionless shear stress: $27 T= N21-aN,) Division of the first of the equations in (b) by the third gives a relation for the dimensionless first normal stress difference in terms of the dimensionless second normal stress difference and the dimensionless shear stress; into that we can substitute the equation just obtained above and get ~_2Na(1- aN) ) 1 a(i=N;) Next, we square the third equation in (b), and then insert the expression for T2, from (*) and the expression for N, from (*), to get N,(1- aN, )[1+(1-20)N,]° a(1-N,)* oy d. To solve the final equation in (c) Giesekus (see p 87 of Ref. 5 on p. 262) suggested making the following change of variable: ee N, =——*—_ 2" 1+(1-2a)x Then the various factors in the final equation in (c) are: _(+x(l-a) TaN, = 1+(1-2a)z’ 1+ (1-2a)N, = 8-28 When the last four equations are substituted into (***) we get ee eke 4a(1-a)xz* This is a quadratic equation for x” which may be readily solved: = =1-4a(1-a@)f? + 8a(i— ai? Then this result along with Eq. 8C.3-7 gives the second normal stress difference as a function of the shear rate. Having this, the shear stress and first normal stress difference can be obtained from Eqs. 8C.3-4 and 5. As the shear rate goes to zero, one gets 1 Mp, ¥; > 2MA, and ¥, > ~amA. The functions given in Table 8.5-1 are given in Giesekus's paper as well as in R. B. Bird, R. C. Armstrong, and O.Hassager, Dynamics of Polymeric Liquids, Vol. 1 (1987). pp. 361- 368. $24 8C.4 Tube flow for the Oldroyd 6-constant model The non-Newtonian viscosity for the Oldroyd model is given in Eq. 8.5-9. Therefore for tube flow , 1+ 1 n= nui 22) 1+ 0,7 From Eq, 2.3-13, we have the expression valid for any kind of liquid a (Po-Pur = aan Combination of these two results gives AngiL (1+ 0,7? =P, \1+0,7 The mass rate of flow through the tube is given by w= ph fread =2np|tv.nir=2np] tug, - fbr? Sear] The first term in the last result is zero at both limits. We next integrate by parts again do,|* Ry 3 a ( dv, ar |y in which 7, is - do, /dr evaluated at the tube wall. This result is good for any non-Newtonian fluid. We now specialize to the Oldroyd model: 2noL in| ( 1+ on7' . niente site) 2h)] 9 5 2Mol_\ ori Lary’ 1 Rp 4 fo bu YdY > ae traf i) (Ay 207 fe] }apR?¥_-4mpf)" [r(V)} ay & 30 in which Y = 0,7? and n=0,/0,. The expression for r given earlier may be evaluated at the tube wall, and this gives pa 2tofel:( 1+ 072) _ Py-P, 1+ 0,72 in which X = 0,73. Next, multiply the equation for w. by 3x0, /mpR? and eliminate P, —P, by using the expression for R just above to get _3Vow_ 1 (1+x)? 7 oe mpR® = Vx aX) f%) ) in which x(1+nY)* f(X)=f} at) vay =}°X? -3n?(n—1)X + 3n(n—-1)(2n-1)In(1 + X) -1 2 -p( 24) [6n+(7n-1)x] (***) This gives, in dimensionless form, the mass rate of flow in terms of the wall shear rate. The latter may be eliminated in favor of the pressure drop by using the expression for R above. That is, to get Q in terms of the pressure drop, the asterisked equation have to be combined. The curves thus obtained may be found in the original publications (refs. 6 and 7). 8-31 8C.5 Chain models with rigid connectors a. The paper by M. Gottlieb is the first example of a molecu- lar dynamics calculation for polymer chain with rigid connectors. He followed the motions of a three-bead-two-rod model as it moves around in a liquid made up of 47 "solvent beads." The solution is presumed to be at macroscopic equilibrium. He showed that one does not get a Gaussian distribution of orientations for the polymer model, in agreement with the previously published theory of H. A. Kramers, Physica, 11, 1-19 (1944). See also, R. B. Bird, C. F. Curtiss, R. C. Armstrong, and O. Hassager, Dynamics of Polymeric Liquids (Vol. 2), Wiley, New York, 2nd edition (1987), pp. 40-41. This book will hereinafter be referred to as "DPL." b. The publication by O. Hassager showed how to get the viscosity and normal stress coefficien‘s for a three-bead-two-rod chain in a flow situation. In a subsequent publication [C. F. Curtiss and R. B. Bird, J. Non-Newtonian Fluid Mechanics, 2, 392-396 (1977)] it was shown how to extend the calculations to models with 4 and 5 beads. Hassager showed that the viscosity and first normal stress coefficient for models with infinitely stiffened Fraenkel springs are different from those of the rigid rod models. This perplexing situation was what led Gottlieb to investigate the same problem by using molecular dynamics. See also DPL (§16.5). c. The paper by X. J. Fan and T. W. Liu dealt with Kirkwood- Riseman chains of 3 to 8 beads. This model has fixed bond lengths and fixed bond angles, that is, two kinds of constraints. They were able to evaluate the equilibrium configurational distribution, and they found that it differs from the classical "Gaussian distribution.” This calculation enabled the authors to relate rheological properties to “chain stiffness," by comparing their results with those for a Kramers freely jointed chain. See also DPL (§16.6). d, The paper by T. W. Liu is a landmark contribution to the theory of Kramers chains. He showed how one can study the configurations and rheological properties of freely-jointed Kramers chains by the use of Brownian dynamics. By this technique he was able to generate movies showing the change of the configurations with respect to time. He also succeeded in taking appropriate averages over the configurations, in order to get the viscosity and first normal-stress coefficient as a function of shear rate, for chains with 3, 5, 10, and 20 beads. He also calculated the elongational viscosity in steady elongational flow and got information on how the $32 polymer molecules “unravel” under the influence of the elongational flow. e. The paper by H. H. Saab, et al., is an extensive comparison between the Curtiss-Bird phase-space theory for polymer melts and the available experimental data on a host of rheological properties. An extensive comparison of the results with those of the theory of Doi and Edwards confirms that in almost every instance, the Curtiss-Bird theory is to be preferred over the Doi-Edwards theory. In the Curtiss-Bird theory, the polymer molecules are modeled as Kramers freely jointed bead-rod chains, whereas in the Doi- Edwards theory, mixed modeling is used. See also DPL Chapter 19. f. The papers by J. D. Schieber show how the extension of the Curtiss-Bird theory to polydisperse polymer melts can be implement- ed. Both the log-normal (Wesslau) distribution of molecular weights and the Flory-Schultz distribution are used. The curves for the viscosity, first normal-stress coefficient, elongational stress growth viscosity, and steady-state elongational viscosity were obtained. These curves often differed appreciably from those for monodisperse samples. Comparisons with experimental data are given in the second of the two papers. See DPL, Example 19.6-1. i. The rodlike connectors are generally much more difficult to deal with than the springlike connectors, because it becomes necessary to use nonorthogonal coordinates to describe the chain space. For Hookean springs, one can perform many of the kinetic theory derivations with relative ease. For non-Hookean springs, it is possible to make some assumptions that enable analytical results to be obtained. But even for rigid dumbbells and three-bead-two-rod models, analytical calculations become prohibitively time consuming. ii, To overcome the problems associated with the use of rigid connectors in modeling, molecular dynamics and Brownian dynamics have proven to be useful. Also, Brownian dynamics can be useful when considering the flow in constrained channels, where the interaction with the containing walls have to be considered. Brownian dynamics proves to be particularly helpful in getting information about the actual motions of the polymer molecules during various types of flow. 8-33 9A.1 Prediction of thermal conductivities of gases at low density. a. Since Argon is monatomic, we use Eq. 9.3-13 to predict its k in the low- density gas region: = 1.9891 x 107% Here T = 100 + 273.15 = 373.15K, and Table E.1 gives M = 39.948, « = 3.432A, /K = 122.4K for Argon. Then KT/e = 373.15/122.4 Table E.2 gives 2, = 1.0344. Equation 9.3-13 then give 049, and interpolation in k = 1.9891 x 107* 499 x 1077 cal/s-em-K 3.432? x 1.0344 which is within 1.5 percent of the observed value. b. Equation 9.3-15, Eucken’s formula, gives k (6 + 1.25R/M) w= (Gp +1.25R)u/M with R = 1.987 cal/g-mole-K. Insertion of the data for G, and jz, and for M from Table E.1, gives 1929 x 10-7 forNO, k= (7.15 + 1.25 1.987) oy = 0.02595 W/m-K vs. 0.02590 W/m-K from Table 9.1-2. = 620 x 1077 cal/s-em-K 1116 x 10-7 16.04 for CHy, k= (8.55 + 1.25 x 1.987) = 0.03212 W/mK vs, 0.03427 W/m-K from Table 9.1-2. 768 x 1077 cal/s-cm-K = ay 94.2 Computation of the Prandtl number for gases at low density. Use of Eq. 9.3-16, with molar heat capacities G, = MC, calculated from the given values C, and molecular weights M from Table E.2, along with R = 8.31451 x 10° J/kg-mol-K from Appendix F, gives the predictions in column (a) of the following table. Computation of Pr from its definition, Eq. 9.1-9, and the tabulated Cy, yz, and k, gives the values in column (4). The predictions are closely confirmed for He and Az, but are less successful for the polyatomic compounds. (0) (8) _ Gas Pr from Eq. 9.3-16 Pr from G,, 1, and k He 0.667 0.670 Ar 0.667 0.665 Hy 0.735 0.714 Air 0.736 0.710 COz 0.782 0.769 HO (M = 18.016) 0.764 0.862 9A.3 Estimation of the thermal conductivity of a dense gas. a. Table E.1 gives the following critical constants for methane (CH): Te = 191.1 K, pe = 45.8 atm, and ke = 158 x 10°® cal/em-s-K. The reduced conditions for the prediction are then T, = (459.7 + 127)/(1.8 x 191.1) = 1.71 and p, 110.4/45.8 = 2.41. From Fig. 9.2-1 we find k, = 0.77 at that state, giving = keke = 0.77 x 158 x 107° .22 x 107* cal/em-s-K = 0.0294 Biu/hrft-F. which is about 4% above the observed value, 6. For this calculation, we need to predict the viscosity of methane at 127 F (325.9 K) and low pressure from Eq. 14-18. We find ¢ = 3.780 and KT/e = 325.9/154 = 2.116 and ©, = 1.153 by use of Tables E.1 and E.2. Hence, 71 x 1077 g/em-s Next we use the Eucken formula, Eq. 9.3-15, to estimate the thermal condue- tivity k° at low pressure and 127 F, where C, = 37.119 J/g-mol-K according to the heat capacity polynomial given for methane in Reid, Prausnitz and Poling (1987): 171 x 1077 16.04 (37.119 + 1.25 « 8.31451) 0.0347 W/m-K .0200 Btu/hr-ft-F = 0.000347 W/em-K Finally, we multiply &° by the ratio of ky at 110.4 atm to the asymptote ky = 0.52 at p, = 0 in Fig, 9.2-1. The resulting predicted k at (110.4 atm, 127 F) is k = 0.0200 x 0.77/0.52 = 0.0297 Btu/hr-ft-F and is just 1% above the measured value. This is unusually good agreement. 9A.4 Prediction of the thermal conductivity of a gas mixture. ‘The data for this problem are as follows: Component M Hx10°, Pas ky W/mK Mole fraction 1H) 2.016 0.8944 0.1789 0.80 2(CO2) 44.01 1.506 0.01661 0.20 Insertion of these data into Eq. 14-16 gives the dimensionless coefficients 1 = by =10 bam Je(1e Bm)" [p+( +(e “aay V8 44, it) 1.506 2.016, = 2.457 _ (1+ 44. a1) a [+ ( (2 ) 1a (2818) “y va \'* 3016 a8) 8044 fa) 01 = 0.1819 Substitution of these results into Eq. 9.3-17 gives 0.8 x 0.1789 0.2 x 0.01661 Keni 08 x10402% 2457 08x 0.1819 +0.2x1.0 1204 W/mK q-4 9A.5 Estimation of the thermal conductivity of a pure liquid. We first calculate the derivative (Op/Op)r required for Eq. 9.3-4: — 1 a (8p/Op)r = e*/ [o-*(p/Ap)r} = (1/0.9938)[38 x 107 = 2.648 x 10"? cm?/s? 1 = 2.648 x 10 megabar em’ /g Inserting this result into Eq. 9.3-4 and setting C, © Cy, we obtain vy = V'2.648 x 1010 = 627 x 10° cm/s Equation 9.4-3 then gives the following estimate of the thermal conductivity: k=2.80(Np/MP kv, 6.0221 28 x 1.38066 x 107" « 1.627 x 10° = 2.80 ie 50 x 10* g cm/s*-K 650 W/m-K = 0.375 Btu/hr-ft-F q-5 9A.6 Calculation of the Lorenz number. a, When x and e in Eq. 9A.6-1 are expressed in terms of the gas constant R and Faraday constant F, the Lorenz number takes the form =(z) Insertion of numerical values for R and F from Appendix E gives pa @ (831451)" ~ F (964853 = 2.44 x 107% volt? /K? b. Insertion of the result just found, and the given ke and T, into Bq. 9.5-1 gives the thermal conductivity estimate k= Lk 2.44 x 10-*volt? /K? x 293.15K = 172x10-Sohmen = 4.16 volt? /K-ohm-em = 416 W/mK for copper at 20°C. qb 9A.7. Corroboration of the Wiedemann-Franz-Lorenz Law. Conversion of the tabulated data into SI units and insertion into Eq. 9.5-1 gives the following results at T=203.15K: Metal Ake, ohm-m —k, W/m-K Lorena number, L = k/keT, volt? /K? Na 4.6x108 133 2.11076 Ni 6.9x10* 59 1.4x10-% Cu 1.69% 10* 385 2.21075 Al 2.62108 209 1.9x10-* ‘The approximate agreement of L for these metals illustrates the Wiedemann-Franz- Lorenz law. 9A.8 Thermal conductivity and Prandtl number of a polyatomic gas. a. To calculate k for a polyatomic gas at moderate pressure we use Eq. 9.3-15, k= (6) + 1.25R/M)u (Gp + 1.25R)u/M along with the viscosity expression in Eq. 14-18: fe = 2.6693 x 107° From Table B.1 we find, for CH,, the values M = 16.04, ¢ = 3.780, «/x = 154K, and from Table E.2 at KT/ = 1500/154 = 9.740 we find 2, = 0.8280. Equation 14-18 then gives the predicted viscosity = 2.6693 x 107% (3.780)? x 0.8280 = 3500 x 1077 g/cm-s and Eq, 9.315 gives the predicted k value k= (20.71 + 1.25 x 1.987) x 3500 x 1077 /16.04 = 5.06 x 107 cal/em-s-K = 2.12 W/mK b, The predicted Prandtl number according to Eq. 9.3-16 is oo 20.71 = 071 + 1.25 x 1.987 = 0.89, dimensionless 4-3 9A.9 Thermal conductivity of gaseous chlorine. For Cl;, Table E.1 gives M = 70.91, ¢ = 4.115A and ¢/x = 357K. Equation 14-14 then gives p= 2.6693 x 107 a0, = 1.34274 x 10° VT with p[=]g/em-s and T[=]K, and Eq, 9.3-15 gives k = (C, +1.25R)u/M. The G, and k in the units of the problem statement: calculated results follow, with 1, TK 198 275 276 276 363 363 395 453 453 495 553 583 583 676 676 T/357 0.5546 0.7703 0.7731 0.7731 1.017 1.017 1.106 1.269 1.269 1.3866 1.549 1.633 1.633 1.884 1.884 Qe 2.0915 1.8217 1.8239 1.8239 1.5799 1.5799 1.5138 1.4172 1.4172 1.3609 1.2974 1.2694 1.2694 1.1994 1.1994 10 0.8931 1.2044 1.2091 1.2091 1.6008 1.6008 1.7427 1.9935, 1.9935, 2.1701 2.406 2.5249 2.5249 2.8775 2.8775 (Gp +1.25R) 10 kprea 10.54 10.59 10.59 10.59 10.81 10.81 10.91 11.02 11.02 11.095 11.17 12 nz 11.32 11.32 1.33 1.80 1.81 1.81 2.44 244 2.68 3.10 3.10 3.40 3.79 3.99 3.99 4.59 4.59 Ave.= Kobs/ Kprea 0.985 1.056 1.066 1.061 1.074 1.070 1.13 114 1.10 1.09 1.09 il 1.12 1.10 1.07 1.084 The predicted & values exceed the observed values by an average of 8.4% in this temperature range. 9A.10 Thermal conductivity of chlorine-air mixtures. Numbering chlorine and air as components 1 and 2, respectively, and inserting their given properties into Bq. 1.4-16, we obtain the following coefficients for Eq. 93-17: i= F2=1; a 1381 (/28.97\1/"]* On = R (@ + a) [ +\ ieer (32) = 0.53899; 2 1 28.97\~1? 4 (70.91) /* maz (: + fat) lt 9) = 1.8105 1 \ 28.97 Equation 9.3-17 gives, for binary mixtures: aiky tak, ee rPit22H12 t2b21 + 22822 Insertion of the coefficients and compositions for this problem gives At 2) = 0.25, 0.25 x 0.0896 0.75 x 0.02614 nix = 9.35 4 0778 x 0.59800 * 0.25 x 1.8105 + 0.75 ~ ~0008 cal/omse At ay = 0.5, Kini = = = 0.0675 cal/em-s-K At 21 =0.75, 75 x 0.0896 0.25 x 0.02614 Kix = O75 + 0.25 x 0.53800 7.75 x 1.8105 + 0.55 ~ 00800 cal/emsK 9A.11 Thermal conductivity of quartz sand. a. For spheres (9g; = gz = g3 = 1/3), Eq. 9A.11-2 reduces to aj = 3/(2 + kj/ko) ‘The resulting a; values from Eq, 9A.11-2 for the water-saturated sand are _ 3 © 2+ (ko/ko) and Eq. 9A-11-1 then gives 3 3 => = 0.183; = 2+ (kh /ho) 5 02 = FF (ka/ko) a ay = 0.433 __ (1)(0.427)(0.00142) + (0.183)(0.510)(0.0204) + (0.433)(0.063)(0.0070) (1)(0-427) + (0-183)(0.510) + (0.433)(0.063) which predicts ke = 6.3 x 10~? cal/em-sK, vs. 6.2 x 10-* observed. For the same sand when completely dry (ky /ky = 332, kz/ko = 114), Eq. 9A-11-2 for spheres gives _3 “247 3 3 = = 0.00808; a2 = Ty aap ~ O08; r= aaa ao = 0.0.0259 m4 and Eq. 9A.11-1 with de Vries’ correction factor of 1.25 for dry sand gives ket __ (1)(0.427)(0.0000615) + 0.00898)(0.510)(0.0204) + (0.0259)(0.063)(0.0070) 125 ~ (1)(0-427) + (0.00898)(0.510) + (0.0259)(0.063) ~ predicting ke = 0.38 x 10-* cal/em-sK, vs. 0.58 x 10~* observed. For the same sand when water-saturated at 20°C, de Vries’ recommended 9; values give pe ie ee eee 1000) = 3 [1+ 0.195[(1.42/1.42)—1] * 1+0.750(0.49/1.42) — 1] ~ 1 - 1 i ee as meee q+ ERT oy z= 2 Z patf ed) isn ma3 li + 0.125(7.0/1.4)— 1] * T+ 0.750(7.0/1.4) — il 0.58: and Eq. 9A.11-1 gives __ (2)(0.427)(0.00142) + (0.280)(0.510)(0.0204) + (0.532)(0.063)(0.0070) a (10-427) + (0.280)(0.510) + (0.532)(0.063) predicting ke = 6.2 x 10~ cal/em-s-K, in still better agreement with the observed value 6.2 x 107%. 9-i For the same sand when completely dry (t1/ko = 332, ko/ko = 114), Eq. 9A-11-2 with de Vries’ recommended gj values gives 1 2 1 mos lease Ij" 1+0.750{1 — 1] 1.000 1 2 1 eo —___]} 00171 : jlipmmae cyt eo 00 1 2 1 ay = =| ___g _____d _ oggg * 3 lo T+ oTo0R4— 1] = 0004S and Eq. 9A.11-1 with de Vries’ correction factor for dry sand gives keg _ (1)(0.477)(0.00006) + (0.0171)(0.510)(0.0204) + (0.0048)(0.063)(0.0070) 1.25 — (1)(0.47) + (0.0171)(0.510) + (0.0048)(0.063) predicting ke = 0.54 x 10-3 cal/em-s-K, vs. 0.58 x 10~? observed. (b) Equation 9.6-1 gives ho G25). Insertion of ¢ = 0.573 and k; = 0.0189 cal/em-s-K for the solids gives, for the water-saturated sand, predicting keg = 5.1 x 107% cal/cm-s-K, vs. 6.2 x 10~? observed. This is not as good as the prediction in (a) from Eq. 9A.6-11 with de Vries’ g; values. For the completely dry sand, insertion of the k value for air as kp into Eq. 9.6-1 gives ken _ 3(0.573) a ko a ‘0.0189 + 2(0.0000615) 0.873 ated 0.0189 — 0.0000615 . predicting ke = 0.30 x 10° cal/em-s-K, vs. 0.58 x 10-* observed. The result in (a), from Eq. 9A.6-11 with de Vries’ gj, is better. Predictions of ke are more difficult for dry sand than for water-saturated sand. An oblate-spheroidal model gives little advantage according to the present data. 9-12 9A.12 Calculat n of molecular diameters from transport properties. a, Equation 1.4-9 and the viscosity value from Problem 9A.2 yield the following molecular diameter calculation for Argon in cgs units: viTm (22)" Nr 39.948 x 1.38066 x 101° x 300\\"/* = V2/3/(2.2' 10-4) (| 0 gives Mo _ qo” Then inserting Fourier's law with a temperature-dependent thermal conductivity, we get d[_,lar d 140 Af _,laT)_ AT _(K,-(k, -k,)o) 122 |= al A ° er zal (ko ~ (ho )9) 3] : Integrating once we get aor 40 (Fo ~ (ko - kx )®) 55 = Ss and a second integration gives k,O-4(ko -k,)O? =C,0+C, When the constants are determined from the boundary conditions, @(0)=0 and ©(m)=1, we get C, =0 and C, =(ky +k,)/2m, so that the temperature distribution in the solid is lo-26 k,@-4 ‘ Then the total heat flow through the surface at @ = 0 is then Lere 1aT orth “i ardz 140 -T, Die (Fa), drdz Tq (lor) drdz os, 7 atk says \0-27 10B.13 Flow reactor with exponentially temperature-dependent source For this problem we may take T, to be zero, since there is no particular need to do otherwise. Then the temperature is made dimensionless by dividing by the inlet temperature: @=T/T,. Also, the quantity S,, may be identified with K of Eq. 10B.13-1. Therefore the quantity 5,,F(@) in Eq. 10.5-7 becomes SaF(@)= Kew(-5) 7 Kex{-4) one Ae Fa We may then proceed to Eqs. 10.b-21 to 23: Zonet: @'=T'/T;=1 or T'=T, Zonet [or pis dO= {" exp( +4 )f0 =z ie a To do the integral we make a change of variables A/@ =x so that dO =—Axdx. Then the integral becomes Al te tax= +A] - A etx tde=NZ h This last integral can be written as a power series. We then get: ale | =NZ or h Aexp(4/0") - Ae- alins+ $2 \0-28 for ©" as a function of Z. However it is easier to calculate Z as a function of ©", since that does not in- volve any trial and error. Zonell: ©" =0"|, Iz=1 Another way to treat Zone Il: If the temperature rise is not too great, we may expand e*x? in a power series about x = 1: etx = e[1-(x-1) +3 (x-1)? -H(x-1)° + B(e-1)" +--+] Then alot -Af ex tax =— Ae [1 (1-x)+3(1-x)*---Jax onl "I a-y+ay? fy af IAG de for ©" as a function of Z. Also, by this method it is easier to calculate Z asa function of ©", since that does not involve any trial and error. 10-29 10B.14 Evaporation loss from an oxygen tank a. The fact that the thermal conductivity varies linearly with temperature can be written as krky _T-Ty | ee Te Tae 8 which also serves to define the dimensionless temperature. From an energy balance on a solid spherical shell of thickness Ar gives (42°°4,)|, -(42r°4,) near Division by Ar and then taking the limit as Ar goes to zero gives a (2 al" q,)=0 Then, introduction of Fourier's law gives (ref) (488). a ad or ae aa) a We now integrate once with respect to r to get 7428. do _C, ar =C, or eo dry? Inserting the expression for the thermal conductivity as a function of temperature, and integrating again, we find G {ba +(k,-h)60=-2+c, or b+ (hy ~hy)O? =, {0-30 Now we apply the boundary conditions: B.C. at ro: B.C.at ry: Subtracting the first equation from the second eliminates C, (which we are not going to need anyway) and gives an equation for C,: Hh +h)=¢(2-2) og or Mt herr Oe Gan) The heat flow inward through the spherical shell at ry is now aT de Qe= aio], 2 snri( +4o(, - 1,2) rato The derivative of the dimensionless temperature can be found thus: do ky, + ky ror by B+ (k, -ho® = Hk +ko)rors 1 (4-7) 7? Evaluating this expression at r = rp gives 2 kth) “athe (i=Fo)¥o Putting this expression into the formula for Qy gives (0-3) aT ko tk (Ti -T, = 4ar3| +k = AoFA |) 7-0 nee] semen EEE) b Since most of the quantities are given in the c.g.s. system, we will convert all quantities into that system. The quantity Q) may be evaluated thus: 16.2 x 10° )(4.136 x 107° O,=antoocnsatancasal CEP eEA D1) = 2.8182 cal/s The factor 4.136 x 10" needed for converting the units of the thermal conductivity is obtained from Table F.3-5. The rate of evaporation of oxygen is then: -(232 nese (2600) = 6.201 g-ml/hr or 6.201)(32) 1000 = 0.198 kg/hr 10-32 10B.15 Radial temperature gradients in an annular chemical reactor a. Consider a cylindrical shell of thickness Ar and length L. We make an energy balance over this shell, by paralleling the derivation in Eqs. 10.2-2 to 6, replacing the electrical heat source by the chemical heat source S,. Hence we have (cf. Eq. 10.2-6): d L(mn,)=82 Into this we substitute Fourier's law for heat conduction in the r- direction to get aera) 1a(,at)_ 5 {ka G)) 8 i bapa a) provided that the effective thermal conductivity does not vary with position. The boundary conditions are: B.C.1: at r=, T=Ty aT Ber: tre, w= atr=n, © b. A natural choice for the dimensionless radial coordinate involves division of r by either the inner or outer radius; we choose the inner radius and write €=r/r,. Then the differential equation becomes: Keg 1d dT) _ 75 €db\° dé) kee 1 { aa! or “54 EA |=-8, rp € ag ag From this, it is evident that ky;(T~-Tp)/S.73 is dimensionless. By inserting a factor of 4, we get @ = 4k, (T -Tp)/S,r3. The insertion of the factor of 4 is arbitrary, but it makes the final dimensionless answer somewhat simpler. In terms of these dimensionless quantities the partial differential equation becomes: [0-33 1d(,do)_ zala) with boundary conditions B.C.1: at€=1, @=0 B.C.2 at€=1, do/dé=0 c. Integration twice leads to @=-£+C,InE+C, Application of the boundary conditions gives C, =2 and C =1, so that @=1-&? +2InE d. The dimensionless temperature at the outer wall is then ©(a)=1-a? +2Ina (a=r,/r9) The volume averaged reduced temperature is: 2m (a 1e2_ 1 pd, cep e_igeyt e=b fG-g +2ing)eagae [25 +8ing- 36] jp frgaeao el ana 4(a? +1)420 e. The temperature at the outer walll is 2 T=T)+ Su (1-4? +2Ina) el 2 (4800 st (397x107 oe )(2.s4x12 = (28) = 900+ rom cal a2 4(0.3) [0-34 x(1- (1.11? +2In(1.11)) = 900 + (0.681)(1- 1.23 +0.21) = 900 + (63.3)(1-1.23 + 0.21) = 89°F fIf the inner and outer radii were doubled, the temperature difference between the walls would be four times as great. 10-35 10B.16 Temperature distribution in a hot-wire anemometer a. We start by writing a shell balance over a segment Az of the wire: al (2D*) ale ae(420")+ ab?) Az h(T-T,)aDAz = 0 Division by }D?Az and taking the limit as Az > 0 gives Gelense Hel, 1? _ AN(T-T1) ano Az k D 2 . : ee po taettTieedn) ela az D k Let us now define the dimensionless quantities z felicity ya the iil L T, KD KAT, Now when the differential equation is multiplied by L’/kT,, it becomes, in dimensionless form, #9 dé z This is a nonhomogeneous, second-order differential equation, the solution of which is the sum of a complementary function and a particular integral: -H@=-J with @=Oat C=41 cp, =C, cosh VHE + C, sinh VHG; 0p, =J/H Since the solution must be symmetric about ¢=0, the constant C, must be zero. Therefore, the complete solution has the form 10-36 = L ___J/H @=C,coshVHE+ A where GQ= chit where the boundary condition that @=0 at ¢=1 has been used. Therefore the solution to the problem is o=L{1_soshvHe or T-T,= LP {1 soshyli/kb= H\ coshvH b 4hk,\~ cosh4/4h/kDL c. First we evaluate the cosh-function in the denominator, creating dimensionless ratios with internally consistent units: z oe L=cosh ae Ge LOO a 5/2. mi He F ee (40.2) 0.0127 S89 = 6 Then, at z = 0, the quantity in the large parentheses is dl 1-—————— | = 0.842 ( cosh/4h/kD i] Then the volume rate of heat production is 2 BP _(T=Ti)(4h) __(80)(4)(100x 5.678) _ 6 971.498 J/m?s Kk, (0.842)D ~ (0.842)(0.127 x 0.001) = 6.371x 10? J/cm*s= 6.371x10" amp? -ohm/cm* since 1 amp-volt=1 amp” ohm =1J/s. Then current (amp) = = [6.371x10? x 10° (}2(0.0127)") = ¥63.71x 10° (1.267 x10“) = 1.01 amp 10-37 10B.17 Non-Newtonian flow with forced-convection heat transfer a. This problem can be solved by modifying §10.8 as follows: First replace (1- £?) by (1- £**) in Eqs. 10.8-19, 25, and 26. Then the power-law analog of Eq. 10.8-27 becomes et gs a6, o=c6re/ -F5) cing +c, in which the constants of integration are determined to be c,=0 Cy = As +3)(2+1) C, =[(s +3°)-8]/4(s + 1)(s+3)(s +5) With these values for the constants, we are led to Eq. 1B.17-1. b. This problem may be solved by modifying Problem 10B.7, part (b), by replacing (1- 0”) by (1- 0**') in the differential equation for ¥ as well as in the integral condition. This leads to the following equation for @(a,¢): 3 ot? “2 (s+2)(s+3). (a,c) =e +c Jecerc, with the constants of integration given as C,=0 _8+2 tw stT 27 ele ee ) 2° st 6(s+3)(s+4)(2s+5) Then we are led to Eq. 10B.17-2. 10-38 10B.18 Reactor temperature profiles with axial heat flux a. The differential equations in Eqs. 10.5-6 to 8, and the boundary conditions in Eqs. 10.5-9 to 14 are still valid, but with a linear form for the function F: T-T, 5. =5, i . (F) in which S,, and T, are constants describing the linear dependence of the reaction rate on the temperature. Then, if we use the dimensionless downstream coordinate Z=2/L, a dimensionless quantity B= pC, 0pL/k_i.., and a dimensionless chemical heat source N=S,L/pC,vq(T,-Tp), the differential equations for the three regions become: a® ido! _do! 1" _ dO" gu, 1 de" _do™ BZ az = Bidz ee " Badz dz The boundary conditions are: B.C.1: at Z=-00, @!=1 B.C.2: at Z=0, oe! =e" B.C.3; at Z=0, d@'/dZ = d0"/aZ B.C.4: at Z=1, oe" =e" BC.5: at Z=1, de" /dZ = do" /dz B.C.6: at Z=, 6" = finite That is, we specify the condition far upstream from the reaction region, and we require that there be continuity of the temperature and the heat flux where the regions join. The solutions of these linear, homogeneous, second-order differential equations are then Region I: @1(Z)=C, +C,e™ Region II: 0" (Z)=Cye™2 +Cye™ for m, # m_ Region II: @™(Z)=C, +Ce™* 10-39 where m, = + {1-(4N78)). When the integration constants are determined, we get: Region I: ol(zyar4{ Mama =™.) (oe, JZ. wont: ape GT) sy 92, m2 eon ez) = mete? mgt ont egion 2) Yl, +m.) : a meme mam) Region III: ° (2)={ a ~— These results correspond to Eqs. 10.5-21 to 23. b. In preparation for taking the limit for B going to infinity, we first find the following Taylor series expansions m, = 4B(1+(1-4(4N/B)+- =t B+N+0(B7) m_ = 4B(1-(1-4(4N/B)+-- N+0(B7) “= It is important to note that the limiting value of m_ for infinite B is not zero, but N. Then, the limiting expressions for the temperature profiles in the three regions are mym_e™ ae mye 1s gm oe oe etn es em m_(mr~Je™e ee im bel em nae ae me" — me™ — yin) nt Jeo ela (since Z is negative) me™ - limo"(Z)= un 2 These results are consistent with Eqs. 10.5-21, 22, and 23. To get the second equation above from Eq. 10.5-22, we have to substitute F(@)=© and integrate. (0-4 10C.1 Heating of an electric wire with temperature-dependent electrical and thermal conductivity a. Rewrite Eq. 10C.1-3 as j Af, ko), | ke (zy EdE\ ky dé J) Ekg AL. This is easily transformed into Eq. 10C.1-4, with B =k, )R?E? /k pT. Use of Eqs. 10C.1-1 ane 2 leads directly to Eq. 10C.1-5. b. When Eq. 1C.1-8 is substituted into Eq. 1C.1-5 we get “te ([1- a {8(- 27)(1+B9,+-)} -a,{4B(1- ¢?)(1+ BO, +- jy Ke e(tall-€ 2\(14 BO, +-- -)) oo +30, + -)f ~B,{4B(1- 22)(1+B0,+-)}'+~) Equating terms containing B', we get a 1 Bil- =B ~pae 8ag#40-®)) which, when the differentiations are performed, gives an identity, B=B, as it should. Then we equate the terms containing B?, which gives AAG ~ gael Fag te, | sap[-aeftal-#)] =~B,B-4B(1- £*) Division by }B? and performing the differentiation in the a, term then gives the differential equation for ©, \o-4! 1d[,d =| (1-2 )e. |-« 1-2&?)+B,(1- €? ral gel 7) |= -a(1-28") +B, (1&7) Performing two integrations then gives (1- £7), =~}, (26? - &*) + £B,(48? - 4) +C,Iné +C, The constant C, must be zero in order to satisfy the boundary condition at the tube axis, and the application of the boundary condition at the tube wall gives O=-$ay +B, +C, Taking the difference between the last two equations gives (after dividing through by (1- &”)) 0, =4ai(1-€)- 48-2) in agreement with Eq. 10C.1-9. c. Rearranging Eq. 10.C-10 we have Reale Ie Ht = (140 ky ko To Ko J Into this equation we substitute Eqs. 10C.1-1 and 2, to get (1-2,e-2,07— =(1-B,0-£,07----)(1+) =1-B,0 - 8,0" —--+0 ~ B,0* - B,0°—~ =1-(6, -1)0-(B, -B,)@?—- Equating the coefficients of equal powers of @then gives a, = B, -1, G = By ~ By, and so on. Then Eq. 10C.1-12 follows directly: @=4B(1-é}{1- £B[(6, +2) + (6, -2)E?}+--} \o- 42 10C.2 Viscous heating with temperature-dependent viscosity and thermal conductivity a. The shell momentum and energy balances lead to d( aT dv.) _ and Ze D) oof ms) =0 Mulitply the first equation by b?/s10, and the second by b?/t1v2 to get Eqs. 10C.2-3 and 4. b. When Eqs. 10C.2-5, 6, 7, and 8 are used in the equation for temperature, and the coefficients of like powers of Br are equated, we get for the terms containing the first power of Br This equation has the solution @,=-4C%?+C,E+C,. The constants of integration are determined from the boundary conditions that @,=0 at &=0,1. Thus the following result is obtained: =4Ch (6-6?) We now tum to the ¢ equation (along with Eq. 10C.2-2) which is @, FE=c(1+ 8,0 +8,07+--) The expansions in Eqs. 10C.2-6, 7, and 8 then give d ae (% + Bro, +--+) = (Cio + BrC,; + Br°C,+---) (1+B,(BrO, + Br°0, +---)+--) Equating the coefficients of the zeroth power of Br, we get 10-43 etn ln Integration then gives ¢)=C\£+C). The boundary conditions (0) =0 and ¢9(1)=1 lead to @=§ andhencealso ©, =}(£-é") These are just the results obtained in §10.4. We next go back to the energy equation and equate the terms containing Br?: d dO, dO, ae 0, Ea S02) +2C, +B,0, é Into this we insert the expression for @, to obtain : Geb = M168 +68") 20, -F0(E- £4) Integration twice gives a(1 i Z 6,=-S(Fer-e det) ce? 2 1 1 (de - het} rceec, Then we use the boundary conditions ©, = 0 at £ = 0,1 leads to 6,--S(2e 8 +de)- a (-6 428-8) +c (6-2) a4 2 24" 7 The velocity distribution that goes along with this approximation is ¢,. Equating the terms linear in Br gives the differential equation B= Cy +B, 0, =Cy + 40,(E- 2) Integration gives lo-ae 10C.3 Viscous heating in a cone-and-plate viscometer a. First we make a table of “translation” to get from the plane Couette flow to the cone-and-plate flow: Plane Couette Flow Cone-and-Plate Flow (Fig. 10.4-2) (Fig. 2B.11) S=x/b $= ¥/Vo 9=2,/v, 9 =0,/rQ 2/6 Qo Br= 0? /kjTy Br(r/R)” = (uR?Q?/kyT,)(r/R)> The torque on the cone-and-plate viscometer is given by the force times the lever arm integrated over the entire plate: T, = 2H [opp gp Po tar But sind I(r \_ | 19% sind Tyg =n S28 O| 09 B56 roy is a suitable approximation for the cone-plate system, with 0 = 7/2. Therefore the torque expression becomes for a fluid at temperature Ty and viscosity Hg tor | () When there is no viscous heating, the tangential velocity is 04 = 1Q(v/¥0) Combining the last two equations gives, for no viscous heating: 10-45" 2mpQR> a (**) When (*) is divided by (“*), we get ee Te 586) 22g Tay nag, b. We now consider the system with viscous heating and use the cone-and-plate variables in the correspondence table. This gives ig = a a = 3f}(1-4BrE"p, +--)E-dE where we have used the dimensionless velocity expression from part (c in Problem 10.2. When the integration is performed we get T. T.9 =1-45%6, +0(Br’) To get the higher-order terms shown in Eq. 10C.3-1, one would have to go back and get the higher order terms in part (c in Problem 10C.2. 10-46 10D.1 Heat loss from a circular fin a. A heat balance over a ring of thickness Ar gives (2a(2B)rq, ), -(22(2B)rq,) pear —2.2nrbr-h(T-T,)=0 Divide by 27(2B)Ar and take the limit as Ar goes to zero to get (ra, tim eA Dreae _ 390 ar B (T-T,)=0 or from which we get the differential equation d ~£(q)-Ber-1,)=0 or £(+82)-Ar-7,)=0 dr dr) Bk In the second form, Fourier's law has been introduced. Next we rewrite the equation in terms of dimensionless variables: § = r/R) and @=(T-T,)/(To-T.): 2 1408) This equation has the following solution (with B? = hR3/Bk): (8) =Cylg(BE)+C,Ko(BE) in which I, and K, are zero-order Bessel functions. The constants are determined by use of the boundary conditions: @(1)=1 and 0/8)... q, = 0: This leads to 1=Cylp(B)+C,K(B) and 0=C,Bl,(BR,/Ro)-C,BK,(BR,/Ry) These two equations can be solved simultaneously to give ae BK,(BR,/Ro) ** TH@)BK(BR,/Ro) *Bl,(BR,/Ro)Ko(B) 10-47 BI,(BR,/Ro) Hence the dimensionless temperature profile is 69(é) = fo(BE)Ki(BR,/Ro) + 1(R,/Ro)Ko( 65) To(B)Ki(BR,/Ro) + 1,(BR,/Ro)Ko(B) b. The total heat loss is then: do = 4nBk(Ty -T,) | E 4mBK(T, TNE 7 1,(BR,/Ro)Ky(B)~ 1(8)Ki(BRi/Ro) = 408007. a Ne BITE BIC IR TE dT| = 2aR,(2B)k—| Q=2ak, (26) lo- 48 10D.2 Duct flow with constant wall heat flux and arbitrary velocity distribution a. The analogs of Eqs. 10.8-19 26 are wo EEE) ow LAC After multiplication by & the second of these can be integrated once to give EGE Col, FOE ME +c, Then division by & and further integration gives Eq. 10D.2-1 (along, with the defintion in Eq. 10D.2-2). The constant C, has to be zero, since the temperature must be finite at the tube axis (B.C. 1). Then from B.C. 2 we get C, = 1/I(1) using the equation immediately above. To get the remaining constant, we use the integral condition analogous to Eq. 10.8-24, to get $= Pols, cio(s)éag = fflCos + Caf? E1( aE +c, ole) sas = CoGI()+ Colo o()e| IS EMU(E)AE fis + Cott) =o + [rey Po eel Fue )az fs + +C.10) From the last line, Eq. 10D.2-3 follows immediately. b. The wall and bulk temperatures, in dimensionless form, are given by f.gegde Soogde 0, = (E =0,6) and Substitution of the result in (a) into these expressions gives 10-45 egy eo, a 10) ross fo] Pag eas +, ab +Cy = Tay rod ‘Toph Subtracting and then changing the order of integration in the second expression, we obtain = 1 ply 1"? E ae pM poe cus} ° ay The inner integral may now be written as the difference of two integrals, the first going from 0 to 1 and the second from 0 to €. But these two integrals are, respectively, just I(1) and I(&). Hence we get finally 7 De 1) gee) eee L@y - oO ashe etal Ee Thus, the first two terms cancel each other, and the last term gives Eq. 10D.2-4. c. From the definition of the dimensionless temperature, we get Teeth GoR/k Taking the reciprocal, and replacing R by D/2, we get Eq. 10D.2-5. d, The quantity I(1) is the ratio of the average to the maximum velocity in the tube. 1(1) = [5 08dE = (1/0, man )fo?2S4S = (0-)/0 an @,-, = (0-50 11A.1 Temperature in a friction bearing. ‘The method given in the solution of Problem 10A.3, based on Eq. 10.4-9, gives the maximum temperature rise in the lubricant as 19? R? Tnax — Th = 3 Sk _ 1(2.0 g/ems)(8000x/60 rad/s)?*(2.54 em)? ~~ 8 ([4.0 x 10-4 x 4.1840 x 107] g-cm/s*-K) = 16.9C and the maximum temperature Tmax a8 217C. Next, we consider the analysis given in §11.4, which includes the curvature effects. Since T; and Ty are equal, Eq. 11.4-14 is applicable and gives ban = [22a meV (Gis? =D as the location where the temperature Tmax occurs. For this problem, « = 1/1.002; hence, fmex = 1 o00)7 0.999001165 which location is essentially in the middle of the gap. To evaluate Tmax, we multiply Eq. 114-13 by (Ty ~ T,) to make each term finite; the result (after division by «* in the numerator and denominator) is ron BE omar l(-2)- (1-2) eel With € = max, we obtain the maximum temperature rise in this system as _ (2.0 g/em-s)(80007/60 rad/s)*(2.54 em)? 1 © “(£0 x 10-4 x 4.1840 x 107] geem/s*K) | (1.0027 1)? + [-0.002000667 + 0.002002667] = 8.438 x 10° - [2.00 x 10°] = 16.90 in agreement with the previous calculation. This good agreement is attributed to the narrowness of the gap relative to the cylinder radii in this problem. 11A.2 Viscosity variation and velocity gradients in a nonisothermal film. a, We begin by determining the temperature at which the logarthmic discrep- ancy, A, between the two viscosity representations in Eq. 11.4-18 is largest. The discrepancy is expressed as follows: A= In(First (2) function) — In(Approximation to first (z) function) = [pat @ - [2 ae @ Use of Eq. 11.41 to express (2/5) in terms of temperatures then gives z®-™(7—z)|-[an®-™ (Fm) B{T-h T-T To A ‘The T-derivative of the logarithmic discrepancy is aa eile a Th | Setting this derivative equal to zero, we get or T=Ty=VTTs sore Poh as the temperature of maximum discrepancy between the two expressions for In y(t). ». For the conditions given, (To) = (80°C) = 0.3548 x 10-* g/cm-s; H(Ts) = u(100°C) = 0.2821 x 10~? g/cm-s; Ta = (273.15 + 80)(273.15 + 100) = 363.01 K or 89.86°C Eqs. 11.4-14 and 11.4-19 then give the viscosity at Ty as neon mool(Z) (=2)| 0.2821 89.86 — 80 = (0.3548 ep)exp [(» 3) Ga) = (0.3548 cp) exp [(—-0.2293)(0.493)] = 0.3169 ep ‘Three-point Newton interpolation of Inj in Table 1.1-2 gives 1 = 0.3151 at 89.86°C, so the largest relative discrepancy is A = —0.0057, or —0.6 percent of (2). 1-2 11A.3 Transpiration cooling. a. In the absence of transpiration, Eq. 114-1 is indeterminate, but its limiting form is obtainable by expressing the exponential functions as first-order Taylor expansions in wy (and thus in Ro): Th This profile, designated as Qp, is tabulated here for the present geometry: 7, microns 100 200 300 400 500 Oo 1.000 0.375 0.1666... 0.0625 o In the presence of transpiration with the given rate w, = 1 x 10°® g/s, the constant Ry in Eq. 11.4-27 is _ (1x 10-5 g/s)(0.25 cal/g-C ) ~ (Ga\(613 10-* calfem-s-0) = 0.003245 cm = 32.45 microns Equation 11.14-27 then gives, with r in microns, T=Ti _ _(exp(—32.45/r) ~ exp(-32.45/500)) =T, ~ ((exp(—32.45/200) — exp(—32.45/500)) A table of this function, here called @y, follows: r, microns 100 200 300 400 500 Ov 1.000 0.406 0.185 0.070 0.000 ¢. The ratio of the heat conduction to the inner surface r = xR with the latter transpiration rate to that with w, = 0 is, from Eq. 11.4-32, Boe Qo exp g=1 (Ro(1~ 8)/«R) exp(Ro(1 — 8)/«R) — _ __(82.45)(0.8)/100) ~~ exp(32.45)(0.8)/100) — 1 0.2596 = Spoaseay ai = O87 ‘Thus, this small rate of transpiration reduces the rate of heat conduction to the inner surface by 12.4 percent. 1-3 11A.4 Free-convection heat loss from a vertical surface. ‘The physical properties for this problem are as follows, evaluated at an average temperature T = (T) + 7;)/2 = 110°F = 43.33°C = 316.5K: B =1/T = 0.00316K~" B= pM/RT = 0.001154g/cm* = 1.923 x 107*g/em-s from Table 1.1-2 1.007 J/g:K from Perry’s Handbook, 6th Ed., Table 3-212 0.2407 cal/g/K 0.0276 W/m-K from Perry's Handbook, 6th Ed., Table 3-212 6.60 x 10~Scal/s-cm-K ‘The Prandtl and Grashof numbers are then _ (0.2407cal/g:K)(1.923 x 10-4 g/em's) _ 7 6.60 x 10-cal/s-em-K) ee _ (0.001154 /em*)?(980.7em/s?)(0.00316K~")({(150 — 70)/1.8]K)(30cm)* ~ (1.923 x 10-#g/em-s)? = 1.34 x 10° ‘The heat loss rate from one side of the plate, according to Eq. 11.4-51, is Q = WHaavg = WC- b((Ty — Ti) (GrPr)/* ‘With Lorenz’ value of C, this gives Q ‘50cm)(0.548)(6.60 x 10Fcal/s-em:s)([80/1.8]k)(1.34 x 10° x 0.701)!/4 7.9 cal/s With the value C= 0.518 recommended by Whitaker for air, the result is Q=75 cal/s However, this value of C is based on a Prandtl number of 0.73 for air, rather than the value 0.701 found at the present conditions from more recent data, Linear extrapolation of the table on page 349 gives a C value of 0.516 at Pr= 0.701, and a corrected heat loss rate of 7.4 cal/s. i+ 114.5 Velocity, temperature and pressure changes in a shock wave. a. From Eq. 11.4-7, the initial air velocity is v1 = May V7RT/M 1.4(530 R)(4.9686 x 10! Ib ft? /s?- Ib-mol-R)(28.97 Ibm /Ib-mol) = (2) = 2256 ft/s b. The final velocity ao is found from Eq. 11.4-75 with € > 00, so that _7-1 2 1 orem sait Tei Ma 0.4 21 =oa + ry 0.375, Then Eq. 11.4-69 gives = (0.875)(2256) = 846 ft/s The final temperature is obtained from the energy balance, Eq. 11.4-65: 0) Toy = Ty + 5 Pee) 2 P (2256? — 846? ft?/s?)/2 el 2 (0.242 Btu/lbm R x 2.5036 x 10* Ibmft?/s*-Btu) = (630 R) + = 891°R The final pressure is calculated from the foregoing results by use Eq. 11.4-61 and the ideal gas law: hence Poove0/Too = prt /Tis man (2) ) = (1 ay (28) (8) = 4.48 atm ¢. The change in specific internal energy is O = GyAT = (Cp/7)AT = ((0.242/1.4] Btu/Ib,_-R)(891 — 530 R) = 624 Btu/lbm, and the change in specific kinetic energy is 1__ (846? — 2256? ft? /s*) 22.5036 x 104 Ib,,ft?/s?-Btu) 7.4 Btu/Ibm AK = 11A.6 Adiabatic frictionless compression of an ideal gas. The states encountered in such a compression satisfy Eq. 11.4-57, Pp a as well as PL pn RM Combining these relations, we get eTp Tp = Cs Hence, the initial and final states in such a compression satisfy B (2)" To Mn and the final temperature in the case considered here is Te = (460 + 100)(10)'4~* = 1407°R = 947°F {1-G 11A.7 Effect of free convection on the insulating value of a horzontal air space. ‘The relevant properties of air at 1 atm and 100°C= 373.15 K are: = 0.02173 cp from Table 1.1-2 = 2.173 x 10~* g/em-s p= pM/RT = 1/(82.0578 x 373.15) = 0.0009461 g/em* 6 = 1/T = 0.002680 K-? Cy = 1.015 J/g-K from CRC Handbook 2000-2001, pp. 6-1, 6-2 k = 31.40 mW/m-K from CRC Handbook 2000-2001, p. 6-185 = 31.40 x 107° W/em-K (1.015 J/g-K)(2.173 x 10-4 g/em-s) 31.40 x 10-5 WemK Pr = Cpu/k = = 0.703 The no-flow state will be stable as long as the Rayleigh number, GrPr, does not exceed its critical value of 1708, given in Ref. 4 of §11.5. This gives the following restriction on the temperature difference: 2 _mnré & 96(F - Toph - <1708 a or a 1708y2 (h— 8) S egies Pe 1708(2.173 x 10-4)? (0.0009461 g/cm*)?(980.665 cm/s*)(0.002680 K~)(2.5 em)?(0.703) .1°C If a very thin metal sheet is placed midway between the two plates, forming two cylindrical chambers of height h/2, then a corresponding calculation for the total temperature difference across the two chambers gives the stability condition (Ti — To) < (2)(8)(3.1) = 49.6°C for absence of free convection. II-7 11B.1 Adiabatic frictionless processes in an ideal gas a. For adiabatic frictionless processes, the fluxes may be set equal to zero. Then the energy equation becomes ¢, DT (322) 2 Bo 6p DT _ (dine) Dp Pe De" OinT M Dt (dint For an ideal gas p= pM/RT, so that (Inp/dInT), = -1. Then a (a+b tet?) DrEDE RT Hence for a given element of moving fluid (Z+o+er)ar- r& T P Integration from the initial state p,,T, gives answer (a) or aint +0(T-1,)+£(T?-T?)=RIn2 T, 2 PA c. For the data given, we have aln 22 = 3,2041n © ~ 3,156 cal /g-mole-K T, 300 1 b(T, —T,) = 18.41 x 10° (800 — 300) = 9.205 cal / g-mole-K = 4.48x10° (tT -T?) =F (800° - 300") = -1.231 cal / g-mole-K Summing these results we get inP2 = 11.124 or P2 = exp 1124 - 970 Pi pr? 1.987 Hence p, = 270 atm W-g 11B.2 Viscous heating in laminar tube flow (asymptotic solutions) a. From the energy equation we have mB) AB) Into this we substitute the expression for the velocity distribution of a Newtonian fluid ina circular tube: 2, = 0, mae [1 = (r/R) }. This leads, then, to Eq. 11B.2-1. b. Integration gives (for the isothermal wall) at large z: ‘i T=—Hiae rt ¢Cylnr+ Cy ‘The constants may be evaluated using the boundary conditions given in the problem, and the result is given in Eq. 11B.2-2. c. To get the z dependence of the temperature we perform an average over the cross-section aT = Fete Ce man ff(1— dg SE = “Him [eee which leads to the result T~T, = (410, nay /OC,R?)z We then make the postulate in Eq. 11B.2-4, and substitute this postulate into Eq. 11B.2-1 to get the following equation for f(r) 4MOzmoe KA ap df), 40% max 0 at £) aa te 8) A This equation may be integrated to give F(7) = (10% mox/k)(S? -$8*) which leads to Eq. 11B.2-5. 11B.3 Velocity distribution in a nonisothermal film a. When x= 6, the two terms inside the large bracket are equal, but with opposite signs, and hence v, = 0. b. The second term in the bracket, being a constant, will not contribute to the derivative. The factors in front of the bracket are also constant. Therefore the derivative of the bracket is: 0+(1/6)In(u5 /Hto) slots au)” w(t} (s/o [usin oe HoJS When we set x equal to zero in this expression we get aay dx t-]=(0/8)In(us/to)~(1/8)In(5 so) =0 Ix=0 Hence at x = 0, we have found that do, /dx =0 c. Let M=In(u5/Ho) and X = x/5. Then Eq. 11.4-20 becomes ScosB( 1, MX eM) gg) ae _ p36” cosB(_1_\( (1+ MX)e™ -(1+ M)e™* ee MEI eR gd? cospl (1+ MX)(1+M+4M?+-.-)-(1+ M)(1+MX +4M?X?+---) = Mo Merny _ pgcosB{1-X?+O(M) te (een When [ls — Ho (or M- 0), the above result simplifies to oy, -9 PSSSB(y_ x2) _ egos 1-(2) ero 2Ho 5 in agreement with Eq. 2.2-18. \|-10 11B.4-Heat conduction in a spherical shell In this problem, the heat flow is in the @ direction. Then Eq. B.9-3 simplifies to 1 a(, ,aT saaaol 8%) = The first integration leads to aie a0 sind fe Ok The second integration gives T =C, Intan$6|+C, =C, Intan}6+C, Since the argument of the tangent function is always less than a right angle, the absolute value sign is not needed. The constants of integration are obtainable from the boundary conditions, which give: T, =C,Intan}0, +C,; T, =CIntan}(2-0,)+C, We next form the following differences: tan}, T, -T, =C,[Intan}6, -Intan}(2z-0,)|=C, Ina) tando T-T, =G[intan} 9-Intand(n—4,))= Cine ay Finally we get for the temperature distribution in the shell: This solution clearly satisfies the two boundary conditions. Heit 11B.5 Axial heat conduction in a wire a. This problem involves purely axial flow of heat (by conduction and convection) so that the energy equation is a aly Cc 2 00,0. a= eee oe alt lz = ir aT_@T e ~Aatt_ ko dz az dz dz? in which v, =-v, and A= pC,v/k. Integration of the differential equation gives ~at= Zee, At z=, we know that T=T_ and dT/dz=0; hence C, =-AT,,. Hence the first-order differential equation becomes 2-0 where @=(T-T..)/(T)-T..) in which (since ©(0) = 1) In@=-Az+InC, or This is just Eq. 11B.5-1. b. For temperature-dependent physical properties we have the following energy equation: A do_d do do_d do 7 eon fas) -A,L(@) 22 = 4 ge PC nL(O) z(EKO) z) or -ALOy Ee KO) z) in which A, = pC,_v/k.. The first integration gives +A, [rue@yQaz x0) Bic, W-iz We now use the boundary conditions that at z=0, @=0 and d@/dz = 0, to find that C,=0. The above result may then be written as 0 (a\ dO 11D) a do +A. [,L(8)4 = (0) or -A.f L(8)d =K(0)E This equation may be integrated to give . K(6)d0 -A, f'dz= [Pp Alief ag “LU This result simplifies to that in (a) when K and L are both equal to unity. Furthermore, it satisfies the boundary conditions at z=0 and z=, c. To show that the last equation in (b) satisfies the differential equation (the first equation in (b)), we differentiate both sides with respect to z (on the right side, we differentiate with repsect to @, using the Leibniz formula, and then multiply by d@/dz): -—K@)_ #8 LUO)e dz A second differentiation with respect to z (once again using the Leibniz formula) gives and this is the differential equation with which we started. I-13 11B.6 Transpiration cooling in a planar system We start with Eq. J of Table 11.4-1, and assume steady-state, negligible change in pressure with distance, and negligible viscous dissipation. Then for constant thermal conductivity this equation simplifies to 2 a, at _,@T do_@o pC, 2, = ay ay? or ain = an in which @=(T-T,)/((T)~T,), n=y/L, and $=pC,v,L/k, a constant. This equation is to be solved with the boundary conditions that ©(0)=1and @(1)=0. Set p= d@/dn to get the first-order separable equation dp dp =P or P= oan PHF 7 i oi which may be integrated to give do Inp = on+InC, or aqnPsce" This is also a first-order separable equation and integrating it gives @=C,fedn+C, =Gen +C The constants of integration are then found from the boundary conditions, and we get finally k(T)-T,)d0| _K(T,-To) zoe oi W-\4 11B.7 Reduction of evaporation losses by transpiration a, Without transpiration, we have from Eq. 11.431 _ 4nKR(T, -T,) cr = 82Btu /hr Q 4n(4ft)(0.02Btu / hr-ft°F)(327°F) 29 2 b. With transpiration, we get from Eq. 11.4-30 AmRyk(T, -T,) . a Q= Rae with Ry =w,C, /4nk We do not know R,, but we can get it by using an energy balance in the form 0-0 AP =| 28h.» P When this is inserted into the left side of Eq. 11.4-30, we get MAsp —_(T1-Tx) Celtel -1 This may be solved for Ry, and then the energy balance may be used to convert the result into an expression for Q = ee) 7 CG, 1-K, AA vay _ 4n(0.02)(91.7 (0.22)(327) 9.22 mf 17) * 1) = (104.76)In(1.7845) = (104.76)(0.5792) = 61 Btu/hr \)-15 11B.8 Temperature distribution in an embedded sphere a. In both regions the partial differential equation is 10/20), 1 a(. ar aa" 5) Fino 308095) = ° b. At the surface of the embedded sphere the boundary conditions are that T, = T, and that k, (IT; /dr) = ky(AT/dr). c. For the temperature field inside the sphere, substitution of Eq. 11B.8-1 into the terms in the above differential equation gives: Tagine | oka 2 aa" ae [pte b+ cos 1 9(, ar 3ky - = S| sino | =-| 2A 0 Fn 705"? 96) [| a Outside the sphere we get for the same two terms 1 A( 2) _ 541-9 of kiko a 7 a2(r 2) -2ar cos 1 F 2k, Ar” cos@ 1 af, par ky-ky YR 4 SZ | = -}1-| 2Ar7 Fano a0 5p) [ lz ae? J] - d. The boundary conditions are also satisfied: 3k larcoso=|1-| = |larcose + 2k k +2k, 3k, k,| 2 Ia Le ceca [rcoso ta ako |scoso 2k, Sky oie +2 [I-16 11B.9 Heat flow ina solid bounded by two conical surfaces a, The temperature has to satisfy the differential equation A {sin of) =0 wo a6 b. Two successive integrations give a and T=C,Injtan}6]+C, 0 Since in this problem, @ may go from 0 to }z, and hence the tangent will not have negative values. Therefore absolute-value signs are not needed. c. The integration constants are determined from the following simultaneous equations: T, =C,In(tand@,)+C, and T, =C,In(tan40,)+C, On solving them, one gets C T%-T T-T “Intan}0,—Intan}O, — In(tan}6,/tan}0,) T, Intan} 6, -T, Intan}@, Intan} 6, ~Intan}6, Q d. The @ component of the heat-flux vector is obtained from the first equation in (b) above plat ___Gk k et rsin 0 In(tan} 6, /tan}0, ) 0 d0 sin e. The total heat flow across the conical surface is then _ Rpm . _ T,-T, R=, Foloao,tSiN 0,dodr = 2aRk Tian} 6, /tan JO) 3, /tan¥0,) Wey 11B.10 Freezing of a spherical drop a. The heat conduction equation for the solid phase is da(,aT)_ rar aT) 0 (Ry srs R) Two integrations lead to T=-(C,/r)+C,. The constants of integration are determined from the boundary conditions B.C.1: Atr=R,T=Ty; B.C.2: Atr=R, «Ze ur-r.) r This leads to the following expressions: ToT. Ty-T, ——0 = __; © = T, + 9 > (R)-(YR,)~(k/nR?) [(VR)-(VR,)=(k/hR?)]R, The total heat flow across the spherical surface at r= R is then Q= sar'(- ft) Ty -T. lk ane'(—3 Fam (yR,)~(k/mR*)] This can be rearranged to give the solution in the text. b. We now have to equate the heat liberated on freezing at r= Ry to the heat flowing out across the surface at r= R: : aR, h-4nR?-(T)-T..) (pati, )(4aRi) ar ian (ik) (uR°7k,) Integration then yields —(eAF, )fe[1= (leR/K) + (HR?/KR, )]RFAR, = HR? (Ty ~T..)f dt where f, is the time for the freezing of the entire droplet. Evaluation of the integrals then leads to the expression in the text. 11B.11 Temperature rise in a catalyst pellet a. We make an energy balance over a spherical shell of thickness Ar: 4nrq,|, -4n(r+ Ar) q,|,,., +4r7Ars, =0 lhear Then division by 47Ar gives ee _ 5. =0 Ar When the limit is taken that Ar 0 and use is made of the definition of the first derivative, we get dia 2g = al 4,)-1°S, =0 Insertion of Fourier's law then gives 4 4) 25 <9 (#* A (re +7°S.=0 (**) or we for the appropriate equation describing the heat conduction with heat generation by chemical reaction and constant k. b. From Eq. B.9=3, with the time=derivative term set equal to zero, and all velocities set equal to zero, and all derivatives other than r derivatives set equal to zero gives the heat conduction equation in spherical coordinates for a system with no chemical reaction. Therefore, we have to add a term describing the heat production per unit volume: 1 d(> a) _ 7 a ee which is the same as the result obtained in (a). c. The above differential equation may be integrated in a sequence of steps as follows: \-14 2a) se aT $7 G : cy _ (" ar) 3k dr 3k re Sat 6k 4 a wee T= 24C, (**) The constant C, must be zero, because neither the temperature nor its gradient are expected to be infinite. The heat loss to the surroundings provides the second boundary condition needed for getting C,: Definition of heat transfer coefficient: 4,|,_,=h(Tx-T,) From Fourier's law: Iher = Equating these expressions: WT, -Ty . a S.R? S.R Inserting T, from (**): - ee +G-T, |= 5 7 S.R? , S.R Sok for C,: C, == +— +T, orig OF 6k 3h & Thus we finally get the temperature profile within the catalyst pellet: S.R2 ry], SR PTE ee [-(5) ‘3h d..When the heat transfer coefficient goes to infinity, the last term in the temperature distribution drops out. e. The maximum temperature in the system is 7 6k 3h 6k SR? , S.R_ SR? (1428) f- In Eq. (**) one would have to leave k inside the differential operator, and insert the specific r dependence of both k and S, . {1-20 11B.12 Stability of an exothermic reaction system a. For the postulated steady-state solution @T a kz tS0exp(A(T~Tp))=0 with T=T, at x=4B b. Using the given dimensionless variables, we may rewrite the problem as a Ge tae? =0 with @=0at E=41 c. Multiply the differential equation by 2d0/dé to get 2 d (do pideeue ° Za) aati Integration then gives do (2) sam =0 We now use the fact that, from the symmetry of the problem, at = 0 we must have d@/dé =0. Then if we let ©=@, at € =0, we can get an expression for C, and then write 2 (2) =2A(exp®, - exp®)=0 Note that we have not “evaluated” the integration constant C,, we have merely replaced it by @g, which has a recognizable physical meaning. c. We next take the square root of both sides: d@ GE TEVA OR, =expO WW 24 The minus sign has been selected, since the left side of the differential equation is inherently negative, and the quantity under the square- root sign is positive. Then we integrate over half the plate 80 do 0 Jexp@, — exp d. The integral can be done analytically by making the change of variable y* = exp(@-,): =-v2A Pde os do ° f1-exp(@-©,) 2d . e(-2 0) boc aed ge =2exp(-40,)arecosh(exp}@,) The integral over y can be found in an integral table. Combining these last two results we get exp(-}0,)arccosh(exp40,) = (Ja e. For A > 0.88 .no value of @, can be found. This means that when S,9 is too large, or B is too large, or k is too small, then the heat cannot be dissipated fast enough. This is an important example, because it illustrates that it is not always possible to get a steady-state solution to a physical problem. To do a complete analysis of this problem, it would be necessary to solve the problem with the time-derivative term included. 11B.13 Laminar annular flow with constant wall heat flux The equation analogous to Eq. 10.8-12 for the annular flow is A r)_i-«? (R)lar_,10(_ aT C 1-( =) -—* ain =) |S = k= re WE Pom (3) in(Vx) (7) & OFS) where Eq. 2.4-14 has been used. We now introduce dimensionless variables: €=1/R, £=z/pC,v, ma¢R?, and © =k(T-T;)/qoR- Then the above energy equation becomes [o-e)-t- ye] 3-762) This is to be solved with the boundary conditions ar 0 Atr=KkR, “ky h or at é=k, = Atr=R, -k2Z=0 or at E=1, ae Atz=0, T=T, or at¢=0, O=0 We seek an asymptotic solution for large downstream distances of the form @(,¢)= Cf + P(E). The function ‘(E) has to satisfy 12 (,a aying ral? a f(t-#)-G- HS] The first and second integrations lead to (cf. Eq. 10.8-27) o[é-4) actin) 9 2 gf 2 w= ol = -£)- ree ing-£)lecing ce 11-23 The constants C, and C, are determined from the boundary conditions at £ = x and €=1: 1 Cy)=——**—_,_ and G=- ce pee) (1-«*)+5 Then the equation for the radial distribution of the temperature is vei =)- i (os-0}- fe )ing We The constant C, can be obtained by using an integral condition, similar to that in Eq. 10.8-24 or 25 2anRe-qy=["[R0C(T-T,)eardrdo or $= fiooEds where @=0,/2. max- This gives \|-24 since the terms containing ¢ just cancel. This last equation has to be solved for C,. This is in principle an easy problem, but extremely time-consuming and unrewarding. It can, however, be solved by using Mathematica, and the result is* -9(1- x?)°(114+ 3x?) -2(1~ x?) (88 +88x7 + 25x*)In x K +3(-25- 24x? +1814 + 24° +7K8)(InK)? ~72K4 (INK)? Q= ° 721-2) (1x?) +(1+ x? )(in x) “The authors wish to thank Mr. Richard M. Jendrejack for his help on this problem. \\-25 11B.14 Unsteady-state heating of a sphere a. The time-dependent heat conduction equation is a -ch a(n a) or a _12a o2 (i aol e or or “BE In the second form, we have introduced the dimensionless variables: @=(T-T))T;-Tp), €=1/R, and t= at/R?. The solution in Eq. 11B.14-1 is, in terms of the same dimensionless variables — pee (gy Sine ate @=1425(-1)" Se b. We do the differentiations; then it is clear that the first and last expression below are the same. Therefore Eq. 11B.14-1 satisfies the differential equation. FO _yS_yyn(_2 2) SINE nade aX 1)" (-0? 0?) nag 2 e =23- yin nf conn _ sinnn€ | iat nn (nay cosnaf naksinnng _cosna 25(-1)" na Sons nagsinna§ cosnng nt BEE B)-28 0 1 -sEseest Le oe mt nag c. When & = 1, we get from Eq. 11B.14-1 @= 14230 1 RAE rime since sinn = 0 for all integral values of n. d. Inasmuch as [26 sinx lim—— x90 x =1 the solution for ¢ = 0 is certainly finite: @=1425(-1N'e""" nl e. In dimensionless form Eq. 11B.14-2 is x sinnne 1-239 Then, multiplying by sin maé and integrating gives ~{igsinmnédé = 2B) a fisin ma€ sinnnéaé The left side can be evaluated as follows: ~ ig sinmngag = - mah” xsinxdx = (sinx-xcosx)[}” eo (ma mn)" 1 m mmcos mn) = — =—(-1 (mxcosmn) = cosmn a(-1) i (mx) The right side may be evaluated thus: 23 (ay Fe ffsinnng sinmagig = 23-0)" at [fsinnesinmxdx es =23 1)" ta ( Foam )= Lay" Thus the two sides are equal, and it is proven that the initial condition is indeed satisfied. W-2] 11B.15 Dimensionless variables for free convection a. When the proposed dimensionless quantities are intro- duced, Eqs. 11B.15-1 to 3 follow directly. Nothing more needs to be said. b. We can convert Eq. 11B.15-1 into Eq. 11.4-44 if we require that P0¥o 4 © all Then, when (1) is substituted into Eq, 11B.15.2, then the latter may be converted into Eq. 11.4-45 if we also require that any} Hop, Yoyo am — °88(To-T1) _p, Vyo?z0 Next we substitute (J) into Eq. 11B.15-3 and further require that w) PC»Yo? yo then Eq. 11B.15-3 becomes Eq. 11.4-35. We thus have four equations from which to determine the three "scale factors" Yo, Vy, and V,. However, it can be seen that Eqs. (II )and (IV) are not independent, since multiplication of Eq. (IV) by Pr just gives Eq. (I). Thus we have three independent equations from which to determine three unknowns. We now eliminate y, by multiplying Eqs. (I) and (I) to get Eq. (V), and by dividing Eq, (III) by Eq. (1) to get Eq. (VI): Pohl s0H = Pr Hp V-210. gB(T)-T))H 2) 7 =Pr Vin Introducing the abbreviation B = pgB(T, ~T;), Eq. (VI) can be solved for v,9 to give Eq. (VII), and then v,9 is obtained from Eq. (V) to give Eq, (VID): BH _ [aBH (va) = PE an _ | 4 [aBH _ Jo3B Came °=\ppe\ nV aH Finally yp may be obtained from Eqs. (IV) and (VIII): k HH _ Jour 1X) == ay = ) ¥ PC, 20 Nose VB The reciprocals of these last three quantities appear in Eqs. 11.B-41 to 43. c. If all the dimensionless groups were set equal to unit, then combination of Eqs. (II) and (IV) would give Pr = 1, thereby severely restricting the applicability of the results. {1-28 11C.1 The speed of propagation of sound waves a, Equation 11C.1-1 can also be written as (2) - AG r) 7 (ap/as), _(AH/9T), (ap/ar), aV)5 (@UTA),\ IV) (av/a8), - (dUjaT), (aV/aT), This may be rearranged to give (3) (30) (22), C32), (33) (Fe) a a8),\ aT), Next we apply the Maxwell relations to the first two factors on the left and right sides, and we apply two of the four "fundamental relations" for pure fluids to modify the third factor as) ° (LalaHealok Then we apply the relation (JAE, to the first and third factors, to obtain (05/4p),. . (as/av),. (aV/aS),- (p/28),, On cross-multipiclations, this gives an identity. b. The equations of continuity and motion are ro 2 (gy +6') )=-(V +(e +P'Y(vo tv’) Zo +0'V(v0 +v')=-V(Po +P’) Since p) and pp are constants and vg is zero, we get (when the products of the primed quantities are neglected) 11-29 op ow Pa-p(V- d pw =-v; BPW -v) and Py>-=—Vp c. Since the momentum and energy fluxes have been taken to be zero), the flow is isentropic. This enables us to rewrite the equation of motion thus: atem(§) 008) To get the second expression, Eq. 11C.1-1 has been used. Use of the definition of the speed of sound then gives Eq. 11C.1-4. d. Next we take the time derivative of the equation of continuity to get a a ov Ce TES iovvi={ Ve Z) or FP -civ%p To get the second form, the equation of motion of Eq. 111C.1-4 has been used. e. From Eq. 11C.1-6 we get 37 oA 2) v? sin| z v,t) _ p 2 ve . ~v?ppA( 2) sin| (2 7 2] Therefore, Eq. 11C.1-6 is satisfied. \1-30 11C.2 Free convection in a slot a. The equations of change are simplified as follows: Continuity: go- .] and at steady state and v, = 0 we get 0=0 Motion: a a _ (av, a, # Soro.) -2-r¢+ S tat +DBg(T-T) or O=n oe, PB3(T -T) since p= —Pgz + constant ae Energy: onk GT whence T=T+Ay b. The boundary conditions are; v,(£B,y)=0 — v, isanoddfunctionofy — T(x,0)-T =0. c. To get the velocity profiles we rearrange the equation of motion as follows: Po, Ox? This may be integrated to give: 2 v,(x,y)= -2BeA xy4+C,x+Cy or v,(x,y)= ee (- (3) d. For water at 20°C , 7 = 0.9982 g/cc and B = 0.00021 (°C). The maximum velocity will occur at x = 0 and y = W, so that I-31 _ DBgAB’ = ane v, Hence the corresponding temperature gradient will be 2H. max PBgB’w 2(1.0019 x 107 ajem-s OSD) . (0.99823 g/cm?)(0.00021 °C*)(980.7 cm/s*)(0.01 cm)*(0.2 em) =0.271 °C/em 11-32 11C.3 Tangential annular flow of a highly viscous liquid First, rewrite Eq. 114-13 to eliminate N in favor of the Brinkman number Br, which appears in Eq. 10.4-9: o-(1-B) em ld Ee To show that this reduces to Eq. 10.4-9 in the limit of a very thin annulus, begin by taking the limit of the term that does not coniain Br and then we treat the term containing Br. Term wii i We let x=1-e and =1-e(1-n), where 17 is the x/b of §10.4. Then using Eq, C.3-2 we find cing _,_r[ed=m+iea-nf+-] : Ink -[e+ie?+-] 1-(1-n) + O(e)= n+ Ole) In the limit of vanishing ¢, this leads to the last term in Eq. 10.4-9. Term wil The coefficient of Br is now (2e-e?)"|\ [1-e@-n)f? {:- 1 | ene (1-ey ethene When everything is expanded in terms of powers of €, we find qe ASS (rf zet1— np +362 (1-0?) ; -(1-[1426 +30? +-))([1- m}+$e(-n4 n?)+--)] Simplification of this expression leads to ((-33 1 qei(l~3e+ He +-)[(-2e(1~n)-3e*(1- n)?) ~(-26-3e?---)([1- n]+4e(-n4 0 )+-)] Then cancellation of some terms gives 1 geil Ber Be? (1m)? — {36?(1- 9) -?(- +?) 4] =ha-ser-)[-a(-nen)+(-n4 +] ul -7n =3(n ”) This is in agreement with the term in Eq. 10.4-9 that is multiplied by Br. {1-34 11C.4 Heat conduction with variable thermal conductivity First we obtain an expression for the gradient of F: VF = V(JkdT + constant) = f(Vk)aT dk =jSvrar Sar The V operator can be taken inside the integral, since it involves only differentiation with respect to position coordinates and therefore commutes with the integration over T. Since k depends solely on T (which may in turn depend on the position coordinates), we must differentiate with respect to T and then perform the gradient operation on T, as shown above. Next, since VT depends on position coordinates, and not on T, the VT may be removed from the integral sign ve=vrj Rar When the integration is performed, we get VF =kVT We now form the Laplacian of F V?F =(V-VF)=(V-kVT) But (V-kVT)=0 by the equation of energy. Therefore, we have finally VF=0 11-35 11C.5. Effective thermal conductivity of a solid with spherical inclusions a. At very large distances from the region containing the inclusions, the coordinates of the various inclusions (r,8) will not be very different from one another. Furthermore, if the density of the inclusions is small, the effect of the various inclusions will be additive. Therefore to get the temperature field far from the region containing the inclusions, we can write eoelhe =a (RY! Ty(r,0)-T' -|: "a Dke ; |Arcos@ in which n is the number of inclusions, each with a thermal conductivity k,. This is the equation that the describes the system in Fig. 11C.5(a). b. For the system in Fig. 11C.5(b), we can apply Eq. 11B.8-2 directly kee ~ko (R’)? To(r,0)-T? = 1 fa () Ai 6 regarding the shaded sphere as a hypothetical material of thermal conductivity ky. c. Next we relate the volume of the inclusions in the true system to the effective volume of the inclusions in the equivalent system: 42R°n=42R'°9. Hence R’? =(n/9)R°. d. We can now equate the right sides of the above two equations for the temperature profiles far from the origin nticke (2) - es = ky (2/2 ky +2ky\r keg t kyr) Oo We can now solve for the effective thermal conductivity, and express the ratio ky/ky as 1 + (deviation resulting from the inclusions). This is exactly Eq. 9.6-1. \1-36 11C7 Effect of surface-tension gradients on a falling film a. If we let I be the gas phase and II be the liquid phase, then the contribution from Eq. 11C.6-4 to the stress component f,, will be the z-component of ao [i-t]=Vio or mas Hence, in Eq, 2.2-13 has to be replaced by Ta =(egcosf)x+A When this is combined with Newton's law of viscosity, Eq. 2.2-14, we get 182s = (pgcosp)x+A When this is integrated, and the no-slip boundary condition at the wall is used, we obtain the velocity profile as follows: SH b Fea) ee lS) fees b. The mass rate of flow in the film is w= fi" [) po,dxdy = pWof,v,dg 5 = | (282 20. nue (¥/3) this last expression is good in the vicinity of the wall, where the quadratic term can be neglected. b. Equation 12B.4-2 presupposes that the heat conduction in the z direction can be neglected relative to the heat convection in the z direction. In addition, laminar, nonrippling flow is assumed. c. The fictitious boundary condition at an infinite distance from the wall may be used instead of the boundary condition at a distance 5 from the wall, since for short contact times the fluid is heated over a very short distance y. Therefore the inifinite boundary condition can be expected to be adequate. d, Equation 12B.4-3 can be written as y(90/dz) = B(9°@/ dy’). Next we have to convert the derivatives to derivatives with respect to the dimensionless variable : 2 _400n 40 _y (-4) az dn az dn¥9pz\ 3z 2 404 _ dO 1, ge. #{# 1 )2-€8 dy dn dy dn¥9pz" dy” dn\ dn ¥9pz) dy dn When these relations are substituted into the partial differential equation and use is made of the defining equation for n we get Eq. 12B.4-7. e. When we set d0/dn = p, we get dp/dn +3n*p =0, which is first-order and separable, and the solution is given in the book. The next integration gives a. i2-1l 12B.5 Temperature in a slab with heat production a. This problem is discussed on pp. 130-131 of the 2nd Edition of Carslaw and Jaeger. The solution in dimensionless form can be obtained from Eq. (7) on p. 130. We must first determine the correspondence between their symbols and ours: ce ot K o x/l K iP Ay BS&L b k T-Ty n= =o So Therefore the temperature rise as a function of position and time is LLU weer sat -{: ? epee’ b, The center-plane temperature is obtained by setting 7 equal to zero. The maximum of the center-plane temperature is then 2 Trax = To + c. According to the figure on p. 131 in Carslaw and Jaeger, 90% of the total temperature rise occurs at about t=1, that is, at about t= b?/a. 12-12 12B.6 Forced convection in slow flow across a cylinder a. First we have to determine the velocity component v, from Eq. 12B.6-1 (here we let €=r/R and measure 9 downstream from the stagnation locus): dy _v.sind 0 oy 28 [(ame-1)+2-2| Then let € =1+ 7 and, for small 1,, introduce the Taylor expansions Ing =(E-1)}= nt y/R+- and -1/€? =-14+29+-. Then we may write 20, sin@ sr Y= Ay v.. sin [4n+- 28 Therefore, B = 2v, sin0/SR. b. We identify the boundary-layer coordinates as x=0, y=r-R, and z=z. Then we recognize that h, =R and set h, =1. Therefore we can get the heat loss from a length L of the cylinder as follows, starting from Eq. 12.4-31: PHT TDL 202)” cana)” 2aPT(8) VSR GIP pony. (SEE) = PUG BP aout, [ rA5 =) : Comparison of this result with the solution to part (b) given in the text allows the constant C to be evaluated. c. The boundary-layer thickness can be obtained from Eq. 12.4-29 as follows: zg 6,1 0 (20, jae 7 Sa Peed RR — eae ao B (es La aao) (ge) / fe At the separation locus, = , the integral is finite (according to Eq. 12B.6-2) and the denominator in f is zero, so that f is infinite. Near the stagnation locus, @= 0, the numerator and denominator of f may be expanded in Taylor series to give: (We 0-2 ~)" (v6 o-10°+ Vo(1- $0? +-) Then in the limit as 6 — 0, this gives the stagnation value f=)” Numerical integration gives for 9= 47 f=1.1981 To get the answers given in the book, we have to recognize that the theta in the problem is being measured from the stagnation point at 6 = 7 to the separation point at @=0. Note that for this flow, just as for the flow around the sphere, the boundary layer thickness increases through finite values to an infinite value at the separation locus. (Note: Eq. 6.2.1 of Abramowitz and Stegun (NBS, Applied Math Series, 55) is a convenient formula from which the integral from zero to 42 can be calculated in terms of gamma functions, which are tabulated.] f2-|4 12B.9 Non-Newtonian heat transfer with constant wall heat flux (asymptotic solution for small axial distances) For Newtonian fluids, Example 12.2-2, we wrote vonafe tal ofl salt) nk where Eqs. 2.3-18 and 19 have been used, as well as the definition y=R-r. The analogous procedure for power-law fluids gives 2.=P.anl (Frm "| ab G | oa) SY alt) v, (sme) a)=k This gives the formula for the v that has to be used for power-law fluids. The quantity vp , with dimensions of velocity, appears only in the dimensionless parameter A. {2-15 The boundary condition at =0 gives C,=1. The boundary condition at =, gives 1 Ce a= pera And the complete solution (in dimensionless form) is eran fePaq- eran freP aa @=1- ig eran fePan r(@) The integral in the numerator cannot be integrated analytically. f. The local wall heat flux (ie., at any position z down the wall) is [=e] oa] ToT ToT Meo atin VBE ECS) APB: In taking the derivative, the Leibniz formula was used. Finally the average heat flux at the wall is =f, =~ Tor 1 a0 = [| 08 TQ) yop L Le 7 a ffePeen? Ta) vac This is the result that is given in Eq. 12B.4-9. rie 12.1 Product solutions for unsteady heat conduction in solids a, We begin by defining a dimensionless temperature difference by T,-T(x,y,2,t) A= T,-To in which T, is the initial temperature of the solid rectangular parallelepiped, and T; is the imposed temperature on the surfaces of the solid. Then the 3-dimensional heat conduction equation for the solid is A _ (PA PA aA tn of $2 $5223) If we now postulate a product solution A(x,y,2,t) = X(x,t)Y(y,t)Z(z,t) then we get (XYZ) _ of 70, 0? (XYZ) ee ot ox? ay Division by XYZ then gives pale 907) 18x oo Cae au FZ XYZ Xor "Yay Za When the product on the left side is differentiated we get VIX, 1a 192_ (1 eX, 19'y 19% X dt Y ot Zot Ores Ye Ovae Zidzy The first term on the left is a function of x and f, as is the first term on the right. Similarly, the functional dependences of the second and [2-17 third terms are the same. Therefore we postulate that these pairs of terms can be equated to give: OR tes Gee 4 ae? Bae! BOR That is, we get three one-dimensional heat-conduction equations. These can be solved according to the method of separation of variables given in Ex. 12.1-2, and all of them have the same initial conditions, and the same kinds of boundary conditions. Therefore, when we combine the three solutions in the product form above we get a] eto) atanyot cos(n + devi] PA Pate cosln sea] ~ (ayn pna(m + 4)(n+ 4)(p +4) ‘[cos(m + 4)xx/allcos(n + 3)ry/b]{cos(p + 3)22/c] elm [22 (ned) /?+(ps3) [2 peat This can be substituted into the differential equation to verify that the latter can indeed be satisfied. Also, the satisfaction of the boundary and initial conditions can be verified. b. This and many other interesting and important product solutions are discussed in Carslaw and Jaeger on pp. 33, 184, and 227. (2-13 12C.2 Heating of a semi-infinite slab with a variable thermal conductivity a. The heat conduction equation is, for variable k, oe E z( 7 -£(«2] or 06,2 = 2 2{ 090) 2) or aol 563] In order to use Eq. 12C.2-2, we need to convert the derivatives: =m 2 _40.9n y a8) _ 40 nd) 6 dt a dn ot dn\ & dt. 20 _ db dn _d1 FO doi Oy dm dy dn OP dg BF b. We now substitute the above derivatives into the heat conduction equation to get (after multiplication by 5”) “al a) ial ae When this is integrated over 7 from 0 to 1, we get + p082)an 6] Zcne)~o |in 0 6H fa ~olia Performing the integrations we finally end up with a dé =n, +[}edn- oa «($2 po a) which is equivalent to Eqs. 12C.2-3 to 5. 12-19 c. We now use ®=1-31+4, which is a good choice for the approximate temperature profile, since it gives ©(0)=1, ®(1)=0, and &’(1) =0 in agreement with our intuition. We then get M and N by substitution into Eqs. 12C.2-4 and 5: M= fodn=[(1-3n+4n°)dn=3 ds d® v=(S2sp082) =[(-3+397)+6(0-3nedn°\-3+3n)] =3(1+) This leads to the differential equation for the boundary layer thickness: 35% = 31+6)a, which when integrated gives 5 = /8(1+ B)aot The time-dependent temperature distribution is then The heat flux at y = 0 is finally [et ee Te) 20 eet) Fy ayl gL) dining 2y8(1+ Bat in which k is given by Eq. 12C.2-1. (2-20 12C.3 Heat conduction with phase change (the Neumann-Stefan problem) a. Using the definition of the dimensionless temperature differences in Eqs. 12C.3-1 and 2, we may write the heat conduction equations for the solid and liquid phases as follows: 2 Solid: Oy gO : e Liquid: a ae The initial and boundary conditions are: LC: at t B.C. 1 at B.C.2: at z=0, B.C.3: at z=Z(t), B.C.4: at z=Z(t), b. The assumed forms for the solution are chosen because of their similarity to other one-dimensional heat-flow problems in a semi-infinite region: Solid: @,=C,+ Crert Tee z vV4at =(C; +C,)-Cyerfe Liquid: @, =C, + Cyerf2— =(C, +C,)- a ~erf V4at The last form for the liquid-phase temperature equation will turn out to be somewhat more convenient to use. c. When we apply B.C. 1, we get C, = 0. Using B.C. 2, we get C,+C,=1; we not that the initial condition is automatically satisfied. Then B.C. 3 gives Z) dat Z(t) Cerf rd 1-Cyerfe 12-21 The only way that this equation can be satisfied, is if Z(t) «Vf. We elect to write, then, Z(t)=AV4at where A is a dimensionless constant that has yet to be determined. Finally we apply B.C. 4 to get ‘4 Next we apply that part of B. C. 3 that deals with the dimensionless melting temperature: CyerfA=0, and 1-Cyerfcd = 0, whence ®, 1-8, Ca 7 7 *) 4 rfl erfcA Combining (*) and (**) and rearranging, we get This gives A in terms of A and @,,. Then we get for the temperature erf(z/4at) erfA erfe(z/4at) Solid: ©; =, aa ; Liquid: ©, =1-(1-0,,) Finally, we have Z(t)=4at, where J is now a known function of Aand ©,,. \e 22 12C.4 Viscous heating in oscillatory flow a. The starting point for this problem is Eq. 4.1-44, and the development up to and including Eq. 4.1-50 can be taken over here. B.C. 1 is also valid, but now B.C. 2 must read: at y=b, v° =0. When Eq. 4.1-50 is solved for these boundary conditions, we get 0°() _sinhe(1- 6) 0% sinhe where c= fiwb?/v = ab?/2v(1+i)=a(1+i) and &=x/b. The above expression for v°(é) can now be substituted into Eq. 4.1-48 to get Eq. 12C.4-1. b. Next we get the dissipation function, which for this problem is &, = 4(dv, /Ax)*. The derivative of the velocity is 0, _ gp {A” sie ox = dx and its square is obtained by using the relation (which should be proven) S{u}S{o} = [M{wo] + R{wo*}], where u and v are complex numbers and the asterisk indicates a complex conjugate: a0,)? _ 1] g|( do?) dv? av’) 2) Tigh] | prior at (%) i (eye Ua la Next we get the time average of the above function; the first term vanishes because of the exponential (which contains sine and cosine functions) and only the second term survives: av, __1 (dv°\( do® dv° __vg ccoshe(1-&) (%) -#(S\¥) ME ry from part (a). Then the time averaged dissipation function is: \2- 23 2° ecoshe(1-£))/ ecoshe(1-€))" (3) ( sinhe it sinhe ) (aay (cc*)(coshe(1- €))(coshe(1~€))* b (sinhe)(sinhc)* (m0) [cosh? a(1- £)cos* a(1-€) + sinh? a(1- £)sin? a(1- €)] 2\b sinh? acos” a+ cosh? asin? a To get the third line of the above we have used the identity cosha(1+i)(1- €) = cosha(1- £)cosa(1~ €) + isinha(1- €)sina(1-£) which should be verified. Then, finally, use sin? 0+cos? @=1 and cosh? 9 ~ sinh? 0 = 1 to simplify the above expression for ®,, thus: oe o(z) (= ~ 6) +sinh? a(1— 2) 5 (22) em sin’ a+sinh?a the last expression being a limiting expression for very large frequencies. c. To get the time averaged temperature distribution, we use Eq. 12C.4-4, thus &T HF o(%) cos? a(1- €) + sinh? a(1- €) de OM sin?a+sinh? a Then introducing a new variable = 1-& we write PT __ Mvp cos*an+sinh? an dank \. sin?a+sinh?a Integration once then gives aT __wv5/sin2an+sinh2a — Ho sin2an + sinn2an) . o d(an) 4k" sin?a+sinh?a 12-24 and a further integration then yields 7 = Hei sin® an + sinh? an 4k \ sin? a+sinh?a }+cansc, The constants of integration may then be determined, so that we get finally: )+sinh? a(1- ?a+sinh?a For the high-frequency limit, we get T-T, = Mil e™)-g1-e)] (Note: The solution given here for finding the dissipation function does not use Eq. 12C.4-1 and is hence somewhat easier than the method suggested in the text. It does, however, require somewhat more familiarity with doing manipulations with complex variables.) 12-25 12C.5 Solar Heat Penetration The basis for this problem was Eq. 5-40, M. N. Oz Ed. , Wiley 1993 -- - p.206. The temperature profile is re-expressed as. T(y,t) - Tw) Trax (0) — T(e) Heat Conduction, 2end and should have been written as T* = exp[- yal 2a]-cos[at — 1 lo /2a] 2_pyivaea va ‘The thermal properties are taken from (Rohsenow, Hartnett and Cho, Handbook of Heat Transfer, McGraw-Hill, Third Ed., p.2.68, 1998): exp[-77]-cos[at ~ y?@ / dan” |dn Density, kgm" ‘Thermal conductivity, Specific heat, Wim. Jikg.K 1515 0.027 800 It follows that 0.027 PW - 361 5m _ mW -8600joules _ g yp 4-5. 800-1515 joule-W — hr hr ) For the assumption of a sinusoidal input with a 24 hour period T(0,t) = asin(n@ t)) Show that the mean temperature, in the absence of day-to-day variation is just ay , and for a periodicity of 24 hrs © t=at/12hrs;@=7/12hrs ‘The amplitude of the fluctuations at depth y relative to those at the surface, “A,” is just Aye exp(-v*o/2a) exp(-40.4-y/m) ‘Then at a depth of 10 cm the relative amplitude 12-26 A, = exp(-4.04) = 0.0176 as required b) The transient term is difficult to evaluate, but this is of no practical importance. The periodically steady oscillations are about the initial temperature of zero, and therefore all transients must be of lesser magnitude than these periodic excursions, Since the excursions are already negligible there is no need to examine this problem further. ©) Only the slowest component of the oscillations need be considered as the others damp. out even faster. 12-2] 12C.6 Heat transfer in a falling non-Newtonian film For the non-Newtonian falling film problem, the velocity distribution has been found in Problem 8B.1: oll fll lo This can be rewritten in terms of the coordinate y, which is the distance from the solid surface (yn) 7 (1-4 - = Pema (1-2) When this expression is expanded in a Taylor series, we get 1 y)_1(1 1) ES 1-14(442)(Z)-2 (142274) 4... rename 1e(Fa) 8) a(S) | For positions very close to the wall, this can be approximated by max i + (2) Inserting the expression for the maximum velocity, we get -(228)" ¥ which simplifies to Eq. 12B.4-1 for a Newtonian fluid. Therefore the solution in Problem 12B.4-1 may be taken over for a power-law fluid by replacing B= uk/p*C,g6 = (k/0C, )(u/pg6) (ust below Eq. 12B.4-3) by B=(k/pC,)(m/pg6)"". The quantity 6 enters into Eq. 12B.4-8 via the variable 7, and explicitly in Eq. 12B.4- 9. j1-28 12D.1 Unsteady-state heating of a slab (Laplace-transform method) a, We take the Laplace transform of Eq. 12.1-14 along with the initial conditions and boundary conditions given in Eqs. 12.1-15 and 16. This gives: po-1=55 with @(41)=0 This second-order, nonhomogeneous, ordinary differential equation can be solved by getting the complementary function (from Eq. C.1- 4a) and the particular integral (by trial and error); this gives 5 =C,cosh\pn+C, sinh jpn +t P When the constants are evaluated using the boundary conditions, we find that osh pn p peoshp Taking the inverse Laplace transform, we get anh peoshfp W215 lll The inversion was performed by using Eq. (40) on p. 259 of Erdély, W. Magnus, F. Oberhettinger, and F. G. Tricomi Tables of Integral Transforms, McGraw-Hill, New York (1954), with @ set equal to zero. Unfortunately, this formula has two misprints in it: the x in the denominator of the first term should be /, and the factor 2p in front of the summation sign should be 2”. The inversion can also be performed by using the complex inversion theorem, as explained in Carslaw and Jaeger (see reference at the bottom of p. 403). 12-29 b. The expression for the transformed function © (in (a) can also be written in terms of exponentials. Then we expand part of the denominator in a binomial expansion to get: oP 4 ern ) 2D _LSayeremrm 1 gyre vrenten) P Pe aX ) P ae’ +e" eF-) 4 @ FO) 5-2) "2m P at ad The inverse Laplace transform is then T,-T _ Qn+1+n Os SF -1-S(-1J"erk (-)"erfc 24 Te ro) ore at -X( eraria This can be rewritten in terms of the dimensionless temperature of Example 12.1-1 thus: T-T, Qn+1-n Qn+1+n Do o 1 fc— -1)" fo = =X Yorke +3) orf Here, the dimensionless temperature ® has been introduced to avoid confusion with the dimensionless temperature @ used in Example 12.1-2. c. To compare the results of Examples 12.1-1 and 2, we replace the position variable y of Example 12.1-1 by x (so that it will not be confused with the y of Exammple 12.1-2). Then the first few terms of the final result in (b) are, written in terms of x =b—y, are: rT rf (wy) L1G) q a \4at/b? vent OO) +t G0)] w4at/o? f4at/0? the subsequent terms being smaller. =erfe {2-30 12D.2 The Graetz-Nusselt Problem a. The partial differential equation to be solved is ermal (5) ESF) When this is put in nondimensional form, using the dimensionless quantities defined in Eq. 12D.2-1, the following equation is obtained for the temperature distribution @(2,¢): & 06)2-73( 6% aC EE 2) with 6(§,0)=1, @(1,¢)=0, and = We now try the method of separation of variables with the dimensionless temperature given by (E,¢)= X(E)Z(¢). This gives, after division by XZ LdZ_ Ad (dx Zdl XP Ed” dé Since the left side is a function of ¢ alone, and the right side is a function of & alone, both sides have to equal a constant. We choose this constant to be — 6”. Hence we get two ordinary differential equations: dZ acm ee or Z(¢)- _R2, O(E,£)= LAX, (E)erl BE) with A; = fxeig The expression for A, is obtained by using the requirement that @(,0)=1, and making use of the orthogonality relations for the eigenfunctions. b. We write for the Newtonian fluid 0(€)= 0, /(2,) =P, max(1- €?)/(v,) = 2(1- ?) so that Eq. 12D.2-3 becomes: d 2B se) +200 (1-€)xX,=0 with X(1)=0 and X’(0)=0 This second-order equation will have two solutions, but one of them will become infinite at €=0 and is therefore unacceptable. The remaining solution we write as a power series X(6)= Sbyé! () When this is substituted into the differential equation, we get 5 jPojg? + 267(1- 2) Sng! = jn io By collecting equal powers of € we get the recursion relation for the coefficients: by = (26? /7)(b, 4-8 “) \2-3t. The Bg are arbitrarily chosen to be unity, and the higher coefficients are obtained from the equation immediately above. The values of 6? are then obtained from the two equations marked with (*), along with the boundary condition at £ = 1, which requires that 3b, =0. pe This involves solving an infinite order algebraic equation by trial and error--clearly a tedious process. Methods are, however, available to get the eigenvalues ? for small and large values of i. For small values there is the method of Stodola and Vianello, and for large values there is the WKB (Wentzel-Kramers-Brioullin) method. The following eigenvalues have been obtained: i 1 2 3 4 5 6 2B? 7.314 44.61 113.9 215.1 348.4 513.8 See B. C. Lyche and R. B. Bird, Chem. Eng. Sci., 6, 35-41 (1956), for the first three values both for Newtonian and power-law fluids; the higher values are obtained from the WKB method as 26? = (4i-4)*. Further discussions and literature references for non-Newtonian flow may be found in the 1st edition (Chapter 5) and the 2nd edition (Chapter 6) of R. B. Bird, R. C. Armstrong, and O. Hassager, Dynamics of Polymeric Liquids, Vol. 1, Fluid Mechanics (published by Wiley-Interscience). 12-33 12D.3 The Graetz-Nusselt problem (asymptotic solution for large z) a. Using the definition of the dimensionless temperature as given in Problem 12D.2-2, we can write el = 22 Mase . k(T, -To) a0| 9 = lew = Roe R dl, RO, a |,., b. For very large z, _ yu PAS) Age exp(-B?S) yi a lea Furthermore, from the definition of the bulk temperature in Eq. 10.8- 33 we find using Eqs. 12D.2-2 and 3 *9OEd = 1 PE 2h yosts=95 Aol Ae oxses 9 =-23.4,exp(-8' Nae Ge ax, was aX, =-2Aen(-oie) (5) = 1dX| _ poe) 1 aX| = EAB) ar ae 2A, exp(-BiS) a7 Get ea the very last expression being valid for large z. When these results are substituted into the result in (a), we get to=-E(t,~To){ 2) pp MET) This is the same as Eq. 12D.3-3 in the text. (see also p. 217 of R. B. Bird, R. C. Armstrong, and O. Hassager, Dynamics of Polymeric Liquids, Vol. 1, Fluid Dynamics, Wiley-Interscience, New York, 2nd edition (1987)). 12-34 12D.4 The Graetz-Nusselt problem (asymptotic expression for small z) a, When Eq. 10.8-12 is written in terms of the dimensionless variables © =(T, -T)/(T, ~Tp), €=1/R, and £=2/R, it becomes 08 _ald{,7@ tomelt-F 32" Reel £38) Next we introduce the three assumptions used for small z in Example 12.2-2, as well as the dimensionless quantities in Eq. 124-1.) D This gives for the partial differential equation for @(o,€) the following problem statement: 20 _ #0 Nose gt with @(¢,0)=0, 0(0,6)=1, O(~,¢)=0 We use the method of combination of independent variables, by introducing the new independent variable: n = (No*/9¢)¥. Then the transformation of the derivatives proceeds as follows: 2) - (41) =#0(No? (Hewe-2 a), anat), dn 9 3)” Sdn (2) -2(2) -# NYY 40.0 na (20) -£9(a) do), dn\do), dn\9t) ~~ dno a0), dn? \o, When these substitutions are made into the partial differential equation in dimensionless form, we get (after some simplification) the following ordinary differential equation and boundary conditions for @(n) ; #8 5p ip igo (With (0)=1and @(<)=0 12-35 To solve this equation, we set (1) = d@/dn, and obtain a separable first-order differential equation for ®: do ' i ae aoe -37°@ with solution in” © =Cyexp(-n°) This first-order separable equation can be integrated to give 0=C, fl exp(-7 i +C, The lower limit of the integral has been chosen arbitrarily. Application of the boundary conditions enables us to determine the constants of integration. The final solution is then Slexp(—7 7 _ freel- where Eq. C.4-3 has been used to write the denominator integral in terms of a gamma function. The integral in the numerator cannot be evaluated analytically. b. To get the wall heat flux, we use Fourier's law of heat conduction: @=1- ar k- k Gen =-F5,| =+ jan RI ~ xl -7( 52) Shere (aa) x-™5e} Sal Into this expression we substitute the final result of (a), to obtain (with the help of the Leibniz formula for differentiating an integral, given in Eq. C.3-2) a 12-36 ak, 1_(4(v,)R R)* =p Mgorg| ag] age k 1_{ Dv,)p CH D =+—(T, -T, 2. ——.. +K(r, set aces c. In (a) we can write the velocity profile for the Newtonian fluid (or any generalized Newtonian fluid) as with (1)=0 — (no-slip condition) That is, for the Newtonian fluid $(€)=2(1-£?). Then when we switched to the dimensionless coordinate ¢ measured from the wall, the velocity profile can been written as 2, =(0,)y(e) with y(0)=0 —(no-slip condition) Then, the velocity profile in the immediate vicinity of the wall is (0, )¢"(o+ and, for the Newtonian fluid, 2, = (0, )W'(O)o+ 2, =-(0,)N(d/ag)2(1- 2], = (0, + Hence, the above solution in (a) can be modified for the generalized Newtonian fluid by replacing N = 4(v,)R/ot by w-(-# ea aé|4) a 12-37 12D.5 The Graetz problem for flow between parallel plates a. Let the flow take place in the positive z direction between plates at x= +B. Then the partial differential boundary equation for T(x,z) and the boundary conditions are: eT or 06,0, wz with T(x,0)=T,, T(4B,z)=Tp We can, if we wish, replace the boundary condition at x=—B by a boundary condition JT/dx=0 at x=0, because of the fact that we know that the distribution of temperature about the center plane will be symmetric. Then we solve the problem for positive x only, knowing that the full problem must be symmetric about the plane x=0. We now introduce the dimensionless variables T-T, T,-T) AC) es (E)= 0.) g Then the problem may be reformulated as @(é,0)= 2 oee=Se , =0 FE leeo with @(é,0)=1, (1,6) =0 We anticipate that the method of separation of variables will be appropriate, and write @(£,¢)= X(£)Z(¢). When this is substituted into the equation above, we get (after division by XZ) 1dZ_ 1 aX zag xp ag? where we have introduced the separation constant ~B?. Hence we get two ordinary differential equations az ae BZ whence — Z(£) exp(-6°) 12-38 @X ory . , eth Ox=0 with X(1)=0 and X’(0)=0 Since this last equation with its boundary conditions is a boundary- value problem of the Sturm-Liouville type, there will be an infinite number of values of the eigenvalues 6? and an infinite number of eigenfunctions X,(&). Furthermore, it is known that the eigen- functions are orthogonal on the range (0,1) with respect to the weight function 9(é). Therefore, the dimensionless temperature profiles must have the form: Lxi(S)o(E)4é x? (E)ae The expression for the A; is obtained by using the requirement that @(é,0)=1 and using the orthogonality relations for the eigen- functions X,(é). It remains to find the eigenfunctions X,(€) and the eigen- values f?. This has to be done by solving the equation for X;(£) by a power-series technique. It is a tedious process. The results are given in the Handbook of Heat Transfer, by Rohsenow, Hartnett, and Cho, cited in a footnote in Table 12D.2 on p. 404. b. In the limit of very large ¢, only one term in the summation in (*) is needed, that is, we need only X,(&) and £7. To get the heat flux into the wall at large distances downstream, we need to find 0(6.£)=SAx%(ZJerp(-BH) (with 4, = rere) uuien)) (00/28). a where 0,22=Te B e, ers lea+B and T, is the bulk fluid temperature. For very large ¢, (2-39 ax, “Ae exp(-B7¢) The bulk temperature is, for very large ¢ obtained as follows: fest fied =-DA exp(-676)f, O(¢)=* =EA exp(-B? o)f ox: dé \f 1 aX, BE ae 1 dx, BF dE |, 1 dX, BP dé |, dé = “EA exp(-B7¢) =-3A exp(-B?6) a5 Substituting the last two results into the expression for the heat flux, we find In terms of the Nusselt number, this result may be written as = AnB___4q0B _ ER) 4p? =7.541 which is the result cited in Eq, (F) in Table 14.2-2. c. For the limiting case of very small values of z, we start with the same equation used in part (a) for T(x,2): aT _ aT : v5 =a5> with T(x,0)=T,, T(£B,z)=Tp Since we are concerned only with the temperature change in the immediate vicinity of the wall, it is convenient to change to a new coordinate y= B—x and to expand the velocity in powers of y thus: where we have used the result from 2B.3(b); this truncated expression is valid in the vicinity of the wall for small values of z. The energy equation for T(y,z) can then be written as rT iar |. (0, = aS with Ty,0)=Ty, 10,2) =To, To2)=T, The last boundary condition is sufficiently good for small values of z, since taking the center plane as being at an infinite distance from the wall will not change the temperature distribution significantly. We now introduce the following dimensionless quantities: Then the differential equation for @(7,¢) and the boundary condi- tions are: : Nnge= Fo with @(n,0)=0, 0(0,.)=1, 0f~,5)=0 We now seek a solution by the method of combination of variables, using the combined variable z = (Nn°/9£)¥°. This is possible since the first and third boundary conditions both state that the dimensionless temperature is zero. We are then led to the ordinary differential equation (for the details, see the solution to Problem 12D.4) £8 5288 _ ge Gyo with @(0)=1 and O(~)=0 for which the solution is 12-41 1 pe ~ Fy or# ke To get the wall heat flux, we use Fourier’s law ar 4 HTi=To)( NY” 20) B 9f) ox =k w=ks, =4k(T=To) yo B a neo =i) Morey" NE) oy The above result can be put in the form of a Nusselt number as follows: ly=0 Nua MB. 408, 4008 2 eee 3 ys (0) RE) oh 4 ((v,)B\” “arr ae) This is in agreement with Eq. (C) of Table 14.2-2. The replacement of the bulk temperature by the entrance temperature in the first line is appropriate for the entry region, inasmuch as heat has had the opportunity to penetrate only a negligible amount in that region. (2-42 12D.6 The constant wall heat flux problem for parallel plates a. Application of §10.8 to parallel-plate system For the laminar flow in the z direction between parallel plates at x = +B, the differential equation is rn (5) Ee We now introduce the dimensionless variables _*x a kz Meu T) ch ae o(é,.)= When the differential equation is multiplied by B/qy, the above differential equation can be rewritten as 2 _ #0 Ae + (1-8)5e= Se ) with the boundary conditions: = _ B.C. 1: at €=0, 567° B.C.2: at €=41, Ba B.C.3: at [=0, e=0 We now seek a solution valid for large downstream distance, and try a solution of the form (8,5) =Coo + ¥(S) *) Since this function cannot satisfy B. C. 3, we replace the latter by B. Condition 4: atany planez, — 2zWay=[" ['3oC,(T-T, o.dxdy (2-43 or, in dimensionless variables atany plane £, ¢=fto(E,¢)(1- €?)dé When the trial function in (*) is substituted into (*), we get ay (reo g Integration of this equation gives Fen GlE-$E) +6, >) Further integration gives W=C,(46?-HE*)+C,E+C, so that (8,6) = Col +#Cy ($6? - HE) +E +O, B.C. 1 leads to C, =0. Furthermore, B. C. 2, with the help of (***), gives Cy =}. Finally Condition 4 leads to: C= fi[3o+3(be? - Be") +c, 1-2 )ae from which we get C, =~ 2%. Therefore, finally O(E.0)= 2542 -284)-& Then the bulk temperature as a function of downstream distance is (2-44 Klt(.2)-T, fp.(x)ax _ 908 LOCE-DL- é?)ae Sree(x)ax ko f-e)ag T,-Ty= (35) and the wall temperature as a function of downstream distance is B Bo T)—T, = ME (GE +40-H-%)= BEG C+H) Therefore or Nu=—4%8_ _ 140 K(T)-T,) 17 in agreement with Eq. (L) of Table 14.2-2 (note the definition of Nu in the table title!). b. For very small values of z the velocity distribution the linear portion very near the wall is the only part of the distribution that is important; if we let y be the distance from the wall at x=B, we have Aside from the replacement of R by B, these equations are identical to Eqs. 12.2-13 and 14, and they have the same boundary conditions. Therefore, the remainder of Example 12.2-2 is valid for the slit flow (with the replacement of R by B). When 1 =0, we also have z= 0, and Eq. 12.2-24 gives for the difference between the wall and bulk temperatures 12-45 Ty-T, _VOA a (a0B/k) TB Wace? * Tater B inasmuch as the bulk temperature is virtually the same as the entrance temperature, the heat having penetrated to only a very small distance into the fluid. Then the Nusselt number is Nua 4HB 4908 _ 40(3) [(v.)B* ~—k K(T)-T,;) V3 Vaz in agreement with Eq. (H) of Table 14.2-2. c. The slit analogs of Eqs. 10.8-12 and 19 are a x) lar _ ar a _ #0 16.0 1-(3) Sookgz or (i- e5e7 ad In the second equation we have introduced the dimensionless variables af-h © 40B/k ce z 7 2 Pomad™ = B From §10.8 we know that the asymptotic solution for large downstream distances is 0.(E,0)= 86448 -284)-& The complete solution is assumed to be of the following form (6,0)=0..(6,6)-@,(8,5) Substituting this into the partial differential equation above tells us that the time-decaying function ©,(,¢) must satisfy the equation 12-46 20, _ 70, (8S ae with the following boundary conditions: 2, B.C. 1: t E=0, ts at €=0, 7 0 2, B.C.2: at €=1, 4-0 até 3 B.C.3: at {=0, 0, =@.(E,0) We try a solution by the method of separation of variables, letting ©,(£,¢)=X(€)Z(¢). The functions X and Z must then satisfy the following ordinary differential equations a2 227 2(1-€2)X=0 oy de age where -c? is the separation constant. We then expect to have a complete solution of the form O(E,£)=0.(E.5)- $B, exp(-c2) (8) where 1 p, = h9=(E-O1Xe(S)I(t- 5?) 48 kis pe\e/a_e2\qe a. Therefore, the problem is reduced to one of finding the eigenfunctions X,(&) of (****), and then getting the eigenvalues ¢,, by applying the boundary condition at the wall € =1. The Nusselt number is shown in Fig. 14.2-1 over the entire range, including the limiting cases given in parts (a) and (b). \2-47 12D.7 Asymptotic solution for small z for laminar tube flow with constant heat flux Exchange the order of integration in Eq. 12D.7-1, recognizing that the area of integration is a triangular area bounded by the lines Y= y, ¥=Z, and Y=. Then the inner integration can be performed analytically: 0(n,a)=99R fo waz = wr sy fale Pale =n 2 a Toh z * Fazlaz =491 re [%-z} a; Next rewrite the result as @(7,2) ete ee rath Wek dy 3x Ze Paz - [He @)| The first integration can be performed analytically, the second integral can be put in the form of a (complete) gamma function, and the third integral can be written as an incomplete gamma function. In the second integral, we set t= %°, so that dt=3%7d% and dy= 4¢8dt. Then a [He (42 Jat = 4fC ee etdt = 41 (2) A similar derivation can be performed for the indefinite integral. Then we are led directly to Eq, 12.2-24. 12-48 12D.8 Forced conduction heat transfer from a flat plate (thermal boundary layer extends beyond the momentum boundary layer) Evaluate the left side of Eq. 12.4-5 by using Eq. 12.4-8: ,2| _2K(T.-To) oy or Next, evaluate the the integral on the right side of Eq. 12.4-5: 5 OC é T-T, ots arte —ravlf 2 1- Ea Jom The dimensionless integral on the right side can now be split into two parts to perform the integration, since the equation for the velocity profile changes before we get to the upper limit: 6 (2rd -2m3.4° + nfA*\(1- 2m, +20} - nf ny 1 +fy4(D(1- 2m +207 ~ nt ny = (GAT - BA? + a4 - ZA) +(B-A7t +47 -ha4 +1 a5) = hh At + RAP - At + har = F(A) We can now write Eq. 12.4-5 as 2k(T.. -To) or 2a p€0.(T.-To)5rF(A) o 2 = F(A)é, 28r dx Integration of this equation then gives 5; = «/4(ax/v..)/F(A). The result in Eq. 12.4-12 is valid for A21, so that the ratio of the two boundary-layer thicknesses is fe Onee,|A( Gx/Us) a o7u (Goes 37 ane F(A) “aSS) BREA) When this is squared, we get Eq. 12D.8-1. \a-4y, 13B.1 Wall heat flux for turbulent flow in tubes (approximate) The integral in Eq. 13.3-6 can be evaluated by making a change of variable. Let Then Eq. 13.3-6 becomes (after setting the upper limit equal to infinity): -aa(_?» Pr (es) Go Toxo 7HTo Tr) The integral may be found in a table of integrals, where we find dx n a _2Qn C1+x3 3si Jr Aa) Hi Hence Eq. 133-6 becomes -p(_% )7" 20 = fot (x) aya MTo-Te) Then the dimensionless wall heat flux is ie et D 14.5v, This now has to be written in terms of dimensionless groups: HRT wate Bd) oe Paar where Eq. 6.1-4a has been used in order to introduce the friction factor. Thus we have obtained Eq. 13.3-7. GD = 33 / K(To-Tg) 2% [3-\ 13B.2 Wall heat flux for turbulent flow in tubes a. The assumptions are, for the asymptotic solution discussed here (the asymptotic form of the solution is introduced in Eq. 13.4- 10): Fully developed turbulent flow Axial heat conduction is presumed to be negligible with respect to axial heat convection iii, The turbulent Prandtl number can be taken to be unity (in developing Eq. 13.4-20) iv. The modified van Driest equaion of Eq. 5.4-7 is used to describe the turbulent velocity profile (in development of Eq. 13.4-20) b. To get the constant C, in Eq. 13.4-10, we use B. C. 2. First we have to get the derivative d0/dé using the Leibniz formula #@ 5 ,¢ 18) de fis (a"/a)] Then at €=1 (the tube wall), the turbulent thermal diffusivity vanishes, and the dimensionless temperature gradient is unity, so that we get the result in Eq. 13.4-14: 1(1) afi +0) or Cy=[1(1)]" ‘The dimensionless wall temperature is obtained from setting €=1 in Eq. 134-12 (and setting C, = 0) y= CoS +Cof qs aera rays The dimensionless bulk temperature is obtained using the definition in Eq. 10.8-33, starting with Eq. 134-12 (and setting C, =0): fooeae tale 18) ae CoS + Cy To? i F(a"7a)] (2a) at fe +C, Then we get the dimensionless temperature difference: ) 1) cya tora Then we exchange the order of integration to get the second line of Eq. 13.4-15: -O,= 16) De “Cohen ray Lo 18) ta ¢ F “raph ler Frs[arnaforee- e845 JE a 1(é) mbar enragt be aia eral 18) ig ie Ooh (aay bet aay] “Taye qa 7 The first two terms cancel, and the third term gives Eq. 13.4-16, when the expression for Cy in Eq. 13.4-14 is used. c. It is not necessary to find the constant C, in this problem because we are interested only in the dimensionless temperature difference. In obtaining this difference, the integration constant C, cancels out. If, however, we want to find the complete temperature profile, then we need an expression for C,. 133 13C.1 Constant wall heat flux for turbulent flow between two parallel walls a. For slit flow, the analog of Eq. 13.4-6 is 92.2 a) 0 ad “al 23] with the boundary conditions J6/9E=0 at £=0, 00/a =1 at £=1, and © =0 at £=0. Here the dimensionless variables are defined as: 90B/k’ x z * 9,7. mmeB? [ke We now try a solution of the form @(E,¢) = Co +'¥(E), which should be asymptotically correct at distances far down the tube. This leads then directly to the following differential equation ove mars zl ard A first integration then gives a) ay fy (» 7 le cl o(E)aé +C, =CoI(E) where we have introduced the abbreviation (£)=[* 9(E)4Z and the constant of integration has been set equal to zero since 0@/dé = 0 at t he tube axis. Then, since @/dE =1 and a =0 at the tube wall, we find that C, = 1/J(1). Thus the dimensionless temperature profile is He) ) in which it is understood that @” is a function of &. Next we form O(E,6)=Cob + Col E+ Cy 13-4 beta] Sl Hhaye is °14(a/a Bhotg)aé Next we exchange the order of integration in the numerator and make use of the definition of J in the denominator to find J(E) 20-O5= Cols Taityay*® “hate ite ile &)as aE The inner integral of the second term may be written as J(1)- J(E), and the J(1) contribution to the second term of @,-®, exactly cancels the first term. The final expression for ©, - ©, is thus oe ver ® oo Feap hr (a fay : Then, since 49B/k(Ty -T,) = /(@p - ©;), Eq. 13C.1-1 follows. b. For laminar Newtonian flow, ¢=1-? and a" = 0; then J(E)=[$(1-E?)a& = €-4€° and J(1)=3. Furthermore SUP ag = fle? 364 +4 6)as = Hence, finally, for laminar Newtonian flow (using the mean hydraulic radius in the expression for the Nusselt number) q(4B)__ 140 eek re=T sez (see the entry in Table 14.2-2) For plug flow, J(£)= ‘a =, J(1)=1, and [i[s(2) dé = [Leaé =3. Hence we finally get Nu = 12, which is the entry in Table 14.2-2. bs 13D.1 The temperature profile for turbulent flow in a tube a. Condition #4 is ['@(é,¢)6(E)édé = ¢. Into this, we now substitute the expression for dimensionless temperature of Eq. 13.4- 12, in order to get C,. We do this term by term: Istterm: [CoCO(E)EdE = CoC o(E)EdE = (Eq. 13.4-14 was used) 4th term: [)C,@EdE = C,I(1) < WE) 2nd term: ef Fra (a/a)} (" a eho EME Co Can he Bio areyl* )éds ae 1) = Of: Hila Ta Arata" o( eas - fF o()eae Ae -p Og 1p “haar (erTaye Taye +(a%Va)| Substitution into the 4th condition above and solving for C, gives ¢-pl@coL OF ep [Et] ae in( (2"/a)] Cepe(a"/e)] 7) which is the same as Eq. 13D.1-1. b, For a Newtonian fluid 1(é)= [*(1- E?)EdE = $4? -4€4and 1(1) = 4. Then [1(€)/I(1)]= 26? - &*, [1(E y/uayP =4et— aehaes, and aE a = fp(46° ~ 46° + 7 Jab - (26 -E° de =-% 14A.1 Average heat transfer coefficients. ‘The total heat transfer rate is: Q= wl, (Toa — Ti) = (10,000 Ibm /hr)(0.6 Btu/lby-F)(200 ~ 100 F) = 600,000 Btu/hr ‘The total inside surface area of the tubes is: A= wDLoe = (n)(1.00 ~ 2 x 0.065 in)(1/12 ft/in)(300 ft) = 68.3 ‘The various temperature differences between the inner tube surfaces and the oil are: (1) -%): = 213-100 = 113 F (To — Tha = (113 + 13)/2 = 63 F (Zo — Th)in = (113 — 13)/1n(113/13) = 46.2 F Insertion of these values into Eqs. 14.1-2,3,4 then gives the heat transfer coefficients: hy = (600, 000)/(68.3 x 113) = 78 Btu/br-ft?-F hg = (600, 000)/(68.3 x 63) = 139 Btu/hr-ft?-F ‘hin = (600, 000)/(68.3 x 46.2) = 190 Btu/hr-ft?-F iat 14A.2 Heat transfer in laminar tube flow. (a) The Prandtl number, based on the property values given, is Ibm /ft-hr) ) _ (0.49 Btu/lbm-F)( (0.0825 Btu/hn a (b) The Reynolds umber is Div)p _ _4w_ “w *Dp _ (4)(100 Thy» /br) © (x)(1/12 ft)(1.42 Ib, /ft-hr) = 1076 Re= (c) From Fig. 14.3-2, at L/D = (20 ft)/(1/12 ft) = 240, we read a (a—t).(B). Cou (2) ean (To-Te)in \40. k Ho - Now, for uniform Ty, "i @ = a) T—Ta, In (FB) = (0.0028)(4 x 240)(8.43)-2/*(1.0) = 0.649, Ta (Tso -Ta) _ -Te Hence, for this problem, giving = exp(—0.649) = 0.523 Insertion of Ty = 215°F and Ty = 100°F gives Ty = 215 — 0.523(215 — 100) = 155°F 4-2 14A.3 Effect of flow rate on exit temperature from a heat exchanger. (a) From the solution of Problem 144.2 we find that Re= 10.76w and that Th-Ta _,, (—0.040¥ TT "? \ 0.0028 ) = exp(-232V) in which w is the mass ow rate in Iby/hr and Y is the ordinate of Fig. 14.3-2 at the prevailing Reynolds number. The exit bulk temperature is then The = Ta + (To — Tn) (1 — exp(-232Y)] (b) The total heat flow through the tube wall is Q = wlp(The — Tar) Caleulations for (a) and (c) are summarized below: w, Re Y 1-6” TT, Thay Q, lbp /hr °F °F Btu/br 100 1076 0.0028, 0.478 54.9 154.9 2690 200 2152 0.00185, 0.349 40.1 140.1 3930 400 4304 0.0036 0.566 65.1 165.1 12760 800 8608 0.0040, 0.605 69.5 169.5, 27260 1600 17216 0.0037 0.576 66.3 166.3 51950 3200 34432 0.0033, 0.535 61.5 161.5 96460 143 14A.4 Local heat transfer coefficient for turbulent forced convection in a tube. Fig. 14.3-2 requires the viscosity values 1s at T, and io at Ty. Interpolation in Table 1.1-1, or on page 6-3 of CRC Handbook of Chemistry and Physics, 81st Edition (2000-2001) gives pp = 1.13 mPa-S at T, = 60°F= 15.56°C, and pip = 0.398 mPa-s at Ty = 160°F= 71.11°C, whence (4/10) = 2.84. The other phys- ical properties in Fig. 14.3-2, including y in the Reynolds and Prandtl numbers, are evaluated at the “film temperature” Ty = (To + Th)/2 = 110°F= 43.33°C, giving 1 = 1.489 lbm/hr-ft, C, = 4.1792 J/g K = 0.99885 Btu/Ibm-F, and k = 0.6348 W/m-K = 0.36679 Btu/hr-ft-F. Then the Prandtl number at Ty is (0.99885 Btu/Iby-F)(1.489 Btu/hr-ft-F) = 0.36679 Btu/hr-ft-F aoe and the Reynolds number calculated at Ty is DG _ 4w Re = —— = —_ RDB 4X (15,000 Ibm/bt) a 79 y x98 © F(2/12 f)(1.489 Ibm /hr-ft) From Fig. 14.3-2 at this value of Re, we get the ordinate expression ng (Cap yy pia ( Set a) = 0.0028 (4444-1) GG \ k Ho in which hj, can be regarded also as hige according to the analysis in Problem 14B.5, and G = 4w/rD? = (4)(15, 000 Ibm /hr)/(2/12 ft)? = 6.88 x 10° lbp, /hr-ft? Insertion of the foregoing results into Eq. 14A.4-1 then gives A \ AS pS 40a hice = 0.00286,G ( SP ay k Ho = 0.0028(0.99885 Btu/Ibm-F)(6.88 x 10° Ibm /hr-ft?)(4.05)~?/9(2,84)+01# =7.8 x 10? Btu/hr-ft?-F as the asymptotic value of the local heat transfer coefficient, and Go = hive(Ts ~ To) = (7.8 x 10°)(60 — 160) cL -7.8 x 10* as the radial heat flux at the inner wall of the pipe. \4-4 14A.5 Heat transfer from condensing vapors. (a) The boundaries of the condensate layer are at Ty = 190°F and Ty = 212°F; thus the film temperature Ty is (190 + 212)/2 = 201°F. The physical properties at this temperature are well approximated by the values at 200°F, given in Ex. 14.7-1: Alfyay = 978 Btu/lbm k= 0.393 Btu/hr-f-F p= 60.1 Ibm /it® 738 Tb, fart B= ‘The resulting abscissa for Fig. 14.7-2 is: kpPl3g'l3(Ty — To) 157 AHfvsp _ (0.393 Btu/hr-ft-F)(60.1 Iban /ft?)?/3(4.17 x 108 ft/hr?)!/3(22 F)(1.0 ft) (0.738 Ibm /ft-hr)*/3(078 Btu/Tbm) = 168 Btu!" hr-!-2/3+5/2g,1-241/94145/3F-141, dimensionless This value falls in the laminar region of Fig. 14.7-2. Extrapolation of the laminar line with a slope of 3/4, consistent with Eq. 14.7-5, gives Dm = 170(0.168)"/4 = 45 The heat transfer rate, neglecting subcooling, is Q = DIA gay = 4(1/12 ft)(45 X 0.738 Ibm /ft-bt)(978 Biu/lbm) = 8400 Btu/hr. A similar result is obtainable from Bq. 14.7-5, once the flow is known to be laminar. (b) Comparison of Eqs. 14.7-5 and 6 gives, for laminar condensate flow: Hence, if the tube were horizontal the heat transfer rate would be: Qhor. = (8400)(0.725/0.943)(12)/* = 12,000 Btu/hr The assumption of laminar condensate flow on the horizontal tube is clearly rea- sonable, given the result of (a) and the still smaller value of I'/u in (b). \4-5 14A.6 Forced-convection heat transfer from an isolated sphere. (a) The physical properties of air at 1 atm and Ty = }(To + Too) 65.56°C= 338.7K are: pM/RTy 0.02023 ep 1.008 J/gK k = 26.9 x 10"? W/m-K from CRC Handbook 2000-2001, p. 6-185 042 x 107° g/cm? .023 x 10~* g/cm-s, from Table 1.1-2 ‘The Reynolds and Prandtl numbers are Re = Dose # _ (2.54 cm)(100 x 12 x 2.54 em/s)(1.042 x 107* g/em' _ 2.023 x 10-* g/em-s = 3.99 x 10* Pr k (1.008 W-s/g-K)(2.023 x 10~* g/em:s) 26.9 x 10-5 W/em-K = 0.703 Substitution of these values into Eq. 14.4-5 gives Num = 2 + 0.60(3.99 x 10*)!/?(0.703)'/* = 108.6 Hence, hy = 108.6k/D = 108.6(26.9 x 10~* W/em-K)/(2.54 em) 0.01150 W/cm?-K and the convective heat loss rate is Q = 2D? hm(To ~ Too) (2.54? em?)(0.01150 W/em?-K)({100/1.8] K) = 12.9 W = 3.1 cal/s according to Eq. 14.45. The radiative lo 150°F= 1.008 W-s/g-K from CRC Handbook 2000-2001, pp. 6-1,6-2. s about 1.0 W for a perfectly black sphere in a large enclosure with walls at 100°F, and can be estimated by the methods of §16.5. 14-6 (b) For Eq. 14.4.6, we need io = 0.02144 ep at T) = 200°F= 93.3°C and the following property values at Ta = 100°F= 37.8°C= 310.9 K: Hoo = 0.01898 ep = 1.898 x 10~* g/em-s p= pM/RT.. = 1.136 x 10-° g/em* 1.007 J/g-K = 1.007 W-s/g-K = 27.0 x 107? mW/m-K = 27.0 x 107° W/em-K » ‘The resulting values of Re and Pr calculated at the upstream state are Re = (2:54 m)(100 x 12 x 2.54 em/s)(1.136 x 10-* g/em*) 1.898 x 10-4 g/emss = 4.63 x 10* pra (1.007 W-s/g-K)(1.898 x 10" g/cs 27.0 x 10-5 WjemK = 0.708 Substitution of these values into Eq. 14.4-6 gives 1A Num = 24 (0.4 Re? 40.06 Re?/*) Pr? (tz) = 2+ 138.1 = 140.1 whence 40.1kao/D = 140.1(27.0 x 10-* W/em-K)/(2.54 em) .0149 W/em?-K and the convective heat loss rate is Q= 2D? hm(To — Too) = (2.54 cm)*(0.0149 W/em?-K)({100/1.8] K) = 16.8 W = 4.0 cal/s according to Eq. 14.4-6. This result is believed to be more accurate than that found in (a). \4-7 14A.7 Free-convection heat transfer from an isolated sphere. For the conditions of this problem, the thermal expansion coefficient 6 = 1/Ty is (1/338.7 K), and the other physical properties are the same as in part (a) of Problem 14A.6. (Note that, for the correlations in §14.6, 8 and p are evaluated at Ty rather than T for calculation of Gr.) Then 229A ‘ GPre ° e a) (%) # k _ (2.54 em)*(0.001042 g/cm-s)?(980.7 cm/s?)(100/[1.8 x 338.7]) ~ (2.023 x 10-# g/em:s)? (0.703) = 4,92 x 10* Eq. 14.6-4 gives 0.878 x 0.671 Num = Teor Perea ( GrPr)/* _ ___(0.878)(0.671)_ (1 ¥ (0.492/0.703"7* 7 =6.73 (4.92 x 104)'/* Hence, him = 6.73k/D = (6.73)(26.9 x 10-° W/em-K)/(2.54 cm) = 0.000712 W/em?-K and the convective heat loss rate is Q=7D*hm(To — Too) (2.54 cm)*(0.000712 W/em?-K)([100/1.8] K = 0.80 W = 0.20 cal/s By the methods of §16.5, one can calculate that the rate of heat loss by radiation is of comparable magnitude: 1.0 W for a perfectly black sphere in a large enclosure with walls at 100°F. 14A.8 Heat loss by free convection from a horizontal pipe immersed in a liquid. From the data provided, we find the following values at Ty = 32.3°C: dp p aT 0,99496(33.3 — 31.3) .27 x 10-* K~? = 1.815 x 1074 Fo? p = 0.99496 g/em*)(12 x 2.54 cm/ft)$ /(453.59 g/lbm) = 62.11 Ibm /it® ).9986 cal/g-C = 0.9986 Btu/Ibm-F ).7632 cp = 1.8463 Ibm /hr-ft k= 0.363 Btu/hr-ft-F (0.9986 Btu/Ibm-F)(1.8463 Ibm /hr-ft) 0.363 Btu/br iF oe = 5.08 Hence, pp (054% (62.11 Bo /f9)2 (4.7 x 10" f/hs?)(1.815 x 104 20) 5 og) (1.8463 Tb /hr-ft)? Gr = 1.088 x 10° ‘Then from Eqs. 14.6-1 to 3 and Table 14.6-1 we get 0.671 [1+ (0.492/5.08) 975/475 0.671 1.112 Num = 0. 2 ) (1.088 x 10°)/4 = 0.772 ( ) (181.6) = 84.6 ‘The heat transfer coefficient is then uke 0.363) _ A hm = Nug = 84.6 CE ) = 614 Btu/hr-ft?-F and the rate of convective heat loss per unit length of the pipe is Q _ hmAAT fr = (61-4 Btu/hr-ft?-F)(3.1415)(0.5 £t)(20°F) = 1930 Btu/hr-ft hy DAT i4-9 14B .1 Limiting local Nusselt numbers for plug flow with constant heat flux (Note: Problem 10B.9-1 should be worked prior do doing this problem) a. For circular tubes with plug flow, the dimensionless temperature distribution, the dimensionless wall temperature, and the dimensionless bulk temperature are obtainable from Eq. 10B.9-1 on p. 325: OZ oR? =K(T=T1) _ e-F ) =204+12?-4 (where =F and C= 0 Oo = Olga = 2044 fj Orosadé freo8dé = 2ff@Edg = 2f'(20 + hE - 3) eds = 26 9, Then the difference between the wall temperature and the bulk temperature is K(T)-T,)_1 @,-0, = MT=%) aR 2 Jo and the Nusselt number is pee CHE es k k(T)-T,) in agreement with Eq. (J) on p. 430. Note that, by convention, the Nusselt number for tubes is defined using the diameter rather than the radius, and this definition introduces the factor of 2. b. For the plug flow in a slit of width 2B, we have for the dimensionless temperature, wall temperature, and bulk tempera- ture, all obtainable from the results of part (b) of Problem 10B.9: 14-10 oz x =C+}o0%-1 (where o= 5 and ¢=—%5) = f[Odo=[i(¢+}07-1)do=6 Then the difference between the wall temperature and the bulk temperature is K(T, -T, : o,-0, Mts! . The Nusselt number is then. — (4B) __qo(4B)__ 3 4_ Mus aT) oe in agreement with Eq. (J) on p. 431. Note that the Nusselt number for slits is defined in terms of 4B, and this is the origin of the factor of 4 which appears here (see heading of Tables 14.2-1 and 2, as well as the caption for Figure 14.2-1 on p. 429. 14-41 14B.2 Local overall heat transfer coefficient. Let 0 and 1 denote the inner and outer surfaces of the tube, and hg and hy denote the local heat transfer coefficients on those surfaces at the cross-section where the oil bulk temperature is 150°F. According to the development in §9.6, the temperature drops within a cross-section have the same ratio as the corresponding resistance terms that sum to 1/(roUo) = Toho ‘The numerator on the right is ae (35/1190) * 220 = 0.1452 + 0.0006 = 0.1458 hr-ft-F/Btu in which a thernal conductivity of ko, = 220 Btu/hr-ft-F has been used for copper at T = 190°F, based on Tables 9.1-5 and F.3-5. To calculate the denominator, we use Eq. 14.7-3 for the heat transfer coefficient for filmwise condensation on horizontal tubes. Iteration is required, since the temperature difference across the condensate film is unknown. As a first approximation, we choose T; = 190°F, and use the physical properties at 200°F from Example 14.7-1: Ablyap = 978 Btu/lbm k = 0.303 Btu/hrft-F p= 60.1 Ibm/ft? H = 0.738 Ibm /hr-ft ‘Then Eq. 14.7-3 gives sang ys by hq =oras| Esa) uD(Ta — Tr) 0.393% 6(4.17 x 10*)(978. (0.738)(0.5/12)(Ta — Ti) = 12,500(213 — Ty)"/4 Equating the heat flow through the numerator and denominator resistances gives (Ty ~ 150)/0.1458 = (213 — T1)ry hy = (213 — Ty)*/4(0.5/12)(12, 500) = 0.725 [ or 213 — Ty = 0.0000183(T; — 150)*/* Successive substitutions of T; in the right-hand term give a rapidly converging se~ quence of left-hand values: to the solution: T, = 212.9975, 212.9954, 212.9954, . ‘Thus, the outer-surface temperature of the tubes at this cross-section is 212.9954° F. The temperature drop through the tube wall is 0.0006/0.1458(212.995 — 150) 0.25F. Thus, the thermal resitances of the tube wall and condensate film are unim- portant here, as assumed in Problem 144.1. l4-He 14B.3 The hot-wire anemometer a. The physical properties of interest at p = 1 atm and a film temperature of 335°F are: p=0.0499 Ib,, /ft? ¢, =0.242 Btu/Ib,,°F 4 = 0.0594 Ib,, /ft-hr = 1.64107 Ib,,/ft-s (from Eq. 1.4.14) 5( 1.986 i= (0.2404 5( 2388))(.0410 (from Eq. 9.3-15) Pr = 0.74 (from Eq. 9.3-16) 5.373 x 10 Btu/ft-s°F (0.01/12)(100)(0.0499) _ (1.64x10") 254 Also the Reynolds number is Re = Then Eq. 14.4-8 gives Nu,, = (5.99 + 2.29)(0.905) +0.92(-3.54 + 1062) *°(6.33)(0.905) = 8.010 Then we get the heat transfer coefficient from 010)(5.373 x10) (0.01/12) =186 Btu/ft? -hr°F =0.0516 Btu/ft? -s°F Finally, the heat loss from the wire is Qz=h, AAT =h,,-2DL-(Ty -T..) = (186 299195) c00 70) Btu/hr =10.75 Btu/hr = 3.15 W=10.75 W b. For an approach velocity of 300 ft/s, Re = 762. Equation 14.4-8 gives Nu,, = 14.20, and Q(300)/Q(100) = 14.20/8.010 = 1.77. This is very close to V3 =1.73 from King's relation. 14-12 14B.4 Dimensional analysis. (a) The left-hand member of Eq. 14B.4-1 is expressible in terms of integrals of the dimensionless product function ¥, T as follows: Ta ™ [EFL )- Tho — Ta z=0 ) Here the angle brackets denote cross-sectional averages as in Eqs. 10.8-32 and 33, whereas overlines denote long-term time averages. Thus the averages on the right depend only on Re, Pr, and L/D, when viscous dissipation and radiant energy ab- sorption and emission are neglected in the energy equation. With these assumptions, and the further neglect of axial heat conduction, the quotient (The ~Ts1)/(Zo— Tn) is equal to the time-average of Q/(wC,(To — Ts). (b) The heat transfer coefficients h, and hi, each differ from hy only by the ratio of the corresponding AT definitions given in §14.1. But AT, and ATig are expressible in terms of AT; and AT», so their ratios depend only on Re, Pr, and L/D according to the result in (a). Hence, Nug and Nujp are functions of the same arguments as Nuy, confirming Eqs. 14.3-12 and 13. Equation 14.3-14 requires extension of Eq. A to a variable upper limit Z, giving Tz) = Ta Tea Ty, = ® function of (Re,Pr,?) (B) Then, according to Eq. 14.1-18, htocD _ wCy dln(To — Th) & xDk dz/D) _(» ) Cyn) din(To — Ts) - Co k de din{(T> — Th)/(To — Tr)] dz = 4 RePr Combining this result with Eq. B, we get (for this special case of uniform wall temperature in the heat-exchange section), Nutoe = Nutoc (Re, Pr, (Z)) in agreement with Eq. 14.3-14, \4-13 14B.5 Relation between /,,. and ht, a. We relate the rate of heat transfer across the increment of surface 2Ddz to the decrease in the internal energy within the volume element 42D?: Age ™Ddz)(T, To) =-($2D) oC, (v)dT This is really an application of the d-form of the energy balance discussed in §15.5, and given specifically in Eq. 15.4-4. It is clear from this equation that the kinetic and potential energy changes are being neglected, and that Eq. 15.4-4 has been multiplied through by w. The above equation may now be integrated over a length L of the tube to get, with Ty = a+ BT, Winde=-4DeG (Nf part = ~1D0G Manama 22H ry p)- ale) 7 PLO er ann. a|-In{T,(0)(1-6)~ al} = PE, (L)-To(L)]-InlT4(0)-To(0)}} _ 106, (0) [7,(L)* To(L)]-[T(0)-To(0)] 1-B (T.-Ton =-4Dp6, ple Atel 2) T,(L)-T,(0) = +4 oC, (oT which agrees with Eq. 15B.5-2. b. Equation 14.1-14 can be written as vers iy, = SPPSHD (ToL) THO] eo L To-To)in from which Eq. 14B.5-3 follows at once: le My = zh Iypedz Then, differentiating the integral in Eq. 14B.5-3 using the Leibniz formula in Eq. C.3-2, we get dhy 1 1 Ht = (Lig) + = Moe! dee En) + Me or dh, Icy thin +e which is Eq. 14B.5-4 14-15 14B.6 Heat loss by free convection from a pipe The properties of air at 1 atm and a film temperature of 190°F (or 650° R) are = 0.216 cp .0173 Btu/hr-ft°F 7/650 (°RY? Since the temperature difference is the same in both the original problem (of Example 14.6-1) and the new problem (Problem 14B.6), it suffices to determine the ratio h’,/l, , where the accent indicates the result for the "new" problem. Next we calculate the ratio of the heat-transfer coefficients: J" [1+(0.49yrr)"*]” pw [1+(0.a937pr""]” -( (2 o190 (1298) 0152) 650 0.241 0.0216) \ 1.299 = (1.102)(0.930)(1.000) = 1.025 Thus, in the "new" problem, the heat-transfer rate is only slightly greater than in the original problem. If the thermal conductivity and viscosity had been assumed to vary with temperature by the simple power-law suggested by the simplified kinetic theory of Chapters 9 and 1, then almost no change would have been predicted. 14-16 14D.1 Heat transfer from an oblate ellipsoid of revolution a. From Eq. A.7-13 we get (with Z=sinhé, K=coshé, S=sinn, C=cosn, s=siny, c= cosy) = 3S) =0 a°[(z8c)? + (85s) + (KC) ]= 0° [(Es)° + (KC)’] =a |(K? -1)s? +K?(1-S*)]=0?(K?-S*) +. hy = aVK? i= 3( = [(KCc)* +(KCs)* +(25)"]=«°[(KC)? +(25)"] =a?[K(1-S?)+(K?-1)s*]=02(K?-S?) +h, =aVK?-S? =aKS i= 3(2) = a°[(K5)? + (KSc)*]=«°K°S* iat These results may be used with the expression for surface elements after Eq. A.7-18 to get Spy = @2-\cosh? € — sin? n(cosh & sin n)dndy b, From Eq. A.7-17, we get Laplace's equation as 1 hy hy @ O=Tly (als hg C2 .}n0 er se(oones a 7 since heat is flowing in the € direction only. This equation may be integrated to give © = K, arctan(sinh €) + K. with the boundary conditions: @(£,)=0 and ©(c)=1. This leads to Eq. 14D.1-7. c.In the limit as €, = 0 (a two-sided disk of radius a = R), the result of part (b), namely Eq. 14D.1-7, simplifies to 14-17 2m~arctan(sinhg) _ 2 in-0 @=1- Garctan(sinh é) The normal dimensionless temperature gradient at the surface is then obtained by using Eq. A.7-15, thus: 1 do 1 4(2 ) _ ti inh ig dilgcy RY osin? y dE MINS) 1 2) 1 } = =| ——__.coshé erat 1+sinh? € eo icanty a) (agers Then the total heat loss through both sides of the disk is Q=2)(n-q)dS =-2kf(n-VT)aS =2K(Ty- 7.) ie coat = 2K(To “rj Bast sin ndn=8kR(T -T...) ? cos nsin ndndy The heat transfer coefficient is then hy, = _ 8AR(Ty-T.) _ 8k "AAT (nR?\(T)-T.) aR and the Nusselt number is 14-13 15A.1 Heat transfer in double-pipe heat exchangers. (a) In the absence of heat loss to the surroundings, Eqs. 15.4-7,8 give weCpe(Tea ~ Ter) = —waCpa(Tia ~ Tar) with each flow rate w expressed from plane 1 toward plane 2. Insertion of the data then gives Qe = (5000)(1.00)(Te2 — 60) = —(—10, 000)(0.60)(200 ~ 100) = 600, 000 Btu/hr whence Tez = 60 + 120 = 180°F ‘The log-mean temperature difference is (AT)a = (20 — 40)/1n(20/40) = 28.85F and the required heat exchange area, from Eq. 15.4-15, is _@e__ . _(600,000 Btu/hr) _ To(AT)in (200 Btu/hr-ft?-F)(28.85F) Ao = 104 ft? (b) Eq. 15B.1-2 gives Q_ U,AT,-U2,AT, A In@,AT,/,AT) 5 (350 x 40) 20)/380 x 40)) = 4926 Biu/br-fl? ‘The required heat exchange area is then Q__ 600,000 = = 2 A= Oya = a6 =H (c) The minimum usable flow rate of water to cool the oil to 100°F in coun- terflow is (10, 000)(0.60)(200 — 100) we = = 4286 Ib fh (1-00)(200 — 60) 86 Ibm/! whereas the minimum usable flow rate of water in parallel flow is | __(210,000)(0.60)(200 — 100) _ ee (1.00)(100-60) 15,000 Ibm /hr (d) If parallel flow is used, with w, =15,500 Ibm /hr of water, the outlet water temperature will be Tex = 60 + (10,000)(0.60)(200 — 100)/(15, 500)(1.00) = 98.71°F Then (AT)in = (140—1.29)/In(140/1.29) = 29.6°F and the required heat exchange area is Q____ (10,000)(0.60)(200 — 100) 4= Taty, = (200)(29.6) = 101 ft? 15A.2 Adiabatic flow of natural gas in a pipeline. (a) The density and mass flux at plane 1 are _ PAM _ (100 x 144 Iby/ft?)(16.04 Ibm /Ib-mol) = = 0.2821 Ibm /ft* = RT (1545 x 530 ft-lb /Ib-mol) H G: p10, = (0.2821 Ibm /ft?)(40 ft/s) = 11.28 lb /ft?-s) Re and f at plane 1 are the same here as in Example 15.4-2, and f is approximated as constant at 0.0025 along the pipeline, giving ey = 4fL/D = (4)(0.0025)(52, 800/2) = 264. Furthermore, Pipr = (100 x 144 Iby/ft?)(32.2 Ibpft/Ib y-s?)(0.2821 Ibm /ft?) = 1.308 x 10° Ib?, /ft*-s?. Eq, 15B3-7 may be rearranged to give e a+ v/2rlins F or, for this problem with +y = 1.3 for methane, 264 —[2.3/2.6]Ins 0.3 _ 1.308 x 10° $ 26° (11.287 1026 This equation has the solution s = 0.74, corresponding to po/pi = V0.74 = 0.86 and pz = 0.86 x 0.2821 = 0.243 Ibm /ft?, (b) Eq. 15B.3-8 and the result of (a) give [ 1 — 0.8672](11.28 Ibm/ft?-s)? (0.3 (1-308 x 105 1b3, 7s?) 2.6 = (100 psia)(0.86) fs + = 86 psia (c) The temperature at the compressor inlet is Ts = Ti(p2/P1)(01/p2) = 530(86/100)/(0.86) = 530°R Eq. 15.4-22 then gives - vf 2 Wr = 2 1 — (£) th-(G ler" = i P2 _ orf, (Ly a) 0.86 (49686 x 530 Ibmft? /s? 16.04 Ibm /Ib-mn0l) = (-282 + 2.520 x 10° £2 /s?)/(32.2 Ibmit/s*-Iby = 7817 ftlby/lbm eo mol) 1.3 [ 100. | 100 5.5/1.3 _ ae and the required compressor power output is, . - aD?) Wn = wy = (3) CWn (2 10) S cee > (11.28 Ibyp/f82-8)(7817 ft-lb /Ibye) = 2.770 x 10° ft-lby/s = 504 hp 15-3 15A.3 Mixing of two ideal-gas streams. (a) The right sides of Eqs. 15.3-6, 7 and 10 are w = Wig + wip = 1000 + 10,000 = 11,000 Ibm /hr = 3.0556 Ibm/s P= viata + Ypw1 + PiaSie + PBS RMa wre , RT wy Mv. M vp = [(1000)(1000/3600) + (100)(10, 000/3600) tbyyft/s?] (8 x 540 ) [sccvseza 4, 10;000/3600 = aie + vw + 40 42 jo? _— 1000 100 155.6 + 25083.5 = 26539.1 Ibm ft/s?) Ibm in] With C, = 6.97 Btu/lb-mole-F, hence Gp = 6.97/28.97 Btu/lbm: side of Eq. 15.3-10 becomes the right z 1 E=CplwreTia + wsTue) + 5 [warts + wrsr7] 1000 3600 6.97 540/28.97 Btu/Ibm )(25036 Ibm ft? /s*- Btu) [ 1 [1000 2 , 10,000 +3 [op(10) + 3600 | = 9.939 x 10° + 1.390 x 10° = 1.0078 x 107 Ibmft?/s* (100)? tnt?/o] and 7 = C,/(C, — R) = 6.97/(6.97 — 1.9872) = 1.399. Then Eq. 15.3-13 yields the solutions 2.399 ) 3.05556 1.399? (26539.1)? = 5065[1 + 0.97838} = 109.5 ft/s and 10020 ft/s 1.399) 26539.1 [| 1.399? — 1) 3.05556 x 1.0078 x 107 v= 14,)1-2(—= |) == the smaller one of which is stable. ‘Then Eq. 15.3-10 gives the temperature of the mixed stream: Te = (Bfw— V6, _ [0078x107 1 a ee = | sosess — ~ 3109.5)? 87/8 = 546.5°R = 86.5°F 28.97 6.97 x 25036 °R-s?/ft? 5-4 ‘The cross-sectional area of the mixed stream is Sy = Suet Sy = Te 4 OH BE [ie 4 al Pratia” pon PM [ve ” ts 49686 x 540 1000/3600 , 10,000/3600 = (Se £3/Ib,, ) |e, ee (ae xo8.g7 ) [ 1000 ~*~ 100 = 0.9816 f Thin is] and the pressure pz of the mixed stream is obtained from Eq. 15.3-7: _ Pow nas = eo 6.86 x 10! Ib, /ft+s? = 1.00 atm (b) If the fluid density were treated as constant, Eq. 15.3-6 would give (Sia + Sta)u2 = Sravia + Stove whence ° Stadia + Savvis Stat Siw _ __vie/o-+ we ~ wia/pria + wie/ pris wie + wis wra/ Ya + wrs/5 11, 000 = Too = 1089 ft/s w= (c) Eq. D of Table 15.3-1 gives, for the conditions of (b), (104/00) "B24 (wre / re) "SE — (2.000% te 1000/1100) + (10,000/11000. 454545 + ts ~ 5995.1 ft? /s = 44004.8 ft/s? x 3.1081 x 10? ft-lbs /(Ibm ft? /s? = 1.368 x 10° ft-lby/Ibm (200? _ Go ) > > 15-5 15A.4 Flow through a Venturi tube. (a) Eq. -34 requires the calculated values So = (1 /4)(Do)* = (x /4)(3/12 ft)? = 0.04909 ft?, P2 ne (0.75) 14 = 0.0610 Ib, /ft*, Pi _ RT (= X 529.7 Ibym-ft? /s?-Ib-mol pa OM 28.97 Ibm, /Ib-mol 1 = (So/ $1)? (p2/p1)?/7 = 1 — (Do/D1)*(p2/p1)*/ = (8/12)*(0.75)*/"* = 0.99990. Substitution of these and known values into Eq. 15.5-34 gives ) = 9.085 x 10° ft?/s*, 3 (0.98)(0.0610)(0.04909) = 2.08 Ibm/s (b) For isothermal flow, poM _ (0.75 x 14.696 x 144 Iby/ft)(28.97 Ibm /Ib-mol) RT, (1544.3 x 529.7 ft-lb r/Ib-mol) 1 —(e2S0/ p11)? = 1 ~ (p2/p1)*(D2/D1)* = 1 ~ (0.75)*((3/12)* = 0.9978, i = 0.0562 Ibm /ft, and le _ RL _ (49686 x 529.7 - 5S 2/2 ik (1/e)dp = FF In(pa/p2) = Fe yze— n(1/0.75) = 2.614 x 10° £2 /s! Then Eq. 15.5-33 gives 7 w = (0.98)(0.0562)(0.04909),/ 2 ne ae (c) For constant density at the entering value, 14.696 x 144 Ibg /ft2)(28.97 Ibm /Ib-mol) (1544.3 x 5297 ft'lb/Ib-mol) 1=(p2So/p1S1)? =1— pay )* = 0.9961 96 Ibm/s = 0.0750 Iby,/ft®, 2.271 x 10° ft? /s? ‘Then Eq. 15.5-33 gives w = (0.98)(0.0750)(0.04909) 44 Ibm/s \5-6 15A.5 Free batch expansion of a compressible fluid. (a) Just after the start of the discharge, p1 is equal to po and p2/p: attains the critical value given in Eq 15.5-43. Hence, 2 1.4/(1.4-1) ‘The expansion from pi to po satisfies p/pT = R/M, hence Inserting p2/po = (p2/po)'/7 according to Eq. 15.5-39, we get Te = To(p2/p0) 9! ala “+ (2) y+ Te Zi Z 50 K = (5) =s00 (7) = 25 (b) When p, = pz = 1 atm, pr = 1/0.528 = 1.893 atm. Then the temperature within the main part of the tank is only capa N=T% (2) = 300 (1:59 = 300 x 0.5078 = 152 K Po 100 opr (c) At the state described in (b), p1/po = (1.893/100)'/1 = 0.0588. The time required to reach this state is computable from Eq. 15.5-46: 7 (V/S2) 2) [(myor? '* [aioe + yore 5(5 )[@) (10/0.1 ft) (a) {(0.0588)- 4] = 0.58 s \o-7 15A.6 Heating of air in a tube. __ For the air contained in the 20-ft length of tube, we write Eq. 15.1-3 with Wr =0 (since this system has no moving surfaces); also we use the approximation A(v? /2) of the kinetic energy change from inlet to outlet. This gives the simplified steady-state energy balance A(a+l)=6 1) for horizontal flow, or 1 5 ; a ar +3 (2) - =6 Poth for horizontal flow of an ideal gas in a tube, Proceeding as in Example 15.3-1, we get the algebraic equation 8.18 1 9.8 7 Zao 39(Tz —T,) + 3% 10-*(T} — T?) — =z x10 8(T} — T}) Btu/Ibm] 1 A = ayy | (4022\* + 5 (75 ft/s)? x 3.9942 x 107° Biu/(Ibm ft?/s2)) |( 2) -1 2 157; = (800 x 20/185 = 86.4865 Btu/lbm) Inserting T; = 5°F, and an initial guess of 800°R for Tz, we get the following Newton iteration sequence, converging to 864°R~ 354°F: Old >, LHS, f=LIS — f' =df/dT,, ATs New Ts, R Btu/lbm -RHS —Btu/lby-R =-f/f' oR 800 82.9245 -3.5620 0.2458 14.49 814.49 814.49 86.5761 40.0896 0.2474 -0.36 814.12 15-8 15A.7 Operation of a simple double-pipe heat exchanger. (a) The two exchangers will work most effectively if connected so as to simulate a single double-pipe exchanger operating in countercurrent flow. That is, the stream to be heated should enter exchanger A through its inner pipe at plane 1, next to the outlet of the twice-cooled hot water stream, and should exit the inner pipe of exchanger B at plane 2, next to the original inlet of the hot-water stream. (b) The Reynolds and Prandtl numbers for the stream being heated are: 4w (4)(5400 Iby,/hr . (S) a (ant f)(L.09 Sm) = 721x108 - (%) eS @ Btu/lbm-F)(1.09 lb) o ok 0.376 Btu/h At these conditions, with jo and jx not distinguished, Fig. 14.3-2 gives hinD k (Re)“(Pr)"¥? = 0.0028 and the insensitivity of hj, to L/D at Re above 8000 gives hoc = hin according to Eq. 14B.5-4. Hence, the local heat transfer coefficient for the inner pipe wall is ie 0.00285 (Re)(Ps)"/* (0.376 Btu/hr-ft-F) (0.0875 ft) = 1237 Btu/hr-ft?-F 0.0028: (7.21 x 10*)(2.90)'/* and the local overall coefficient Uj based on Dj satisfies 1 1 = ————— + Wall and deposit resistance based on D; DiGi ~ Dake * Dihanaaies +! O24 SPO8 1 1 0.001 = (osrs x1237) * (00875 x 2000) * O.os7s ee aad lea 1 @, * waa * 3000 +001 = 0.000808 + 0.0005 + 0.001 = 0.002308 hr-ft?-F/Btu Hence, U; = (1/0.002308) Btu/hr-ft?.F, Equation 15.4-13 then gives In (® a ) The ~ Teo 1 1 U; = = DL 15A.7-1 (ae ] am) ae : ; 15-4 1 1 1 st ) (nr x 0.0875 x 40 0.002308 ( B400)(.00) + sara) EHP) 0.2941 in which L, wy and we are measured in the direction of the cold water flow. ‘The computation of Tuy is summarized below. For each trial value of Tez, an energy balance with heat capacities of 1.00 Btu/Ibm-F gives 5400(1.00)(Ze2 ~ 70) = 8100(1.00)(200 — Tax) (154.7 — 2) from which the outlet temperature Th of the cooled hot water can be calculated for testing against Eq. 15A.7-1. The final Thr is very close to 136°F. Thy — 70 Ta Tr =200- Ty —70 200 — Ter Te 212-70) 130 160, 90 70 0.2513 132 158.667 88.667 68 0.2654 135 156.667 (86.667 65 136 156 86 64 135.8, 156.133 86.133 64.2 [5-10 15B.1 Performance of a double-pipe heat exchanger with variable overall heat-transfer coefficient a. We start with Eq. 15.4-12 oof (0, -T.) — Jem )al WiC Cpe Since U is a linear function of T,, ~T,, we write U=a+A(T,-T,) and we choose a and B by requiring that the conditions at planes 1 and 2 are satisfied: Atplane 1: U, =@+B(T,-Ta) Atplane 2: U, = a+ B(Ty2-T.2) This gives us two equations and for finding the two constants a and B: (Uy ~Us)(Tm = Ter) (Tia =Tea)= (Tm =Ta) (U,-U,) (Tro -Te2)- (Tin -Ter) a=U,- Then the overall heat-transfer coefficient can be written as: U-U _ (Tr=T)-(Ta=Ta) . AT-AT, Uy=-Uy (Tig=Te2)-(Tn -Ter) AT2- AT, b. The first equation in (a) may now be written as: aT ~Fe a (a+ BAT) E Jers i pt Part of the right side may be rewritten by using Eqs. 15.4-7 and 8: a -(ag Ta-Ta WiC WC pe = #5 (Cn -Ta)- Cra Tad) = (ATs AT) The differential equation now becomes daT 1 ~Wia+ pat) (0% -AT,)(2a1,)dl Integration then gives: wm dAT 1 “han; BAe pat) 7,47 =AT,)(2279) foal or \aT, +—In| = aT 1 era a lar where A is the area of the heat-transfer surface. Then, introducing. the expression for a developed in (a), we get u, AT, oum{ 1) =u AT, - AT;)- (Uz -U, )AT, U, aT, [U, (47; - AT,)- (Up -U,)AT,] Finally, solving for Q. we find U,AT, -U,AT, = A—_UsAT, =U, AT, Q.= AT UAT, )=In(U,AT,) This is the same as Eq. 15B.1-2. 15-12 15B.2 Pressure drop in turbulent flow in a slightly converging tube a. We begin with the d-form of the mechanical energy balance as given in Eq. 15.4-2 odo gdh 3 lay -aW- pita When this is integrated from z=0 to z=L for flow of an incompressible fluid in a horizontal tube and no mechanical work term, we get (after using Eq. 6.2-16) 7 5 (8-vf) + (02 ~p)= 2 Lae b. Since w = puS = pv(xD*) is constant for each cross section, we have p,0,D} = poD? so that, for an incompressible fluid By D Furthermore, this result can be used to rewrite the local friction factor thus fa O71. __0.0791 (2) (Day (Oy) : 0.0791 ( D y" __0.0791 ( D) \" (/y)*(oY"(DyD)"\P) (Driv): c. Use Eq. 15B.2-1 to make a change of variable in the integral in Eq. 15B.2. Lf, pp.v°f( dz ain ta (hae = ple The limits on the integral over D are fixed by using Eq. 15B.2-1. 15-13 d, When the expressions for the velocity and friction factor in (b) are substituted into the integral in (c), the integration can be performed to get p, (D,/D)* (oyo,)" Lv”, 0.0791Lv? "oop e D (D, -D,)(Diei/v) 0.079110? DalP 19/8 gy (D.-D,(Dies/v)™ i ___ (0.079107 | ay ee tel(2) 1 Then Eq. 15B.2-3 becomes By 1] tenor, D, -D,)(Dy»,/v)"" (2) | e. From Eqs. 6.1-4 and 15B.2-2, we get for D, =D, =D and = 0) =0 Gone a _2Lv* _ 0.0791 D (Dop/u)" We now have to show that the result in (d) simplifies to this result when -D, =D, =D and 0, = 0; =. Clearly the first term on the right side of Eq. 15B.2-6 vanishes when D, = D, = D. The next term gives 0/0, and hence we have to apply L'Hépital's rule: ' [(2/P.)**-1] intl tim tt ok pb, (D,/D,)-1 1 x-1 et ‘ This, along with the statements that D,=D,=D and 0, completes the proof of equivalence. 5-14 15B.3 Steady flow of ideal gases in ducts of constant cross section a, In the absence of work terms associated with moving mechanical parts and for a duct which is horizontal, the second term on the left side of 15.4-2 and the first term on the right side can be omitted. For a circular tube, 4 times the mean hydraulic radius equals the tube diameter, we get for a differential length dL of the tube 1 f 1 vdo+—dp+20°LdL=0 or —vdv+—dp+4u%de, =0 arts naa b. Use of the product rule for differentiation, Aa () ad =|==ap-| 4 la, (3) pp)? leads directly from Eq. 15B.3-1 to Eq. 15B.3-2. The d-form of the mass balance is dw=0 or d(vS)=0 or pdv+vdp=0 or dp=p@ For an ideal gas, pV = RT or pMV = RT or pM =pRT. Hence for the isothermal flow of an ideal gas, the second term in Eq. 15B.3-2 is zero, and that equation becomes, after multiplication by 2/0” 2 2\pRT\( 1 a(S Then, combining the last two equations, we get Eq. 15B.3-3. c. When the result in Eq. 15B.3-3 is integrated from "1" to "2" encompassing a length L of the pipe, we get 2 = 2RT/ ve : ~inv,)= BE (1-2) ¢tn{ 2 = ar a = AInv, ine,)= eines __Rtp? 1-() sin) =BB(.-») nr Mp) | ra) & - 15-15 de, on 2 e VS This may be solved for the "mass velocity” G to give - ot) -0 vInr (e,—Inr)? so that the critical value of r is given by the solution of the equation e,-Inr, d. From Eq. 15.4-4, we find that the right side is zero (for an adiabatic system with no work done by moving parts), and that the ideal gas law makes the fourth term on the left be zero. In addition, since the tube is horizontal, the second term on the left is also zero. Hence we are left with n CG, Ry vdv+C,aT=0 or vdv+—dT=0 or vdv+———dT=0 a M My-1 The third expression comes from the use of the ideal-gas expression C,-Cy =R and the definition y = C, /C, , which give Next the energy equation can be integrated to give yoto()REeconst. or gu? +(—2.)P got (Lu y-U) M y-p 15-16 where the ideal gas law has been used to get the second form. Next, after solving the integrated form of the energy equation for p/p, we substitute this expression for p/p into the second and third terms of Eq. 15B.3-2 to get (after multiplying the latter by 2/v”) ft _f to), 3(B tN ypt) Pt v y jv vp, p Then we use the d-form of the mass balance (pdo + udp = 0) to get {2\2-3 3 (2+ toot That} ae, yo Plo” 0 or, when some terms are combined, (eI)#- 3 (B+ aa "a ode yjv lp This equation can be integrated to give (Ze inde (2+ tga) d 7 a Py ts Using the macroscopic balance, 03/0, = p,/2, we get 2 a (nde way (FAR) y J" er (poyl J 207 Ne We can now solve for G=,v, (using s=(p,/p,)*) to get ee PP Cape | Tysiyrlins ya l-s 2y IS-17 e. The macroscopic energy balance is ae Then using the ideal gas law in the first term and the macroscopic mass balance on the second term gives MC, 11 “a (B-B}+4e(4-4)=0 Rip py Pr Pi Then, since MC, /R = 7/(1-7), this equation can be rearranged thus 2 BB-(-1)6 1-(4] Pp, mx \ 2y Jer] le And this expression can be rearranged further to give Eq. 15B.3-8. Let the right side of the above equation be designated by X and proceed as follows: PrP Pry op P2_ Po PiP2 Pr ee ea PPL 2X or Px a( 1421) Pi Inserting the expression for X will then lead directly to Eq. 15B.3-8. {5-18 15B.4 The Mach number in the mixing of two fluid streams s. Requiring that the radicand in Eq. 15.3-13 be zero is equivalent to the statement that 1.(EAV2)(2) 2 y? KP) wo The factor (w/P) can be obtained from Eq. 15.3-11, and the factor (E/w) can be obtained from Eq. 15.3-10. This gives We now move the second factor on the right side to the left side, multiply the entire equation by v3, and replace C, by (R/M)[y/(y -1)] to get 1 RT,\ R (298) (SG We next multiply by 2 and rewrite the equation thus: 2 2 of Rhy (2) ={ 41) 2RT 5 [7 M M y )™M 7 Then we collect terms in the same powers of the velocity at plane "2" to get 1 2RT, (1 RT,\? 1 (Be-MEe BY -0 we (be: This equation may be solved for the velocity at plane "2" to give v= yk 15-19 This is exactly the speed of sound, given in Problem 11C.1(c). b. To describe the behavior of a gas passing through a sudden enlargement, we can set wa=w, and wy = and also Pu=Pr Tw=T, Sa=S, Sy Then when w, P, and E are defined analogously to the quantities in (a), we find that Eqs. 15.3-11, 12, and 13 remain unchanged. {5-20 15B.5 Limiting discharge rates for Venturi meters a. First, we take the square of Eq. 15.5-34 ? (1 — plr-dir v-cs( BY |e in which f =(5)/S,)? and r=p,/p,. Next we set dw?/dr =0 to get : [= (1-1!) ar|1-pr' and, performing the differentiation, we have (2/y)r2 —((yy) 41)" Pirie rir) — Bi Bly jr 1- prt (- Briry [(2/ryr2 —((y7) +1)" 1-67) P(e rt wr B(2/v)r reir) = 0 When this is multiplied out, two terms cancel, and the remaining terms are: Byars sf 2 npr (3 si}p 62 <0 7 Y Y Y Multiplication by yr°% finally yields 2 _(lty)_ ptr Bly-1)+ =0 This is equivalent to Eq. 15B.5-1. b. First we solve Eq, 15B.5-1 for B =0 and get j5-21 2 Vth) r=|— (Fa) We substitute this expression for r into the first equation above in (a) in for B=0 and then rearrange it as follows: _ mmc Tren) 2(y-1), M 2 awe vil ale (- zal a V7 \ a= + eal 1 yy yM. consi a2) in agreement with Eq. 15B.5-2. c. For isothermal flow, we get from Eq. 15.5-33 2ff*(RT/pM) dp (@RT/MVin(ps IPs) ar = Cah 2804} 1-(P2S0/P1S:) 1-(P250/Ps51) (2RT/M)In(1/r) 2 w=C,pr5 = Cyp,Sor in which r =p, /p, and B =(5,/5,)°. Now for negligibly small 8, we can find the maximum discharge rate by setting, dw®/dr = 0 which gives (Pint) =2rnt-1-0 dr r. r This gives the value of r for the maximum discharge rate as \5-22 It is interesting to compare the results for air (7 =1.4) for the two cases. For adiabatic flow, we get w, 2 rey 2 \aot a mx ff 2 =,f14 = = Vi4(0.8333) CaP:SoVM/RT, Vea) y (zi) wn = (1.183)(0.5787) = 0.6846 For isothermal flow, we have —— Wms 11 9.6065 CipiSyVM/RT Ve 2.718 The ratio of these two results is 1.13. We estimate that this ratio will amost never be greater than 25%. Thus, the simpler isothermal expressions are frequently useful for preliminary estimates of flow behavior 15-23 15B.6 Flow of a compressible fluid through a convergent-divergent nozzle a, From Eq, 15.2-2 with no work done by moving parts and with viscous heating neglected, we get for the assumption of flat velocity profiles: (08-08) +f Sap =0 Then, we set v; =0 and, assuming adiabatic flow, we can use Eq. 15.2-5 to evaluate the integral. This results in (yr pop +t (2) -1|=0 ay-lp Then the ideal gas law may be used to rewrite this expression as ly 492 RTL 1-(22)" : ue M y-ll py b. We may start with Eq. 15.5-34 with S, considered to be very large, and S, replaced by 5, w=Cyps; fete”) We next replace p; by p;(P2/p,)"" and solve for S,, assuming that C, =1. This gives, with r= p,/p, w oar - rrr) 15-24 The minimum cross-section for given values of w, p,, and p, will occur when a Any?” (r+ 1/7") (ri pyr yy (r" = rrr When this equation is solved for r, we get (zy PL -( y+ ) which is Eq. 15B.6-2. c. Combining the results from (a) and (b) we get RT, 7 2 dy? =A 1-— ae | al RT, y ()-4 y M y-lly+1) M y+1 Then, from Eq. 11.4-56 and Eq. 15B.6-2, we find (ry T= r.(2) = uh Po 2 From the last two equations we get ws) oe Therefore, the velocity at surface "2" is sonic (cf. Eq. 9.4-4). It is clear that this problem is very similar to that in Problem 15B.5. Here, however, we are considering the effect of varying S,, whereas in Problem 15B.5 the reverse procedure was used. d. Since we now want to get the velocity as a function of the local pressure, we have to replace the first equation in (a) by s(0*-ef) +f Fap=o 15-25 We then use the fact that 0, = 0, and perform the integration to get the velocity at any value of the local pressure for an ideal gas in adiabatic flow vy Bape {2 he (" Yr ph a) sy] ‘(ileal where r=p/p, (!). Then taking the square root and using the ideal gas law, we get r = [PRD _y_\q_ poo ae (le rm) The cross-section at any value of the local pressure can then be obtained from (with 7 =p/p,): eGo )(5) | =p,Sr"" (SBS )e- rin) Then, solving for the cross-section S, we get we Cail)" Is-26 Next, for the conditions of the problem given in part (d) of the problem, we have R= (1544)(32.17) = ft?lb,, /Ib - mole-s?-R RT,/M = (1544)(32.17)(560)/29 = 9.592 x 10° ft?/s® QRT; 2 A(4) S esx 10° u w = (10 Ib- mol /s)(29 Ib,, /Ib- mol _ pM __ (10 atm)(6.8087 x 10* (1b,, / ft-s*) per atm.) 1'RT, — (4.9686 10" Ib,,ft? / s? Tb - mol°K)(560°K) =0.710 Ib, /ft® = 2591 ft/s We may now summarize the calculations of v, T, and $ thus: -on4 v=2591¥1-7°; T= 560r°%; Ale p(atm) + 70286 v T s 10 1.0 1.000 0000 560 oo 9 0.9 0.970 449 543 0.977 8 0.8 0.938 645 525 0.739 7 0.7 0.903 807 506 0.650 6 0.6 0.864 956 484 0.613, 5.28* 0.528 0.833 1058 466 0.606 a 0.5 0.820 1099 459 0.607 4 0.4 0.769 1245 431 0.628 3 03 0.709 1398 397 0.688 2 0.2 0.631 1574 353 0.816 i 0.1 0.518 1798 290 1.171 0 0.0 0.000 2591 0 ~ *Pressure at the minimum cross-section 15-27 15B.7 Transient thermal behavor of a chromatographic device We start with the energy equation of Eq. 15.1-2, which we apply to the gas inside the chromatographic column. We are concerned only with the change in time (and not distance) within the column. Since there are no moving mechanical parts, the equation simplifies to futg =Q=UA(T, -7) in which the total kinetic and potential energy in the system are considered constant, and U is the overall heat transfer coefficient between the external gas stream and the gas within the chromatographic device. From thermodynamics we can write ati = (# | ar (s fF) a= Guar s|-vo T(E). fv The quantity inside the brackets is zero for an ideal gas and is neglected here. Then U-U°=C,(T-T°), where the superscripts ° stand for the reference state, and Usa = fold v = plieViu, + PC (T-T Van where the density and heat capacity are taken to be constants. The energy balance becomes Z aT t Vip S- = UA(T, - == T,o| 1+—|-T PCV Gell) or are f 5) We now introduce the dimensionless temperature © = T/T. and the dimensionless time 7= t/t) and rewrite the energy balance as dO _ aa Bi(1+7)-O] with BeUAty/pCy Vex \5-28 This differential equation can be put into the form 4 Be=B(1+2) with @=1at r=0 This is a linear first-order differential equation (see Eq. C.1-2), which with the indicated initial condition has the solution o-(1+ eoh)eper B)'B This is probably the simplest solution that can be offered for this problem. We are now in a position to answer the questions in the problem: a. The difference between the external gas temperature and the temperature within the chromatographic device is 1,1 pe 11 ee Ty-TaT oll 2)-Te Lt 2S de ms )=T {4-46 a ) b. The limiting value of the temperature difference will be 1 (47). =Tyo5 c. The time required for the temperature difference to be within 1% of the limiting value is given by the solution of the equation e“** = 0.01 or t= (2.303)(2)/B. d. Constant physical properties; no dependence of temperature as a function of distance down the column; the neglecting of potential energy and kinetic energy effects. e. All the quantities that are contained in B. 15-29 15B.8 Continuous heating of a slurry in an agitated tank a. The starting point is Eq. 15.1-1 or 2. On the left side the potential energy term ©,,, is omitted because for a full tank the ©, is a constant. Similarly K,,, is time-independent and can be omitted. The term U,. =f, pUdV will be retained. Since Ui = Ai-(p/p), and p and p are time-independent, we can replace the internal energy by enthalpy, which is given by Eq. 9.8-8, the second term on the right side of that equation being negligible. Furthermore, since the heat capacity is considered constant, Eq. 9.8-8 becomes simply H=H,+¢,(T-T,) where the inlet slurry temperature T, has been chosen to the datum plane for the enthalpy. On the right side of Eq. 15.1-1, the mechanical work term and the kinetic energy and potential energy terms can be disregarded. But the heat addition term Q and the outlet enthalpy term must be retained. Therefore, Eq. 15.1-1 becomes d 4 =u dt 2w+Q or pC,V— =-wC,(T-T;)+UA(T, -T) which is Eq. 15B.8-1. b. When the system has attained steady state operation, the left side of the above equation becomes zero, and the slurry temperature has leveled off to T.. This quantity is then defined by the equation 0=-w¢,(T..-T,)+UA(T, -T..) or (with the definition that UA/wC, =2): 15-30 which is needed presently. Now the differential equation can be rewritten in terms of the dimensionless variables given in Eqs. 15B.8-2,3: otra) 4b) =0o{-2-0)-(0-1)=-0(1+9) Hence the differential equation in dimensionless form is do & =-O(1+2 de 7 O(1+ 2) c. This equation is solved to give od® . 00) Pg 7 lt Q)iyae or =e) This is just the dimensionless form of the solution given in the text. d. When the solution is written in the dimensionless form, it is easy to see that it satisfies the differential equation and initial condition. It is also evident that @—> 0 (or T->T.,), which is to be expected. \5-38 15C.1 Parallel-counterflow heat exchangers a. From Eg. 15.1-3, with only the enthalpy terms contribut- ing, and Eq. 9.3-8 with only the first term being important (and the heat capacity constant), we get for the region between a and b 5Cy(Ts ~Ta2)~ WaCpa(TA~Th)=0 from which Eq. 15C.1-1 follows. b. Then the application of Eq. 15.4-4 gives, for the region between the dashed lines in Fig. 15C.1, three differential energy balances over the heat-transfer surface dA: wyCpadTh = Sut, -T)dA A il waCpadT 4 = => U(T,-Th)dA W5CypAT 5 -[Sutrs -T))+3u(TE = T.) a Then introducing the ratio R, and the dimensionless differential area da (defined in the problem statement) leads directly to Eqs. 15C.1-2, 3, and 4. c. First we differentiate Eq. 15C.1-4 with respect to @ to get R da "da da da 1d°T, +f Fa TE -0 Then using Eqs. 15C.1-2 and 3 this becomes AGT, aT, I Rde® da 4 1 (To -Th)-4(TH -T,)=0 Next use Eq. 15C.1-1 to rewrite this as 2 aT , pay _ 1 da? da 70, do 1 a +R? -=0=0 da? Ga 4 Ty-Tp)=0 or 15-32 In the second equation we have used the dimensionless temperature. This differential equation may be solved (see Eq. C.1-7a) as follows: © = Cexp[-3(R-VR?= Aa] +C, exp[-3(R VRP? =1)a] 8 ee, Application of the boundary conditions gives two equations for the integration constants: 1=C,+C, O=Cye™F + Cye™er The integration constants are therefore: ssp On -a (naa Oo rear gmt TE d. Next we obtain dO/da:: dO __me™" mye" da yea; yay Then evaluate this at a =0 to get 1 aT, m. m. a) 5 Tar ~Ta2 AQ lang 1 VR tar yg RA a; Then eliminate the derivative by use of Eq. 15C.1-4 (also evaluated at o=0) to obtain: {5-33 oar y oy Ty -Tyl Ta ttatTall= — aa tae, We now replace the denominator on the left side by using Eq. 15C.1- 1, and then we have to manipulate the left side in such a way as to obtain the ratio ¥ defined after Eq. 15C.1-8; thus the left side becomes: (Tay =Tor)+3(Ta2 = Tar) _ (Tan = Tn) + (Toa =Ts - Ta + Tr) Ta2-Tar With this substitution Eq. (*) gives ¥ as a function of a. But we would like to have ay as a function of Y. That is, we have to solved the following equation for x: cot with A=m., Bem, x=e\® lar T-x 1- m9 But this is just a quadratic equation, which can be solved by the standard method, which yields two solutions: =GeA =1 d =a oC an CoB The solution x=1 is physically uninteresting. The other solution gives Via, Cl tea +i)-m. (24 W)-2¥m__2-¥(1-2m_) (¥)¥4)=m, (2+ ¥)-2¥m, ~ 2-¥(1=2m,) 2-¥(1-1+R-VR?+1 ce 2-¥(1-1+R+VR* +1) e Taking the logarithm of this equation gives Eq. 15C.1-8. 15-34 15D.1 The macroscopic entropy balance a, First rewrite Eq. 11D.1-3 as (¥-ev8)-7(V-4)-7 v-pv8)-(v-2q)-Ze(q-v1)-2(e:Wy) (t:Vv) We now integrate this over the entire macroscopic flow system, which is presumed to have some moving parts in it so that the volume and surface of the system are time dependent: ae g dl Jae fpr oslo [ea a Hla-vinr) + (e:vv)]av vey Since the last term is the integral of the entropy production terms in Eq. 11D.1-4, we can label this term gs... We may now use the Leibniz rule to move the time-derivative operator to the left of the integral sign and the Gauss divergence theorem to transform the volume integral into a surface integral: a8 d sia a d a = pSdV =— | pSdV -pv SS = —S, “pv 5S dS Baier gS” vorsSis— Sn Jlocorsy When this is inserted into the preceding equation and the Gauss theorem applied to the integrals containing divergences, we get: d Q 7 Sosy == [(n-olv-vs)5}s “Sf Aa}s +10 se) We now divide the surface into four parts: $(t)=$, +S, +S; +S, the cross-section of the entrance (1), the cross-section of the exit (2), the fixed solid surfaces (f, and the moving surfaces (m). The first surface integral above will contribute only at I and 2: \5-35 ~ J(n-p(v—vs)8)AS = 9,0,8,5; - 9,025,5, = S(t) The second surface integral will, however, contribute at all four surfaces: M51 _ 52 1 Wi __ War -f{n- fa}is= fae. _ (n-4 i= Pale +Q. J ( T a a T * sto) Ty PRAT, P2027 Hence the macroscopic entropy balance becomes: which is Eq. 15D.1-1. b. The macroscopic energy balances states that the increase in the total entropy of the system results from (1) the convection of entropy in and out of the system (w,8,-w,S,); (2) the entropy transport at the inlet and outlet of the system by heat conduction ((4:5,/T;)- (q2S2/T))--presumably very small compared with the entropy convection terms; (3) the entropy transport through the walls of the system, and (4) the entropy production within the system, which consists of a term involving the heat flux and a term involving the momentum flux. Entropy is thus produced by the dissipative effects of heat conduction and viscous flow. These terms are positive for linear flux laws. c. The term in the entropy production term involving the stress tensor can be written a f(u:Vv)dv = avg Vie) if the temperature variation throughout the system is not too large with respect to the absolute temperature. Then the integral is indeed the energy dissipation by viscous heating divided by the average temperature. 15-36 15D.2 Derivation of the macroscopic energy balance When Eq. (N) of Table 11.4-1 is integrated over the entire volume of the flow system we get: j alton +l + pO)av=— J (¥-(o0? + pt +6)v}iv vit) - f(V-q)av- J(e-peyv Je [t-v])av vo We now apply the Leibniz formula to the integral on the left side of the equation to get J ater + pli + pb)av vo oa {(sev* sell +pb}iv + {| pv? + pl + p@)v.)ds S(t) Hk + Ugg + Py) + J (n-(o0 +pul +p@)v,.)ds so This is now substituted into the preceding equation (in which the volume integrals have been converted into surface integrals by using the Gauss divergence theorem) to get: (Ku + Uy + iy) =— J (o-(ee0" +pll + pb)(v—vs)}as = f(n-q)as- dlrprids- J(n-[t-v])as s(t) s(t) We now divide the surface into four parts: S(t) =S, +S, +5, +5,,, the cross-section of the entrance (1), the cross-section of the exit (2), the fixed solid surfaces (f, and the moving surfaces (m). The first surface integral above will contribute only at I and 2: “Jf . (}p0? +pl+ pe\(v “Vs ))as = ($0,(0?) + 0,0, (2) + 0,4, (v1))S, ~ ($2(02) + P22 (v2) + P22(02))S» 15-37 The contribution of the second surface integral at 1 and 2 is presumed to be negligible with respect to the convective energy transport, but will in general have a nonzero value on the surfaces; we call this Q. The third integral will contribute both at "1" and "2" and on the moving surfaces: ~ {(r-pr)ds = +P.(01)S, ~Palea)Ss + Wm? it) where W\?) is the pressure contribution to the work done on the system by the via the moving surfaces. The fourth integral will be: - f(n-[e-v] ds +w so which is the stress contribution to the work done on the system by the via the moving surfaces. Of course there are also contributions at 1 and 2, but these are considered to be negligible to the pressure forces. When all of the above contributions are assembled, we get F Kee + Ug + Pea)= (bos? )+ xt, (7) + 014, (04))S; ~($P2(v8) + Palla (o>) + P22 (22))So +Q4+P1(24)S; — Pr (02)S, + WY + WY? which is the macroscopic energy balance given in Eq. 15.1-1. \5-38 15D.5 The classical Bernoulli equation For an isentropic flow, dll =-pdV = pa or vii=-pvV =-pv, Then Eq. (E) of Table 11.4-1, with the heat and momentum fluxes set equal to zero, becomes for a steady-state system p(v-V4u") + p(v-(-pVp"))=-(V-pv) +o(v-g) Using g=—gVh and some standard vector identities p(v-V}0")+pp"(v-Vp)=—p(V-v)-(v-Vp)—p(v-Vh) The second term on the left and the second term on the right can be shown to be equal, using the equation of continuity (appropriately simplified for steady-state flow). Next, consider that the del operators are acting only along a streamline, so that V = (v/v)(d/ds), and hence we get 4 (492) 2-04 yg eva(te")= pas? Pe Gel When this equation is divided through by pv, we get Eq. 3.5-11. Then Eq. 3.5-12 follows at once. 15-39 16A.1 Approximation of a black body by a hole ina sphere Use Eq. 16.2-12 e..<— 2 — hoe e+ f(1—e) or _2(1erote) ) Into this we insert the values ¢= 0.57 and ey. = 0.99 and get 0.57(1-0.99) _ 0.57(0.01) _ 0.0057 Ue 0.99(1-0.57) 0.99(0.43) 0.4257 = 0.01339 Then using the definition of f we write Triote 4n(3*) Then solving for the hole radius we get f= 0.01339 = Thoie = 2-V0.01339 (3) = 0.69 in. 16A.2 Efficiency of a solar engine The area of the mirror is (#R*)=25zft?. Since the solar constant (heat flux entering the earth's atmosphere) is, according to Example 16.4-1, 430 Btu/hr- ft, the energy input to the solar device is (430Btu/hr- ft?)(252 ft*)(3.93x104 hp/Btu-hr?)= 13.3 hp Therefore the efficiency of the solar device is Efficiency = ra = 0.150 = 15% \G-2 16A.3 Radiant heating requirements The heat required is the sum of the radiant heat-transfer rates betwen the floor and each of the other surfaces. Since no pertinent data are supplied for its estimation, the convective heat transfer will be neglected. It can be expected to be appreciable, however. The total radiant heat-transfer rate is then Quad = Ancor (Tiooe ~ Tats Fi Floor to ceiling: F, =0.49 Floor to large walls: F, =F =0.17 Floor to small walls: F, =F, =0.075 Summing the contributions, we get: YF; = 0.49 + 2(0.17) + 20.075) = 0.49 + 0.34 + 0.15 = 0.98 Alternatively, we may consider the floor to be completely surrounded by black surfaces. From that point of view we know that the sum of the F, should be 1.00. A cumulative error of 2% has thus resulted from considering each of the cold surfaces separately. Then we get Qua = (1.712 x 10° Btu/hr-- ft?R*)(450ft? ) -[(75+460)* - (-10 +460)" ]R* = (1.712 10" )(450)(8.19 x 10" - 4.1010") =3.15x10* Btu/hr Here we have used the value of the Stefan-Boltzmann constant given on p. 867. 16A.4 Steady-state temperature of a roof Since June 21 is (conveniently) very close to the summer solstice, the angle of incidence of the sun's rays on a flat roof may be calculated quite simply. We know that the earth's axis is tilted at an angle of about 23.5 degrees. Thus the angle of incidence of a flat roof at 45 degrees north latitude will be about 45-23.5= 21.5 degrees, and the heat received by the roof will be given by the solar constant multiplied by the cosine of the angle of incidence and then further multiplied by the absorptivity of the surface: (430)(cos 21.5°)a. = (430)(0.9304)a., in units of Btu/hr.ft2, We now equate the radiant energy received from the sun by the roof to the radiant energy emitted by the roof plus the heat lost by convective heat transfer for the two cases given in parts (a) and (b): (430)(0.9304)a = (Troy Tac) + 0°T 5 a, For a perfectly black roof, we have (430)(0.9304)(1.00) = 2.0(Tyage ~ 560) +(1.712%10")(.00)T 4, This equation may be solved by trial and error to get about 625°R or 10°F b. For the flat roof with a = 0.3 and ¢ = 0.07 we get (430)(0.9304)(0.3) = 2.0(T,.o¢ ~ 560) + (1.712x 10” )(0.07)T4,4 which may be solved by trial and error to give 610°R or 150°F. ib-4 16A.5 Radiation errors in temperature measurements Assume that the thermocouple behaves as a gray body in a large black enclosure, and equat the net radiation loss to the convective heat input: e0(Ti, - Tan) = (Tyas - Tu) in which T,, is the thermocouple junction temperature. For the conditions of this problem (0.8)(1.712 10 )[(960)* - (760)* | = (50)(',., - 960) Solving this for T,,, we get 705 ° 0. Tyys = 960 + So = 971°R = 514°F There is thus a 14°F difference between the calculated gas temperature and the thermocouple reading. 16A.6 Surface temperatures on Earth’s moon. (a) A quasi-steady-state energy balance on a lunar surface element that directly faces the sun gives Joa = coTS max in which Jp is the solar constant, Tmax is the temperature of that surface element, and a and ¢ are its total absorptivity and emissivity. Setting a = ¢ for a gray surface, and using the value calculated in Example 16.4-1 for the solar constant, we obtain the quasi-steady-state estimate Tamax = (Ioo)'/* = (430/1.7124 x 10-°)*/4 = 708°R of the maximum temperature on the moon. (b) For a spherical lunar surface, receiving radiation from the sun only, the local intensity of incident radiation is _ flocos6, for0< 6 < 1/2; 10)={j for r/2S01, then we cannot set F,, = 0 in this problem, inasmuch as light leaving surface "2" is not all intercepted by surface "1". In fact, it is not difficult to show that Fp, = A,/A2. Then if the inner cylindrical surface is only slightly smaller than the outer cylindrical surface, then F,, will be just slightly smaller than 1, and if the inner cylinder shrinks to a wire, then F,, will approach zero. IL-8 16B.3 Multiple radiation shields. (a) Equation 16.5-1 gives the radiant heat flow between successive planes in the series as o(Tf - Ths) Wer+ leis Riga Qin = in which Ries = [Les + Veiga ~ WY /Ai Summing the thermal resistances Rj,iz1 from i= 1 to i= n— 1, and equating the heat flows through all of them, gives Q DY Ris =o(T} - TH) whence i-Ts) Riot (b) The ratio of the radiant heat flow through n identical sheets to that between two is Q(n sheets) siz Q(2 sheets) ~ (@—1)Riz m1 (c) The ratio of the heat flow with three non-identical sheets to the heat flow without the middle sheet is Ris [fer + 1/es — 1] Ria + Res (I/er + 1/e2— 1} + (1/ez + es — 1 in agreement with the result of Example 16.5-1. \o-4 16B.4 Radiation and conduction through absorbing media a. We begin by combining Eqs. 16.6-5 and 6 to get 0= 2a = mq? This may be integrated to give Ing? =-m,z+C The constant of integration may be obtained from the boundary condition at z= 0, so that =qe™ and (from Eq. 16.6-6) GQ=mgPen Next, use Eq. 16.6-4 to get a o- eT a maper Integration twice with respect to z gives tr) T(z)= eo +C,z+C, Next we apply the boundary conditions that T(0)=Ty and T(5)=Ts to get = at age +k iz ee For very large values of m,, the first and third terms become negligibly small, and we are left with Tete Very large m,: 4. ~-k é For very small values of m, we can expand the first and third terms in a Taylor series and get: Ty-Ts Very small m,: 9. =— qh (1-m, 2+) -k Thus in both limits, the conductive heat flux is virtually unaffected by the radiation. \b-l! 16B.5 Cooling of a black body in vacuo ‘An energy balance over the black body in the enclosure is d qu =2 or vv = Agi? When the Stefan-Boltzmann law is inserted, we get a aT _ 4 00 Ve Ao(T}-T*) This separable, first-order equation may be integrated as follows + dT _ Aor Sm 1 “evi The integral on the left side may be integrated with the help of an integral table to give IT, +T, 3 + 2aretan ~2aretan 21 = SAGTaE 1 2 T, pC,V The left side gives T =T, as ¢ goes to infinity, and T=T, as # goes to zero. The left side is dimensionless, and the right side is also, as can be seen by using the table of notation given on pp. 872 et seq. eee ro (lee) = dimensionless WG-12 16B.6 Heat loss from an insulated pipe. (a) Eq. 10.6-29 gives for this problem, Qe /L = 2n(To — Tr) ee with ro = 1.0335 in., r) = 1.0335 + 0.154 = 1.1875 in.and rz = 1.1875 +2 = 3.1875 in. With the given thermal conductivity values, we then obtain agen) 2n(To ~ Ta) [L= ME Ro _ 2(250 ~ Ta) ~ 0.0053 + 2.8 - Nea Btu/hr-ft if Ty = 100°F; oO if T, = 250°F (b) The net radiative heat loss is given by Eq. 16.5-3. Setting e = the aluminum foil, we get 0.05 for Qh) /L = oxD2(0.05)(T3 ~ TS) = 0.1712 x 10~*x(2 x 3.1875)(0.05)(Tj — 540*) _ [23 Btu/brft if Ty = 100°F = 560°R; 290 Btu/hrét if T) = 250°F = 710°R ‘The free-convective heat loss is predictable as in §14.6. For T, = 100°F, Ex- ample 14.6-1 gives Qo") /E = 18 Btu/hr-ft For T, = 250°F, Eqs. 14.6-4,5 and Tables 14.6-1,2 give Nuls™ = 0.772(0.515)(GrPr)!/* = 0.398(GrPr)!/* ‘The needed properties of air at Ty = (80 + 250)/2 = 165°F= 74°C are obtained from the ideal gas law, from Table 1.1-2, and from CRC Handbook of Chemistry ond Physics, 81st Ed., 2001-2002, pp. 6-1, 6-2, and 6-185. 1 = 0.0206 mPa's = 0.0498 lb /ft-hr pM/RT = 1.017 x 107? g/cm? = 0.0634 Ibyn /ft? = 1.012 J/g/K = 0.2420 Btu/Ibm /edotR 29.5mW/m-K = 0.0170 Btu/hrft-F B=1/T, = (1/625)R™* 16-13 Hence, (2 x 3.1875/12)° (0.0634)? GrPr = ee ( (0.0498)? = (2.76 x 10")(0.709) = 1.96 x 10” 17x wsr0/62) (¢ 1420)(0.0498 (0.0170) and Num = (0.398)(GrPr)!/* (0.398)(1.96 x 107)'/4 = 26.5 Therefore, QE”) ]L = ht D(To ~ Ta) = (Ntimk/D)(#D)(To ~ To) jumtk(To — Ta) = (26.5)x(0.0170)(250 — 80) = 241 Btu/hr-ft giving 93 Btu/hrft if Ty = 100°F; 531 Btu/hr-ft if Ty = 250°F. Qieona) — Qld) — Qleom) (33493 -18 ~ (0-290 — 241 (c) Linear interpolation to zero heat accumulation at the outer surface gives the steady-state values To = 100 + 293/824(250 — 100) = 153°F Qherr) (= QUad+(om) 17, = 334 + (293/824)(0 — 334) = 215 Btu/hr-ft |ol4 17A.1 Prediction of a low-density binary diffusivity. (a) We begin by looking up the needed properties of the species from Table EB. Species M,g/g-mol — T., K Pe, atm A: CHy 16.04 191.1 45.3 B: CoHs 30.07 305.4 48.2 Equation 17.2-1 then gives the following prediction of Day for methane-ethane (treated here as a nonpolar gas-pair) at p= 1 atm and T = 293K: a Daw =0( Fre) Caren)" TeaTen)C0/Ma + 1/Mo)"/p ate nog = 2.745 x 1074 ( * ( V191.1 x 305. ) + (45.8 x 48.2)1/3(191.1 x 305.4)°/22(1/16.04 + 1/30.07)"/?/1 atm = 0.152 em?/s (b) Equation 17.2-3 gives 1 6 (cDap)e = 2.96 x 10 (4 1 (eapen)!? (LeaTew PP? M2 (45.8 x 48.2)9 (911 x 305.477 = 2.96 x 107% (aa a 16.04 * 30.0 = 4.78 x 10~* g-mol/em-s ‘The reduced conditions for Fig. 17.2-1 for this problem, calculated as described on page 522, are Te 293 ren JT-aTen VI91.1x 3054 7 P _ 1.0 POS Preabon VSS x ABD At this reduced state, Fig. 17.2-1 gives (CDau)r = 1.20. Hence, the predicted value of cD4p is 1.20(cDag). = 5.74 x 10-®, Dividing this result by the ideal-gas prediction ¢ = p/RT = 4.16 x 10-* g-mol/em™* at this low-density condition gives Dap = 0.138 cm?/s. = 0.021 (c) Equations 17.3-14 and 15 give the binary interaction parameters caw = (3.780 + 4.388)/2=4.084A and cap/x = Vind x 25 = 189 K \T-\ when the Lennard-Jones parameters of Table E.1 are used for the individual species. Then, at KT/e4p = 293/189 = 1.550, Table B.2 gives 2p,ap = 1-183, and Eq. 17.3- 12 gives the prediction 7 1 1 Dap = 0.0018583,/T? ( — + —— } ——_—_ AB (Gk “ in) wee 1 1 1 = 2 —— eae) (a 04 * 30. of) EO 83) = 0.146 cm?/s (d) Use of Eqs. 1.4-11a,c with the combining rules of Eqs. 17.3-14,15 gives the estimates €ap/K =0.77/191.1 x 305.4 = 186.0 K Ws pp vat , ei () + () = 4.2224 2 Pc PcB Then at KT/eap = 203/186.0 = 1.575, Table B.2 gives p,ap = 1.1755, whereupon Eq. 17.3-12 gives 1 1 1 ' = too3)s Pap = 0.00185834/ (293) Gdn 0. z) (1)(4-222)7 (1.1755) = 0.138 em?/s 12 17A.2 Extrapolation of binary diffusivity to a very high temperature. (a) Equation 17.2-1 in the nonpolar form gives Dawlisoox = DAB| 299K (1500/293)' "7° = 2.96 cm?/s (b) Equation 17.3-10, with n = p/KT, predicts Dap « T°/?, thus giving Daal, soox = PABlya9x (1500/293)'* = 1.75 cm?/s (c) Equation 17.3-12 and Table E.2, with €4p = 135.8 from Table E.1 and Eq. 17.3-15, gives 2 Dap ysoox = DAB |gyqy¢(1500/293)"/? (Feszbeesas) = 251 em?/s AB |se00%/ean The superior agreement of Method (c) with the experimental value of 2.45 cm?/s illustrates the wider range provided by the Chapman-Enskog theory when combined with the Lennard-Jones potential-energy model. (1-3 1TA.3 Self-diffusion in liquid mercury. Equation 17.4-5 gives or in cgs units, Daas = (1.38066 x 107"erg/K)(7,K) ( 6.02214 x 10°: 2n(ua,g/ems) 1s cules/g-mol (200.61¢/g-mol)(V-a,cm*/g) Insertion of the values tabulated for this problem, with (cp) divided by 100 to get HA,g/em-s, gives the following results: T(K) Daas,obs. Dagar, pred. —_Ratio, pred. /obs. 275.7 152x107 1.2410" 0.82 289.6 1.68x10- = 1.40x 1070.84 364.2 257x107 2.16 x10" 0.84 The predicted self-diffusivities are about 5/6 of the measured values. 17A.4 Schmidt numbers for binary gas mixtures at low density. ‘We begin by tabulating the needed molecular parameters for species A and B from Table E.1, and estimating the binary parameters ¢4g and ¢4g/K from Eqs. 17.3-14 and 15: Species M,e/gmol 0, A e/K,K A: Hy 2.016 2.915 38.0 B:CChF: 120.92 5.116 280. AB: 4.0155 103.15 Equation 17.3-11 and Table E.2 then give the following prediction of cx for binary mixtures of Hy and Freon-12 at T = 25°C=208.15 K: cDap = 2.2646 x 1075 7( Z + a) a Ma * Ma) 0% 500,48 1 1 1 = 2.2646 x 107,/298.15 ( = + ——_- J __-_ : (sare * mim) (E055 F(0.959 = 1.794 x 107 g-mol/em-/s With this prediction of cD and the viscosity data of Problem 14.4, the Schmidt number can then be calculated as Se=—H- = HK pPap- McDap (taMa+zpMp)cDap in accordance with Eqs. G and L of Table 17.7-1. Results are as follows: ta=zH, 0.00 0.25 0.50 0.75 1.00 Hy g/ems 124.0 x 10-® 128.1 x 10-® 131.9 x 10- 135.1 x 10-® 88.4 x 107% M, g/g-mol 120.92 91.194 61.468 31.742 2.016 Se 0.057 0.078 0.120 0.237 2.44 We see that the Schmidt number depends strongly on the composition when My and Mz differ greatly. This fact is also illustrated in Table 17.3-1. VFS 17A.5 Estimation of diffusivity for a binary mixture at high density. The following properties of Np and CzHg for this problem are obtained from Table El: Species M,g/g-mol Te, K Pe, atm No 28.01 126.2 33.5 SoH 30.07 305.4 48.2 (a) The measured value Dag = 0.148 m?/s at T = 298.2 K permits calcula tion of an experimentally-based value of (eD4p)c- The reduced conditions for this measurement are T. 298.2 T= ee = ees = VTcaTep V126.2x 3054 P 1.0 p= ee = ee = 0.025 POS Preaton V5.5 x 482 On Fig. 17.2-1, this state lies essentially at. the low-pressure limit, with (CDs), = 147. Accordingly expressing ¢ by the ideal gas law, we find _ Latm YP = G20 x 208.2 em®-atm/gamol O18 m/s) = 6.05 x 10~* g-mol/em-s Hence, the critical Dap value is (cCDan)e = CDan/(cDap)r = 6.05 x 107° /1.47 = 4.12 x 107% g-mol/em-s Now, the reduced conditions for the desired prediction are = T2882 " VteaTea 126.2 x 305.4 7 rp _ 40 De® rcabes S555 x ABD. at which state Fig. 17.2-1 gives (CDap)r = 1.42. The resulting prediction is then cDap = (cDaz)-(cDaz)r = (4.12 x 107°)(1.42) = 5.8 x 107% g-mol/em-s. 995 (b) Equation 17.2-3 gives the predicted critical value 1 1 yn (33.5 x 48,2)1/9 ( cD, = 2. 8 ( gg Seen een ae (Daz). = 2.96 x 10 ( i 126.2 x 305.47 28.01 © 30.07, = 3.78 x 107 g-mol/emss. Multiplication by (Daz), = 1.42 as in (a) gives cDap = (3.78 x 107*)(1.42) = 5.4 x 107* g-mol/em-s. \7-6 17A.6 Diffusivity and Schmidt number for chlorine-air mixtures. (a) We begin by tabulating molecular parameters for chlorine and air from Table E.1, and estimating the binary parameters ¢4g and €am/X from Eqs. 17.3- 14 and 15: Species M,g/emol 0, A e/k, K A: Cl 70.91 4.115 357. B: Air 28.97 3.617 97.0 AB: 3.866 186.1 Equation 17.3-12 and Table E.2 then give the following prediction of Dap for chlorine-air mixtures at T = 75°F = 23.89°C = 297.04 K: Dap = 0.018583, /7? Gz 1 1 Ma 5) ata 1 1 1 . 5 2 ee Dee ae (3 7 aa TERRI = 0.120 em?/s (b) Equation 17.2-1 needs the following values from Table E.1: Component My g/g-mol Te, K Pe, atm A: Cl 70.91 417. 76.1 B: Air 28.97 132 36.4 ‘The nonpolar version of Eq. 17.2-1 then gives the prediction 1.823 Dap = 2.745 x 10+ ( + (76.1 x 36.4)/3(417. x 132.)§/7(1/70.91 + 1/28.97)!/?/1 atm = 0.123 em?/s (c) The result of (a), and the ideal gas expression for ¢, give cDap = pan = 4.92 x 107® g-mol/em-s With this prediction of Dag and the viscosity predictions of Problem 14.4, the Schmidt number can be calculated as Se= —f a pDap McDap (taMa+zpMp)cDap in accordance with Eqs. G and L of Table 17.7-1. Results are as follows: AI Th 0.00 0.25 0.50 0.75 1.00 #, g/ems 0.000183 0.000164 0.000150 0.000139 0.000131 M, g/g-mol 28.97 39.455 49.94 60.425 70.91 Se 1.28 0.84 0.61 0.47 0.375, We see that the Schmidt number depends strongly on the composition when. Mz and Mg differ greatly. This fact is also illustrated in Table 17.3-1 and in Problem 174.4. 18 17A.7 The Schmidt number for self-diffusion (a) Equation 1.3-1b, written for non-tracer species A, gives He = 7.10 x 107° M4? pT NE for the critical viscosity in g/em's. Here My is in g/g-mol, pea in atm and T.4 in K. Eq. 17.2-2 gives ea Ne op (Pane)e=2.96 x10-* (+ se) allt Tea ‘The resulting critical Schmidt number for self-diffusion with Ma ~ Ma, is a 7.70 —#__) - 1 ois (wren), 2.96 V2 (b) Figs. 1.3-1 and 17.2-1, with the result in (a) for Ma * Ma, give Sc for self-diffusion as the following function of T, and p,: Hr(TrsPr) Se e = 1.84-——— a Daas)e(TrsPr) Calculations from this formula are summarized below: Phase> Gas Gas Gas_— Liquid Gas. Gas T, 0.7 1.0 5.0 07 1.0 2.0 Pr 0.0 0.0 0.0 sat. 1.0 1.0 Be 032 04516278 1.00 0.94 Gaon Wl eG Scase 0.82 082,082.39 1790.94 17A.8 Correction of high-density diffusivity for temperature. ‘The following properties of CHy and CzHg for this problem are obtained from Table Bl Species M,s/gmol T.,K Pey atm A: CHy 16.04 191.1 45.8 B: CoH 30.07 305.4 48.2 ‘The reduced conditions (State 1) for the given cDap value, calculated as de- scribed on page 522, are 313 VISIT x 3054 and Fig, 17.2-1 gives (Dap), = 1.27 at this state. The reduced conditions (State 2) for the desired prediction are T, 1.30; pe 2.89 351 136 Te = ee pe = Vioi.d x 305. Pe Tis BAB. and Fig. 17.2-1 gives (-D4g), = 1.40 at this state. The prediction of cDag is then obtained as follows: = 2.89 (Dr|state 2 (P)rIgtate 1 40 = eo > g-mol/es = 6.0 x 10-57 = 6.6 x 10° g-mol/ (Dap)|state 2 = (Pav)|state 1 ‘The observed cDap value at State 2 is 6.3 x 10-* g-mol/cm's, in fair agreement with this prediction. : 17-10 17A.9 Prediction of critical cDaz values. (a) Equation 17A.9-1 gives KT/eaae = 1/0.77 = 1.2987 at T = Te, and Table B.2 gives Qp,44e = 1.2746 at this argument value. Insertion of this result, along with Eq. 17A.9-2, into Eq. 17A.9-1 gives 2.2646 x 10-® ( 1 1 ) 1 Ma * Mae) GAS Tea/PeayPP.2746) ae pie ne which verifies Eq. 17.2-2, within the uncertainty of the coefficient 1.01 determined from low-density self-diffusion data. (cDaaze = 1.01 (b) Evaluation of the component parameters in Eqs. 17A.9-4,5 according to Eqs. 1.4-11a,c gives rr 1B 1/6 oan = 2.44 (®) (2) =244 (222) pea) \ pew PeaPeo 5AB 1 4.7000E-01 4.7273E-01 --2.7282E-03 0.58 -2.8533E-01 -1.22428-03 2 6.9000E-01 6.9072E-01 -7.2080E-04 — -0.10 -4.1690E-01 =1.6487E-03 3 '9.4000E-01 9.3925E-01 7.5217B-04 0.08 «= -5.6691E-01 -2. 1098-03 a 1.2200E+00 1.21618+00 3.8625E-03, 0.32 -7.3404E-01 -2.6078E-03 5 1.5200E+00 1.5198E+00 2.24345-04 0.01 --9.1731E-01 —-3.1524E-03 6 1.8500E+00 1.8489E+00 1. 0664E-03 0.06 = -1.1160E+00 © -3.7483E-03 7 2.20008+00 2.2026E+00 -2.6318E-03 0.12 -1.3295B+00 -4.3978E-03 3 2.5800E+00 2.5801E+00 -5.1116E-05 0.00 © -1.5573E+00 5. 1009E-03 NUMBER OF MODEL CALLS . 1B NUMBER OF ITERATIONS. 4 11-23 18A.1 Evaporation rate. Let A denote chloropicrin and B denote air; then Eq. 18.2-14 gives, for constant total pressure p and ideal gas behavior, = PAB iy (2) a7 Goa) Vee = P_Dap wm (222) RT (22 — 21) PBI Inserting the values T = 15°C = 298.15K, pp: = p = 770 mm Hg and pp, = 770 — 23.81 = 746.19 mm Hg, we get (770/760) atm (0.088 em?/s) | (_770_ (82.06 x 298.15) cm@atm/g-mol (11.14 cm) 746.19 = 1.03 x 107* g-mol/cm?-s Finally, the evaporation rate in g/hr is Wa =Na:MaS = (1.03 x 107* g-mol/em?-s)(164.4 g/g-mol)(2.29 cm?)(3600 s/hr) = 0.0139 g/hr [S-1 18A.2 Sublimation of small iodine spheres in still air. (a) From Table E.1 and Eqs. 17.3-14,15, we get the following values for the system Ip-air: Species M aA elk, K Ak 253.81 4.982 550. B: air 28.97 3.617 97.0 AB 4.2995, 231.0 Thus, at T = 40°C = 313.15K, we get the argument value KT/eap =313.15/231.0 = 1.356, at which Table E.2 gives Qp,ap = 1.251. Equation 17.3-12 then gives 1 1 1 = 5 % Dap = 0.0018583,/T" Gar + is) _ 1 1 = = 0.00185 5 a 9.00185834/ (313.15) (aa si) CATO EFS = 0.0888 em?/s (b) Equation 18.2-27, with rz + oo, gives Wa = 4aricDap in ( (747/760 atm)(0.0888 em?/s) ©™) 9.06 x $13.15 em*atm/g-mol) .95 x 10~* g-mol/s x 3600 s/hr .06 x 10-4 g-mol/hr {8-2 18A.3 Estimating the error in calculating the absorption rate. Equation 18.5-18 gives Wa = KesoVDap in which K is a product of known quantities. Then the error in W, resulting from small errors Acay and ADagp is AW, = (4) Acao+ ( Wa ) ADap dcao Dap Keao =KVDapA ap, awAcaot 9 ay ADas Division by W then gives the fractional error expression AWa ., Scan 14D ap Wa cao 2 Dap Hence, the maximum absolute percentage error in the calculation of W, under the given conditions is = 5% + 3(10%) = 10% 13-3 18A.4 Chlorine absorption in a falling film. ‘The absorption rate is predicted by Eq. 18.5-18, which may be rewritten in terms of the average film velocity by use of Bq. 2.2-20: Wa = 2nRLego\) ‘The solubility is p2ao/Ma (0.998 g soln/cm®)(0.00823 g Cl2/g soln)/(70.91 g Clo/g-mol Cle) 1.16 x 10°* g-mol Cl/cm* ca ‘Then the predicted absorption rate is Wa = 2n(1.4 x 13 cm?)(1.16 x 10~* g-mol Clz/cm*) 6(; em?/s)(17. (G3 em) = 7.58 x 1075 g-mol/s = 0.273 g-mol/hr 18-4 18A.5 Measurement of diffusivity by the point-source method. (a) Directly downstream of the source, the distance s from the source reduces to z; hence, Eq. 18C.1-3 takes the form __ Wa © 4nDapz Therefore, the injection rate Wa required to produce a mole fraction x4 * 0.01 at p=1atm and T = 800°C= 1073K at a point 1 cm downstream of the source is Wa = 4nDap(0.01c)z = 4nDap(0.01p/RT)z = 4x(5 cm?/s)(0.01)[(1 atm) /(82.06 x 1073 em*atm/g-mol)](1 em) = 71x 107% g-mol/s cA (b) Expansion of s in powers of r? at constant z gives, to second order in r, and and z 1 2ah-i5 s [ zat ] through order r Wa 4nDapzs =ca(0,2) bass | [2~ co/2Danie [35 ca(r2) = Zexpl- (v0/2D.p)(s — 2)) = ea(02) [1-35 (+ 02/2Dan)) +0 (5)] ‘Thus, c4(r,z) and z4(r,z) will be within 1% of their centerline values as long as the second-order term within the square brackets does not exceed 0.01. The deviation 1, of the sample collector from the axis, at the given conditions, therefore must satisfy 22 1+ v07/2Dap _ (0.02)(1 em?) ~ T+ (50 em/s)(1 em)/(2 x 5 cm?/s) 0.00333 cm? re $ V'0.00333 cm? = 0.058 em ry < (0.01) Hence, 18-5 18A.6 Determination of diffusivity for ether-air system. (a) Equation 18.2-17 gives Dap Ma At’ ”p in(en2/eB1) pMAn RT (2-4) Ma At p In(p/(p ~ pa,vap)) for each finite increment At of time and ~Az; of decrease in liquid height. Insertion of the given constants gives (0.712 g/em*) (0.2 cm) 82.06 x 295.15 cm*atm/g-mol) Dap, em?/s = + (74.12 g/g-mol) (At,s) (747/760 atm) (2A, em) In(747/(747 — 480)) (24642 cm’ /g-mol gas) (0.2 em) (z2 — 21, cm) ~ (041 em*/g-mol liquid A) (At, s) In(747/267) _ : (22-1, cm) = eee (Ais 's for 0.2 cm level decrease) and the following calculated values for the six time intervals: (22 — a1), av., em 1.0 15 2.0 2.5, 3.5, 45, Atfor Az =—02cm 590 895 118514802055 2655 Dap, cm?/s 0.0780 0.0771 0.0779 0.0777 0.0784 0.0780 ‘The average of these values is 0.07785 cm?/s. (b) Conversion of the above result to 760 mm Hg and 0°C, using Eq. 17.2-1 in its nonpolar form, gives Dap = 0.07785 x (747/760) x (273.15/295.15)!*"* = 0.0664 cm?/s This result appears preferable to the one reported by Jost. 18-6 18A.7 Mass flux from a circulating bubble. (a) With the data provided, Eq. 18.5-20 gives the surface-average mass flux 4ADapir (Nadeg = Vp ea (4)(0.46 x 10-* em?/s)(22 em/s) .041 x 107° g-mol/cm*) =1.17x 107 g-mol/em?-s (b) Equation 18.1-2, with the surface-averaged k, obtained by Hamerton and Gamer, gives (Na)arg = Fe(c40 ~ eve) = (117 em/hr)(1 hr/3600 s)(0.041 x 107 g-mol/em*) = 1.14 x 107° g-mol/em?-s \8-7 18B.1 Diffusion through a stagnant film—alternate derivation From Equation 18.2-1 we get di, Naw ine Oe or 1 te Na Xp dz Dap Integration gives — At I? de pete Hence Which is in agreement with Eq. 18.2-14 13-8 18B.2 Error in neglecting the convection term in evaporation a. Without the convection term, Eq 18.2-1 and 4 become N, ‘Az 2 = Hcy He and ne =0 Integration of the latter equation twice then gives x, =C2+C, Then application of the boundary conditions gives two equations: xm eC tC, and x4) =C\z, +C, These equation may be solved simultaneously to give Xa -%, Xa -X, Xa =%a2 and C,= x4, +247 2a2z, -% y-% Gq Therefore, in the approximation being considered here, the mole- fraction profile in the system is given by Xm—Xa _ 2% Xa~Xan 2; b. To get the result in (a) from Eq. 18.2-14, we can expand the latter in a Taylor series, as was done in getting Eq. 18.2-16. c. To get the solution of Example 18.2-2 by using the result in (a), we make the following calculation NgRT _Na,RT(z)—%)_ (7-26%10~ )(82.06)(273)(17-1) By, 5 48 p(-dx,/d2) plea) (755/760)[(33/755) -0] =0.0641 cm?/s Hence the error is 2-2641= 0.0686 ,. 199 - 0.79% 0.0636 18B.3 Effect of mass transfer rate on the concentration profiles a. Rewrite Eq. 18.2-14 as Neg = £248 Jy Lo tan or 1242 «exp Nal“) Bom TxXq Tim Dag When this is substituted into the right side of Eq. 18.2-11, we get waa) og Nale=)) CD ag cBay b. Starting with Eq. 18.2-1 and integrating directly we get 1d_ oN x 1 Naz fe 4_ Na d a dy 2 Na pg, T-x, dz cB VnTmay re Se When the integrals are evaluated we get dx, _ Na _ Toag Dag 2) Changing signs and taking the antilogarithm of both sides then gives the result in (a). c. Expanding the right side of Eq. 18B.3-1 in a Taylor series in the argument of the exponent, we get Nez cBap (z-2) If we retain just two terms in the Taylor series, and bring the "1" on the right side over to the left side, we get Naa(1-¥ CD sg -%) X4=Xy- which is of the form x, = mz +b, that is, a straight line function. \g-10 18B.4 Absorption with chemical reaction a. Equation 18.4-8 remains valid, but now the boundary conditions are: at =1, T=1; and at £=0, dl'/d¢=0. Hence the boundary conditions lead to a pair of simultaneous equations: 1=C,cosh+C,sinh@ and 0=04+0,6 from which it follows thatC, = 0 and C, = 1/cosh@. Then the analog of Eq, 18.4-9 is T =cosh ¢¢/cosh@ The mass flux at the liquid gas interface at z= L is then: which leads directly to the result in Eq. 18.4-12. b. We start with Eq. 18.4-7, the solution of which can be written in the form of a superposition of exponentials T=Cye% +C,e-% which is to be solved with the boundary conditions given just above Eq. 18.4-8. This leads to the following equations for the constants of integration: 1=C,+C, and 0=C,ge* -C,ge* These two simultaneous equations can be solved to give 1 e? Cad ef = and C= = +e + Tre? P +e q Therefore , the dimensionless concentration profile is \8-il eM-s) erate] cosh[9(1- £)] 4fe? +e? ] cosh @ Thus we are led to the same result that we obtained in Eq. 18.4-9. c. Equation 18.4-12 can be written thus 8.) (e . lee Nol, =( £40@ae ) [Ri sea a Therefore, for very large L we get (using an expansion appropriate for large values of the argument) ee Nala can sak; (1 2exp(-2k/1? [®an)*-) > cao (Baak” For very small values of L we get (using an expansion appropriate for very small values of the argument) Natl ira md x0 V Beak (3) Dap As L becomes infinite, this becomes ca ig alte (Ee, —4 =cosh,|——z ~ sinh Z=exp| — cao YB as Bea Be As L becomes zero, the dimensionless concentration becomes unity. (8-12 18B.5 Absorption of chlorine by cyclohexene a. For a second-order reaction, Eq. 18.4-4 has to be replaced by Pe, 1» Saget keh with the same boundary conditions as before. Introduce the dimensionless variables P=¢,/¢4q and ¢ = kiE,9 /6D45z. Then the differential equation becomes 2) ao =0 with boundary conditions [(0)=1 and I'()=0. We now let aT /dg= p(T), so that d?*P/dg? = dp/dg = (dp/dl)(aP/d€) = p(dp/ar). Then we obtain a differential equation that is first order and separable 4p _ 6p? ® or _ This may be inegrated, and we use the boundary condition that at ¢=00, P=0 and also that dP'/d¢ = p(T) =0: i. oe Sfrdp =|, rar from which ar poet =r" P or i Here we must choose the minus sign, since the slope of the concentration vs. distance curve is negative. Then using the first boundary condition, we can integrate this equation to get: firvar=-ffag or T4=(14e) 18-13 Hence the final expression for the concentration profile is 2 Cao [Eno 14 [Heso ce-[te (caus) b. From the result of (a) we get the absorption rate at the liquid gas interface: de, Nazlno =~ SanGe| = (ES c. The equation to be solved is 4+ f(ca)=0 or pth. flea) dc -8 ABT? Pac, 7 where we have introduced the variable p as before. The resulting equation is integrated, as before, to give a 1 pea pre ye Sipap = pb Seales or A second integration yields (* V(2/Baa)f, Then we differentiate both sides with respect to z to get 1 dey (as) fea)aen de =-1and 4] ne kz ~\(2/B an)" Flea )dca This together with N jz|,. =~ Sn (de4/d2)),_, gives Eq. 18B.5-2. S$-\4 18B.6 Two-bulb experiment for measuring gas diffusivity--quasi- steady-state analysis a. The molar shell balance on Az gives for species A SN shane =0 lnsae Division by Az and taking the limit as Az goes to zero gives aN ,,/dz=0 or N 4,= constant. b, Equation 18.0-1, for this problem, may be simplified thus: N pg= ~CB gp (A 4 /dz) +x 4(N g.t N p.)= ~CB gp (dx 4 /dz) since N 4,=—N ,,; this is true, because in a system at constant c for every molecule of A that moves to the right, a molecule of B must move to the left. c. The equation in (b), with N ,,= constant, then becomes dx ,/dz=-(N 4, /C® 4g), which when integrated becomes N Niaz X 4a z+C, "CDs 1 d. Atz=L, x ,=x4, So that neNericg, cD ap Subtracting the last two equations gives then xa- eel z) e. At z=—L, we know that x ,= x, =1- x4, so that N x, See ee (1-x3)-x4 = (L-(-0) from which Eq. 18B.6-2 follows directly. 1$-15 f. A mass balance over the right bulb states that the time rate of change of moles within V must exactly equal the rate at which moles enter V by diffusion at the end of the tube. That is (vex) =5N a or volta =5(4- x4) Das L in which S is the cross-section of the connecting tube. g. The equation in (f) may be integrated J ‘The integration constant may be obtained by using the fact that at = 0, we know that the mole fraction of A in the right bulb will be zero, or C, =—In(}—0). Therefore dog In(J-x3)-Ing=-SSa0; or 2 4 = exp SBat) h. If we plot the slope will be the diffusivity, B 4p. 3-16 18B.7 Diffusion from a suspended droplet a. A mass balance on A over a spherical shell of thickness Ar is (in molar units) 4m? Nal, -4n(r+ Ary -Ngy|,.4, =0 or, equivalently (427°N,,), - (4m? N4,)) near Now divide by 4Ar and take the limit as Ar goes to zero to get d FA (PN 4,)=0 This may be integrated to give r?N,, = C,. We may use the boundary condition that N4,=Nj; at r=7, (the gas liquid interface) to evaluate the constant and obtain r?N4, =1?Nan- b. Equation 18.0-1, written for the radial component in spherical coordinates, is =-cf, A+ x4(Na, +No,) 0 If gas B is not moving, then N,, may be set equal to zero and the equation may be solved for the molar flux of A: CD gy dx Ny, =—o 4 Ba a Y-x, dr Multiplying by r? and using the result obtained in (a), we get Eq. 19B.7-1: Disp 2 in FIN gy = ~ 2 2 lin non XB When r, — © (and presumably also x, — 0), this last result gives 18-18 18B.8 Method for separating helium from natural gas Let A be helium and B be pyrex. Then a shell mass balance gives the following equation d ap 'Nar)=0 Insertion of Eq. 18.0-1. with N,, =0 and x4 <<1, gives for constant diffusivity 7. wl aa Integrating twice we get c,=C,Inr+C, The boundary conditions are: at r=R,, c,=C,4, and at r=R,, C4 =C4y- Evaluation of the constants of integration gives for the concentration profile: fa ~Cap _ In(Ro/r) €a~Ca2 In(R2/R;) Then the molar rate of diffusion through the wall is de, _ + Bae(Ca ~ a2) Na =—Ba8 Ge = rIn(R;/R,) and ee 2k Da (cuca) Wasdale Ne= a(R /R) is the molar flow rate of the helium through the pyrex tube. 18-19 18B.9 Rate of leaching a. The molar balance for substance A over a thin slab of thickness Az is Division by SAz and letting the slab thickness Az go to zero yields aN 4, Then inserting an approximate version of Fick's law gives de, N, ABT 2D which is good for a dilute solution of A in B. Thus the diffusion equation becomes @ey =0 de b. The above differential equation may be integrated to give ec, =C\z+C, The constants are determined from the boundary conditions that C4 =Cqq at z= 0, and Cy = Cys at z= 5. The final expression is then C4 -¢, z 4-6, Af 22 op A ~Ca5 Cas emo 8 Cao Cas oN c. The rate of leaching (per unit area) is then de 1 Dap (Cao - ©. 18-20 18B.10 Constant-evaporating mixtures. (a) For this one-dimensional, steady-state, nonreactive system, the species con- servation equations take the form dNq:/dz = 0, and give Naz = const. for each species. The coefficients cDyg depend only on T in low-density systems, according to Eq. 17.3-16 and the corresponding formula of Mason and Monehick for polar gas mi tures; thus, each of these coefficients is predicted to be constant over this isothermal system. Assuming insolubility of nitrogen (3) in the liquid mixture, we obtain Nz then Eq. 17.9-1 gives dus _ ee dz Dis cDaz as the Maxwell-Stefan equation for ys. Integration and use of the boundary condi- tion at z= L give Nis | Nos Diy | Das tay = [ Je-o = A(z —L) whence tn = exp[A(z ~ L)] (18B.10~ 1) Equation 17.9-1 and the condition y + yz + ys = 1 then give as the differential equation for the toluene mole fraction profile y:(2). Insertion of the notations B, C and D, and the solution for ys, gives \g-2! which has the form of Eq. C.1-2, with f(z) = —B and g(z) = —C + Dexp[A(z— L)]. Using the solution indicated there, we get un = exp(Bz) [f exp(—Bz){—C + Dexp[A(z ~ E]}d2 + x] a iet™ 06m" fects pete f dominate 7 = KP CoE 4 paloean f (A-De ‘Thus, the toluene mole-fraction profile is D Cc nz) = fae—ny) _ [CE wl2)= Ze [5 in agreement with Eq. 18B.10-1. (b) Numerical results of the suggested calculation procedure are shown in the following graph, prepared by Mike Caracotsios. Cubic spline interpolation was used to get equilibrium vapor compositions over the tabulated range of liquid composi- tions. The interfacial vapor composition yio calculated from the equilibrium data ‘becomes equal to that calculated in step (iv) at a liquid composition 210 = 0.192. 18-22 Trterfacial Interfecial ‘4 Vapor molar toluene fraction + Vapor molar toluene fraction from Diffusion from Equilibrium 0.0 O41 0.2 03 04 Liquid molar toluene fraction 18-23, 18B.11 Diffusion with fast second-order reaction a. A shell balance on species A plus Fick's first law of diffusion leads to , 4's <9 which has the solution Xp=Cz+Cy a When the constants of integration are determined from the boundary conditions that x4(0)=xj,. and x,(8)=0, the mole-fraction profile becomes x4=4a( 1-3) Then the rate of dissolution of A at the solid-liquid interface is 2 dz =D astao <0 5 Nash =~€®as b. For the system pictured in Fig. 18B.11, we have the following differential equations ax, az 2 =0 (for 0 = Jim|W |= lim n(24R?8 40,4 tanh(Ab/n)) = lim: (22RD 4c4,A[(Ab/n)+---]) = 2AR?D 4c4,A°D = 2AR* OK {Ac 5, Then the effectiveness factor is 2nR29,cq,A tanh Ab _ tanh Ab Ma = lin W/O) Rc ab {8-30 18B.15 Diffusion and heterogeneous reaction in a slender cylindrical tube with a closed end a., The mass balance over a small segment of the tube is Maz], S—Mazleyae5~ PASF(@a0)=0 eras Division by SAz and letting the segment thickness go to zero gives _ diy, de ~FFlO40)=0 b. The mass-average velocity is given by VD, = O40, + Opp, At steady state, B is not moving so that vz, is zero; if substance A is present only in very small concentrations, then is quite small. Hence v, =0. c. Because of the result in (b), we can write the mass flux as = By 24 = Then the diffusion equation becomes @o, PBan pa Ef (o4)=0 or Po, PL = Kio, = dg? SpB,,' “ Here we have set the wall mass fraction @4y equal to that in the main stream @, and we have introduced explicitly the first order reaction kinetics. We have also switched over to a dimensionless coordinate {= 2/L. \$-31 The differential equation is to be solved with the boundary conditions: BCd: at€=0, @,=04, B.C.2 at =1, da,/dz=0 The general solution is 4 =C,coshNC +C, sinhNE in which N = \PL’k//Sp,,. When the constants are determined from the boundary conditions we get a —UrUr”—— On coshN _ cosh NC coshN ~sinhN sinhNE coshN _coshN(1-¢) ~~ coshN d. The mass flow rate of A into the capillary is do, = PD ans do,| Lal len __ P® 4x54; (~N)sinh N(1- 6) - L cosh N = PD apSO gi L Waleag = ~PDaB5 <0 (NtanhN) 18-32 18B.16 Effects of temperature and pressure on evaporation rate a. For ideal gas behavior, x41 =Py,pi/P, a8 pointed out several paragraphs before Eq. 18.2-1. Since the vapor pressure increases with temperature, according to the Clausius-Clapeyron equation, x4; will also increase. When the total pressure increases, X41 Will decrease. b. The evaporation rate is given by Eqs 18.2-14 and 15. Since, at constant temperature, cs p and 4, «1/p, the evaporation rate should be nearly independent of the imposed pressure. c. The temperature dependence of the evaporation rate can be estimated as follows: ce 1/T and (for the simple kinetic theory of rigid spheres) © ,, «T°. This suggests that (Evaporation rate at T’ : Evaporation rate at T ) = (T’/T)” However, it is known that the kinetic theory of rigid spheres underestimates the temperature dependence. If one reads off the slope of the curve in Fig. 17.2-1, we estimate that (Evaporation rate at I’ : Evaporation rate at T) ~(T’/T)"* which is a more believable result. [8-33 18B.17 Reaction rates in large and small particles a, Equation 18.7-11 for small R becomes, on using the expansion of the hyperbolic cotangent for small arguments Wag = 42RD 4cag[1— (14 4 (kia/-,)R?+-)] = (dR Rem where we have retained only the leading term. For large R, we use the expansion of the hyperbolic cotangent for large arguments (which may be found in mathematics handbooks) to get W ap = 42RD 4c an[1- (AR + 2ARexp(-2AR)+---)] =-(4AR?) [KAD scan where A = /kja/®, , and only the leading term has been retained. Note that for small R the rate of disappearance of A is proportional to the volume of the catalyst, whereas for large R the rate is proportional to the surface area. b. Equation 18B.14-2 for small R becomes, using the expansion for the hyperbolic tangent for small arguments [Wa]=2-AR?c4,9,4(Ab-42°D? +---) = (2baR*)(kya)cg, Here, again, we let A = /k/a/®, , and we have kept only the leading term. For large R we use the expansion for the hyperbolic tangent for large arguments to get [W,|=2-R2c4,8.44(1-2exp(-2Ab)+---) =(2aR?) {kia aca. Here again |W ,|e volume for small R, and |W |e: surface area for large R. \G-34 18B.18 Evaporation rate for small mole fraction of the volatile liquid According to Eq. 18.2-13 1 _In(tro/tn)_ y= tan _ In(1-242)-In(1- m1) (x5), Xp —XB1 X41 Xaz 1 —Xar X41 ~Xa2 We now wish to expand the logarithms for the situation where the mole fractions are much less than unity. To do this we may use Eq. C.2-3. This gives Xa2 (aaa) #H(0hs ha) (ha) Xar~Xa2 naa) (ch +Xatan tha} This is the expression in brackets in Eq. 18.2-16. 18-35 18B.19 Oxygen uptake by a bacterial aggregate a. A mass balance over a thin spherical shell gives (4r?n,,)], -(427?n4,)| hea + (Anr?Ar)r, =0 in which A is stands for oxygen. Dividing by 4Ar and taking the limit as the spherical-shell thickness goes to zero, we get d LP ng) =P Then inserting Fick's first law of diffusion and introducing the zero- order chemical reaction rate constant to describe the disappearance of the oxygen, we have 1 d(_2dp, » A Abe a” dr ) The B in 45 stands for the medium through which the oxygen is diffusing. We then multiply the equation by R?/p) 4, and define the dimensionless quantities: =1/R, %=(,/Py, and N=R?ki/p Bx. Then the diffusion equation becomes, in dimensionless form ax rales) b. If we assume the presence of an anoxic (oxygen-free) core, then we have the following boundary conditions: at £=1, %=1; at &=0, y=finite; and at =), 7=0 and dy/df =0. The physical significance of the last boundary condition is that there must be no diffusion across the boundary between the anoxic core and the outer region of the aggregate. This may occur if all the material that enters across the outer surface of the aggregate is completely used up by the time it reaches the spherical surface at & = £5. c. Two integrations of the above equation give: ry = ke 18-36 BELAINE AC, and = 4NE-CE"4C, The constant C, can be found from the boundary condition at § =1, so that x= 4N(E*-1)-C,(E4-1)+1 We now have to apply the two boundary conditions at & = &, in order to determine the two quantities C, and & (which as yet is unspecified): a $3 L=0at E=E, 0=EN(&-1)-C,(E51-1) +1 dy/aé =0 at £ = &4: O=4NG +E or 4NG} Elimination of C, between the last results gives, after rearranging (#84) If N is very large, the cubic relation has the roots 1, 1, and ~}. The last of these is physically impossible, and the other two roots indicate that if the reaction rate is exceedingly large, there the anoxic region will occupy the entire sphere. If we set €, equal to zero, we find the minimum value of N, namely N = 6. Below this value, there will be no anoxic region. Thus the final expression for the concentration profile is: 4 =1-4N(1- 6?) +4NE}(E7-1) for 12€2&)20 and 7=0 for 0<é<£, Equation (**) above gives the boundary of the anoxic region in terms of N. 18-37 18C.1 Diffusion from a point source in a moving stream a. Make a mass balance (in molar units) over the ring-shaped area element shown in Fig. 18C.1: (2ardrN 4.)|, -(2a7ArN4,)],,., +(2aAzrN 4, ), -+(2aAzrN 4, |, = 0 Divide by 2AzAr and take the limit as the dimensions of the ring- shaped element go to zero: (NasJsege (Nach. 4, (Nardheae ~(7Narl, = + lim =0 a0 Az a0 Ar Then using the definition of the partial derivative, we get aNac , 2, 2Nae) _ 12, ac), ae. Ne Ar Nar) <0 a 13( 1M +a <0 We now use the expression for the molar flux given in Eq. (D) of Table 17.8-2: In getting the approximate expressions, we have assumed that there is negligible convective diffusion in the r direction, and that the distinction between molar and mass average velocity is unimportant in this system. These assumptions would be valid if the concentration of A in the mixture is small. When these expressions are inserted into the equation for the molar flux, we get Eq. 18C.1-1. b. To make the change of variable, we use ¢4(7,z)=C4(5,2), and apply the chain rule of partial differentiation as follows: (F)-(BE8-B)-BH%), (3-38 z 2%) } (a(% ) Peyz 1d, , Peg 24(Fazside, oe (% 1a re) Va (%) edie) su = +3( Ha) st+z? rarL ar Jz} slas| Var J. |). (as) 2 "sas J. # When these are substituted into Eq. 18C.1-1, we then get Eq. 18C.1-2. c. Let AnD 1 a f(s,z)=¢, Ww, a toa - 38, Then Bee Qebe Qx (#) =0'f; 23( 22) = {208 - 20°F) When these expressions are substituted into Eq. 18C.1-2, an identity is obtained. c. B. C. 1 is clearly satisfied. To examine B.C. 2, we first have to calculate the derivative [8-39 a, __Wa ( Jews Learn) a AnD s,\ 8? s Then AnD yy a =We%)(1—a8) When s is made to go to zero (which implies that z goes to zero as well), the right side of the above equation goes to W.4, and therefore the second boundary condition is satisfied. To examine B. C. 3 we have to calculate the derivative with respect to r: (8) tagoren: 3 ).8 4tDay where s= Vr? +2”. When r—>0,s goes to z, and B. C.3is satisfied. The meanings of the three boundary conditions are: B. C. 1: The concentration of A on a spherical surface at infinite distance from the injection point must be zero (since A is diffusing in all directions) B.C. 2: This is a statement that W., is the injection rate of A. B.C.3: This means that the maximum in the concentration must be on the z axis. e. To determine the diffusivity, one can write Eq. 18C.1-3 as In(c4s) = In(W 4/429 45)-(%/2B48)(8-2) Hence, if c,s is plotted vs. (s-z) on semi-logarithmic paper, the slope will be — (0/29 4g) and the intercept will be (W ,/478 45). For more on this subject, see T. K. Sherwood and R. L. Pigford, Absorption and Extraction, McGraw-Hill, New York (1952), pp. 42-43; H. S. Carslaw and J. C. Jaeger, Heat Conduction in Solids, Oxford University Press, 2d ed. (1959), Eq. 2; H. A. Wilson, Proc. Camb. Phil. Soc., 12, 406 (1904). }8-40 18C.2 Diffusion and reaction in a partially impregnated catalyst Use the notation of §18.7 as well as the following dimension- less quantities: E=r/R, T=ca/cag, and ¢=kgaR?/4. Super- scripts I and Il indicate the two regions. a. In Region I, the diffusion equation and its solution are “ 7 rales \-0 and r=-Gach In Regiion II, the diffusion equation and its solution are (cf. Eqs. 18.7- 6and 9) A F er and re = Feoshos + Fsinh og The boundary conditions that have to be satisfied are: B.C: at&=0, Mis finite (whence Cj = 0) B.C.2: até=x«K, rY=r) B.C.3: até=x, dO /dé=ar/dé BC4: até=1, rM=1 Boundary conditions 4 and 3 give us the following two equations: 1=Cll cosh ¢ +C}! sinh 1 ~Heosnge +S “L gsinh ox - Z sinh ox + = gcosh ox These two equation may be solved simultaneously to get i ~sinh ox + 9K coshoK | “sinh 6(1- x) - «cosh 6(1+ kK) \$-41 u_ cosh ¢x - ¢xsinh ox 2 “sinh 9(1— K) - «cosh 9(1+ x) Therefore the concentration profiles in the two regions are: (‘nn OK+ ieee pelaange | pity LL +(cosh 6x ~ gxsinh ox)sinh 6 E sinh 9(1— x)- ox cosh (1+ x) r= on © €sinhg(1— x)—cosh (1+ x) To get the concentration profile for region I, we used B.C. 2. b. To get the molar flux at the outer surface, we need dr ar”) W, =-40R°D, = AARD sean Ge | =k Ise After evaluating the derivative at the surface, we get finally _ __ (cosh o(1- x)++ pxsinh o(14+ x) ae 41k én) sinh 6(1— x) - oxcosh (1+ »)| In the limit that k > 0, this result simplifies to Eq. 18.7-11. 18-42 18C.3 Absorption rate in a falling film a. The total moles of A transferred per unit time across the gas-liquid interface is W ,. This has to be equated to the amount if A that is leaving in the film of finite thickness 5: W, =Whecalea),_v.(xddx When it is assumed that A diffuses only a very short distance into the film, then v, (x) may be set equal to the fluid velocity at the gas-liquid interface, V, m4, and taken outside the integral. Furthermore, since c, (2,2) is virtually zero beyond a distance small compared to 5, the integration can be extended to infinity. This reasoning leads to Eq. 18C.3-1, b. Inserting c4(x,z) into Eq. 18C.3-1 and changing to the variable u requires no further explanation. c. Changing the order of integration requires specifying the region of integration. In this case it is a triangular region extending from u=0 to u=-,and from the diagonal line € = u across to € =<. When the order of integration is reversed, it is necessary to integrate over exactly the same region, but this time from €=0 to =e and from u=0 up to the diagonal u = &. This leads to Eq. 18C.3-3. Having done this, the inside integral can be performed analytically to give 4D 450: max Wa = WLe go |“ f° Eexp(—E?)dé Now the remaining integral can be done also to give which is in agreement with Eq. 18.5-18. 18-43 18C.4 Estimation of the required length of an isothermal reactor a. The steady state mass balance over a length Al of the reactor is WO go; ~ WO ao),,9) ~ (SAl)artg = 0 Dividing by wAl and letting Al go to zero gives dora, __Sany dl w or doy) _ SAN,M4 dos b. Next we want to use the result of Eq. 18.3-9 to write MyMs(dtqo/dl) _ SaM4( 2cBp 1 (Max40+MyxXm) © 8 1-440 Then integration gives ( M,wd ) a(L) AX py 28ac 4s )"*4[M4x49 +Mg(1-X40)} In(1- 4x40) is-44 18C.5 Steady-state evaporation a, We start with the second form of Eq. 17.9-1, written for diffusion of a 3-component mixture in the z direction. For convenience, we omit the index z on the species molar fluxes: dx, 1 me x Ng -X,N, dz fACDap 2B, (%eN0 0a) Consider now the concentration gradient of species 3, taking into account that N, and N, are constants, and that N, is zero: dx. Be 1 Ny, No fo cpg (eNt oe apy (eN2)= Oba}? This first-order, separable differential equation is easily integrated to give the result in Eq. 18C.5-1: X3 = X39 exp( Vins + Vans )O = X59 EXPAT in which the notation v,,, is a symbol defined in the textbooi. b, Next we consider the concentration gradient for species 2: ee de apg tN —*N2)+ 1 (x2N3 ~¥5N2) 23 or, in dimensionless form, Yina¥2 — Vara%i)~ Vaas¥a= Vira%2 ~ Vara (1- ¥2— 3) Vazss = (Visa + Vate )¥a + (Vara ~ Vaag)3 — Yarz When we use the abbreviations given in the text, this equation can be recast in the form dx, Fe 7 (Mua + Vara) ae (Yara ~ Voas X30 €XP( Vira + Vaaa)$— Ya. OF [3-45 dx Te B= Cea PAL Man This is a first-order linear equation, which is readily solved x =e [[ger (Ceme*S Yaa ag oC (C, = X39 from B.C.) emmy eM = (Cry (a= nye re a = CX gat #(29- Cx59 _ Vor \yay 4 Yar A-B A-B B B in which Yorn No 900 = yp) (Nit No)? |, gag Ni, No BON,+N, °°? cw, fF © PLC, eB)” 23 Dp Nx a. 8, -8 N 13) 4 22-8 Pap) Pa) c. The results in (2) and (b) above are in agreement with Eqs. (2) and (3) of the article by Carty and Schrodt, cited in the text. d. Figure 3 in the article by Carty and Schrodt indicates good agreement between the above equations and the experimental data. \8-46 18D.1 Effectiveness factors for long cylinders Instead of Eq. 18.7-6 (for spheres) we have now (for cylinders): hr) «kay with c4(R)=cg4n and c4(0)= finite The differential equation may also be put into the form 1d te) kia =| r—4]- ae ra - This may be recognized as the equation for a first-order modified Bessel function. Therefore, the solution is i.) Inasmuch as Ky cannot satisfy the boundary condition at r = 0, we must set C, equal to zero. The boundary condition at r = R then gives the constant C,. The concentration profile is therefore ta (kva/® 41) Can To(Vkya/®,R) c= a Ve ; O a A This is the cylindrical analog of Eq. 18.7-9. The molar flow of A at the surface is then War= anki, ta) __ (22RL) 8. Gg - —R). Re - Caro) { ica) | ka], R)-\kya7® which is the cylindrical analog of Eq. 18.7-11. The analog of Eq. 18.7- 12is then \%41 W ano = (#R7L)(a)(-kica) Next we evaluate the effectiveness factor _ Wan 21, (k7a/D,R) * Wana ([kia/8,R)lo(Vk7a/,R) The generalized modulus for the cylinder is aw, |HaVe _ [eva mR?L _ [fa R DB, 5, \Oq 2aRL \B, 2 Therefore the argument of the Bessel functions is \Ra]®,R=2A We may now express the effectiveness factor in terms of the generalized modulus thus: n, 1,(2A) AIy(2A) Na which is the desired result. 18-4? 18D.2 Gas absorption in a falling film with chemical reaction The problem to be solved is a4 Fey Pp GA = Bas Got Kb with boundary conditions ¢4(x,0)=0, ¢4(0,z)=Cagy and c, (%,2)=0, or, in dimensionless notation $a with ¢(£,0)=0, e(0,£)=1, e(s»,£)= in which ¢ = ¢4/ 49, = 2/L, €=XDing/Dagh, and a= KIL] max We now take the Laplace transform the partial differential equation and the boundary conditions to get st-(£,0)= 4 — ae with @(0)= Ys, (ee) =0 oe Because of the boundary condition at £= 0, c(E,0) = 0. The solution of the ordinary differential equation for ¢ with its boundary conditions is e= texp(-y (s+aé) We will not invert this, since all we need is the molar flux at the wall, de =D 4x09 | 2 0 ee Dol dE The Laplace transform of this expression is de Noes =-S ase gee Dalal, a Hence the molar flux at the wall is a, Naz(2)|ep =~ Sas = Tan Veta = +9 gla | ge a 13-49 Ax(Z) yao = + DasCa Dal 3 os pal sta =F ao (es . Batol pee" ekg } ed fe Te iI = +8 aio Be [ee at), eee | The first transform was obtained from a table of transforms, and the second by using the convolution theorem. The total molar flow rate through the interface is then coe pe Wa =WEN ax (2g =WLD ssC a9 Fa (ered. eae aah 7 Umm erfva , (erfVa | exp(-a) _erfva oe al ae ae ~ 1.0 are: Dap = 0.246 cm?/s from Eq. 17.2-1 c= p/RT = 4.18 x 1075 g-mol/cm? k = 25.5 x 107° W/em-K from CRC Handbook 2000-2001, p. 6-185 The molar heat capacity of the transferred vapor at Ty is Oya = 8.00 cal/g-mol-K = 33.47 J/g-mol-K from O. A. Hougen and K. M. Watson, CPP Charts, Wiley, New York (1943), Fig, 26. Substituting into Eq. 19.4-5 we get: NayOpad _ (Navd\ (ePanCya) _ (,,, 1-248) ( DanCya k” \cDap k 7 ANT = ta0 k 10.0180 ~ G 7 ais) 0.000418 x 0.246 g-mol/em-s)(33.47 J/g-mol-K) -( 25.5 x 10-5 W/em-K 0.0081 —0.00599)(1.350) = Then the right-hand member of Eq. 19.4-9 takes the value —(NayCpa/k)5 0.0081 0.0081 ‘exp(NayCpa/k) 1 —exp(—0.0081) ~ 1— {1 — 0.0081 + 4(0.0081)? + = 1.004... roosty+... 0 (b) From Eq. 19.4-9 we see that the quotient just calculated is the ratio of the interfacial conduction fluxes calculated with and without allowance for diffusive energy flux. The diffusive energy flux is evidently unimportant in this problem. 9-1 19B.1 Steady-state evaporation a. From Eq. (M) of Table 17.8-1 and the fact that B is stagnant leads to N,, =cv,. Equation (D) of Table 17.8-2 states that N4, =C,0, ~ C94 (dx,/dz). Equating the right sides of these two expressions and dividing by c gives dx, + Bay dts apt oor Ut dz I-x, dz x,0,-8 b. When this expression for the molar average velocity is combined with Eq. 19.1-17 (when simplified for steady state and unidirectional diffusion with no chemical reactions), we get Dap ary ay _ ax, dx, 1 (Ss) =0 Tox, dz dz" dz? j-x,\ dz which is just Eq. 19B.1-1. c. Equation 19B.1-1 can also be written as (cf. Eq. 18.2-5) One integration of this equation gives (cf. Eq. 18.2-6) 1 dx, ix, 1 and the second integration gives (cf. Eqs. 18.2-7 and 8) =In(1-x4)=Cz+C Then one can follow the text in §18.2 until Eq. 18.2-11 is obtained. 19B.2 Gas absorption with chemical reaction Equation 19.1-16 is ( 224. v-vo, For constant mass density, p can be taken inside the time and space derivatives. This gives: ( Ws a PB yV?O, +14 4¥-¥p) = 85904 +14 We now divide by the molecular weight of species A to get (4+ v-Vea] = DasV2eq4 + Ry For steady-state diffusion, the time-derivative term can be omitted. Since the bulk motion in the direction of diffusion is presumed to be small, the v term can be omitted. Then we further simplify the equation to diffusion in the z direction. This gives de, O= Sane Ra where R, is the molar rate of production of species A per unit volume. If A is disappearing by an irreversible, first-order reaction, then Ry =-K/%,, so that we arrive at @c, 0= 8, 4 - ke, a5 gat — Kita which is just Eq. 18.4-4. 19B.3 Concentration-dependent diffusivity a.. For this problem the diffusion equation simplifies to (Sula) $4) b. One integration of this differential equation gives de Baalea) Gee Ci and a second integration gives f° Balen )den ioe The integration constant may be obtained by applying the boundary condition at 2 =b: [08 Baal cade =C, ee Taking the quotient of these two equations gives Je 80 eqldcn ae 2 Baa(ca)den which is equivalent to Eq. 19B.3-1. c. The molar flux at the solid-liquid interface is then tea 4] =-& 43} ——[ “D4, (c4, de’ : a Gals an(Ca de, which is equivalent to Eq. 19B.3-2. The quantity in parentheses is the concentration gradient, obtained by differentiating the result in (b) with respect to z using the Leibniz formula. d. When Eq. 19B.3-3 is inserted into Eq. 19B.3-2, we get 19-4 = Bape 6 €4~ Eq) + Bo(ca a) + fea © Bala calttiles-t a In the second term on the right side we have (cata) = (a ~ Dena +88)- (Ca Zena +28) = (Co ~€hs)- 2m eu = (cho ~ 4») 2(cao — Ca») (Cao + ¢an) = 9 In the third term on the right side we have (ca-ea) [* ie ean 223 = (Cho - Be4oE4 + BC 407% - C2) ~(chy -3c%pF4 + 3cq404 - C2) se JEa + 3(Ca0 ~ Cas )EA = (Cao ~Can){ (cho + Ca0%an + Can) ~ 3(Cao + Cas) #(Ca0 + Can) +3(4) (cao + Can)’ | = (Cao Cav) 4¢A0 ~ F406 a0 + a6] 2 = 4 (Cao ~Cas)(C40 ~ Can) When this is substituted into the molar-flux expression above, we get the result in Eq. 19B.3-4. e. If the diffusivity is linear in the concentration, so that the terms in Eq. 19B.3-3 containing terms higher than the quadratic term may be omitted, then the result in Eq. 19.3-4 is valid, but the expression in brackets is just unity. This means that one gets a valid expression for the mass flux by using the formula for constant diffusivity, but using the diffusivity at the average concentration. 19-5 19B.4 Oxidation of silicon a. Solution of Fick's second law (Eq. 19.1-18) for unidirectional diffusion with the quasi-steady-state assumption gives c4(z)=C,z+C>. The integration constants are determined from the boundary conditions that c,(0)=C4p and c,(5)=c,4s- This gives Eq. 19B.4-1. b. The unsteady molar balance on oxygen gives -ien) The left side is the time rate of change of total number of moles of oxygen within the region. The first term on the right side is the rate at which oxygen enters an area $ of the gas-solid interface, and the second term is the rate at which oxygen leaves the system through an area of the silicon-silicon-dioxide interface, by virtue of surface reaction. The left side can be rewritten (using the Leibniz formula) as de, f fox, HdV = {- Sao iz at yy ase { Save f(n-vsjats S(t) at very ao The first term on the left can be neglected, because of the quasi- steady-state assumption. When the right side of this expression and the right side of the previous equation are equated (and divided through by S) we getEq. 19B.4-2. c. The unsteady molar balance on the silicon dioxide gives = £ Sesten t)dV = S(k’cq5) fey This states that the mass of silicon oxide increases because of the surface reaction of oxygen with silicon and also because of the change in volume of the silicon oxide region. Once again the Leibniz formula is used to rewrite the left side as { Seav §( J(n-v}ndS = ¢55822 = 3 - vit) S(t) iit Vp at i9-6 where V; is the molar volume of the silicon dioxide. Equating the right sides of the last two equations gives Eq. 19B.4-3. d. Following the directions given in the text we get Cao ~ Cas wn, a( fA) — beg and this may be rearranged to give Eq. 19B.4-4. e. The differential equation for the movement of the boundary is Oktrc2 Voki'cas = a ean 1+ (k75/®,z) or ki \ dé Vc aoky’= -(vi \¢ This is a separable first-order differential equation, which may be solved with the initial condition that 5(0)=0: Vacaokif,at= fp (1 + #2 Dap and this yields kyo? 2D ap Vacaok’t= 6+ which may be rearranged to give Eq. 19B.4-5. 19-1 19B.5 The Maxwell-Stefan equations for multicomponent gas mixtures a. If we start with the first form of the Maxwell-Stefan equations, we have Vx, =-24%4(v,-vp) AB From this, we get, by rearranging cD olva-va)= Sat Vx, which is just Eq. (F) of Table 17.8-2. b. If we start with the second form of the Maxwell-Stefan equation we have Vx4=- a (x)Na-x4Na) or XgNq—X4Ny =-c® Vy If now we add and subtract x,N., on the left side, we get (x4 4Xp)Ny —%4 (Ny +Ny)=-COgpVX 4 or Ng =%4(Na +Ng)- CB agV24 which is just Eq. (D) of Table 17.8-2. 19-8 19B.6 Diffusion and chemical reaction in a liquid a. The differential equation for the steady-state diffusion from a sphere is Alida) _ yr 2 ope Sunde a) kre or rales @) eto According to Eq, C.1-6b, the solution to this equation is Cra p= 4G a § Application of the boundary conditions that I'(1)=1 and (ec) =0 determines the constants, and the final result is (cf. Eq. 19B.6-1) _1e% “ber The mass flux at the sphere surface is then DapCao 1+b) R (1+b) and the total loss of A from the sphere in moles per unit time is 2 Pateae) kyR? = Saslao | 4 W, = 4aR’ ( 7 b. The unsteady-state mass balance on the dissolving sphere of species A is rae DapCaoM, KR? 3 2( BancaoMa i £5 mR Pan) = AnR’ ( R \« Das } \9-9 Since we have used here the steady-state expression for the molar flux at the surface of the sphere, this is a quasi-steady-state treatment. The integration of this equation can be accomplished as follows: First we divide through by 47 p..,, to get Re AR _ BaslaoMa ph 4, i dt Psph AB Next, put all the factors containing R on the left side rr 1+(K/R?/®4p) dt Pagh Next we introduce a new dimensionless variable Y = R,[ky’/ 5 and write 1+Y dt Pon Y_a¥ _kibaoMa Then we integrate y Y¥ kb soMa ot “pt gy -KlaMy Iie jot and finally ae Ki aoM, Y-Y,)-In UT (7-Y,) in E = Hala) oe Kr 14+ JR7DapR __kiyoM Re Ra) in eae AO App Bg RRM EIB eR Pan From this one can get the dependence of the sphere radius on the time. 19-10 19B.7 Various forms of the species continuity equation a. To get Eq. (A) of Table 19.2-1 from Eq. 19.1-7, use Eq. (C) of Table 17.7-1, in the form p, = pw, on the left side of Eq. 19.1-7, and Eq. (S) of Table 17.8-1 on the right side. To get Eq. (b) of Table 19.23 from Eq. 19.1-7, move the term ~(V-p,v) to the left side of the equation, and then use Eq. 3.5-4 (with fidentified as @,). b. Rearrange Eq. 19.1-11 to get Ha 4 (9-c4v*) i (¥-Ju)+Ra Then rewrite the divergence term on the left side by differentiating the product Se eg V-v 9) +(v*Veq)=[V-Tu)+Re Then write c, = cx, and once again differentiate the products, thus: Oe x M4 x gc(V-v*) tev *-VIq)+tq(V*-VC) -(V-Ji)+Ry or, on rearranging {3 +(v “vs,)) + o( E00 ») =-(V-Ju)+Ra Next we may make use of the overall equation of continuity in Eq. 19.1-12 to modify the second term on the left (with the prefactor x,.) a x : {Se + vr-v5,)) +03 Ry =-(V-Ju)+Ra This equation can be put into the form given in Eq. 19.1-15. No assumptions have been made in getting to this result from Eq. 19.1- 11. (9-11 19C.1 Alternate form of the binary diffusion equation Equation 19.1-17, in the absence of chemical reactions, is ax, a a (v*-Vx,)= B4,V?x, Equation 19C.1-1 can be written as ome 1 Moar ty = =, V -v) = Sal Vv *M-3(VM- vm) To compare this with Eq. 19.1-17, we multiply through by M and then replace M by its definition: M = x,M, +X,M,=x4M, +(1-x4)Mg =x,(M,4 ~My) + Mg. This leads to Ox, a +(v-Vx4) =D ,V 24 $B gp Maa Ma) Wa Vt) M To get this result, we have also divided through by the factor - Mg (which is never exactly equal to zero). When this last equation is compared to the first equation above, we see that the two results are the same, if +(1/M)(M, ~ Mp) 8 45V% 4 To show this we introduce the definitions of v and v*, and also Fick's first law in the form of Eq. (F) of Table 17.8-2. This yields Bava HV =e (Mata¥a + Marga) MA =O srg —¥0) 1 = ppl Ma — Mae + Mars) av +(My~ Mpg + Mat4)82v5] = (YM)[(Maxa + Mp%p)xa¥4 +(Matp +Max,)Xa¥e] Since this is an identity, the proof is completed. 19-12 19D.1 Derivation of the equation of continuity a. The equation of continuity for species A states that for an arbitrary fixed volume V the time rate of change of the total mass of species A must equal the net rate of addition of A over the bounding surface $ (by convection and diffusion) plus the rate of production of species A by chemical reactions. This statement in integral form is given by Eq. 19D.1-1. The surface integral may be transformed into a volume integral by using the Gauss divergence theorem to give [Seay =-[(v-m)aV + [rad v v v Here we have also moved the time-derivative operator inside the integral, inasmuch as the volume is not changing with time. Since the volume is completely arbitrary, we may remove the integral signs to get the equation of motion in the form of Eq. 19.1-6: b. For an arbitrarily chosen "blob" of fluid, contained within the volume V(t), whose boundaries are moving with the mass- average velocity, we may write the mass conservation statement as follows: the time rate of change of A within the volume equals the rate of addition of A by diffusion across the surface S(t), plus the rate of production of A by chemical reaction within V(t) : 4 fpsdV=- f (m-jq)dS+ fradV at ye) sy vo Next we apply the Leibniz formula to the left side to get a . J send + Jea(n-vsaS=- J (n-j,)dS+ frydV vn Ht st) se) vo This may be rearranged to give (9-13 a : J Pad =~ J(n-(in +Pav) WS + fradV vi) S(t) vit) where use has been made of the fact that the surface of the blob is everywhere moving with the mass-average velocity, i.e., vs =v. Then, we make use of the Gauss divergence theorem for the surface integral, to obtain a J SppadV =~ J(V-(ja tPav) AV + fradV vey vit) vay Since the volume element V(t) was chosen arbitrarily, we may remove the integrals to get the species equation of continuity for A in the form of Eq. 19.1-7: Wa. (-(j4 + D48)) #14 19-14 19D.2 Derivation of the eqution of change for temperature for a multicomponent system a. Applying the chain rule for the functions related in Eq. 19D.2-1, we get oH es (mE) Ig A a(mAl) am Fg Jy, Ph Ap J, LAM), am J, Ama), ems\ 7) | So 0 | (2) ( atts) (2) (su-“t} +4 (for @#N) le 8 (=) -5| amt) ($e a Aix Jn, BA Oy J, Amy), aim), ain J, oF \(_ mg) XY a \/_ ms) a _ 3 2\- 4) +='$f 2) ve) +8 (for @=N) Subtraction of these two results gives Eq. 19D.2-4. b. Regard H as a function of p, T, and the first (N - 1) mass fractions, so that on on d= (¢ } »(F J. aT+ “32 a.) do, 9 boy a), When this is applied to a fluid element moving with the fluid, Eq. 19D.2-5 results. Equations 19D.2-6 and 7 are standard thermo- dynamic results. c. Because of the relation m,=n,M,, the differential quotients in the last term in the last equation can be rewritten, with the help of Eq. 19D.2-4, in terms of partial molar quantities: 19-15 (3), “(ae =-1(9H) _1/(@H\ Hy Hy Mal Oita Jy, My Ory ME Mn my In Eq. 19D.2-5, we use Eq. (E) of Table 19.2-4 for the substantial derivative of the enthalpy, and we use Eq. 19.1-14 for the substantial derivative of the mole fractions. We also use Eqs. 19D.2-6 and 7 for the other differential quotients. Then Eq. 19D.2-5 becomes (amd) bp, ye DT ' (sir), [Breee Dr Sf Fe Fa) j.)+re) GAM, My a)-(e:vv) +22 = d, Rearranging we get a DT anv Dp = =+(V-q)-(t:V Copia da (Sir, Dt My SFP \(r-i0)-re) The last term in this equation can be simplified as follows: + SHE a)-ra)-( S(7-ie)-re) at at rz = = DHa((¥Ja)-Ra)+ Ayv((V-In)-Rw) Be 8 i iMe= This, then, leads to Eq. (F) of Table 19.2-4. 19-1 19D.3 Gas separation by atmolysis or "sweep diffusion" a, The Maxwell-Stefan equation for a three-component mixture are dx, 1 1 a= eon (xaNoe ~XNaz) + ay (x4Nez~%cN a2) dx, 1 1 Se oon (XpNac -XaNpe)+ Be (xsNo. -%cN gs) We now make the substitution x- =1-x, Xp in order to have just two independent variables. Then by introducing the indicated dimensionless variables, we get Eqs. 19D.3-1 and 2. b. Next we take the Laplace transform of Eqs. 19D.3-1 and 2: PRa~Xar=YaaFa + Y an%o + PY 4 = _ ee PXp~ Xp. = YoaXa + YonXs +P Yop This set of equations can be solved simultaneously to give for 4 -— in which p, and p_ are given by Eq. 19D.3-4, and X4(X41/%miP) by Eq. 19D.3-5. Similar equations can be given for ¥, by interchanging the subscripts A and B. The transformed expression ¥, can be inverted by using the Heaviside partial fractions expansion theorem if we exclude the relatively unimportant cases p, = p_and p, (or p_) = 0. Thus we get _X(Xar iO) X(Xa ois de® | X(Xar-Xorsp_ler> a PE, SMa API *4(6) PsP P.(P, -P-) p-(p_-P,) The expression for x, is obtained by interchanging all indices A and B. 19-17 ¢. Finally, at €=1, x4 = 249/80 that = X(taveteri0) , X(xa-torip. Je | X(xar tiple PsP p.(p.-p-) p-(p-~Ps) and a similar expression for xs. We hence have a pair of equations giving the relations among X,1, X42, X1/ aNd Xpz, the dimensionless fluxes v,, and the dimensionless diffusivity ratios. Keys and Pigford go further and give a plot of the separation factor ora talon Xa/Xe for the special case of very small mole fractions. Note that the choice of notation in this problem has the advantage that there is symmetry between A and B, which makes it easy to give the results for both species after the result for one species has been worked out. 19-18 19D.4 Steady-state diffusion from a rotating disk a, Equation 19.1-16 for steady-state diffusion in the absence of chemical reactions is (v-Ven)=Ba¥°P4(2) For the special case that p4 depends on z alone, the diffusion equation simplifies further to q 1a or mee dp, @p 2. = Ba Gt in which the dimensionless coordinate ¢=z/Q/v has been introduced. b. The differential equation is solved by setting do, /d¢ =p, so that we get the first-order separable equation dp/d¢=ScH(£)p, which has the solution Inp=SefSH(Cdo+InC, or Fe =p=C, exp(Se{SH(Z}#2) A further integration gives pa =Ci[perw( Sely H(E IE HE +c, The boundary conditions that (4 (0) = P4g and p,(c»)=0 then give Pao ee sane Kexp| iS oe the second expression being the high Schmidt number limit, i.e., taking only the first term in the expansion for H. The integral in the denominator can be evaluated analytically 19-19 Jp exp(3Scat? at Hence the final expression for the concentration profile is 4 i = 7 loewel-38cat? jag oa c. The mass flux in the z direction is then 34Sca di rayne soe lie in which d¢/dz = /QJ¥. At the surface of the disk the mass flux is Sca B ‘ aa Pao®an “ - ssi) dp po 8 Seay # = gl 1g)? Y me 7 dy jac =~ Ban PA = +010D 5 Tac which is the result in Eq. 19D.4-7. 19-20 20A.1 Measurement of diffusivity by unsteady-state evaporation. Assuming ideal gas behavior, and insolubility of species “B” in the liquid, we esti- mate the mole fraction of ethyl propionate in the interfacial vapor as Pawap _ 41.5 tao = 0.0545 Eq. 20.1-1-22 gives AV(t) Linear interpolation to 4 = 0.0545 in Table 20.1-1 gives 0235, whence 7 Avi) ]? Pan ole ges ag [eV AV(t) Vi- V0 with AV in cm® and ¢ in s. Application of this formula to the tabulated data gives the following results: vi HS = OE eK | Ce ei Pap 12 0.0281 0.0278 0.0272 0.0273 0.0270 0.0273 0.0269 = 16.48 [ ‘The average of the last seven determinations of Dag is 0.0274 em?/s. 20-\ 20A.2 Absorption of oxygen from a growing bubble. ‘The interfacial molar flux of oxygen into the liquid is given by Eq. 20.1-75 as 2s 1)D, Nault) = cao) 2+ DP as for a bubble with interfacial area S(t) = at” and interfacial liquid concentration cao, when a, n and c4o are constants. The solubility wao corresponds to a molar concentration cao = proao/Ma 1.0 g soln/em*)(7.78 x 10™* g O2/g soln)/(32 g O2/g-mol O2) = 2.43 x 107° g-mol O2/cm* which is used here as the interfacial concentration of dissolved Oo. (a) For constant growth rate of the bubble volume, r3 o t, so that ry o 1/9 and S(t) o 73 « #/3, giving n = 2/3 in Bq, 20.1-75. Then at t = 2s, the interfacial molar flux of O2 into the liquid is Naol(t) = cao (7/3)(2.60 x 10-5 (3.1416)2 5) cm? /s) = (2.43 x 1075 g-mol/cm'), 7.55 x 107* g-mol/em?-s The total absorption rate in g/s is then walt) = 40r3()Nao(t)Ma = 4n(0.1/2 cm)*)(7.55 x 10-* g-mol/cm?-s)(32 g/g-mol) =7.6 x 107 g/s (b) For constant radial growth rate, ry ot and S(t) o #, giving n = 2 in Bq. 20.1-75. Then the interfacial flux at time t = 2 s is (4+ 1)D, Nao = exo) & a Ap = (2.43 x 107° g-mol/em*, = 1.11 x 1077 g-mol/em*-s and the total absorption rate in g/s is wa = 4nri(t)Nao(t)Ma = 4n(0.1/2 em)*(1.11 x 107? g-mol/cm?-s)(32 g/g-mol) =1.11x 1077 g/s 20-2 20A.3 Rate of evaporation of n-octane. Table E-1 gives the following Lennard-Jones parameters: Species M oA 2K A:mCsHig 114.23 7.035 361. B: No 28.013 3.667 99.8 Eqs. 17.3-14,15 then give the interaction parameters 4g = 5.351A and cap = 189.8 K, and Eq. 17.3-10 gives, 1 1 cm?/s = 5 5)? ae Daw, ema? /s = 0.00185834/ (295.18) ( mae atm)(5.351)*(1.185) = 0.0580/(p, atm) (a) If p= 1 atm, then Dap = 0.0580 cm?/s and x49 = 10.45/760 = 0.01375. Interpolation in Table 20.1-1 gives y = 0.00859, and Eq. 20.1-20 gives the volume of vapor produced in 24.5 hr as Va = SpV4Dant = (1.29 cm?)(0.00859) \/4(0.0580 cm?/s)(24.5 x 3600 s) 1.585 cm* The mass of vapor produced in 24.5 hr is pVaMa RT _ (1 atm)(1.585 cm?)(114,23 g/g-mol) ~~ 82.0578 x 293.15 cm® atm/g-mol = 0.0075 g (b) If p = 2 atm, then Dap = 0.0290 em?/s and x49 = 0.01375/2 = 0.0688. Table 20.1-1 then gives yp = 0.00430, whence Va = SpV4Dapt = (1.29 em?)(0.00438) y/4(0,00290 em?/s)(24.5 x 3600 s) 561 em and the mass of vapor produced in 24.5 hr is PVaMa RE _ 2 atm)(0.561 cm®)(114.23 g/g-mol) ~~ 82.0578 x 293.15 cm® atm/g-mol = 0.0053 g ma 203 20A.4 Effect of bubble size on interfacial composition. Equations 20A.4-1 and 2 give int pat] Assuming H to be independent of rs, the ratio of wa to its value for a very large bubble is waolr) _ Poot 2e/rs 1, 20 wao(oo) Poo Pools ‘Thus, the bubble radius corresponding to a 10% increase of wo over its value for a very large bubble is given by 2a) aoe —t=01 oo on Pools Poo For a gas bubble in water at 25°C, with pay = 1 atm, this gives the required bubble radius as 2 (2)(72 dynes/em) © (0.1)(1.0133 x 10° dyne/cm’ = 0.00142 cm = 14 microns The (normally minor) dependence of liquid-phase free energy on total pressure has been neglected here. To include this effect, one would need partial molar volume data for the particular system. 20-4 20A.5 Absorption with rapid second-order reaction a. The first thing one has to do is to determine the parameter y from Eq. 20.1-37, using the concentrations and diffusivities given in Fig. 20.1-2. By a trial-and-error procedure we have found that y=4.9x10% mm?/s. We now verify this by substituting into Eq. 20.1-37. We first have to convert the diffusivities in ft?/hr into units of mm2/s: 2.2.54. - 45 =(3.910" fre) 2.5410 mm/ft)’ 1 096x 10 mm?/s 3600 s/hr Qz55 = 1.95 10° ft?/hr = 0.503 x10 mm?/s Substituting into Eq. 20.1-37 now gives: 4 ={4),/0:50310? 6 | 49x10" o,f 49x10" (3) 1.006x10>" Vi.006x10 “P{ 1.006x10 — 0.503x10" or [49x10 oon joan 1~erf 0.3121 = (2.828)(erf 0.2207)(0.9525) When the error functions are evaluated, using a table, we get 10.341 = (2,828)(0.245)(0.9525) This gives 0.66 = 0.66. Therefore, zp(t)=/47t gives the location of the reaction zone as a function of f. Thus we get the following table of results that can be compared with Fig. 19.1-2: t za()= art 0.625 s 0.011 mm 25 0.022 10.0 0.044 Q0-5 This is not in very good agreement with the graph in Fig. 19.1-2. The reason for this may be that Eq. 415 on p. 336 of Absorption and Extraction by Sherwood and Pigford (which corresponds to our Eq. 20.1-37), contains two errors: the r on the left side should be Vr and the D, in the argument of the second error function should be D5. b. To get Nao at 2.5 seconds, we use Eq. 20.1-38, as follows: £40. Das erf/7/D as _ (1. gmol/dm?)(10 cm/dm)’ 1,006 x10 mm’ erf, (4.9 10)/(1.006 x 10°) | (3-14159)(2.5 s)(100 mm?/cm?) 1000 3 = aoa 13x10*) = 4.61 gmol/cm?s Nao = 20A.6 Rapid forced-convection mass transfer into a laminar boundary layer. Equation (20.2-51) gives, for the conditions of this problem, (wo = Woo) BR, = a Hen) nao/(na0 + nB0) — Ao _ (0.9-0.1) =oyp 780 Fig. 22.8-5 gives a dimensionless mass flux ¢ of 1.55 for mass transfer at Ry, = 8.0 and Se 0. With this result, Eq. 22.8-21 and Table 20.2-1 give KA K x 2.0 TF(O,4,0) 0.5972 so that 55 x 0.5972/2 = 0.463 The definition of K in Eq. 20.2-48 then gives the total interfacial mass flux nao(2) + nB0(2) = povo(2) = Kpveey/ 5 38. PVH] In this problem ngo(z) is stated to be zero, so the previous result reduces to ngo(2) = 083/ pau] 2 20-7 20A.7 Slow forced-convection mass transfer into a laminar boundary layer. (a) Equation (20.2-51) gives, for the conditions of this problem, the binary mass flux ratio (wo = woo) Ry = — eee) nao/(n40 + 2B0) — YA0 _ (0.05 0.01) _ = 00 = 0.0421 Eq. (20.2-55) and Table 20.2-2, with Se= 0.6, then give Ro = aA7/8 Kea FoR 0.0421 5 -2ysy__ 0.0421 _ 04082 x (0.8) TF 0768 x 0.0 Equation 20.2-48, with the specification npo(x) = 0, then gives the evaporative A gi iP mass flux |.0266 nao(t) = povo(z) = K pro Woot = 0.0266 /pvacu/2e = 0.0188 ,/ pron /t (b) Eq. 20.2-57 gives nao ~wao(nao + Bo) ee (wao — Wace)Se?/? = 0.332 which gives, since nao = 0, nag = 0.332S¢72/3 240 Ace yy Hie = wao Voor es 0.932(0.6)-79 Oo Oe pcan = 0.0196 /praou/e (c) At the value K = 0.0266 found in (a), Table 20.2-1 gives the interpolated interfacial gradient I'(0,Sc, K) = 0.3797. Then Eq. 20.2-47, with nao = 0, gives the reference solution agg = Bee H'(0,Se,K) |, Vv A Ty (Se Doar __ 0.05 — 0.01 0.3797 10.05 06 VPen/2e = 0.0188y/ pracy /z in excellent agreement with the truncated expansion used in part (a). 20-8 20B.1 Extension of the Arnold problem to account for interphase transfer of both species Equations 20.1-1, 2, and 3 are still valid if both species are crossing the interface. In Eq.20.1-3 we now eliminate c by using Eq. (D) of Table 17.8-2 (evaluated at the interface) to get (NaotNeo) xa! ——(Nao+Nowo) (itr) _ ax, Nao *a0(Nazo+Nao) 0 lo 1-X49(1tr) & lea When Np.o =0 (or the ratio r= Ni,9/N azo B0eS to zero), this equation simplifies to Eq. 20.1-4. Equation 20.1-5 is then replaced by ox, x, sone 7 Ox, (_ (ltr) ox, a Ot \T=xag(147) Oz We now introduce the dimensionless variables II (defined in Eq. 20.1-23) and Z (defined just below Eq. 20.1-8); note that when X42 =0, II is the same as -(X -1). Then the combinarion of variables method gives (cf. 20.1-9) the ordinary differential equation @r +2(Z- eet A og along with ap b Bao X4n)(14+7) a] (40:7) = #9 ler) dL eg This is Eq. 20.1-24. If r=0 and x,., =0, it reduces to Eq. 20.1-10. The sign discrepancy comes about because (in the limit that x,., is zero, IT is the same as -(X-1). The solution of the ordinary differential equation for IT proceeds exactly as that for X in the text, and the final result is erf(Z- 9) +erfp T+erfo T= 20-9 for the concentration profile. If r=0 and x,.=0, this reduces exactly to Eq. 20.1-16. When this concentration profile is used to evaluate dII/dZ at Z =0, then we get an expression for @(x4o,7): 1 (X40 - X40 )(1 +7) P80) = 45 Fo er) This may be rearranged to give (x40 = X40 )(A+7) - 2 tox,(ls7) Vi(1+erfe)pexp(+9") in agreement with Eq. 20.1-25. 20-10 20B.2 Extension of the Arnold problem to nonisothermal diffusion a, Equation (M) of Table 19.2-4 without the last two terms is: 330 H, (wd SNF, |= (v-kvT) Replacing the partial molar quantities by quantities per mole (see comment after Eq. 19.3-6), and assuming that k is constant then gives ax No wee fi, +(V- SNH. |=4°T Ot ae at Differentiating the products in the first and second terms allows us to rewrite this as Naf N on Yee Bet SS Sa H+3(V-Ny)Ag +2 (Ne ‘VA,)=kV°T a = mt a In the second term of this equation, we replace the derivative of the concentration by using Eq. 19.1-10 (omitting the reaction-rate term), and then we see that the second and third terms just exactly cancel. Next, we replace the enthalpy by the heat capacity multiplied by a temperature difference in accordance with Eq. 9.8-8, to get 30,36 pe (T T)+3(Ne ‘VC yq(T-T°)) = kV?T aa a) For constant heat capacities this then becomes: N 2 oO Be Leola oe Cra (N,-VI)=kV°T If all the heat capacities are alike, then they can be taken outside the summation to get .(% t dq= (# 2) a+ (22) ae at). az), @-@)-8@ at), ae). az ),\ at), =( 24 32) ' -(#) +(# or) (2) -(2) { 24) _( a4 du), \az),’\ du" 82" where q is any scalar, here < >. Putting these results into Eq. 20.5-17 gives (2) =K 0° at), au’), which is the desired result aq (b) Our differential equation is of the form Be 2 Koa 62? where q is

and the proposed solution can be written as =A emp{ Et : Gas tag 2 a 242 du 2KtAK*t Cie ag ade ou? Qt “TKe The differential equation is satisfied. (c) Simple inspection shows that these conditions are met. 20-24 20C.1 Order-of-magnitude analysis of gas absorption from a growing bubble (a) The volumetric flow rate for this system is constant at a value The same volumetric flow must occur at any distance, and this constraint requires the suggested result. (b) This is obtained simply by putting the above result into the spherically symmetric continuity equation. (©) The expression (y-n) (1-9/rF The desired expression can be obtained simply by division, and it is also available in many standard tables, for example, H. B. Dwight, Tables of Integrals and Other Mathematical Data, Macmillan, #9.06. (a) Terms (5), (6) and (7) are second, third and forth order respect ely. 20-25 20C.3 Absorption with chemical reaction in a semi-infinite medium a. First, introduce the following dimensionless variables: P=cq/cao, €= \K{[Dqnx and T= kit; also, define the combinations m=|(E/2vz)-e] and p=[(¢/2Vz)+7]. Then Eqs. 200.31 and 2 are 7 . Rae and 2r=e“erfom+eterfcp Then we calculate the derivatives as follows: sort) (ae) =~eerfem— e+ Fertep- 8 0 Vat vat er ef stem et é } oot s ag Vat nt Vat (555 ag Fer eke eke? eer fe" ete ( E +) Vat Vat Vat \2Vr When these contributions are substituted into the diffusion equation, it may be seen that the equation is satisfied. At £=0, we find that m =o and p =<, so that erfc m and erfe p are both equal to zero, and hence the initial condition is satsified. At x=0, P=3ferfc(-k/'t)+erfe(+k/t)] which may also be written as 3[1—erf(-k/t)+1-erf(+k/t)]; however erf(-u)=-erf(u), so that we are left with 4-2=1, which indicates that the boundary condition at x =0 is satisfied. At x=e, erfcm and erfcp both go to zero, and e”§ also. Hence we have to prove thate‘erfep goes to zero: 20-26 erfe(E/2F) am (2/v/m)(exp(- £?/41))(1/2VE) be et -e ~ which shows that the boundary condition at x =e is satisfied. b. To get the molar flux at the wall, we once again make use of dimensionless variables: ky’ or =- B50, [ar WON Dag = a ~Dasla05,| Ix=0 g=0 e en Base m— = fea Beak ete) ent) - 55] a0 VDaak? [pave | which is equivalent to Eq. 20C.3-3. Actually a neater form of the result is: D5) 7 ort IRE + eH = C40) [Jakitert kit +e] In this form, the quantity in the brackets is the correction factor for the absorption rate because of the chemical reaction that is occurring. c. The total moles absorbed through an area A is Apa oft = Fda E 4 Fao Bak tv + i Mg = Af Nar Naghaolt We now perform the two integrations; in both of them we make the change of variable Vt = y: 20-2] [ae ele ¥ dy =erf kt Wetear- 1." (erty) 2uay =f aC ffer*as) 2ydy “5 rele ( 1 yay er? as = oil (kit—-s?))e* ds aK Pee -# ds “Zh otett EE —2 wane 2b ae = kytert /kyt +L [kte*? — Lert. JRE Va a se" When these integrals are substituted into the expression forM,, we end up with Eq. 20C.3-4. d. For large values of kt, the exponential in the last term goes to zero, the error function goes to unity, and the only remaining terms are those in the large parentheses in Eq. 20C.3-4. In this way, Eq. 20C.3-5 is obtained. 20-28 20C.4 Design of fluid control circuits The response to a pulse input to a “long” tube can be expressed as m/A (t-L/op Vicki” | Bt /v*) Here mis the mass of solute “i” fed to a tube of length L and cross-sectional area A. p,(L,t) = This is just 20.5-18, modified to be explicit in time, and using a generalized dispersion coefficient E. For a short pulse most of the solute leaves near the mean residence time t=L/v where Vis the flow average velocity, . The term “E” is the sum of Taylor dispersion and molecular diffusivity: B=K+2, If'we then approximate t as # the exit composition has the form of a normal probability distribution with variance of =2Et /v* and Gis the standard deviation of the distribution. A reasonable set of design criteria is then that V2EE /v < 0.05t, L >> 2uR? /(3.8)’ B,, Eq. 205-4) In practice the “>>” requirement need not be very large. It is really only necessary that there not be much tailing. A discussion of this point is beyond the scope of the present discussion. In any event for most “long” ducts we may approximate “E” as the “K” of Eq. 20.5-15: K = Rv’ / 48D_,; 2Rv/v < 2,100 For higher Re turbulence results in much smaller dispersion coefficients, K = 10Rv* 20-24 We will not explore this turbulent region here. We thus find 2 7 # > 8.99971 5 2.89 [ER ° Cea = 22, with the constraint on length introduced above. This in tum means that ; z 21g >> NE = tlh Oe =2.7-10%em® /D,, ‘Then for typical liquids, with diffusivities on the order of 10° cm’/s, t, >> 2.7-10°s = 4.5min whereas for gases with diffusivities of, let us say 0.27, t, >> 0.018 20-30 20C.5 Dissociation of a gas caused by a temperature gradient [The small Roman numerals after the equations correspond to the equations given by Dirac in his paper.] a. The equation of state It if assumed that the diffusing mixture is at constant temperature and that the ideal gas law is applicable, then we have p=(m,+n,)KT or (n, +1, )T = p/k = constant Here n, and n, are the number densities of the single and double species at any point in the tube. Then if n? and n are the values of n, and n, at the same pressure and temperature in a system with no temperature gradient, then we have also (nf +9) = p/« = constant Then Dirac defines a quantity o by the following: m-n?=o and 1-1 =-0 @ Thus o gives the deviation from the equilibrium state, resulting from the temperature gradient. b. The equation of continuity for the mixture Equation 3.1-4 (or Eq. (D) of Table 19.2-1) gives, for steady state, £ dz po or pv, = constant where 2, is the mass-average velocity. Therefore, if we make use of Eq. (B) of Table 17.7-2, and if we make use of the fact that there is no net movement of mass across any plane z = constant, then we have P10, + P2V, =0 or nymv, + NyMyv, =0 where mand m, are the masses of single and double molecules, respectively. Then mm, +1,(2m,)v,=0 or m4, +270, =0 di) 20-31 c. The energy equation for the mixture For a steady-state system with all species subject to the same external force, the one-dimensional energy equation of Eq. (C) gives e, = E, a constant. Then we use the approximate expression for the energy flux (also used by Dirac) given in Eq. 19.3-6 dT a gO dT “ ~ke + (NH, +NoHa)=E ork + ah + cy0F, E 2 where the partial molar enthalpies have been replaced by the enthalpies per mole, as is normally done for ideal-gas systems. Then switching to quantities per molecule, we may write 0D + no,hy +MVh=E or 0 5 pmo, (2hy —hy)=E Then if we define the heat of dissociation by Altjiuce = 2i, ~ I, the energy equation becomes Ds $0 Ase = E co) d. The expression for the molar flux If we write Eq. 24.2-8 for species "1" in a 2-component mixture, it becomes: Gaga ( BEDE) a Bl Mm) & This equation is a generalization of Eq. (F) of Table 17.8-2, which takes into account the extra term needed to describe thermal diffusion. Since D7 +D] =0 (see paragraph just above Eq. 24.2-3), the above equation may be written as 82 4%, _ Di din? xx, dz p dz (iv) Y- dQ = 20-32 This is exactly Dirac’s equation (iv) if - DT /p is relabeled as D’. e. The equation of continuity for species "1” Equation 19.1-10 at steady state is so a 4 dz the second form being in molecular, rather than molar, quantities. Dirac then postulates that, for systems not too far removed from equilibrium, first order kinetics can be assumed so that (v) The above set of equations (i to v) are the equations that Dirac derived, and these have to be solved simultaneously for ™, tz, 0, U2, and T. for temperature dependent thermal conductivity, diffusivity, and thermal diffusion coefficient. First he shows how to reduce the set of five equations to a set of two equations. Then he resorts to numerical methods to complete the calculation for some sample values of the constants in the system. 20-33 20D.1 Two-bulb experiment for measuring gas diffusivities analytical solution The diffusion equation to be solved is Hy at ae, = Dap a with the initial conditions that c,(z,0)=cy for 0 a and Ply ~ Cy (0) = By Then using the initial conditions and rearranging, we get ae 7 &e, ° SiG P= and Dur Pen = eh These ordinary differential equations can be solved to give & = C, exp([p/B,2) + Co exp(-y/p/B,z) ++ tu =C; exp( \P/9uz)+C, exp(-p/Buz) +a The boundary conditions at plus and minus infinity require that C, and C, be zero. The boundary conditions at the interface (z = 0) yield: ° ° 5 c +2 )m-(e +4) and Cyn] Lc, ( 7 “Pp 8, * From these two equations we get the integration constants 1 cy-mep and Coen Ge-4 cya mecy ¢, =-— a 1 pm+JB,/By \Qu pm B,/8, Then the expressions for the transformed concentrations are 1 ch—mee z-4 ont a * Fine By Be SPOPTRA)* 20-38 _ __1 ch-mep cf 2 =- tS _exp(-,/p/Byz) +t np mY By/B; (-P/8 22) Pp Then, taking the inverse transform (using a table of transforms), we get _(_ca- me? z c — me? z = -— tt eerf ———] +5 m/®,/®, - { se] “a Then using the fact that erfex=1-erfx, and that ~erf (—x)=erfx, we finally get or ch 1 0 — Si ort 2 | 1/9; +1 V48ut cy — 08 _Leerf (2//4But) ep —(Ym)es, (Ym) + Y@u/B, b. To get the flux at the interface, we can differentiate the concentration profiles: ch-mep_) 2 =-§, —t tt |—— —z?/40,t (ite Feo : af ch — mc? } S, "int (9,/ 8; Vat Alternatively one can take the transform of the molar flux expression, differentiate ¢, with respect to z, then take the inverse transform to get the molar flux expression just found. 20-34 20D.3 Critical size of an autocatalytic system (a) The general non-reactive transient solution for cylinders is DAI, (ayr)emp (a2 Dt / R°) 4 and the reactive solution is just Crane = Ando") (b) Initial concentrations will decrease whenever all exponentials are negative, up (kM — at. / R*) and the first term will be the most likely to be positive. Critical mass is then reached R=a,J9/K (©) This answer is given in the problem statement. when 20-40 20D.4 Dispersion of a broad pulse in steady laminar axial flow in a tube (a) The approach here is to take advantage of the problem linearity and to add weighted pulse response solutions, or transfer functions, which are called here h(z,t), For our situation the transfer function is 1 L-t? 1 agp] (Eas 02th V4arKt 4Kt It only remains now to weight individual differential pulses, by f(t’), each introduced A(L,t) = at a time t’, and add them up by integration from time zero to observer time t. ‘The response to an input of mass my added over a cross-sectional area of column A, in a short time period is just (m, / A)h(L,t) The mass of solute per unit area can in tun be written as dm, / A= p,|,,, dt = fat! for uniform concentration over the inlet cross-section. Summing, or more specifically integrating over the time interval of interest then leads directly to the solution suggested (le s2207] op| fESeety a [ ‘Kt! and change the lower limit of integration to “0” 20-41 20D.5 Velocity divergence in interfacially embedded coordinates. (a) Integration of Eq. 20.4-7 over the boundary Sp of any closed domain D(u,w,y) in the boundary layer gives Jor easo= [, woaso)+ f (Zee) asp) (20D.5 — 1) (b) Since the third dot product is the outward normal component of the velocity of the boundary clement dSp relative to stationary coordinates, its integral is the rate of change of the volume Vp of the domain D(u,w,y). With the aid of Eq. 20.4-3, we can rewrite this term as follows, i Or(urwiyrt) , ag = s/f Valu, w,y,t)dudwdy sp a @ Jp = [ VG) sdwdy (200.52) ID a using Leibniz’ rule to differentiate the second integral. (c) Application of Eq. A.5-1 (in which » is any differentiable vector function) to Eq. 20D.5-1, after use of Eq. 20D.5-2, gives the following equation in terms of volume integrals: (VeV)dVp = [ (Vev)d¥p + f oy Su 29) dudwdy Vp Expressing each integral in the coordinates of Eq. 20.4-3, we then obtain —— ‘g(u,w,¥,t) f [ eV)-Vev)— SERRE Vale, w, y,tdudwdy = 0 In order that this equation hold for every domain D of nonzero g(u,1,y,t), the quantity in square brackets must vanish identically in such regions, giving (ev) = (on) 4 evan there in accordance with Eq. 20.4-8. 20-42 21A.1 Determination of eddy diffusivity In(sc,)= InW,/4D4y)—(V9/2D4y)(5-2) Assume that Weo, = 0.001¢W yy, (a) Begin by examining the behavior at r= 0, and note that xD? rane) so that the volume fraction (equal to the mole fraction) of carbon dioxide 2. boo) =e 2 -0.001 16 2K sco, s the effective diffusivity of carbon dioxide in air. Here we have assumed the ideal gas law to hold. We must now estimate the volume fractions, and here they will be taken as 0.0105 and 0.007 for distances of 112.5 and 152.7 cm respectively. Solving the above equation for the effective diffusivity then gives: K4-co, =18.6, 21.9 cm? /s respectively. a 21A.2 Heat and mass transfer analogy Begin by defining the parameters =a Tea 5 us © Gao! Day Yack?! Day The desired solution then follows directly Dap(Co =>) _ 9 Ymax_\ t UOP JaoP ) Sl + (Vv fv\(Se/ Se) 4B dé QN-2 21B.1 Wall mass flux for turbulent flow with no chemical reactions. a. The Blasius formula for turbulent flow in a tube may be found in Eq. 6.2-12: 0.0791 Re¥# The mass-transfer analog of Eq. 13.3-7 is as le Rese” When these are combined, we get Sh =0.0114Re™* Sc¥3 b. If on the other hand, we use the mass-transfer analog of Eq. 13.4-20, fee fe Ih = ——.,|+ Re y3 Nae Se’ Sh= sayy gz Rese”? — Clara Se) then we get Sh = 0.0160 Re”® Sc’ which is in better agreement with experimental data. a2\-3 21B.2 Altemate expressions for the turbulent mass flux. Seek an asymptotic expression for the turbulent mass flux for long circular tubes and a boundary condition of constant wall mass flux. Assume net mass transfer across the wall is negligible. a) Parallel the approach to laminar flow heat transfer in §10.8 to write TG,0) = Co +1, (6) + Ce and show that 4 Here T= (0, ~ @jo)/igoD! Dw); § = 7! D3 § = (z/ D)/ReSe and the subscript “0” indicates conditions at the wall. This is given in Section 10.8 b) Put these results into the species continuity equation to obtain This equation is to be integrated with the boundary conditions AtE=1/2 T.=0 and dIL/dé=-1 ) Integrate once with respect to & to obtain: 12 ~4f eae z #5] Se op Here Se” =u / pD®,. a-4 21B.3 An asymptotic expression for the turbulent mass flux. Start with Eq, 21B.3-1 in the form dam, jdt 1 _ “Dae a “DT SOr /4.5v)"] Making the suggested substitution yields at _ dit Sel dy dy 14.5v so that CE dn Dv,Se'? 1+1° and 14.5v po 1 T1(00) -11(0) = > —. (o)—T10) Dse™! Tg ‘The integral is equal to eee 5 = 1.209 It follows that Sh=————_ = 0.057Re Se! [f/2 T(20) —11(0) tf If the Blasius expression is used for the friction factor we obtain Sh=0.0108Re’’* Sc!’ which is within about 5% of the above value. This is good agreement. 2-5 21B.4: Deposition of silver from a turbulent stream. (a) We may write immediately from the results of the previous problem that dil 14.5 1 dn ReSe!? [f/2 147° where 1 is defined as in the last example. In our situation. However, for our initial purposes it is most convenient to write (Dayo Iv)= Re f/2 = 2.54cme11.Acm/s = 2,863 0.0101em?/s The corresponding Schmidt number is 952 as stated in Fig.21B.3(b). The concentration gradient is then defined by ‘dn 1942" 14 and we are now able to calculate the concentration gradient explicitly in terms of n, ‘We may now calculate the concentration gradient, and the results are shown in Fig. 21B.3(b). However, the abscissa is written in terms of si =sy/v where s is distance measured into the fluid from the surface under consideration. This is done to facilitate comparison with velocity profiles. We can calculate the concentration. profile by integration of the concentration gradient with respect to s*, but this result is not shown on the figure. It rises so rapidly to a limiting value T1(co) - 11(0) = 1.209/1942 = 6.2310 that the initial slope appears infinite on the scale of the figure, and the ordinate is 0.000623 for all readable positions. _ 2864-9.84 1.209¢14.5 polarized, i.e. the silver ion concentration there is essentially. This is also the situation of 1606 Under these conditions the cathode is completely maximum silver flux and the corresponding maximum, or /imiting current. ‘The corresponding Sherwood Number is and the mass transfer coefficient for Ag’ is 1 -(Pae ‘The corresponding silver ion flux 5 1.06110 21606 = 6.710% m/s 254s 12 g—eqts TP = 6.7910"? SA Ni gge = 6.710 Maj? Seats Ss cm cms: uU-7 21B.5 Mixing-length expression for the velocity profile. (a) For steadily driven turbulent flow in a tube, Eq. 5.5-3 gives _ (PoP) in which 7 = 7”) + 7, Integration gives wr, = Po= Pu) oor! in which the integration constant Cy is zero because the other terms vanish at r = 0. Hence, +Co =P (Po— Pr) and the wall shear stress is R(Po-Pu) 2 L Combining the last two equations, we get nr Ry T= Fre = TOG = TO R R (b) Inserting the viscous momentum-flux expression a, 7) = ye Sl oe dy dv, 2 Oa) Tre = 0F (F) into the result of (a), we get the differential equation =. _ for0 =0.412Gr"" Sy © aio, 58" =0.379Gr"> and hl ky, = 9412 Se 0: 85C, 0.379 Pr? It follows that 22-11 Heat of vaporization should be calculated at the freezing point and is about A = 19,313 Btu/1b— mol =10,73Scal / g— mol ‘The molar heat capacity should be evaluated at a somewhat higher but unknown temperature, Since its temperature sensitivity is small we shall use a value of €, » 8Btu/Ib- mole, F = 8cal/ g ~mole,F which corresponds to about 40 F or approximately 5 C. Using the vapor pressure data of Fig. 228.6 we find the relation etween surrounding mole fraction and temperature is 0.00626-%y0 _y.gs 8 T,(1-0.00626) 10,735 or Xyuo = 0.00676 — 6.292107 x, = 0.00626 — 6.299107 ‘This equation is plotted on the attached psychrometric chart, It may be seen here that the maximum possible temperature is about 5.5 C Or about 42 F, and that even for extremely dry air, the maximum is a bit below 10 C or 50 F. We must now see how effective radiation to the night sky can be, and we start with a result already obtained in Prob. 22B.2: a osans26 Xo _o.g5_—8 (+ of, -Ty } T,,(1- 0.00626) 10,735 HT, Note though that radiation now opposes convective heat transfer, and that the radiation flux is independent of local ambient temperature. For the given conditions and a grey surface, with absorptivity equal to emissivity of 0.93, as in Prob. 22B.2, we find cal rag = 1-355+0.93(0.15* 0.273") =-5.04410 cm’ ,s 22-12 ‘We must now calculate the free convection heat transfer coefficient from K Gy02 =0.412—Gr' bn D 3 ‘G with Gr = Depr Vv k= 5.9010" cal/em,s,K (Bejan), D = 50 cm, g = 980 cm/s’, B = I/T = 1/273 = 0.00366, v = 0.133 cm’/s Gr nue 2.54010’ AT 273(0.133)" and Gr? =30.2(AT)? (°C? -5 then h= 0.41210 30,217)? =1.4640°°— oe _ om’ ,s,C°CY and the convective heat transfer rate is eony =1.46e10°(AT)'* cal /cm?,s,(OCY'? ‘We may now calculate the local ambient temperature for which no net evaporation takes place: where radiation out just balances convection in to the water surface. From the left side of the wet-bulb expression this is the temperature at which 30% saturated air has a water vapor mole fraction of 0.00626. From our psychrometric chart this is about 20 C. However, at this temperature 2610 cal/cm*,s conv = which is insufficient to supply the radiative loss. Water vapor must now condense to supply the radiative loss, and local ambient temperature must be well above 20 Cc We could go further, but already it is clear that radiation is very powerful, and that the prediction is greatly at variance with observation. Ice is never found at ambient temperatures even of 20 C. There are many possible reasons for this discrepancy between this simple theory and actual observation. They include 20-13 conduction from the ground, forced convection in the form of wind and turbulence in the boundary layer. This last is unlikely as flows about a cold plate facing upwards are very stable (Rohsenow et al, p. 4.17). However, one of the ‘most important is fog formation (see Ex. 19.5-2),very often seen on still nights. Fog very effectively inhibits radiation. Ice can form at very high ambient temperatures in a desert. Finally we note that the efficiency of our device can be significantly improved by the presence of a small lip as this adds a convective heat transfer resistance without appreciably affecting radiation. A stagnant film has a thermal resistance which can be calculated from Nu=hD/k=D/6 where 8 is the film thickness. Then for 2 mm film h=k/65=(5.9010° cal/em,s.°C)/0.2cm or Npim = 2.95510 cal !em?,s,°C and its resistance is the reciprocal of this quantity. 29-\4 Water Mile Fraction 0.06 0.02 Evaporative Ice maker (no radiation) | ! ] | ] t—Prob. 22B.4 as | 0.04 + L | I T —}—-—Saturati _Energy balance t (no radiation) ~ | 0 5 -__—__| 10 15 20 25 30 35 Temperature, C 22-15 22B.5 Oxygen stripping Once again we must begin by determining terminal velocity, and we begin here with the fundamental momentum balance F= SaR'Apeg = srk py f and rearranging as before we find 4D’ pA Re? p= 4 P Poog 3 My Looking ahead to the inaccuracies involved in using the graphical form of the f(Re) relation shown in Fig. 6.3-1 we make the approximations AD ® Py: Hal pyAp x vz ~10%cm' |? Tt follows that 410° +980 10+ Sketching this f-Re relation on Fig.6.3-1 shows that the Reynolds number is of the order =1.3610* of 10, where f ¥18.5/Re*> Eliminating the friction factor between these two expressions yields Re”* =1.3+10*/18.5 ; Re~ 108 We are now ready to estimate ass transfer rates, and we begin by anticipating the results of Prob. 22B.8 by assuming that essentially all mass transfer resistance occurs in the water phase. The best available correlation is then that Shy © 2+ 0.6Re"”? Sel’? = KD! Day and it remains to determine magnitudes of key system properties. Here we shall assume ‘oxygen solubility to be given by =[2.17-0.0507T +5.604+10%17}{ p/latm) 22-16 where concentration is mmols/L, and temperature is in degrees centigrade [regression of data in J. E. Bailey and D. F, Ollis , Biochemical Engineering Fundamentals, 2end Ed, ‘McGraw-Hill, 1986 — - p. 463}. Then for our circumstances, the entering oxygen concentration is about Co, *1.38mmols/ L = 1.3810 g—mols/ em? For oxygen-water diffusivity we shall use Doyy * (1.04 +0.0537)-10° em? /s [This isa regression of data at 37 C by E. E, Spaeth and S. K. Friedlander, Biophys. J. 1967, p.827 and data at 25 C reported in T. K. Sherwood, et al, Mass transfer 1975, Wiley] It follows that Dow © 2.110% cm? /s Then the Schmidt number Se = 0.0100137 [2.1610°]=476 and 2.1610 cm? /s 0.lem [2+0.6+108"'-476"" |= 0.0106em/s The inward directly oxygen flux at the bubble surface is then Now|, = KeACo, = 0.010106e1.3810 * gmoles/cm*,s 2-1]

Anda mungkin juga menyukai